Sunteți pe pagina 1din 164

Set V Corporation Code * Doctrine of Peircing Corpo Veil Cases*Page 1 of 164

SECOND DIVISION G.R. No. 182770 September 17, 2014 Thereafter, the respondent instituted a complaint for damages against the petitioners, WPM and Manlapaz. The
respondent alleged that in Civil Case No. Q-90-7013, she was adjudged liable for a contract that she entered
WPM INTERNATIONAL TRADING, INC. and WARLITO P. MANLAPAZ, Petitioners, vs. FE CORAZON into for and in behalf of the petitioners, to which she should be entitled to reimbursement; that her
LABAYEN, Respondent. participation in the management agreement was limited only to introducing Manlapaz to Engineer Carmelo Neri
(Neri), CLNs general manager; that it was actually Manlapaz and Neri who agreed on the terms and conditions
of the agreement; that when the complaint for damages was filed against her, she was abroad; and that she
DECISION
did not know of the case until she returned to the Philippines and received a copy of the decision of the RTC.

BRION, J.:
In her prayer, the respondent sought indemnification in the amount of P112,876.60 plus interest at 12%per
annum from June 18, 1990 until fully paid; and 20% of the award as attorneys fees. She likewise prayed that
We review in this petition for review on certiorari1 the decision2 dated September 28, 2007 and the an award ofP100,000.00 as moral damages and P20,000.00 as attorneys fees be paid to her.
resolution3dated April 28, 2008 of the Court of Appeals (CA) in CA-G.R. CV No. 68289 that affirmed with
modification the decision4 of the Regional Trial Court (RTC), Branch 77, Quezon City.
In his defense, Manlapaz claims that it was his fellow incorporator/director Edgar Alcansajewho was in-charge
with the daily operations of the Quickbite outlets; that when Alcansaje left WPM, the remaining directors were
The Factual Background compelled to hire the respondent as manager; that the respondent had entered intothe renovation agreement
with CLN in her own personal capacity; that when he found the amount quoted by CLN too high, he instructed
The respondent, Fe Corazon Labayen, is the owner of H.B.O. Systems Consultants, a management and the respondent to either renegotiate for a lower price or to look for another contractor; that since the
consultant firm. The petitioner, WPM International Trading, Inc. (WPM), is a domestic corporation engaged in the respondent had exceeded her authority as agent of WPM, the renovation agreement should only bind her; and
restaurant business, while Warlito P. Manlapaz (Manlapaz) is its president. that since WPM has a separate and distinct personality, Manlapaz cannot be made liable for the respondents
claim.

Sometime in 1990, WPM entered into a management agreement with the respondent, by virtue of which the
respondent was authorized to operate, manage and rehabilitate Quickbite, a restaurant owned and operated by Manlapaz prayed for the dismissal of the complaint for lack of cause of action, and by way of counterclaim, for
WPM. As part of her tasks, the respondent looked for a contractor who would renovate the two existing the award of P350,000.00 as moral and exemplary damages and P50,000.00 attorneys fees.
Quickbite outlets in Divisoria, Manila and Lepanto St., University Belt, Manila. Pursuant to the agreement, the
respondent engaged the services of CLN Engineering Services (CLN) to renovate Quickbite-Divisoria at the cost The RTC, through an order dated March 2, 1993 declared WPM in default for its failure to file a responsive
ofP432,876.02. pleading.

On June 13, 1990, Quickbite-Divisorias renovation was finally completed, and its possession was delivered to The Decision of the RTC
the respondent. However, out of the P432,876.02 renovation cost, only the amount of P320,000.00 was paid to
CLN, leaving a balance of P112,876.02.
In its decision, the RTC held that the respondent is entitled to indemnity from Manlapaz. The RTC found that
based on the records, there is a clear indication that WPM is a mere instrumentality or business conduit of
Complaint for Sum of Money (Civil Case No. Q-90-7013) Manlapaz and as such, WPM and Manlapaz are considered one and the same. The RTC also found that Manlapaz
had complete control over WPM considering that he is its chairman, president and treasurer at the same time.
On October 19, 1990, CLN filed a complaint for sum of money and damages before the RTC against the The RTC thus concluded that Manlapaz is liable in his personal capacity to reimburse the respondent the
respondent and Manlapaz, which was docketed as Civil Case No. Q-90-7013. CLN later amended the complaint amount she paid to CLN inconnection with the renovation agreement.
to exclude Manlapaz as defendant. The respondent was declared in default for her failure to file a responsive
pleading. The petitioners appealed the RTC decision with the CA. There, they argued that in view of the respondents act
of entering into a renovation agreement with CLN in excess of her authority as WPMs agent, she is not entitled
The RTC, in its January 28, 1991 decision, found the respondent liable to pay CLN actual damages inthe amount to indemnity for the amount she paid. Manlapaz also contended that by virtue ofWPMs separate and distinct
of P112,876.02 with 12% interest per annum from June 18,1990 (the date of first demand) and 20% of the personality, he cannot be madesolidarily liable with WPM.
amount recoverable as attorneys fees.
The Ruling of the Court of Appeals
Complaint for Damages (Civil Case No. Q-92-13446)
Set V Corporation Code * Doctrine of Peircing Corpo Veil Cases*Page 2 of 164

On September 28, 2007, the CA affirmed, with modification on the award of attorneys fees, the decision of the Generally, factual findings of the lower courts are accorded the highest degree of respect, if not finality. When
RTC.The CA held that the petitioners are barred from raising as a defense the respondents alleged lack of adopted and confirmed by the CA, these findings are final and conclusive and may not be reviewed on
authority to enter into the renovation agreement in view of their tacit ratification of the contract. appeal,7save in some recognized exceptions8 among others, when the judgment is based on misapprehension
of facts.
The CA likewise affirmed the RTC ruling that WPM and Manlapaz are one and the same based on the following:
(1) Manlapaz is the principal stockholder of WPM; (2) Manlapaz had complete control over WPM because he We have reviewed the records and found that the application of the principle of piercing the veil of corporate
concurrently held the positions of president, chairman of the board and treasurer, in violation of the fiction is unwarranted in the present case.
Corporation Code; (3) two of the four other stockholders of WPM are employed by Manlapaz either directly or
indirectly; (4) Manlapazs residence is the registered principal office of WPM; and (5) the acronym "WPM" was On the Application ofthe Principle of Piercing the Veil of Corporate Fiction
derived from Manlapazs initials. The CA applied the principle of piercing the veil of corporate fiction and
agreed with the RTC that Manlapaz cannot evade his liability by simply invoking WPMs separate and distinct
The rule is settled that a corporation has a personality separate and distinct from the persons acting for and in
personality.
its behalf and, in general, from the people comprising it. 9 Following this principle, the obligations incurred by
the corporate officers, orother persons acting as corporate agents, are the direct accountabilities ofthe
After the CA's denial of their motion for reconsideration, the petitioners filed the present petition for review on corporation they represent, and not theirs. Thus, a director, officer or employee of a corporation is generally
certiorari under Rule 45 of the Rules of Court. not held personally liable for obligations incurred by the corporation; 10 it is only in exceptional circumstances
that solidary liability will attach to them.
The Petition
Incidentally, the doctrine of piercing the corporate veil applies only in three (3) basic instances, namely: a)
The petitioners submit that the CA gravely erred in sustaining the RTCs application of the principle of piercing when the separate and distinct corporate personality defeats public convenience, as when the corporate fiction
the veil of corporate fiction. They argue that the legal fiction of corporate personality could only be discarded is used as a vehicle for the evasion of an existing obligation; b) in fraud cases, or when the corporate entity is
upon clear and convincing proof that the corporation is being used as a shield to avoid liability or to commit a used to justify a wrong, protect a fraud, or defend a crime; or c) is used in alter ego cases, i.e., where a
fraud. Since the respondent failed to establish that any of the circumstances that would warrant the piercing is corporation is essentially a farce, since it is a mere alter ego or business conduit of a person, or where the
present, Manlapaz claims that he cannot be made solidarily liable with WPM to answerfor damages allegedly corporation is so organized and controlled and its affairs so conducted as to make it merely aninstrumentality,
incurred by the respondent. agency, conduit or adjunct of another corporation.11

The petitioners further argue that, assuming they may be held liable to reimburse to the respondentthe amount Piercing the corporate veil based on the alter ego theory requires the concurrence of three elements, namely:
she paid in Civil Case No. Q-90-7013, such liability is only limited to the amount of P112,876.02, representing
the balance of the obligation to CLN, and should not include the twelve 12% percent interest, damages and (1) Control, not mere majority or complete stock control, but complete domination, not only of
attorneys fees. finances but of policy and business practice in respect to the transaction attacked so that the
corporate entity as to this transaction had at the time no separate mind, will or existence of its own;
The Issues
(2) Such control must have beenused by the defendant to commit fraud or wrong, to perpetuate the
The core issues are: (1) whether WPM is a mere instrumentality, alter-ego, and business conduit of Manlapaz; violation of a statutory or other positive legal duty, or dishonest and unjust act in contravention of
and (2) whether Manlapaz is jointly and severally liable with WPM to the respondent for reimbursement, plaintiffs legal right; and
damages and interest.
(3) The aforesaid control and breach of duty must have proximately caused the injury or unjust loss
Our Ruling complained of.

We find merit in the petition. The absence of any ofthese elements prevents piercing the corporate veil. 12

We note, at the outset, that the question of whether a corporation is a mere instrumentality or alter-ego of In the present case, the attendantcircumstances do not establish that WPM is a mere alter ego of Manlapaz.
another is purely one of fact.5 This is also true with respect to the question of whether the totality of the
evidence adduced by the respondentwarrants the application of the piercing the veil of corporate fiction Aside from the fact that Manlapaz was the principal stockholder of WPM, records do not show that WPM was
doctrine.6 organized and controlled, and its affairs conducted in a manner that made it merely an instrumentality, agency,
Set V Corporation Code * Doctrine of Peircing Corpo Veil Cases*Page 3 of 164

conduit or adjunct ofManlapaz. As held in Martinez v. Court of Appeals,13 the mere ownership by a On the award of moral damages, we find the same in order in view of WPM's unjustified refusal to pay a just
singlestockholder of even all or nearly all of the capital stocks ofa corporation is not by itself a sufficient ground debt. Under Article 2220 of the New Civil Code, 16 moral damages may be awarded in cases of a breach of
to disregard the separate corporate personality. To disregard the separate juridical personality of a corporation, contract where the defendant acted fraudulently or in bad faith or was guilty of gross negligence amounting to
the wrongdoing must be clearly and convincingly established.14 bad faith.

Likewise, the records of the case do not support the lower courts finding that Manlapaz had control or In the present case, when payment for the balance of the renovation cost was demanded, WPM, instead of
domination over WPM or its finances. That Manlapaz concurrentlyheld the positions of president, chairman and complying with its obligation, denied having authorized the respondent to contract in its behalf and accordingly
treasurer, or that the Manlapazs residence is the registered principal office of WPM, are insufficient refused to pay. Such cold refusal to pay a just debt amounts to a breach of contract in bad faith, as
considerations to prove that he had exercised absolutecontrol over WPM. contemplated by Article 2220. Hence, the CA's order to pay moral damages was in order. WHEREFORE, in light
of the foregoing, the decision dated September 28, 2007 of the Court of Appeals in CA-G.R. CV No. 68289 is
In this connection, we stress thatthe control necessary to invoke the instrumentality or alter ego rule is not MODIFIED and.that petitioner Warlito P. Manlapaz is ABSOLVED from any liability under the renovation
majority or even complete stock control but such domination of finances, policies and practices that the agreement.SO ORDERED.
controlled corporation has, so tospeak, no separate mind, will or existence of its own, and is but a conduit for
its principal. The control must be shown to have been exercised at the time the acts complained of took place. SECOND DIVISION G.R. No. 174938 October 1, 2014
Moreover, the control and breach of duty must proximately cause the injury or unjust loss for which the
complaint is made. GERARDO LANUZA, JR. AND ANTONIO O. OLBES, Petitioners, vs.
BF CORPORATION, SHANGRI-LA PROPERTIES, INC., ALFREDO C. RAMOS, RUFO B. COLAYCO, MAXIMO
Here, the respondent failed to prove that Manlapaz, acting as president, had absolute control over G. LICAUCO III, AND BENJAMIN C. RAMOS, Respondents.
WPM.1wphi1 Even granting that he exercised a certain degree of control over the finances, policies and
practices of WPM, in view of his position as president, chairman and treasurer of the corporation, such control DECISION
does not necessarily warrant piercing the veil of corporate fiction since there was not a single proof that WPM
was formed to defraud CLN or the respondent, or that Manlapaz was guilty of bad faith or fraud.
LEONEN, J.:

On the contrary, the evidence establishes that CLN and the respondent knew and acted on the knowledgethat
Corporate representatives may be compelled to submit to arbitration proceedings pursuant to a contract
they were dealing with WPM for the renovation of the latters restaurant, and not with Manlapaz. That WPM
entered into by the corporation they represent if there are allegations of bad faith or malice in their acts
later reneged on its monetary obligation to CLN, resulting to the filing of a civil case for sum of money against
representing the corporation.
the respondent, does not automatically indicate fraud, in the absence of any proof to support it.

This is a Rule 45 petition, assailing the Court of Appeals' May 11, 2006 decision and October 5, 2006 resolution.
This Court also observed that the CA failed to demonstrate how the separate and distinct personalityof WPM
The Court of Appeals affirmed the trial court's decision holding that petitioners, as director, should submit
was used by Manlapaz to defeat the respondents right for reimbursement. Neither was there any showing that
themselves as parties tothe arbitration proceedings between BF Corporation and Shangri-La Properties, Inc.
WPM attempted to avoid liability or had no property against which to proceed.
(Shangri-La).

Since no harm could be said to have been proximately caused by Manlapaz for which the latter could be held
In 1993, BF Corporation filed a collection complaint with the Regional Trial Court against Shangri-Laand the
solidarily liable with WPM, and considering that there was no proof that WPM had insufficient funds, there was
members of its board of directors: Alfredo C. Ramos, Rufo B.Colayco, Antonio O. Olbes, Gerardo Lanuza, Jr.,
no sufficient justification for the RTC and the CA to have ruled that Manlapaz should be held jointly and
Maximo G. Licauco III, and Benjamin C. Ramos. 1
severally liable to the respondent for the amount she paid to CLN. Hence, only WPM is liable to indemnify the
respondent.
BF Corporation alleged in its complaint that on December 11, 1989 and May 30, 1991, it entered into
agreements with Shangri-La wherein it undertook to construct for Shangri-La a mall and a multilevel parking
Finally, we emphasize that the piercing of the veil of corporate fiction is frowned upon and thus, must be done
structure along EDSA.2
with caution.15 It can only be done if it has been clearly established that the separate and distinct personality of
the corporation is used to justify a wrong, protect fraud, or perpetrate a deception. The court must be certain
that the corporate fiction was misused to such an extent that injustice, fraud, or crime was committed against Shangri-La had been consistent in paying BF Corporation in accordance with its progress billing
another, in disregard of its rights; it cannot be presumed. statements.3However, by October 1991, Shangri-La started defaulting in payment.4

On the Award of Moral Damages


Set V Corporation Code * Doctrine of Peircing Corpo Veil Cases*Page 4 of 164

BF Corporation alleged that Shangri-La induced BF Corporation to continue with the construction of the After the Regional Trial Court denied on February 11, 1994 the motion for reconsideration of its November 18,
buildings using its own funds and credit despite Shangri-Las default. 5 According to BF Corporation, ShangriLa 1993 order, Shangri-La, Alfredo C. Ramos, Rufo B. Colayco,Maximo G. Licauco III, and Benjamin Ramos filed a
misrepresented that it had funds to pay for its obligations with BF Corporation, and the delay in payment was petition for certiorari with the Court of Appeals.17
simply a matter of delayed processing of BF Corporations progress billing statements. 6
On April 28, 1995, the Court of Appeals granted the petition for certiorari and ordered the submission of the
BF Corporation eventually completed the construction of the buildings. 7 Shangri-La allegedly took possession of dispute to arbitration.18
the buildings while still owing BF Corporation an outstanding balance.8
Aggrieved by the Court of Appeals decision, BF Corporation filed a petition for review on certiorari with this
BF Corporation alleged that despite repeated demands, Shangri-La refused to pay the balance owed to it. 9 It court.19 On March 27, 1998, this court affirmed the Court of Appeals decision, directing that the dispute be
also alleged that the Shangri-Las directors were in bad faith in directing Shangri-Las affairs. Therefore, they submitted for arbitration.20
should be held jointly and severally liable with Shangri-La for its obligations as well as for the damages that BF
Corporation incurred as a result of Shangri-Las default.10 Another issue arose after BF Corporation had initiated arbitration proceedings. BF Corporation and Shangri-La
failed to agree as to the law that should govern the arbitration proceedings. 21 On October 27, 1998, the trial
On August 3, 1993, Shangri-La, Alfredo C. Ramos, Rufo B. Colayco, Maximo G. Licauco III, and Benjamin C. court issued the order directing the parties to conduct the proceedings in accordance with Republic Act No.
Ramos filed a motion to suspend the proceedings in view of BF Corporations failure to submit its dispute to 876.22
arbitration, in accordance with the arbitration clauseprovided in its contract, quoted in the motion as follows: 11
Shangri-La filed an omnibus motion and BF Corporation an urgent motion for clarification, both seeking to
35. Arbitration clarify the term, "parties," and whether Shangri-Las directors should be included in the arbitration proceedings
and served with separate demands for arbitration. 23
(1) Provided always that in case any dispute or difference shall arise between the Owner or the Project Manager
on his behalf and the Contractor, either during the progress or after the completion or abandonment of the Petitioners filed their comment on Shangri-Las and BF Corporations motions, praying that they be excluded
Works as to the construction of this Contract or as to any matter or thing of whatsoever nature arising there from the arbitration proceedings for being non-parties to Shangri-Las and BF Corporations agreement. 24
under or inconnection therewith (including any matter or thing left by this Contract to the discretion of the
Project Manager or the withholding by the Project Manager of any certificate to which the Contractor may claim On July 28, 2003, the trial court issued the order directing service of demands for arbitration upon all
to be entitled or the measurement and valuation mentioned in clause 30(5)(a) of these Conditions or the rights defendants in BF Corporations complaint.25 According to the trial court, Shangri-Las directors were interested
and liabilities of the parties under clauses 25, 26, 32 or 33 of these Conditions), the owner and the Contractor parties who "must also be served with a demand for arbitration to give them the opportunity to ventilate their
hereby agree to exert all efforts to settle their differences or dispute amicably. Failing these efforts then such side of the controversy, safeguard their interest and fend off their respective positions." 26 Petitioners motion
dispute or difference shall be referred to arbitration in accordance with the rules and procedures of the for reconsideration ofthis order was denied by the trial court on January 19, 2005. 27
Philippine Arbitration Law.

Petitioners filed a petition for certiorari with the Court of Appeals, alleging grave abuse of discretion in the
xxx xxx xxx issuance of orders compelling them to submit to arbitration proceedings despite being third parties to the
contract between Shangri-La and BF Corporation.28
(6) The award of such Arbitrators shall be final and binding on the parties. The decision of the Arbitrators shall
be a condition precedent to any right of legal action that either party may have against the other. . . . In its May 11, 2006 decision, 29 the Court of Appeals dismissed petitioners petition for certiorari. The Court of
12
(Underscoring in the original) Appeals ruled that ShangriLas directors were necessary parties in the arbitration proceedings. 30 According to
the Court of Appeals:
On August 19, 1993, BF Corporation opposed the motion to suspend proceedings. 13
[They were] deemed not third-parties tothe contract as they [were] sued for their acts in representation of the
In the November 18, 1993 order, the Regional Trial Court denied the motion to suspend proceedings. 14 party to the contract pursuant to Art. 31 of the Corporation Code, and that as directors of the defendant
corporation, [they], in accordance with Art. 1217 of the Civil Code, stand to be benefited or injured by the result
On December 8, 1993, petitioners filed an answer to BF Corporations complaint, with compulsory counter of the arbitration proceedings, hence, being necessary parties, they must be joined in order to have complete
claim against BF Corporation and crossclaim against Shangri-La. 15 They alleged that they had resigned as adjudication of the controversy. Consequently, if [they were] excluded as parties in the arbitration proceedings
members of Shangri-Las board of directors as of July 15, 1991. 16 and an arbitral award is rendered, holding [Shangri-La] and its board of directors jointly and solidarily liable to
private respondent BF Corporation, a problem will arise, i.e., whether petitioners will be bound bysuch arbitral
award, and this will prevent complete determination of the issues and resolution of the controversy. 31
Set V Corporation Code * Doctrine of Peircing Corpo Veil Cases*Page 5 of 164

The Court of Appeals further ruled that "excluding petitioners in the arbitration proceedings . . . would be BF Corporation further argued that because petitioners were impleaded for their solidary liability, they are
contrary to the policy against multiplicity of suits."32 necessary parties to the arbitration proceedings. 53 The full resolution of all disputes in the arbitration
proceedings should also be done in the interest of justice. 54
The dispositive portion of the Court of Appeals decision reads:
In the manifestation dated September 6, 2007, petitioners informed the court that the Arbitral Tribunal had
WHEREFORE, the petition is DISMISSED. The assailed orders dated July 28, 2003 and January 19, 2005 of public already promulgated its decision on July 31, 2007. 55 The Arbitral Tribunal denied BF Corporations claims
respondent RTC, Branch 157, Pasig City, in Civil Case No. 63400, are AFFIRMED. 33 against them.56Petitioners stated that "[they] were included by the Arbitral Tribunal in the proceedings
conducted . . . notwithstanding [their] continuing objection thereto. . . ." 57 They also stated that "[their]
unwilling participation in the arbitration case was done ex abundante ad cautela, as manifested therein on
The Court of Appeals denied petitioners motion for reconsideration in the October 5, 2006 resolution. 34
several occasions."58 Petitioners informed the court that they already manifested with the trial court that "any
action taken on [the Arbitral Tribunals decision] should be without prejudice to the resolution of [this] case." 59
On November 24, 2006, petitioners filed a petition for review of the May 11, 2006 Court of Appeals decision
and the October 5, 2006 Court of Appeals resolution.35
Upon the courts order, petitioners and Shangri-La filed their respective memoranda. Petitioners and Maximo G.
Licauco III, Alfredo C. Ramos, and Benjamin C. Ramos reiterated their arguments that they should not be held
The issue in this case is whether petitioners should be made parties to the arbitration proceedings, pursuant to liable for Shangri-Las default and made parties to the arbitration proceedings because only BF Corporation and
the arbitration clause provided in the contract between BF Corporation and Shangri-La. Shangri-La were parties to the contract.

Petitioners argue that they cannot be held personally liable for corporate acts or obligations. 36 The corporation In its memorandum, Shangri-La argued that petitioners were impleaded for their solidary liability under Section
is a separate being, and nothing justifies BF Corporations allegation that they are solidarily liable with Shangri- 31 of the Corporation Code. Shangri-La added that their exclusion from the arbitration proceedings will result in
La.37Neither did they bind themselves personally nor did they undertake to shoulder Shangri-Las obligations multiplicity of suits, which "is not favored in this jurisdiction." 60 It pointed out that the case had already been
should it fail in its obligations.38 BF Corporation also failed to establish fraud or bad faith on their part.39 mooted by the termination of the arbitration proceedings, which petitioners actively participated in. 61 Moreover,
BF Corporation assailed only the correctness of the Arbitral Tribunals award and not the part absolving Shangri-
Petitioners also argue that they are third parties to the contract between BF Corporation and Shangri- Las directors from liability.62
La.40Provisions including arbitration stipulations should bind only the parties. 41 Based on our arbitration laws,
parties who are strangers to an agreement cannot be compelled to arbitrate. 42 BF Corporation filed a counter-manifestation with motion to dismiss 63 in lieu of the required memorandum.

Petitioners point out thatour arbitration laws were enacted to promote the autonomy of parties in resolving In its counter-manifestation, BF Corporation pointed out that since "petitioners counterclaims were already
their disputes.43 Compelling them to submit to arbitration is against this purpose and may be tantamount to dismissed with finality, and the claims against them were likewise dismissed with finality, they no longer have
stipulating for the parties.44 any interest orpersonality in the arbitration case. Thus, there is no longer any need to resolve the present
Petition, which mainly questions the inclusion of petitioners in the arbitration proceedings." 64 The courts
Separate comments on the petition werefiled by BF Corporation, and Maximo G. Licauco III, Alfredo C.Ramos decision in this case will no longer have any effect on the issue of petitioners inclusion in the arbitration
and Benjamin C. Ramos.45 proceedings.65

Maximo G. Licauco III Alfredo C. Ramos, and Benjamin C. Ramos agreed with petitioners that Shangri- The petition must fail.
Lasdirectors, being non-parties to the contract, should not be made personally liable for Shangri-Las
acts.46 Since the contract was executed only by BF Corporation and Shangri-La, only they should be affected by The Arbitral Tribunals decision, absolving petitioners from liability, and its binding effect on BF Corporation,
the contracts stipulation.47 BF Corporation also failed to specifically allege the unlawful acts of the directors have rendered this case moot and academic.
that should make them solidarily liable with Shangri-La for its obligations. 48
The mootness of the case, however, had not precluded us from resolving issues so that principles may be
Meanwhile, in its comment, BF Corporation argued that the courts ruling that the parties should undergo established for the guidance of the bench, bar, and the public. In De la Camara v. Hon. Enage, 66 this court
arbitration "clearly contemplated the inclusion of the directors of the corporation[.]" 49 BF Corporation also disregarded the fact that petitioner in that case already escaped from prison and ruled on the issue of
argued that while petitioners were not parties to the agreement, they were still impleaded under Section 31 of excessive bails:
the Corporation Code.50 Section 31 makes directors solidarily liable for fraud, gross negligence, and bad
faith.51Petitioners are not really third parties to the agreement because they are being sued as Shangri-Las
While under the circumstances a ruling on the merits of the petition for certiorari is notwarranted, still, as set
representatives, under Section 31 of the Corporation Code. 52
forth at the opening of this opinion, the fact that this case is moot and academic should not preclude this
Set V Corporation Code * Doctrine of Peircing Corpo Veil Cases*Page 6 of 164

Tribunal from setting forth in language clear and unmistakable, the obligation of fidelity on the part of lower the asserted dispute, an order to arbitrate should be granted. Any doubt should be resolved in favor of
court judges to the unequivocal command of the Constitution that excessive bail shall not be required. 67 arbitration.74(Emphasis supplied)

This principle was repeated in subsequent cases when this court deemed it proper to clarify important matters A more clear-cut statement of the state policy to encourage arbitration and to favor interpretations that would
for guidance.68 render effective an arbitration clause was later expressed in Republic Act No. 9285: 75

Thus, we rule that petitioners may be compelled to submit to the arbitration proceedings in accordance with SEC. 2. Declaration of Policy.- It is hereby declared the policy of the State to actively promote party autonomy
Shangri-Laand BF Corporations agreement, in order to determine if the distinction between Shangri-Las in the resolution of disputes or the freedom of the party to make their own arrangements to resolve their
personality and their personalities should be disregarded. disputes. Towards this end, the State shall encourage and actively promote the use of Alternative Dispute
Resolution (ADR) as an important means to achieve speedy and impartial justice and declog court dockets. As
This jurisdiction adopts a policy in favor of arbitration. Arbitration allows the parties to avoid litigation and such, the State shall provide means for the use of ADR as an efficient tool and an alternative procedure for the
settle disputes amicably and more expeditiously by themselves and through their choice of arbitrators. resolution of appropriate cases. Likewise, the State shall enlist active private sector participation in the
settlement of disputes through ADR. This Act shall be without prejudice to the adoption by the Supreme Court
of any ADR system, such as mediation, conciliation, arbitration, or any combination thereof as a means of
The policy in favor of arbitration has been affirmed in our Civil Code, 69 which was approved as early as 1949. It
achieving speedy and efficient means of resolving cases pending before all courts in the Philippines which shall
was later institutionalized by the approval of Republic Act No. 876, 70 which expressly authorized, made valid,
be governed by such rules as the Supreme Court may approve from time to time.
enforceable, and irrevocable parties decision to submit their controversies, including incidental issues, to
arbitration. This court recognized this policy in Eastboard Navigation, Ltd. v. Ysmael and Company, Inc.: 71
....

As a corollary to the question regarding the existence of an arbitration agreement, defendant raises the issue
that, even if it be granted that it agreed to submit its dispute with plaintiff to arbitration, said agreement is void SEC. 25. Interpretation of the Act.- In interpreting the Act, the court shall have due regard to the policy of the
and without effect for it amounts to removing said dispute from the jurisdiction of the courts in which the law in favor of arbitration.Where action is commenced by or against multiple parties, one or more of whomare
parties are domiciled or where the dispute occurred. It is true that there are authorities which hold that "a parties who are bound by the arbitration agreement although the civil action may continue as to those who are
clause in a contract providing that all matters in dispute between the parties shall be referred to arbitrators and not bound by such arbitration agreement. (Emphasis supplied)
to them alone, is contrary to public policy and cannot oust the courts of jurisdiction" (Manila Electric Co. vs.
Pasay Transportation Co., 57 Phil., 600, 603), however, there are authorities which favor "the more intelligent Thus, if there is an interpretation that would render effective an arbitration clause for purposes ofavoiding
view that arbitration, as an inexpensive, speedy and amicable method of settling disputes, and as a means of litigation and expediting resolution of the dispute, that interpretation shall be adopted. Petitioners main
avoiding litigation, should receive every encouragement from the courts which may be extended without argument arises from the separate personality given to juridical persons vis--vis their directors, officers,
contravening sound public policy or settled law" (3 Am. Jur., p. 835). Congress has officially adopted the stockholders, and agents. Since they did not sign the arbitration agreement in any capacity, they cannot be
modern view when it reproduced in the new Civil Code the provisions of the old Code on Arbitration. And only forced to submit to the jurisdiction of the Arbitration Tribunal in accordance with the arbitration agreement.
recently it approved Republic Act No. 876 expressly authorizing arbitration of future disputes. 72 (Emphasis Moreover, they had already resigned as directors of Shangri-Laat the time of the alleged default.
supplied)
Indeed, as petitioners point out, their personalities as directors of Shangri-La are separate and distinct from
In view of our policy to adopt arbitration as a manner of settling disputes, arbitration clauses are liberally Shangri-La.
construed to favor arbitration. Thus, in LM Power Engineering Corporation v. Capitol Industrial Construction
Groups, Inc.,73 this court said: A corporation is an artificial entity created by fiction of law. 76 This means that while it is not a person, naturally,
the law gives it a distinct personality and treats it as such. A corporation, in the legal sense, is an individual
Being an inexpensive, speedy and amicable method of settling disputes, arbitration along with mediation, with a personality that is distinct and separate from other persons including its stockholders, officers, directors,
conciliation and negotiation is encouraged by the Supreme Court. Aside from unclogging judicial dockets, representatives,77 and other juridical entities. The law vests in corporations rights,powers, and attributes as if
arbitration also hastens the resolution of disputes, especially of the commercial kind. It is thus regarded as the they were natural persons with physical existence and capabilities to act on their own. 78 For instance, they have
"wave of the future" in international civil and commercial disputes. Brushing aside a contractual agreement the power to sue and enter into transactions or contracts. Section 36 of the Corporation Code enumerates
calling for arbitration between the parties would be a step backward. some of a corporations powers, thus:

Consistent with the above-mentioned policy of encouraging alternative dispute resolution methods, courts Section 36. Corporate powers and capacity. Every corporation incorporated under this Code has the power and
should liberally construe arbitration clauses. Provided such clause is susceptible of an interpretation that covers capacity:
Set V Corporation Code * Doctrine of Peircing Corpo Veil Cases*Page 7 of 164

1. To sue and be sued in its corporate name; Hence, a corporations representatives are generally not bound by the terms of the contract executed by the
corporation. They are not personally liable for obligations and liabilities incurred on or in behalf of the
2. Of succession by its corporate name for the period of time stated in the articles of incorporation and corporation.
the certificate ofincorporation;
Petitioners are also correct that arbitration promotes the parties autonomy in resolving their disputes. This
3. To adopt and use a corporate seal; court recognized in Heirs of Augusto Salas, Jr. v. Laperal Realty Corporation 79 that an arbitration clause shall not
apply to persons who were neither parties to the contract nor assignees of previous parties, thus:

4. To amend its articles of incorporation in accordance with the provisions of this Code;
A submission to arbitration is a contract. As such, the Agreement, containing the stipulation on arbitration,
binds the parties thereto, as well as their assigns and heirs. But only they. 80 (Citations omitted)
5. To adopt by-laws, not contrary to law, morals, or public policy, and to amend or repeal the same in
accordance with this Code;
Similarly, in Del Monte Corporation-USA v. Court of Appeals, 81 this court ruled:

6. In case of stock corporations, to issue or sell stocks to subscribers and to sell treasury stocks in
accordance with the provisions of this Code; and to admit members to the corporation if it be a non- The provision to submit to arbitration any dispute arising therefrom and the relationship of the parties is part of
stock corporation; that contract and is itself a contract. As a rule, contracts are respected as the law between the contracting
parties and produce effect as between them, their assigns and heirs. Clearly, only parties to the Agreement . . .
are bound by the Agreement and its arbitration clause as they are the only signatories thereto. 82 (Citation
7. To purchase, receive, take or grant, hold, convey, sell, lease, pledge, mortgage and otherwise deal
omitted)
with such real and personal property, including securities and bonds of other corporations, as the
transaction of the lawful business of the corporation may reasonably and necessarily require, subject
to the limitations prescribed by law and the Constitution; This court incorporated these rulings in Agan, Jr. v. Philippine International Air Terminals Co., Inc. 83 and Stanfilco
Employees v. DOLE Philippines, Inc., et al. 84

8. To enter into merger or consolidation with other corporations as provided in this Code;
As a general rule, therefore, a corporations representative who did not personally bind himself or herself to an
arbitration agreement cannot be forced to participate in arbitration proceedings made pursuant to an
9. To make reasonable donations, including those for the public welfare or for hospital, charitable,
agreement entered into by the corporation. He or she is generally not considered a party to that agreement.
cultural, scientific, civic, or similar purposes: Provided, That no corporation, domestic or foreign, shall
give donations in aid of any political party or candidate or for purposes of partisan political activity;
However, there are instances when the distinction between personalities of directors, officers,and
representatives, and of the corporation, are disregarded. We call this piercing the veil of corporate fiction.
10. To establish pension, retirement, and other plans for the benefit of its directors, trustees, officers
and employees; and
Piercing the corporate veil is warranted when "[the separate personality of a corporation] is used as a means to
perpetrate fraud or an illegal act, or as a vehicle for the evasion of an existing obligation, the circumvention of
11. To exercise such other powers asmay be essential or necessary to carry out its purpose or
statutes, or to confuse legitimate issues."85 It is also warranted in alter ego cases "where a corporation is
purposes as stated in its articles of incorporation. (13a)
merely a farce since it is a mere alter ego or business conduit of a person, or where the corporation is so
organized and controlled and its affairs are so conducted as to make it merely an instrumentality, agency,
Because a corporations existence is only by fiction of law, it can only exercise its rights and powers through conduit or adjunct of another corporation."86
itsdirectors, officers, or agents, who are all natural persons. A corporation cannot sue or enter into contracts
without them.
When corporate veil is pierced, the corporation and persons who are normally treated as distinct from the
corporation are treated as one person, such that when the corporation is adjudged liable, these persons, too,
A consequence of a corporations separate personality is that consent by a corporation through its become liable as if they were the corporation.
representatives is not consent of the representative, personally. Its obligations, incurred through official acts of
its representatives, are its own. A stockholder, director, or representative does not become a party to a
Among the persons who may be treatedas the corporation itself under certain circumstances are its directors
contract just because a corporation executed a contract through that stockholder, director or representative.
and officers. Section 31 of the Corporation Code provides the instances when directors, trustees, or officers
may become liable for corporate acts:
Set V Corporation Code * Doctrine of Peircing Corpo Veil Cases*Page 8 of 164

Sec. 31. Liability of directors, trustees or officers. - Directors or trustees who willfully and knowingly vote for or The corporations distinct personality is disregarded, and the corporation is seen as a mere aggregation of
assent to patently unlawful acts of the corporation or who are guilty of gross negligence or bad faith in persons undertaking a business under the collective name of the corporation.
directing the affairs of the corporation or acquire any personal or pecuniary interest in conflict with their duty
as such directors or trustees shall be liable jointly and severally for all damages resulting therefrom suffered by Hence, when the directors, as in this case, are impleaded in a case against a corporation, alleging malice orbad
the corporation, its stockholders or members and other persons. faith on their part in directing the affairs of the corporation, complainants are effectively alleging that the
directors and the corporation are not acting as separate entities. They are alleging that the acts or omissions
When a director, trustee or officer attempts to acquire or acquires, in violation of his duty, any interest adverse by the corporation that violated their rights are also the directors acts or omissions. 90 They are alleging that
to the corporation in respect of any matter which has been reposed inhim in confidence, as to which equity contracts executed by the corporation are contracts executed by the directors. Complainants effectively pray
imposes a disability upon him to deal in his own behalf, he shall be liable as a trustee for the corporation and that the corporate veilbe pierced because the cause of action between the corporation and the directors is the
must account for the profits which otherwise would have accrued to the corporation. (n) same.

Based on the above provision, a director, trustee, or officer of a corporation may be made solidarily liable with In that case, complainants have no choice but to institute only one proceeding against the
it for all damages suffered by the corporation, its stockholders or members, and other persons in any of the parties.1wphi1 Under the Rules of Court, filing of multiple suits for a single cause of action is prohibited.
following cases: Institution of more than one suit for the same cause of action constitutes splitting the cause of action, which is
a ground for the dismissal ofthe others. Thus, in Rule 2:
a) The director or trustee willfully and knowingly voted for or assented to a patently unlawful
corporate act; Section 3. One suit for a single cause of action. A party may not institute more than one suit for a single
cause of action. (3a)
b) The director or trustee was guilty of gross negligence or bad faith in directing corporate affairs; and
Section 4. Splitting a single cause of action;effect of. If two or more suits are instituted on the basis of the
c) The director or trustee acquired personal or pecuniary interest in conflict with his or her duties as same cause of action, the filing of one or a judgment upon the merits in any one is available as a ground for the
director or trustee. dismissal of the others. (4a)

Solidary liability with the corporation will also attach in the following instances: It is because the personalities of petitioners and the corporation may later be found to be indistinct that we rule
that petitioners may be compelled to submit to arbitration.

a) "When a director or officer has consented to the issuance of watered stocks or who, having
knowledge thereof, did not forthwith file with the corporate secretary his written objection thereto"; 87 However, in ruling that petitioners may be compelled to submit to the arbitration proceedings, we are not
overturning Heirs of Augusto Salas wherein this court affirmed the basic arbitration principle that only parties to
an arbitration agreement may be compelled to submit to arbitration. In that case, this court recognizedthat
b) "When a director, trustee or officer has contractually agreed or stipulated to hold himself personally
persons other than the main party may be compelled to submit to arbitration, e.g., assignees and heirs.
and solidarily liable with the corporation";88 and
Assignees and heirs may be considered parties to an arbitration agreement entered into by their assignor
because the assignors rights and obligations are transferred to them upon assignment. In other words, the
c) "When a director, trustee or officer is made, by specific provision of law, personally liable for his assignors rights and obligations become their own rights and obligations. In the same way, the corporations
corporate action."89 obligations are treated as the representatives obligations when the corporate veil is pierced. Moreover, in Heirs
of Augusto Salas, this court affirmed its policy against multiplicity of suits and unnecessary delay. This court
When there are allegations of bad faith or malice against corporate directors or representatives, it becomes the said that "to split the proceeding into arbitration for some parties and trial for other parties would "result in
duty of courts or tribunals to determine if these persons and the corporation should be treated as one. Without multiplicity of suits, duplicitous procedure and unnecessary delay." 91 This court also intimated that the interest
a trial, courts and tribunals have no basis for determining whether the veil of corporate fiction should be of justice would be best observed if it adjudicated rights in a single proceeding. 92 While the facts of that case
pierced. Courts or tribunals do not have such prior knowledge. Thus, the courts or tribunals must first prompted this court to direct the trial court to proceed to determine the issues of thatcase, it did not prohibit
determine whether circumstances exist towarrant the courts or tribunals to disregard the distinction between courts from allowing the case to proceed to arbitration, when circumstances warrant.
the corporation and the persons representing it. The determination of these circumstances must be made by
one tribunal or court in a proceeding participated in by all parties involved, including current representatives of Hence, the issue of whether the corporations acts in violation of complainants rights, and the incidental issue
the corporation, and those persons whose personalities are impliedly the sameas the corporation. This is of whether piercing of the corporate veil is warranted, should be determined in a single proceeding. Such
because when the court or tribunal finds that circumstances exist warranting the piercing of the corporate veil, finding would determine if the corporation is merely an aggregation of persons whose liabilities must be treated
the corporate representatives are treated as the corporation itself and should be held liable for corporate acts. as one with the corporation.
Set V Corporation Code * Doctrine of Peircing Corpo Veil Cases*Page 9 of 164

However, when the courts disregard the corporations distinct and separate personality from its directors or AGUSTIN, EFREN RIVERA, CRISALDO VALERO, SAFIA HANDANG, LUCENA R. MEDINA, DANNY BOY B.
officers, the courts do not say that the corporation, in all instances and for all purposes, is the same as its PANGASIAN, ABDURASA HASIL, ROEL ALTA, JOBERT BELTRAN, EDNA FAUSTO, TAJMAHAR HADJULA,
directors, stockholders, officers, and agents. It does not result in an absolute confusion of personalities of the ELENA MAGHANOY, ERIC B. QUITIOL, JESSE D. FLORES, GEMMA CANILLAS, ERNITO CANILLAS,
corporation and the persons composing or representing it. Courts merely discount the distinction and treat MARILOU JAVIER, MARGANI MADDIN, RICHARD SENA, FE D. CANOY, GEORGE SALUD, EDGARDO
them as one, in relation to a specific act, in order to extend the terms of the contract and the liabilities for all BORGONIA, JR., ANTONIO ATILANO, JOSE CASTRO, and LIBERATO BAGALANON, Petitioners,
damages to erring corporate officials who participated in the corporations illegal acts. This is done so that the vs. MAR FISHING CO., INC., MIRAMAR FISHING CO., INC., ROBERT BUEHS AND JEROME
legal fiction cannot be used to perpetrate illegalities and injustices. SPITZ.Respondents.

Thus, in cases alleging solidary liability with the corporation or praying for the piercing of the corporate veil, DECISION
parties who are normally treated as distinct individuals should be made to participate in the arbitration
proceedings in order to determine ifsuch distinction should indeed be disregarded and, if so, to determine the SERENO, J.:
extent of their liabilities.

Before this Court is a Petition for Review on Certiorari under Rule 45 of the Revised Rules of Court, seeking a
In this case, the Arbitral Tribunal rendered a decision, finding that BF Corporation failed to prove the existence review of the Court of Appeals (CA) 19 March 2004 and 12 May 2005 Resolutions in CA-G.R. SP NO. 82651. The
of circumstances that render petitioners and the other directors solidarily liable. It ruled that petitioners and appellate court had dismissed the Petition for Review on the ground that it lacked a Verification and
Shangri-Las other directors were not liable for the contractual obligations of Shangri-La to BF Corporation. The Certification against forum shopping.
Arbitral Tribunals decision was made with the participation of petitioners, albeit with their continuing objection.
In view of our discussion above, we rule that petitioners are bound by such decision.WHEREFORE, the petition
The pertinent facts are as follows:
is DENIED. The Court of Appeals' decision of May 11, 2006 and resolution of October 5, 2006 are AFFIRMED.SO
ORDERED.
On 28 June 2001, respondent Mar Fishing Co., Inc. (Mar Fishing), engaged in the business of fishing and canning
of tuna, sold its principal assets to co-respondent Miramar Fishing Co., Inc. (Miramar) through public
SECOND DIVISIONG.R. No. 168208 June 13, 2012
bidding.1 The proceeds of the sale were paid to the Trade and Investment Corporation of the Philippines
(TIDCORP) to cover Mar Fishings outstanding obligation in the amount of P 897,560,041.26.2 In view of that
transfer, Mar Fishing issued a Memorandum dated 23 October 2001 informing all its workers that the company
would cease to operate by the end of the month. 3 On 29 October 2001 or merely two days prior to the months
VIVIAN T. RAMIREZ, ALBERTO B. DIGNO, DANILO M. CASQUITE, JUMADIYA A. KADIL, FAUJIA SALIH, end, it notified the Department of Labor and Employment (DOLE) of the closure of its business operations. 4
ANTONIO FABIAN, ROMEL DANAG, GINA PANTASAN, ARTHUR MATUGAS, VIRGILIA OSARIO,
ORLANDO EBRADA, ROSANA CABATO, WILFREDO LUNA, LILIA BARREDO, ISABEL ALBERTO, NORA Thereafter, Mar Fishings labor union, Mar Fishing Workers Union NFL and Miramar entered into a
BONIAO, PILAR OSARIO, LYDIA ESLIT, AMMAN SALI, AKMAD AKIL, ROGELIO LAZARO, ISABEL Memorandum of Agreement.5 The Agreement provided that the acquiring company, Miramar, shall absorb Mar
CONCILLADO, MARLON ABIAL, HERMOCILLO NAPALCRUZ, WALTER BUHIAN, ELISEO AMATORIO, JOSE Fishings regular rank and file employees whose performance was satisfactory, without loss of seniority rights
CASTRO, JAMIL LAGBAY, MA. EVELYN SANTOS, LEDENIA T. BARON, ELSA AMATORIO, SARAH F. and privileges previously enjoyed.6
BUCOY, EXPEDITO L. RELUYA, ARNULFO ALFARO, EDGARDO F. BORGONIA, DANILO R. MANINGO,
ABDUSAID H. DAMBONG, LORINDA M. MUTIA, DOMINADOR DEL ROSARIO, JOEL E. TRONO, HUSSIN
Unfortunately, petitioners, who worked as rank and file employees, were not hired or given separation pay by
A. JAWAJI, JUL-ASNAM JAKARIA, LUZVIMINDA A. NOLASCO, VILMA G. GASCO, MORITA S. MARMETO,
Miramar.7 Thus, petitioners filed Complaints for illegal dismissal with money claims before the Arbitration
PROCESA JUANICO, ANTONIO A. MONDRAGON, JR., JESSICA F. QUIACHON, PACITA G. MEDINA, ARNEL
Branch of the National Labor Relations Commission (NLRC).
S. SANTOS, ANECITA T. TARAS, TOMINDAO T. TARAS, NULCA C. SABDANI, AKMAD A. SABDANI,
ROWENA J. GARCIA, LINA P. CASAS, MARLYN G. FRANCISCO, MERCEDITA MAQUINANO, NICOLAS T.
RIO, TERESITA A. CASINAS, VIRGILIO F. IB-IB, PANTALEON S. ROJAS, JR., EVELYN V. BEATINGO, In its 30 July 2002 Decision, the Labor Arbiter (LA) found that Mar Fishing had necessarily closed its operations,
MATILDE G. HUSSIN, ESPERANZA I. LLEDO, ADOLFINA DELA MERCED, LAURA E. SANTOS, considering that Miramar had already bought the tuna canning plant. 8 By reason of the closure, petitioners
ROGACIANA MAQUILING, ALELIE D. SAMSON, SHIRLEY L. ALVAREZ, MAGDALENA A. MARCOS, were legally dismissed for authorized cause. 9 In addition, even if Mar Fishing reneged on notifying the DOLE
VIRGINIA S. ESPINOSA, ANTONIO C. GUEVARA, AUGUSTA S. DE JESUS, SERVILLA A. BANCALE, within 30 days prior to its closure, that failure did not make the dismissals void. Consequently, the LA ordered
PROSERFINA GATINAO, RASMA A. FABRIGA, ROLANDO D. GATINAO, ANALISA G. MEA, SARAH A. Mar Fishing to give separation pay to its workers.10
SALCEDO, ALICIA M. JAYAG, FERNANDO G. CABEROY, ROMEO R. PONCE, EDNA S. PONCE, TEODORA
T. LUY, WALDERICO F. ARIO, MELCHOR S. BUCOY, EDITA H. CINCO, RUDY I. LIMBAROC, PETER The LA held thus:11
MONTOJO, MARLYN S. ATILANO, REGIDOR MEDALLO, EDWIN O. DEMASUAY, DENNIS M. SUICANO,
ROSALINA Q. ATILANO, ESTRELLA FELICIANO, IMELDA T. DAGALEA, MARILYN RUFINO, JOSE
Set V Corporation Code * Doctrine of Peircing Corpo Veil Cases*Page 10 of 164

WHEREFORE, in view of the foregoing considerations, judgment is hereby rendered in these cases: Anent the liberality in application of the rules, as alleged by petitioners, the same deserves scant
consideration. x x x.
1. Ordering Mar Fishing Company, Inc., through its president, treasurer, manager or other proper
officer or representative, to pay the complainants their respective separation pay, as computed in xxx. While litigation is not a game of technicalities, and that the rules of procedure should not be enforced
page 12 to 33 hereof, all totaling SIX MILLION THREE HUNDRED THIRTY SIX THOUSAND FIVE HUNDRED strictly at the cost of substantial justice, still it does not follow that the Rules of Court may be ignored at will
EIGHTY SEVEN & 77/100 PESOS (P 6,336,587.77); and at random to the prejudice of the orderly presentation, assessment and just resolution of the issues. xxx.

2. Dismissing these case [sic] as against Miramar Fishing Company, Inc., as well as against Robert Before this Court, 124 petitioners raise the issue of whether the CA gravely erred in dismissing their Petition for
Buehs and Jerome Spitz, for lack of cause of action; Review on the ground that their pleading lacked a Verification and Certification against forum shopping. 23

3. Dismissing all other charges and claims of the complainants, for lack of merit.SO ORDERED. The Rules of Court provide that a petition for certiorari must be verified and accompanied by a sworn
certification of non-forum shopping.24 Failure to comply with these mandatory requirements shall be sufficient
ground for the dismissal of the petition. 25 Considering that only 3 of the 228 named petitioners signed the
requirement, the CA dismissed the case against them, as they did not execute a Verification and Certification
against forum shopping.
Aggrieved, petitioners pursued the action before the NLRC, which modified the LAs Decision. Noting that Mar
Fishing notified the DOLE only two days before the business closed, the labor court considered petitioners
dismissal as ineffectual.12 Hence, it awarded, apart from separation pay, full back wages to petitioners from the Petitioners invoke substantial compliance with procedural rules when their Manifestation already contains a
time they were terminated on 31 October 2001 until the date when the LA upheld the validity of their dismissal Verification and Certification against forum shopping executed by 161 signatories. They heavily rely on Jaro v.
on 30 July 2002.13 Court of Appeals,26 citing Piglas-Kamao v. National Labor Relations Commission and Cusi-Hernandez v. Diaz, in
which we discussed that the subsequent submission of the missing documentary attachments with the Motion
for Reconsideration amounted to substantial compliance.
Additionally, the NLRC pierced the veil of corporate fiction and ruled that Mar Fishing and Miramar were one
and the same entity, since their officers were the same. 14 Hence, both companies were ordered to solidarily pay
the monetary claims.15 However, this very case does not involve a failure to attach the Annexes. Rather, the procedural infirmity
consists of omission the failure to sign a Verification and Certification against forum shopping. Addressing this
defect squarely, we have already resolved that because of noncompliance with the requirements governing the
On reconsideration, the NLRC modified its ruling by imposing liability only on Mar Fishing. The labor court held
certification of non-forum shopping, no error could be validly attributed to the CA when it ordered the dismissal
that petitioners had no cause of action against Miramar, since labor contracts cannot be enforced against the
of the special civil action for certiorari. 27 The lack of certification against forum shopping is not curable by mere
transferee of an enterprise in the absence of a stipulation in the contract that the transferee assumes the
amendment of a complaint, but shall be a cause for the dismissal of the case without prejudice. 28 Indeed, the
obligation of the transferor.16 Hence, the dispositive portion reads:17
general rule is that subsequent compliance with the requirements will not excuse a party's failure to comply in
the first instance.29 Thus, on procedural aspects, the appellate court correctly dismissed the case.
WHEREFORE, foregoing premises considered, the assailed resolution is MODIFIED in that only Mar Fishing
Company, Inc. through its responsible officers, is ordered to pay complainants their separation pay, and full
However, this Court has recognized that the merit of a case is a special circumstance or compelling reason that
backwages from the date they were terminated from employment until 30 July 2002, subject to computation
justifies the relaxation of the rule requiring verification and certification of non-forum shopping. 30 In order to
during execution stage of proceedings at the appropriate Regional Arbitration Branch.SO ORDERED.
fully resolve the issue, it is thus necessary to determine whether technical rules were brushed aside at the
expense of substantial justice.31 This Court will then delve into the issue on (1) the solidary liability of Mar
Despite the award of separation pay and back wages, petitioners filed a Rule 65 Petition before the CA. This Fishing and Miramar to pay petitioners monetary claims and (2) the reckoning period for the award of back
time, they argued that both Mar Fishing and Miramar should be made liable for their separation pay, and that wages.
their back wages should be up to the time of their actual reinstatement. However, finding that only 3 of the 228
petitioners18signed the Verification and Certification against forum shopping, the CA instantly dismissed the
For a dismissal based on the closure of business to be valid, three (3) requirements must be established. Firstly,
action for certiorari against the 225 other petitioners without ruling on the substantive aspects of the case. 19
the cessation of or withdrawal from business operations must be bona fide in character. Secondly, there must
be payment to the employees of termination pay amounting to at least one-half (1/2) month pay for each year
By means of a Manifestation with Omnibus Motion, 20 petitioners submitted a Verification and Certification of service, or one (1) month pay, whichever is higher. Thirdly, the company must serve a written notice on the
against forum shopping executed by 161 signatories. In the said pleading, petitioners asked the CA to employees and on the DOLE at least one (1) month before the intended termination. 32
reconsider by invoking the rule that technical rules do not strictly apply to labor cases. 21 Still, the CA denied
petitioners contentions and held thus:22
Set V Corporation Code * Doctrine of Peircing Corpo Veil Cases*Page 11 of 164

In their Petition for Review on Certiorari, petitioners did not dispute the conclusion of the LA and the NLRC that the lack of appropriate remedies, this Court will no longer dwell on the exact computation of petitioners claims
Mar Fishing had an authorized cause to dismiss its workers. Neither did petitioners challenge the computation for back wages, which have been sufficiently threshed out by the LA and the NLRC. Judicial review of labor
of their separation pay. cases does not go beyond an evaluation of the sufficiency of the evidence upon which labor officials' findings
rest.41
Rather, they questioned the holding that only Mar Fishing was liable for their monetary claims. 33
While we sympathize with the situation of the workers in this case, we cannot disregard, absent compelling
Basing their conclusion on the Memorandum of Agreement and Supplemental Agreement between Miramar and reasons, the factual determinations and the legal doctrines that support the findings of the courts a quo.
Mar Fishings labor union, as well as the General Information Sheets and Company Profiles of the two Generally, the findings of fact and the conclusion of the labor courts are not only accorded great weight and
companies, petitioners assert that Miramar simply took over the operations of Mar Fishing. In addition, they respect, but are even clothed with finality and deemed binding on this Court, as long as they are supported by
assert that these companies are one and the same entity, given the commonality of their directors and the substantial evidence.42
similarity of their business venture in tuna canning plant operations. 34
On a final note, this Court reminds the parties seeking the ultimate relief of certiorari to observe the rules,
At the fore, the question of whether one corporation is merely an alter ego of another is purely one of fact since nonobservance thereof cannot be brushed aside as a "mere technicality." 43 Procedural rules are not to be
generally beyond the jurisdiction of this Court. 35 In any case, given only these bare reiterations, this Court belittled or simply disregarded, for these prescribed procedures ensure an orderly and speedy administration of
sustains the ruling of the LA as affirmed by the NLRC that Miramar and Mar Fishing are separate and distinct justice.44IN VIEW THEREOF, the assailed 19 March 2004 and 12 May 2005 Resolutions of the Court of Appeals in
entities, based on the marked differences in their stock ownership. 36 Also, the fact that Mar Fishings officers CA-GR SP NO. 82651 are AFFIRMED. Hence, the 04 July 2005 Petition for Review filed by petitioners is hereby
remained as such in Miramar does not by itself warrant a conclusion that the two companies are one and the denied for lack of merit.SO ORDERED.
same. As this Court held in Sesbreo v. Court of Appeals, the mere showing that the corporations had a
common director sitting in all the boards without more does not authorize disregarding their separate juridical SECOND DIVISION G.R. No. 185280 January 18, 2012
personalities.37
TIMOTEO H. SARONA, Petitioner, vs. NATIONAL LABOR RELATIONS COMMISSION, ROYALE SECURITY
Neither can the veil of corporate fiction between the two companies be pierced by the rest of petitioners AGENCY (FORMERLY SCEPTRE SECURITY AGENCY) and CESAR S. TAN, Respondents.
submissions, namely, the alleged take-over by Miramar of Mar Fishings operations and the evident similarity of
their businesses. At this point, it bears emphasizing that since piercing the veil of corporate fiction is frowned DECISION
upon, those who seek to pierce the veil must clearly establish that the separate and distinct personalities of the
corporations are set up to justify a wrong, protect a fraud, or perpetrate a deception. 38 This, unfortunately,
REYES, J.:
petitioners have failed to do. In Indophil Textile Mill Workers Union vs. Calica, we ruled thus: 39

This is a petition for review under Rule 45 of the Rules of Court from the May 29, 2008 Decision 1 of the
In the case at bar, petitioner seeks to pierce the veil of corporate entity of Acrylic, alleging that the creation of
Twentieth Division of the Court of Appeals (CA) in CA-G.R. SP No. 02127 entitled "Timoteo H. Sarona v. National
the corporation is a devi[c]e to evade the application of the CBA between petitioner Union and private
Labor Relations Commission, Royale Security Agency (formerly Sceptre Security Agency) and Cesar S.
respondent company. While we do not discount the possibility of the similarities of the businesses of private
Tan" (Assailed Decision), which affirmed the National Labor Relations Commissions (NLRC) November 30, 2005
respondent and Acrylic, neither are we inclined to apply the doctrine invoked by petitioner in granting the relief
Decision and January 31, 2006 Resolution, finding the petitioner illegally dismissed but limiting the amount of
sought. The fact that the businesses of private respondent and Acrylic are related, that some of the employees
his backwages to three (3) monthly salaries. The CA likewise affirmed the NLRCs finding that the petitioners
of the private respondent are the same persons manning and providing for auxiliary services to the units of
separation pay should be computed only on the basis of his length of service with respondent Royale Security
Acrylic, and that the physical plants, offices and facilities are situated in the same compound, it is our
Agency (Royale). The CA held that absent any showing that Royale is a mere alter ego of Sceptre Security
considered opinion that these facts are not sufficient to justify the piercing of the corporate veil of Acrylic.
Agency (Sceptre), Royale cannot be compelled to recognize the petitioners tenure with Sceptre. The
(Emphasis supplied.)
dispositive portion of the CAs Assailed Decision states:

Having been found by the trial courts to be a separate entity, Mar Fishing and not Miramar is required to
WHEREFORE, in view of the foregoing, the instant petition is PARTLY GRANTED, though piercing of the corporate
compensate petitioners. Indeed, the back wages and retirement pay earned from the former employer cannot
veil is hereby denied for lack of merit. Accordingly, the assailed Decision and Resolution of the NLRC
be filed against the new owners or operators of an enterprise.40
respectively dated November 30, 2005 and January 31, 2006 are hereby AFFIRMED as to the monetary
awards.SO ORDERED. 2
Evidently, the assertions of petitioners fail on both procedural and substantive aspects.1wphi1 Therefore, no
special reasons exist to reverse the CAs dismissal of the case due to their failure to abide by the mandatory
Factual Antecedents
procedure for filing a petition for review on certiorari. Given the correctness of the appellate courts ruling and
Set V Corporation Code * Doctrine of Peircing Corpo Veil Cases*Page 12 of 164

On June 20, 2003, the petitioner, who was hired by Sceptre as a security guard sometime in April 1976, was "It is clear, deliberate and unjustified refusal to severe employment and not mere absence that is required to
asked by Karen Therese Tan (Karen), Sceptres Operation Manager, to submit a resignation letter as the same constitute abandonment." x x x" (De Ysasi III vs. NLRC, 231 SCRA 173).
was supposedly required for applying for a position at Royale. The petitioner was also asked to fill up Royales
employment application form, which was handed to him by Royales General Manager, respondent Cesar Aside from lack of proof showing that complainant has abandoned his employment, the record would show that
Antonio Tan II (Cesar).3 immediate action was taken in order to protest his dismissal from employment. He filed a complaint [for] illegal
dismissal on October 4, 2004 or three (3) days after he was dismissed. This act, as declared by the Supreme
After several weeks of being in floating status, Royales Security Officer, Martin Gono (Martin), assigned the Court is inconsistent with abandonment, as held in the case of Pampanga Sugar Development Co., Inc. vs.
petitioner at Highlight Metal Craft, Inc. (Highlight Metal) from July 29, 2003 to August 8, 2003. Thereafter, the NLRC, 272 SCRA 737 where the Supreme Court ruled:
petitioner was transferred and assigned to Wide Wide World Express, Inc. (WWWE, Inc.). During his assignment
at Highlight Metal, the petitioner used the patches and agency cloths of Sceptre and it was only "The immediate filing of a complaint for [i]llegal [d]ismissal by an employee is inconsistent with
when he was posted at WWWE, Inc. that he started using those of Royale. 4 abandonment."7

On September 17, 2003, the petitioner was informed that his assignment at WWWE, Inc. had been withdrawn The respondents were ordered to pay the petitioner backwages, which LA Gutierrez computed from the day he
because Royale had allegedly been replaced by another security agency. The petitioner, however, shortly was dismissed, or on October 1, 2003, up to the promulgation of his Decision on May 11, 2005. In lieu of
discovered thereafter that Royale was never replaced as WWWE, Inc.s security agency. When he placed a call reinstatement, the respondents were ordered to pay the petitioner separation pay equivalent to his one (1)
at WWWE, Inc., he learned that his fellow security guard was not relieved from his post. 5 month salary in consideration of his tenure with Royale, which lasted for only one (1) month and three (3) days.
In this regard, LA Gutierrez refused to pierce Royales corporate veil for purposes of factoring the petitioners
On September 21, 2003, the petitioner was once again assigned at Highlight Metal, albeit for a short period length of service with Sceptre in the computation of his separation pay. LA Gutierrez ruled that Royales
from September 22, 2003 to September 30, 2003. Subsequently, when the petitioner reported at Royales corporate personality, which is separate and distinct from that of Sceptre, a sole proprietorship owned by the
office on October 1, 2003, Martin informed him that he would no longer be given any assignment per the late Roso Sabalones (Roso) and later, Aida, cannot be pierced absent clear and convincing evidence that
instructions of Aida Sabalones-Tan (Aida), general manager of Sceptre. This prompted him to file a complaint for Sceptre and Royale share the same stockholders and incorporators and that Sceptre has complete control and
illegal dismissal on October 4, 2003.6 dominion over the finances and business affairs of Royale. Specifically:

In his May 11, 2005 Decision, Labor Arbiter Jose Gutierrez (LA Gutierrez) ruled in the petitioners favor and To support its prayer of piercing the veil of corporate entity of respondent Royale, complainant avers that
found him illegally dismissed. For being unsubstantiated, LA Gutierrez denied credence to the respondents respondent Royal (sic) was using the very same office of SCEPTRE in C. Padilla St., Cebu City. In addition, all
claim that the termination of the petitioners employment relationship with Royale was on his accord following officers and staff of SCEPTRE are now the same officers and staff of ROYALE, that all [the] properties of
his alleged employment in another company. That the petitioner was no longer interested in being an employee SCEPTRE are now being owned by ROYALE and that ROYALE is now occupying the property of SCEPTRE. We are
of Royale cannot be presumed from his request for a certificate of employment, a claim which, to begin with, not however, persuaded.
he vehemently denies. Allegation of the petitioners abandonment is negated by his filing of a complaint for
illegal dismissal three (3) days after he was informed that he would no longer be given any assignments. LA It should be pointed out at this juncture that SCEPTRE, is a single proprietorship. Being so, it has no distinct and
Gutierrez ruled: separate personality. It is owned by the late Roso T. Sabalones. After the death of the owner, the property is
supposed to be divided by the heirs and any claim against the sole proprietorship is a claim against Roso T.
In short, respondent wanted to impress before us that complainant abandoned his employment. We are not Sabalones. After his death, the claims should be instituted against the estate of Roso T. Sabalones. In short, the
however, convinced. estate of the late Roso T. Sabalones should have been impleaded as respondent of this case.

There is abandonment when there is a clear proof showing that one has no more interest to return to work. In Complainant wanted to impress upon us that Sceptre was organized into another entity now called Royale
this instant case, the record has no proof to such effect. In a long line of decisions, the Supreme Court ruled: Security Agency. There is however, no proof to this assertion. Likewise, there is no proof that Roso T. Sabalones,
organized his single proprietorship business into a corporation, Royale Security Agency. On the contrary, the
"Abandonment of position is a matter of intention expressed in clearly certain and unequivocal acts, however, name of Roso T. Sabalones does not appear in the Articles of Incorporation. The names therein as incorporators
an interim employment does not mean abandonment." (Jardine Davis, Inc. vs. NLRC, 225 SCRA 757). are:

"In abandonment, there must be a concurrence of the intention to abandon and some overt acts from which an Bruno M. Kuizon - [P]150,000.00
employee may be declared as having no more interest to work." (C. Alcontin & Sons, Inc. vs. NLRC, 229 SCRA
Wilfredo K. Tan - 100,000.00
109).
Set V Corporation Code * Doctrine of Peircing Corpo Veil Cases*Page 13 of 164

Karen Therese S. Tan - 100,000.00 On the other hand, while complainant is entitled to backwages, We are aware that his stint with respondent
Royal (sic) lasted only for one (1) month and three (3) days such that it is Our considered view that his
Cesar Antonio S. Tan - 100,000.00 backwages should be limited to only three (3) months.

Gabeth Maria K. Tan - 50,000.00


Backwages:
Complainant claims that two (2) of the incorporators are the granddaughters of Roso T. Sabalones. This fact
even give (sic) us further reason to conclude that respondent Royal (sic) Security Agency is not an alter ego or
conduit of SCEPTRE. It is obvious that respondent Royal (sic) Security Agency is not owned by the owner of [P]5,200.00 x 3 months = [P]15,600.0012
"SCEPTRE".
The petitioner, on the other hand, did not appeal LA Gutierrezs May 11, 2005 Decision but opted to raise the
It may be true that the place where respondent Royale hold (sic) office is the same office formerly used by validity of LA Gutierrezs adverse findings with respect to piercing Royales corporate personality and
"SCEPTRE." Likewise, it may be true that the same officers and staff now employed by respondent Royale computation of his separation pay in his Reply to the respondents Memorandum of Appeal. As the filing of an
Security Agency were the same officers and staff employed by "SCEPTRE." We find, however, that these facts appeal is the prescribed remedy and no aspect of the decision can be overturned by a mere reply, the NLRC
are not sufficient to justify to require respondent Royale to answer for the liability of Sceptre, which was owned dismissed the petitioners efforts to reverse LA Gutierrezs disposition of these issues. Effectively, the petitioner
solely by the late Roso T. Sabalones. As we have stated above, the remedy is to address the claim on the estate had already waived his right to question LA Gutierrezs Decision when he failed to file an appeal within the
of Roso T. Sabalones.8 reglementary period. The NLRC held:

The respondents appealed LA Gutierrezs May 11, 2005 Decision to the NLRC, claiming that the finding of illegal On the other hand, in complainants Reply to Respondents Appeal Memorandum he prayed that the doctrine of
dismissal was attended with grave abuse of discretion. This appeal was, however, dismissed by the NLRC in its piercing the veil of corporate fiction of respondent be applied so that his services with Sceptre since 1976 [will
November 30, 2005 Decision,9 the dispositive portion of which states: not] be deleted. If complainant assails this particular finding in the Labor Arbiters Decision, complainant should
have filed an appeal and not seek a relief by merely filing a Reply to Respondents Appeal Memorandum. 13
WHEREFORE, premises considered, the Decision of the Labor Arbiter declaring the illegal dismissal of
complainant is hereby AFFIRMED. Consequently, the petitioner elevated the NLRCs November 30, 2005 Decision to the CA by way of a Petition
forCertiorari under Rule 65 of the Rules of Court. On the other hand, the respondents filed no appeal from the
NLRCs finding that the petitioner was illegally dismissed.
However[,] We modify the monetary award by limiting the grant of backwages to only three (3) months in view
of complainants very limited service which lasted only for one month and three days.
The CA, in consideration of substantial justice and the jurisprudential dictum that an appealed case is thrown
open for the appellate courts review, disagreed with the NLRC and proceeded to review the evidence on record
1. Backwages - [P]15,600.00 to determine if Royale is Sceptres alter ego that would warrant the piercing of its corporate veil. 14 According to

2. Separation Pay - 5,200.00 the CA, errors not assigned on appeal may be reviewed as technicalities should not serve as bar to the full
adjudication of cases. Thus:
3. 13th Month Pay - 583.34

[P]21,383.34 Attorney's Fees - 2,138.33 In Cuyco v. Cuyco, which We find application in the instant case, the Supreme Court held:

Total [P]23,521.67
"In their Reply, petitioners alleged that their petition only raised the sole issue of interest on the interest due,
The appeal of respondent Royal (sic) Security Agency is hereby DISMISSED for lack of merit. SO ORDERED.10 thus, by not filing their own petition for review, respondents waived their privilege to bring matters for the
Courts review that [does] not deal with the sole issue raised.
The NLRC partially affirmed LA Gutierrezs May 11, 2005 Decision. It concurred with the latters finding that the
petitioner was illegally dismissed and the manner by which his separation pay was computed, but modified the Procedurally, the appellate court in deciding the case shall consider only the assigned errors, however, it is
monetary award in the petitioners favor by reducing the amount of his backwages from P95,600.00 equally settled that the Court is clothed with ample authority to review matters not assigned as errors in an
to P15,600.00. The NLRC determined the petitioners backwages as limited to three (3) months of his last appeal, if it finds that their consideration is necessary to arrive at a just disposition of the case."
monthly salary, considering that his employment with Royale was only for a period for one (1) month and three
(3) days, thus:11
Therefore, for full adjudication of the case, We have to primarily resolve the issue of whether the doctrine of
piercing the corporate veil be justly applied in order to determine petitioners length of service with private
respondents.15 (citations omitted)
Set V Corporation Code * Doctrine of Peircing Corpo Veil Cases*Page 14 of 164

Nonetheless, the CA ruled against the petitioner and found the evidence he submitted to support his allegation decision.17 The petitioner would likewise have this Court review and examine anew the factual allegations and
that Royale and Sceptre are one and the same juridical entity to be wanting. The CA refused to pierce Royales the supporting evidence to determine if the CA erred in its refusal to pierce Royales corporate mask and rule
corporate mask as one of the "probative factors that would justify the application of the doctrine of piercing the that it is but a mere continuation or successor of Sceptre. According to the petitioner, the erroneous
corporate veil is stock ownership by one or common ownership of both corporations" and the petitioner failed computation of his separation pay was due to the CAs failure, as well as the NLRC and LA Gutierrez, to
to present clear and convincing proof that Royale and Sceptre are commonly owned or controlled. The relevant consider evidence conclusively demonstrating that Royale and Sceptre are one and the same juridical entity.
portions of the CAs Decision state: The petitioner claims that since Royale is no more than Sceptres alter ego, it should recognize and credit his
length of service with Sceptre.18
In the instant case, We find no evidence to show that Royale Security Agency, Inc. (hereinafter "Royale"), a
corporation duly registered with the Securities and Exchange Commission (SEC) and Sceptre Security Agency The petitioner claimed that Royale and Sceptre are not separate legal persons for purposes of computing the
(hereinafter "Sceptre"), a single proprietorship, are one and the same entity. amount of his separation pay and other benefits under the Labor Code. The piercing of Royales corporate
personality is justified by several indicators that Royale was incorporated for the sole purpose of defeating his
Petitioner, who has been with Sceptre since 1976 and, as ruled by both the Labor Arbiter and the NLRC, was right to security of tenure and circumvent payment of his benefits to which he is entitled under the law: (i)
illegally dismissed by Royale on October 1, 2003, alleged that in order to circumvent labor laws, especially to Royale was holding office in the same property used by Sceptre as its principal place of business; 19 (ii) Sceptre
avoid payment of money claims and the consideration on the length of service of its employees, Royale was and Royal have the same officers and employees; 20 (iii) on October 14, 1994, Roso, the sole proprietor of
established as an alter ego or business conduit of Sceptre. To prove his claim, petitioner declared that Royale is Sceptre, sold to Aida, and her husband, Wilfredo Gracia K. Tan (Wilfredo), 21 the property used by Sceptre as its
conducting business in the same office of Sceptre, the latter being owned by the late retired Gen. Roso principal place of business;22 (iv) Wilfredo is one of the incorporators of Royale; 23 (v) on May 3, 1999, Roso
Sabalones, and was managed by the latters daughter, Dr. Aida Sabalones-Tan; that two of Royales ceded the license to operate Sceptre issued by the Philippine National Police to Aida; 24 (vi) on July 28, 1999, the
incorporators are grandchildren [of] the late Gen. Roso Sabalones; that all the properties of Sceptre are now business name "Sceptre Security & Detective Agency" was registered with the Department of Trade and
owned by Royale, and that the officers and staff of both business establishments are the same; that the heirs of Industry (DTI) under the name of Aida; 25 (vii) Aida exercised control over the affairs of Sceptre and Royale, as
Gen. Sabalones should have applied for dissolution of Sceptre before the SEC before forming a new she was, in fact, the one who dismissed the petitioner from employment; 26 (viii) Karen, the daughter of Aida,
corporation. was Sceptres Operation Manager and is one of the incorporators of Royale; 27 and (ix) Cesar Tan II, the son of
Aida was one of Sceptres officers and is one of the incorporators of Royale. 28

On the other hand, private respondents declared that Royale was incorporated only on March 10, 2003 as
evidenced by the Certificate of Incorporation issued by the SEC on the same date; that Royales incorporators In their Comment, the respondents claim that the petitioner is barred from questioning the manner by which
are Bruino M. Kuizon, Wilfredo Gracia K. Tan, Karen Therese S. Tan, Cesar Antonio S. Tan II and [Gabeth] Maria K. his backwages and separation pay were computed. Earlier, the petitioner moved for the execution of the
Tan. NLRCs November 30, 2005 Decision 29 and the respondents paid him the full amount of the monetary award
thereunder shortly after the writ of execution was issued. 30 The respondents likewise maintain that Royales
separate and distinct corporate personality should be respected considering that the evidence presented by
Settled is the tenet that allegations in the complaint must be duly proven by competent evidence and the
the petitioner fell short of establishing that Royale is a mere alter ego of Sceptre.
burden of proof is on the party making the allegation. Further, Section 1 of Rule 131 of the Revised Rules of
Courtprovides:
The petitioner does not deny that he has received the full amount of backwages and separation pay as
provided under the NLRCs November 30, 2005 Decision.31 However, he claims that this does not preclude this
"SECTION 1. Burden of proof. Burden of proof is the duty of a party to present evidence on the facts in issue
Court from modifying a decision that is tainted with grave abuse of discretion or issued without jurisdiction. 32
necessary to establish his claim or defense by the amount of evidence required by law."

ISSUES
We believe that petitioner did not discharge the required burden of proof to establish his allegations. As We see
it, petitioners claim that Royale is an alter ego or business conduit of Sceptre is without basis because aside
from the fact that there is no common ownership of both Royale and Sceptre, no evidence on record would Considering the conflicting submissions of the parties, a judicious determination of their respective rights and
prove that Sceptre, much less the late retired Gen. Roso Sabalones or his heirs, has control or complete obligations requires this Court to resolve the following substantive issues:
domination of Royales finances and business transactions. Absence of this first element, coupled by
petitioners failure to present clear and convincing evidence to substantiate his allegations, would prevent a. Whether Royales corporate fiction should be pierced for the purpose of compelling it to recognize
piercing of the corporate veil. Allegations must be proven by sufficient evidence. Simply stated, he who alleges the petitioners length of service with Sceptre and for holding it liable for the benefits that have
a fact has the burden of proving it; mere allegation is not evidence. 16 (citations omitted) accrued to him arising from his employment with Sceptre; and

By way of this Petition, the petitioner would like this Court to revisit the computation of his backwages, claiming b. Whether the petitioners backwages should be limited to his salary for three (3) months.
that the same should be computed from the time he was illegally dismissed until the finality of this
Set V Corporation Code * Doctrine of Peircing Corpo Veil Cases*Page 15 of 164

OUR RULING resolutions of the CA in any case, i.e., regardless of the nature of the action or proceedings involved, which
would be but a continuation of the appellate process over the original case. 38 Since an appeal to this Court is
Because his receipt of the proceeds of the award under the NLRCs November 30, 2005 Decision is qualified not an original and independent action but a continuation of the proceedings before the CA, the filing of a
and without prejudice to the CAs resolution of his petition for certiorari, the petitioner is not barred from petition for review under Rule 45 cannot be barred by the finality of the NLRCs decision in the same way that a
exercising his right to elevate the decision of the CA to this Court. petition for certiorari under Rule 65 with the CA cannot.

Before this Court proceeds to decide this Petition on its merits, it is imperative to resolve the respondents Furthermore, if the NLRCs decision or resolution was reversed and set aside for being issued with grave abuse
contention that the full satisfaction of the award under the NLRCs November 30, 2005 Decision bars the of discretion by way of a petition for certiorari to the CA or to this Court by way of an appeal from the decision
petitioner from questioning the validity thereof. The respondents submit that they had paid the petitioner the of the CA, it is considered void ab initio and, thus, had never become final and executory. 39
amount of P21,521.67 as directed by the NLRC and this constitutes a waiver of his right to file an appeal to this
Court. A Rule 45 Petition should be confined to questions of law. Nevertheless, this Court has the power to resolve a
question of fact, such as whether a corporation is a mere alter ego of another entity or whether the corporate
The respondents fail to convince. fiction was invoked for fraudulent or malevolent ends, if the findings in assailed decision is not supported by
the evidence on record or based on a misapprehension of facts.

The petitioners receipt of the monetary award adjudicated by the NLRC is not absolute, unconditional and
unqualified. The petitioners May 3, 2007 Motion for Release contains a reservation, stating in his prayer that: The question of whether one corporation is merely an alter ego of another is purely one of fact. So is the
"it is respectfully prayed that the respondents and/or Great Domestic Insurance Co. be ordered to question of whether a corporation is a paper company, a sham or subterfuge or whether the petitioner adduced
RELEASE/GIVE the amount of P23,521.67 in favor of the complainant TIMOTEO H. SARONA without prejudice to the requisite quantum of evidence warranting the piercing of the veil of the respondents corporate
the outcome of the petition with the CA."33 personality.40

In Leonis Navigation Co., Inc., et al. v. Villamater, et al.,34 this Court ruled that the prevailing partys receipt of As a general rule, this Court is not a trier of facts and a petition for review on certiorari under Rule 45 of the
the full amount of the judgment award pursuant to a writ of execution issued by the labor arbiter does not Rules of Court must exclusively raise questions of law. Moreover, if factual findings of the NLRC and the LA have
close or terminate the case if such receipt is qualified as without prejudice to the outcome of the petition been affirmed by the CA, this Court accords them the respect and finality they deserve. It is well-settled and
for certioraripending with the CA.1avvphi1 oft-repeated that findings of fact of administrative agencies and quasi-judicial bodies, which have acquired
expertise because their jurisdiction is confined to specific matters, are generally accorded not only respect, but
finality when affirmed by the CA. 41
Simply put, the execution of the final and executory decision or resolution of the NLRC shall proceed despite
the pendency of a petition for certiorari, unless it is restrained by the proper court. In the present case,
petitioners already paid Villamaters widow, Sonia, the amount of P3,649,800.00, representing the total and Nevertheless, this Court will not hesitate to deviate from what are clearly procedural guidelines and disturb and
permanent disability award plus attorneys fees, pursuant to the Writ of Execution issued by the Labor Arbiter. strike down the findings of the CA and those of the labor tribunals if there is a showing that they are
Thereafter, an Order was issued declaring the case as "closed and terminated". However, although there was unsupported by the evidence on record or there was a patent misappreciation of facts. Indeed, that the
no motion for reconsideration of this last Order, Sonia was, nonetheless, estopped from claiming that the impugned decision of the CA is consistent with the findings of the labor tribunals does not per se conclusively
controversy had already reached its end with the issuance of the Order closing and terminating the case. This demonstrate the correctness thereof. By way of exception to the general rule, this Court will scrutinize the facts
is because the Acknowledgment Receipt she signed when she received petitioners payment was without if only to rectify the prejudice and injustice resulting from an incorrect assessment of the evidence presented.
prejudice to the final outcome of the petition for certiorari pending before the CA.35
A resolution of an issue that has supposedly become final and executory as the petitioner only raised it in his
The finality of the NLRCs decision does not preclude the filing of a petition for certiorari under Rule 65 of the reply to the respondents appeal may be revisited by the appellate court if such is necessary for a just
Rules of Court. That the NLRC issues an entry of judgment after the lapse of ten (10) days from the parties disposition of the case.
receipt of its decision36 will only give rise to the prevailing partys right to move for the execution thereof but
will not prevent the CA from taking cognizance of a petition for certiorari on jurisdictional and due process As above-stated, the NLRC refused to disturb LA Gutierrezs denial of the petitioners plea to pierce Royales
considerations.37 In turn, the decision rendered by the CA on a petition for certiorari may be appealed to this corporate veil as the petitioner did not appeal any portion of LA Gutierrezs May 11, 2005 Decision.
Court by way of a petition for review on certiorari under Rule 45 of the Rules of Court. Under Section 5, Article
VIII of the Constitution, this Court has the power to "review, revise, reverse, modify, or affirm on appeal In this respect, the NLRC cannot be accused of grave abuse of discretion. Under Section 4(c), Rule VI of the
orcertiorari as the law or the Rules of Court may provide, final judgments and orders of lower courts in x x x all NLRC Rules,42 the NLRC shall limit itself to reviewing and deciding only the issues that were elevated on appeal.
cases in which only an error or question of law is involved." Consistent with this constitutional mandate, Rule The NLRC, while not totally bound by technical rules of procedure, is not licensed to disregard and violate the
45 of the Rules of Court provides the remedy of an appeal by certiorari from decisions, final orders or implementing rules it implemented. 43
Set V Corporation Code * Doctrine of Peircing Corpo Veil Cases*Page 16 of 164

Nonetheless, technicalities should not be allowed to stand in the way of equitably and completely resolving the sole control of Sceptres finances and business affairs, Aida took over as early as 1999 when Roso assigned his
rights and obligations of the parties. Technical rules are not binding in labor cases and are not to be applied license to operate Sceptre on May 3, 1999. 50 As further proof of Aidas acquisition of the rights as Sceptres sole
strictly if the result would be detrimental to the working man. 44 This Court may choose not to encumber itself proprietor, she caused the registration of the business name "Sceptre Security & Detective Agency" under her
with technicalities and limitations consequent to procedural rules if such will only serve as a hindrance to its name with the DTI a few months after Roso abdicated his rights to Sceptre in her favor. 51 As far as Royale is
duty to decide cases judiciously and in a manner that would put an end with finality to all existing conflicts concerned, the respondents do not deny that she has a hand in its management and operation and possesses
between the parties. control and supervision of its employees, including the petitioner. As the petitioner correctly pointed out, that
Aida was the one who decided to stop giving any assignments to the petitioner and summarily dismiss him is
Royale is a continuation or successor of Sceptre. an eloquent testament of the power she wields insofar as Royales affairs are concerned. The presence of
actual common control coupled with the misuse of the corporate form to perpetrate oppressive or manipulative
conduct or evade performance of legal obligations is patent; Royale cannot hide behind its corporate fiction.
A corporation is an artificial being created by operation of law. It possesses the right of succession and such
powers, attributes, and properties expressly authorized by law or incident to its existence. It has a personality
separate and distinct from the persons composing it, as well as from any other legal entity to which it may be Aidas control over Sceptre and Royale does not, by itself, call for a disregard of the corporate fiction. There
related. This is basic.45 must be a showing that a fraudulent intent or illegal purpose is behind the exercise of such control to warrant
the piercing of the corporate veil.52 However, the manner by which the petitioner was made to resign from
Sceptre and how he became an employee of Royale suggest the perverted use of the legal fiction of the
Equally well-settled is the principle that the corporate mask may be removed or the corporate veil pierced
separate corporate personality.lavvphil It is undisputed that the petitioner tendered his resignation and that he
when the corporation is just an alter ego of a person or of another corporation. For reasons of public policy and
applied at Royale at the instance of Karen and Cesar and on the impression they created that these were
in the interest of justice, the corporate veil will justifiably be impaled only when it becomes a shield for fraud,
necessary for his continued employment. They orchestrated the petitioners resignation from Sceptre and
illegality or inequity committed against third persons.46
subsequent employment at Royale, taking advantage of their ascendancy over the petitioner and the latters
lack of knowledge of his rights and the consequences of his actions. Furthermore, that the petitioner was made
Hence, any application of the doctrine of piercing the corporate veil should be done with caution. A court to resign from Sceptre and apply with Royale only to be unceremoniously terminated shortly thereafter leads to
should be mindful of the milieu where it is to be applied. It must be certain that the corporate fiction was the ineluctable conclusion that there was intent to violate the petitioners rights as an employee, particularly
misused to such an extent that injustice, fraud, or crime was committed against another, in disregard of rights. his right to security of tenure. The respondents scheme reeks of bad faith and fraud and compassionate justice
The wrongdoing must be clearly and convincingly established; it cannot be presumed. Otherwise, an injustice dictates that Royale and Sceptre be merged as a single entity, compelling Royale to credit and recognize the
that was never unintended may result from an erroneous application. 47 petitioners length of service with Sceptre. The respondents cannot use the legal fiction of a separate corporate
personality for ends subversive of the policy and purpose behind its creation 53 or which could not have been
Whether the separate personality of the corporation should be pierced hinges on obtaining facts appropriately intended by law to which it owed its being.54
pleaded or proved. However, any piercing of the corporate veil has to be done with caution, albeit the Court will
not hesitate to disregard the corporate veil when it is misused or when necessary in the interest of justice. After For the piercing doctrine to apply, it is of no consequence if Sceptre is a sole proprietorship. As ruled in Prince
all, the concept of corporate entity was not meant to promote unfair objectives. 48 Transport, Inc., et al. v. Garcia, et al.,55 it is the act of hiding behind the separate and distinct personalities of
juridical entities to perpetuate fraud, commit illegal acts, evade ones obligations that the equitable piercing
The doctrine of piercing the corporate veil applies only in three (3) basic areas, namely: 1) defeat of public doctrine was formulated to address and prevent:
convenience as when the corporate fiction is used as a vehicle for the evasion of an existing obligation; 2)
fraud cases or when the corporate entity is used to justify a wrong, protect fraud, or defend a crime; or 3) alter A settled formulation of the doctrine of piercing the corporate veil is that when two business enterprises are
ego cases, where a corporation is merely a farce since it is a mere alter ego or business conduit of a person, or owned, conducted and controlled by the same parties, both law and equity will, when necessary to protect the
where the corporation is so organized and controlled and its affairs are so conducted as to make it merely an rights of third parties, disregard the legal fiction that these two entities are distinct and treat them as identical
instrumentality, agency, conduit or adjunct of another corporation. 49 or as one and the same. In the present case, it may be true that Lubas is a single proprietorship and not a
corporation. However, petitioners attempt to isolate themselves from and hide behind the supposed separate
In this regard, this Court finds cogent reason to reverse the CAs findings. Evidence abound showing that and distinct personality of Lubas so as to evade their liabilities is precisely what the classical doctrine of
Royale is a mere continuation or successor of Sceptre and fraudulent objectives are behind Royales piercing the veil of corporate entity seeks to prevent and remedy.56
incorporation and the petitioners subsequent employment therein. These are plainly suggested by events that
the respondents do not dispute and which the CA, the NLRC and LA Gutierrez accept as fully substantiated but Also, Sceptre and Royale have the same principal place of business. As early as October 14, 1994, Aida and
misappreciated as insufficient to warrant the use of the equitable weapon of piercing. Wilfredo became the owners of the property used by Sceptre as its principal place of business by virtue of a
Deed of Absolute Sale they executed with Roso.57 Royale, shortly after its incorporation, started to hold office in
As correctly pointed out by the petitioner, it was Aida who exercised control and supervision over the affairs of the same property. These, the respondents failed to dispute.
both Sceptre and Royale. Contrary to the submissions of the respondents that Roso had been the only one in
Set V Corporation Code * Doctrine of Peircing Corpo Veil Cases*Page 17 of 164

The respondents do not likewise deny that Royale and Sceptre share the same officers and employees. Karen As to reinstatement of petitioners, this Court has already ruled that reinstatement is no longer feasible,
assumed the dual role of Sceptres Operation Manager and incorporator of Royale. With respect to the because the company would be adjustly prejudiced by the continued employment of petitioners who at present
petitioner, even if he has already resigned from Sceptre and has been employed by Royale, he was still using are overage, a separation pay equal to one-month salary granted to them in the Labor Arbiter's decision was in
the patches and agency cloths of Sceptre during his assignment at Highlight Metal. order and, therefore, affirmed on the Court's decision of 15 March 1996. Furthermore, since reinstatement on
this case is no longer feasible, the amount of backwages shall be computed from the time of their illegal
Royale also claimed a right to the cash bond which the petitioner posted when he was still with Sceptre. If termination on 25 June 1990 up to the time of finality of this decision. 63 (emphasis supplied)
Sceptre and Royale are indeed separate entities, Sceptre should have released the petitioners cash bond when
he resigned and Royale would have required the petitioner to post a new cash bond in its favor. A further clarification was made in Javellana, Jr. v. Belen:64

Taking the foregoing in conjunction with Aidas control over Sceptres and Royales business affairs, it is patent Article 279 of the Labor Code, as amended by Section 34 of Republic Act 6715 instructs:
that Royale was a mere subterfuge for Aida. Since a sole proprietorship does not have a separate and distinct
personality from that of the owner of the enterprise, the latter is personally liable. This is what she sought to Art. 279. Security of Tenure. - In cases of regular employment, the employer shall not terminate the services of
avoid but cannot prosper. an employee except for a just cause or when authorized by this Title. An employee who is unjustly dismissed
from work shall be entitled to reinstatement without loss of seniority rights and other privileges and to his full
Effectively, the petitioner cannot be deemed to have changed employers as Royale and Sceptre are one and backwages, inclusive of allowances, and to his other benefits or their monetary equivalent computed from the
the same. His separation pay should, thus, be computed from the date he was hired by Sceptre in April 1976 time his compensation was withheld from him up to the time of his actual reinstatement.
until the finality of this decision. Based on this Courts ruling in Masagana Concrete Products, et al. v. NLRC, et
al.,58 the intervening period between the day an employee was illegally dismissed and the day the decision Clearly, the law intends the award of backwages and similar benefits to accumulate past the date of the Labor
finding him illegally dismissed becomes final and executory shall be considered in the computation of his Arbiter's decision until the dismissed employee is actually reinstated. But if, as in this case, reinstatement is no
separation pay as a period of "imputed" or "putative" service: longer possible, this Court has consistently ruled that backwages shall be computed from the time of illegal
dismissal until the date the decision becomes final. 65 (citation omitted)
Separation pay, equivalent to one month's salary for every year of service, is awarded as an alternative to
reinstatement when the latter is no longer an option. Separation pay is computed from the commencement of In case separation pay is awarded and reinstatement is no longer feasible, backwages shall be computed from
employment up to the time of termination, including the imputed service for which the employee is entitled to the time of illegal dismissal up to the finality of the decision should separation pay not be paid in the
backwages, with the salary rate prevailing at the end of the period of putative service being the basis for meantime. It is the employees actual receipt of the full amount of his separation pay that will effectively
computation.59 terminate the employment of an illegally dismissed employee.66 Otherwise, the employer-employee relationship
subsists and the illegally dismissed employee is entitled to backwages, taking into account the increases and
It is well-settled, even axiomatic, that if reinstatement is not possible, the period covered in the computation of other benefits, including the 13th month pay, that were received by his co-employees who are not
backwages is from the time the employee was unlawfully terminated until the finality of the decision finding dismissed.67 It is the obligation of the employer to pay an illegally dismissed employee or worker the whole
illegal dismissal. amount of the salaries or wages, plus all other benefits and
bonuses and general increases, to which he would have been normally entitled had he not been dismissed and
With respect to the petitioners backwages, this Court cannot subscribe to the view that it should be limited to had not stopped working.68
an amount equivalent to three (3) months of his salary. Backwages is a remedy affording the employee a way
to recover what he has lost by reason of the unlawful dismissal. 60 In awarding backwages, the primordial In fine, this Court holds Royale liable to pay the petitioner backwages to be computed from his dismissal on
consideration is the income that should have accrued to the employee from the time that he was dismissed up October 1, 2003 until the finality of this decision. Nonetheless, the amount received by the petitioner from the
to his reinstatement61 and the length of service prior to his dismissal is definitely inconsequential. respondents in satisfaction of the November 30, 2005 Decision shall be deducted accordingly.

As early as 1996, this Court, in Bustamante, et al. v. NLRC, et al.,62 clarified in no uncertain terms that if Finally, moral damages and exemplary damages at P25,000.00 each as indemnity for the petitioners dismissal,
reinstatement is no longer possible, backwages should be computed from the time the employee was which was tainted by bad faith and fraud, are in order. Moral damages may be recovered where the dismissal of
terminated until the finality of the decision, finding the dismissal unlawful. the employee was tainted by bad faith or fraud, or where it constituted an act oppressive to labor, and done in
a manner contrary to morals, good customs or public policy while exemplary damages are recoverable only if
Therefore, in accordance with R.A. No. 6715, petitioners are entitled on their full backwages, inclusive of the dismissal was done in a wanton, oppressive, or malevolent manner. 69
allowances and other benefits or their monetary equivalent, from the time their actual compensation was
withheld on them up to the time of their actual reinstatement. WHEREFORE, premises considered, the Petition is hereby GRANTED. We REVERSE and SET ASIDE the CAs May
29, 2008 Decision in C.A.-G.R. SP No. 02127 and order the respondents to pay the petitioner the following
Set V Corporation Code * Doctrine of Peircing Corpo Veil Cases*Page 18 of 164

minus the amount of (P23,521.67) paid to the petitioner in satisfaction of the NLRCs November 30, 2005 On August 2, 1993, Ma. Concepcion Lacsa (Concepcion) and her sister, Miriam Lacsa (Miriam), boarded a
Decision in NLRC Case No. V-000355-05: Goldline passenger bus with Plate No. NXM-105 owned and operated by Travel &Tours Advisers, Inc. They were
enroute from Sorsogon to Cubao, Quezon City. 6 At the time, Concepcion, having just obtained her degree of
a) full backwages and other benefits computed from October 1, 2003 (the date Royale illegally Bachelor of Science in Nursing at the Ago Medical and Educational Center, was proceeding to Manila to take the
dismissed the petitioner) until the finality of this decision; nursing licensure board examination.7 Upon reaching the highway at Barangay San Agustin in Pili, Camarines
Sur, the Goldline bus, driven by Rene Abania (Abania), collided with a passenger jeepney with Plate No. EAV-
313 coming from the opposite direction and driven by Alejandro Belbis. 8 As a result, a metal part of the jeepney
b) separation pay computed from April 1976 until the finality of this decision at the rate of one month
was detached and struck Concepcion in the chest, causing her instant death. 9
pay per year of service;

On August 23, 1993, Concepcions heirs, represented by Teodoro Lacsa, instituted in the RTC a suit against
c) ten percent (10%) attorneys fees based on the total amount of the awards under (a) and (b) above;
Travel & Tours Advisers Inc. and Abania to recover damages arising from breach of contract of carriage. 10 The
complaint, docketed as Civil Case No. 93-5917 and entitled Heirs of Concepcion Lacsa, represented by Teodoro
d) moral damages of Twenty-Five Thousand Pesos (P25,000.00); and Lacsa v. Travel & Tours Advisers, Inc. (Goldline) and Rene Abania, alleged that the collision was due to the
reckless and imprudent manner by which Abania had driven the Goldline bus. 11
e) exemplary damages of Twenty-Five Thousand Pesos (P25,000.00).
In support of the complaint, Miriam testified that Abania had been occasionally looking up at the video monitor
This case is REMANDED to the labor arbiter for computation of the separation pay, backwages, and other installed in the front portion of the Goldline bus despite driving his bus at a fast speed; 12 that in Barangay San
monetary awards due the petitioner.SO ORDERED. Agustin, the Goldline bus had collided with a service jeepney coming from the opposite direction while in the
process of overtaking another bus; 13 that the impact had caused the angle bar of the jeepney to detach and to
go through the windshield of the bus directly into the chest of Concepcion who had then been seated behind
FIRST DIVISION G.R. No. 159108 June 18, 2012
the drivers seat;14 that concerned bystanders had hailed another bus to rush Concepcion to the Ago
Foundation Hospital in Naga City because the Goldline bus employees and her co-passengers had ignored
GOLD LINE TOURS, INC., Petitioner, vs. Miriams cries for help;15 and that Concepcion was pronounced dead upon arrival at the hospital. 16
HEIRS OF MARIA CONCEPCION LACSA, Respondents.

To refute the plaintiffs allegations, the defendants presented SPO1 Pedro Corporal of the Philippine National
DECISION Police Station in Pili, Camarines Sur, and William Cheng, the operator of the Goldline bus. 17 SPO1 Corporal
opined that based on his investigation report, the driver of the jeepney had been at fault for failing to observe
BERSAMIN, J.: precautionary measures to avoid the collision;18 and suggested that criminal and civil charges should be
brought against the operator and driver of the jeepney. 19 On his part, Cheng attested that he had exercised the
required diligence in the selection and supervision of his employees; and that he had been engaged in the
The veil of corporate existence of a corporation is a fiction of law that should not defeat the ends of justice.
transportation business since 1980 with the use of a total of 60 units of Goldline buses, employing about 100
employees (including drivers, conductors, maintenance personnel, and mechanics); 20 that as a condition for
Petitioner seeks to reverse the decision promulgated on October 30, 2002 1 and the resolution promulgated on regular employment, applicant drivers had undergone a one-month training period and a six-month
June 25, 2003,2 whereby the Court of Appeals (CA) upheld the orders issued on August 2, 2001 3 and October probationary period during which they had gotten acquainted with Goldlines driving practices and
22, 20014 by the Regional Trial Court (RTC), Branch 51, in Sorsogon in Civil Case No. 93-5917 entitled Heirs of demeanor;21 that the employees had come under constant supervision, rendering improbable the claim that
Concepcion Lacsa, represented by Teodoro Lacsa v. Travel & Tours Advisers, Inc., et al. authorizing the Abania, who was a regular employee, had been glancing at the video monitor while driving the bus; 22 that the
implementation of the writ of execution against petitioner despite its protestation of being a separate and incident causing Concepcions death was the first serious incident his (Cheng) transportation business had
different corporate personality from Travel & Tours Advisers, Inc. (defendant in Civil Case No. 93-5917). encountered, because the rest had been only minor traffic accidents; 23 and that immediately upon being
informed of the accident, he had instructed his personnel to contact the family of Concepcion. 24
In the orders assailed in the CA, the RTC declared petitioner and Travel & Tours Advisers, Inc. to be one and the
same entity, and ruled that the levy of petitioners property to satisfy the final and executory decision rendered The defendants blamed the death of Concepcion to the recklessness of Bilbes as the driver of the jeepney, and
on June 30, 1997 against Travel & Tours Advisers, Inc. in Civil Case No. 93-5917 5 was valid even if petitioner had of its operator, Salvador Romano;25 and that they had consequently brought a third-party complaint against the
not been impleaded as a party. latter.26

Antecedents After trial, the RTC rendered its decision dated June 30, 1997, disposing:
Set V Corporation Code * Doctrine of Peircing Corpo Veil Cases*Page 19 of 164

ACCORDINGLY, judgment is hereby rendered: Thereafter, the plaintiffs moved for the issuance of a writ of execution to implement the decision dated June 30,
1997.30 The RTC granted their motion on January 31, 2000,31 and issued the writ of execution on February 24,
(1) Finding the plaintiffs entitled to damages for the death of Ma. Concepcion Lacsa in violation of the 2000.32
contract of carriage;
On May 10, 2000, the sheriff implementing the writ of execution rendered a Sheriffs Partial Return, 33 certifying
(2) Ordering defendant Travel & Tours Advisers, Inc. (Goldline) to pay plaintiffs: that the writ of execution had been personally served and a copy of it had been duly tendered to Travel & Tours
Advisers, Inc. or William Cheng, through his secretary, Grace Miranda, and that Cheng had failed to settle the
judgment amount despite promising to do so. Accordingly, a tourist bus bearing Plate No. NWW-883 was levied
a. P30,000.00 expenses for the wake;
pursuant to the writ of execution.

b. P 6,000.00 funeral expenses;


The plaintiffs moved to cite Cheng in contempt of court for failure to obey a lawful writ of the RTC. 34 Cheng filed
his opposition.35 Acting on the motion to cite Cheng in contempt of court, the RTC directed the plaintiffs to file a
c. P50,000.00 for the death of Ma. Concepcion Lacsa; verified petition for indirect contempt on February 19, 2001. 36

d. P150,000.00 for moral damages; On April 20, 2001, petitioner submitted a so-called verified third party claim, 37 claiming that the tourist bus
bearing Plate No. NWW-883 be returned to petitioner because it was the owner; that petitioner had not been
e. P20,000.00 for exemplary damages; made a party to Civil Case No. 93-5917; and that petitioner was a corporation entirely different from Travel &
Tours Advisers, Inc., the defendant in Civil Case No. 93-5917.

f. P8,000.00 for attorneys fees;


It is notable that petitioners Articles of Incorporation was amended on November 8, 1993, 38 shortly after the
filing of Civil Case No. 93-5917 against Travel & Tours Advisers, Inc.
g. P2,000.00 for litigation expenses;

Respondents opposed petitioners verified third-party claim on the following grounds, namely: (a) the third-
h. Costs of suit.
party claim did not comply with the required notice of hearing as required by Rule 15, Sections 4 and 5 of the
Rules of Court; (b) Travel & Tours Advisers, Inc. and petitioner were identical entities and were both operated
(3) Ordering the dismissal of the case against Rene Abania; and managed by the same person, William Cheng; and (c) petitioner was attempting to defraud its creditors
respondents herein hence, the doctrine of piercing the veil of corporate entity was squarely applicable. 39
(4) Ordering the dismissal of the third-party complaint. SO ORDERED. 27
On August 2, 2001, the RTC dismissed petitioners verified third-party claim, observing that the identity of
The RTC found that a contract of carriage had been forged between Travel & Tours Advisers, Inc. and Travel & Tours Adivsers, Inc. could not be divorced from that of petitioner considering that Cheng had claimed
Concepcion as soon as she had boarded the Goldline bus as a paying passenger; that Travel & Tours Advisers, to be the operator as well as the President/Manager/incorporator of both entities; and that Travel & Tours
Inc. had then become duty-bound to safely transport her as its passenger to her destination; that due to Travel Advisers, Inc. had been known in Sorsogon as Goldline.40
& Tours Advisers, Inc.s inability to perform its duty, Article 1786 of the Civil Code created against it the
disputable presumption that it had been at fault or had been negligent in the performance of its obligations Petitioner moved for reconsideration, 41 but the RTC denied the motion on October 22, 2001. 42Thence, petitioner
towards the passenger; that Travel & Tours Advisers, Inc. failed to disprove the presumption of negligence; and initiated a special civil action for certiorari in the CA,43 asserting:
that a rigid selection of employees was not sufficient to exempt Travel & Tours Advisers, Inc. from the obligation
of exercising extraordinary diligence to ensure that its passenger was carried safely to her destination.
THE RESPONDENT HONORABLE RTC JUDGE HAD ACTED WITHOUT JURISDICTION OR COMMITTED GRAVE ABUSE
OF DISCRETION AMOUNTING TO LACK OF JURISDICTION IN ISSUING THE: (A) ORDER DATED 2 AUGUST 2001,
Aggrieved, the defendants appealed to the CA. COPY OF WHICH IS HERETO ATTACHED AS ANNEX A, DISMISSING HEREIN PETITIONERS THIRD PARTY CLAIM;
AND (B) ORDER DATED 22 OCTOBER 2001, COPY OF WHICH IS HERETO ATTACHED AS ANNEX B DENYING SAID
On June 11, 1998, 28 the CA dismissed the appeal for failure of the defendants to pay the docket and other PETITIONERS MOTION FOR RECONSIDERATION; AND THAT THERE IS NO APPEAL, OR ANY PLAIN, SPEEDY AND
lawful fees within the required period as provided in Rule 41, Section 4 of the Rules of Court (1997). The ADEQUATE REMEDY AVAILABLE TO SAID PETITIONER.
dismissal became final, and entry of judgment was made on July 17, 1998. 29
On October 30, 2002, the CA promulgated its decision dismissing the petition for certiorari, 44 holding as follows:
Set V Corporation Code * Doctrine of Peircing Corpo Veil Cases*Page 20 of 164

The petition lacks merit. The main contention of Third Party Claimant is that it is the owner of the Bus and therefore, it should not be
seized by the sheriff because the same does not belong to the defendant Travel & Tours Advises, Inc.
As stated in the decision supra, William Ching disclosed during the trial of the case that defendant Travel & (GOLDLINE) as the third party claimant and defendant are two separate corporation with separate juridical
Tours Advisers, Inc. (Goldline), of which he is an officer, is operating sixty (60) units of Goldline buses. That the personalities. Upon the other hand, this Court had scrutinized the documents submitted by the Third party
Goldline buses are used in the operations of defendant company is obvious from Mr. Chengs admission. The Claimant and found out that William Ching who claimed to be the operator of the Travel & Tours Advisers, Inc.
Amended Articles of Incorporation of Gold Line Tours, Inc. disclose that the following persons are the original (GOLDLINE) is also the President/Manager and incorporator of the Third Party Claimant Goldline Tours Inc. and
incorporators thereof: Antonio O. Ching, Maribel Lim Ching, witness William Ching, Anita Dy Ching and Zosimo he is joined by his co-incorporators who are "Ching" and "Dy" thereby this Court could only say that these two
Ching. (Rollo, pp. 105-106) We see no reason why defendant company would be using Goldline buses in its corporations are one and the same corporations. This is of judicial knowledge that since Travel & Tours
operations unless the two companies are actually one and the same. Advisers, Inc. came to Sorsogon it has been known as GOLDLINE.

Moreover, the name Goldline was added to defendants name in the Complaint. There was no objection from This Court is not persuaded by the proposition of the third party claimant that a corporation has an existence
William Ching who could have raised the defense that Gold Line Tours, Inc. was in no way liable or involved. separate and/or distinct from its members insofar as this case at bar is concerned, for the reason that
Indeed, it appears to this Court that rather than Travel & Tours Advisers, Inc., it is Gold Line Tours, Inc., which whenever necessary for the interest of the public or for the protection of
should have been named party defendant.
enforcement of their rights, the notion of legal entity should not and is not to be used to defeat public
Be that as it may, We concur in the trial courts finding that the two companies are actually one and the same, convenience, justify wrong, protect fraud or defend crime.
hence the levy of the bus in question was proper.WHEREFORE, for lack of merit, the petition is DISMISSED and
the assailed Orders are AFFIRMED.SO ORDERED. Apposite to the case at bar is the case of Palacio vs. Fely Transportation Co., L-15121, May 31, 1962, 5 SCRA
1011 where the Supreme Court held:
Petitioner filed a motion for reconsideration,45 which the CA denied on June 25, 2003.46
"Where the main purpose in forming the corporation was to evade ones subsidiary liability for damages in a
Hence, this appeal, in which petitioner faults the CA for holding that the RTC did not act without jurisdiction or criminal case, the corporation may not be heard to say that it has a personality separate and distinct from its
grave abuse of discretion in finding that petitioner and Travel & Tours Advisers, Inc., the defendant in Civil Case members, because to allow it to do so would be to sanction the use of fiction of corporate entity as a shield to
No. 5917, were one and same entity, and for sustaining the propriety of the levy of the tourist bus with Plate further an end subversive of justice (La Campana Coffee Factory, et al. v. Kaisahan ng mga Manggagawa, etc.,
No. NWW-883 in satisfaction of the writ of execution. 47 et al., L-5677, May 25, 1953). The Supreme Court can even substitute the real party in interest in place of the
defendant corporation in order to avoid multiplicity of suits and thereby save the parties unnecessary expenses
and delay. (Alfonso vs. Villamor, 16 Phil. 315)."
In the meantime, respondents filed in the RTC a motion to direct the sheriff to implement the writ of execution
in view of the non-issuance of any restraining order either by this Court or the CA. 48 On February 23, 2007, the
RTC granted the motion and directed the sheriff to sell the Goldline tourist bus with Plate No. NWW-883 through This is what the third party claimant wants to do including the defendant in this case, to use the separate and
a public auction.49 distinct personality of the two corporation as a shield to further an end subversive of justice by avoiding the
execution of a final judgment of the court.50

Issue
As we see it, the RTC had sufficient factual basis to find that petitioner and Travel and Tours Advisers, Inc. were
one and the same entity, specifically: (a) documents submitted by petitioner in the RTC showing that William
Did the CA rightly find and conclude that the RTC did not gravely abuse its discretion in denying petitioners
Cheng, who claimed to be the operator of Travel and Tours Advisers, Inc., was also the President/Manager and
verified third-party claim?
an incorporator of the petitioner; and (b) Travel and Tours Advisers, Inc. had been known in Sorsogon as
Goldline. On its part, the CA cogently observed:
Ruling
As stated in the (RTC) decision supra, William Ching disclosed during the trial of the case that defendant Travel
We find no reason to reverse the assailed CA decision. & Tours Advisers, Inc. (Goldline), of which he is an officer, is operating sixty (60) units of Goldline buses. That
the Goldline buses are used in the operations of
In the order dated August 2, 2001, the RTC rendered its justification for rejecting the third-party claim of
petitioner in the following manner: defendant company is obvious from Mr. Chengs admission. The Amended Articles of Incorporation of Gold Line
Tours, Inc. disclose that the following persons are the original incorporators thereof: Antonio O. Ching, Maribel
xxx Lim Ching, witness William Ching, Anita Dy Ching and Zosimo Ching. (Rollo, pp. 105-108) We see no reason why
Set V Corporation Code * Doctrine of Peircing Corpo Veil Cases*Page 21 of 164

defendant company would be using Goldline buses in its operations unless the two companies are actually one RESOLUTION
and the same.
VELASCO, JR., J.:
Moreover, the name Goldline was added to defendants name in the Complaint. There was no objection from
William Ching who could have raised the defense that Gold Line Tours, Inc. was in no way liable or involved. For resolution are the (1) Motion for Clarification and Partial Reconsideration dated July 21, 2011 filed by
Indeed it appears to this Court that rather than Travel & Tours Advisers, Inc. it is Gold Line Tours, Inc., which petitioner Hacienda Luisita, Inc. (HLI); (2) Motion for Partial Reconsideration dated July 20, 2011 filed by public
should have been named party defendant. respondents Presidential Agrarian Reform Council (PARC) and Department of Agrarian Reform (DAR); (3) Motion
for Reconsideration dated July 19, 2011 filed by private respondent Alyansa ng mga Manggagawang Bukid sa
Be that as it may, We concur in the trial courts finding that the two companies are actually one and the same, Hacienda Luisita (AMBALA); (4) Motion for Reconsideration dated July 21, 2011 filed by respondent-intervenor
hence the levy of the bus in question was proper. 51 Farmworkers Agrarian Reform Movement, Inc. (FARM); (5) Motion for Reconsideration dated July 21, 2011 filed
by private respondents Noel Mallari, Julio Suniga, Supervisory Group of Hacienda Luisita, Inc. (Supervisory
The RTC thus rightly ruled that petitioner might not be shielded from liability under the final judgment through Group) and Windsor Andaya (collectively referred to as "Mallari, et al."); and (6) Motion for Reconsideration
the use of the doctrine of separate corporate identity. Truly, this fiction of law could not be employed to defeat dated July 22, 2011 filed by private respondents Rene Galang and AMBALA. 2
the ends of justice.
On July 5, 2011, this Court promulgated a Decision 3 in the above-captioned case, denying the petition filed by
But petitioner continues to challenge the RTC orders by insisting that the evidence to establish its identity with HLI and affirming Presidential Agrarian Reform Council (PARC) Resolution No. 2005-32-01 dated December 22,
Travel and Tours Advisers, Inc. was insufficient. 2005 and PARC Resolution No. 2006-34-01 dated May 3, 2006 with the modification that the original 6,296
qualified farmworker-beneficiaries of Hacienda Luisita (FWBs) shall have the option to remain as stockholders of
HLI.
We cannot agree with petitioner. As already stated, there was sufficient evidence that petitioner and Travel and
Tours Advisers, Inc.1wphi1 were one and the same entity. Moreover, we remind that a petition for the writ of
certiorari neither deals with errors of judgment nor extends to a mistake in the appreciation of the contending In its Motion for Clarification and Partial Reconsideration dated July 21, 2011, HLI raises the following issues for
parties evidence or in the evaluation of their relative weight. 52 It is timely to remind that the petitioner in a Our consideration:
special civil action for certiorari commenced against a trial court that has jurisdiction over the proceedings
bears the burden to demonstrate not merely reversible error, but grave abuse of discretion amounting to lack A
or excess of jurisdiction on the part of the respondent trial court in issuing the impugned order. 53 The term
grave abuse of discretion is defined as a capricious and whimsical exercise of judgment so patent and gross as IT IS NOT PROPER, EITHER IN LAW OR IN EQUITY, TO DISTRIBUTE TO THE ORIGINAL FWBs OF 6,296 THE
to amount to an evasion of a positive duty or a virtual refusal to perform a duty enjoined by law, as where the UNSPENT OR UNUSED BALANCE OF THE PROCEEDS OF THE SALE OF THE 500 HECTARES AND 80.51 HECTARES
power is exercised in an arbitrary and despotic manner because of passion or hostility. 54 Mere abuse of OF THE HLI LAND, BECAUSE:
discretion is not enough; it must be grave. 55Yet, here, petitioner did not discharge its burden because it failed to
demonstrate that the CA erred in holding that the RTC had not committed grave abuse of discretion. A review
(1) THE PROCEEDS OF THE SALE BELONG TO THE CORPORATION, HLI, AS CORPORATE CAPITAL AND
of the records shows, indeed, that the RTC correctly rejected petitioners third-party claim. Hence, the rejection
ASSETS IN SUBSTITUTION FOR THE PORTIONS OF ITS LAND ASSET WHICH WERE SOLD TO THIRD
did not come within the domain of the writ of certioraris limiting requirement of excess or lack of
PARTY;
jurisdiction.56WHEREFORE, the Court DENIES the petition for review on certiorari, and AFFIRMS the decision
promulgated by the Court of Appeals on October 30, 2002. Costs of suit to be paid by petitioner.SO ORDERED.
(2) TO DISTRIBUTE THE CASH SALES PROCEEDS OF THE PORTIONS OF THE LAND ASSET TO THE FWBs,
WHO ARE STOCKHOLDERS OF HLI, IS TO DISSOLVE THE CORPORATION AND DISTRIBUTE THE
EN BANC G.R. No. 171101 November 22, 2011
PROCEEDS AS LIQUIDATING DIVIDENDS WITHOUT EVEN PAYING THE CREDITORS OF THE
CORPORATION;
HACIENDA LUISITA, INCORPORATED, Petitioner, LUISITA INDUSTRIAL PARK CORPORATION and RIZAL
COMMERCIAL BANKING CORPORATION,Petitioners-in-Intervention,
(3) THE DOING OF SAID ACTS WOULD VIOLATE THE STRINGENT PROVISIONS OF THE CORPORATION
vs.
CODE AND CORPORATE PRACTICE.
PRESIDENTIAL AGRARIAN REFORM COUNCIL; SECRETARY NASSER PANGANDAMAN OF THE
DEPARTMENT OF AGRARIAN REFORM; ALYANSA NG MGA MANGGAGAWANG BUKID NG HACIENDA
LUISITA, RENE GALANG, NOEL MALLARI, and JULIO SUNIGA 1 and his SUPERVISORY GROUP OF THE B
HACIENDA LUISITA, INC. and WINDSOR ANDAYA, Respondents.
Set V Corporation Code * Doctrine of Peircing Corpo Veil Cases*Page 22 of 164

IT IS NOT PROPER, EITHER IN LAW OR IN EQUITY, TO RECKON THE PAYMENT OF JUST COMPENSATION FROM THE MAJORITY OF THE MEMBERS OF THE HONORABLE COURT, WITH DUE RESPECT, ERRED IN HOLDING THAT
NOVEMBER 21, 1989 WHEN THE PARC, THEN UNDER THE CHAIRMANSHIP OF DAR SECRETARY MIRIAM ONLY THE [PARCS] APPROVAL OF HLIs PROPOSAL FOR STOCK DISTRIBUTION UNDER CARP AND THE [SDP]
DEFENSOR-SANTIAGO, APPROVED THE STOCK DISTRIBUTION PLAN (SDP) PROPOSED BY TADECO/HLI, BECAUSE: WERE REVOKED AND NOT THE STOCK DISTRIBUTION OPTION AGREEMENT (SDOA).

(1) THAT PARC RESOLUTION NO. 89-12-2 DATED NOVEMBER 21, 1989 WAS NOT THE "ACTUAL TAKING" III
OF THE TADECOs/HLIs AGRICULTURAL LAND;
THE MAJORITY OF THE MEMBERS OF THE HONORABLE COURT, WITH DUE RESPECT, ERRED IN APPLYING THE
(2) THE RECALL OR REVOCATION UNDER RESOLUTION NO. 2005-32-01 OF THAT SDP BY THE NEW DOCTRINE OF OPERATIVE FACTS AND IN MAKING THE [FWBs] CHOOSE TO OPT FOR ACTUAL LAND
PARC UNDER THE CHAIRMANSHIP OF DAR SECRETARY NASSER PANGANDAMAN ON DECEMBER 22, DISTRIBUTION OR TO REMAIN AS STOCKHOLDERS OF [HLI].
2005 OR 16 YEARS EARLIER WHEN THE SDP WAS APPROVED DID NOT RESULT IN "ACTUAL TAKING" ON
NOVEMBER 21, 1989; IV

(3) TO PAY THE JUST COMPENSATION AS OF NOVEMBER 21, 1989 OR 22 YEARS BACK WOULD BE THE MAJORITY OF THE MEMBERS OF THE HONORABLE COURT, WITH DUE RESPECT, ERRED IN HOLDING THAT
ARBITRARY, UNJUST, AND OPPRESSIVE, CONSIDERING THE IMPROVEMENTS, EXPENSES IN THE IMPROVING THE ECONOMIC STATUS OF FWBs IS NOT AMONG THE LEGAL OBLIGATIONS OF HLI UNDER THE SDP
MAINTENANCE AND PRESERVATION OF THE LAND, AND RISE IN LAND PRICES OR VALUE OF THE AND AN IMPERATIVE IMPOSITION BY [RA 6657] AND DEPARTMENT OF AGRARIAN REFORM ADMINISTRATIVE
PROPERTY. ORDER NO. 10 (DAO 10).

On the other hand, PARC and DAR, through the Office of the Solicitor General (OSG), raise the following issues V
in their Motion for Partial Reconsideration dated July 20, 2011:

THE HONORABLE COURT, WITH DUE RESPECT, ERRED IN HOLDING THAT THE CONVERSION OF THE
THE DOCTRINE OF OPERATIVE FACT DOES NOT APPLY TO THIS CASE FOR THE FOLLOWING REASONS: AGRICULTURAL LANDS DID NOT VIOLATE THE CONDITIONS OF RA 6657 AND DAO 10.

I VI

THERE IS NO LAW OR RULE WHICH HAS BEEN INVALIDATED ON THE GROUND OF UNCONSTITUTIONALITY; AND THE HONORABLE COURT, WITH DUE RESPECT, ERRED IN HOLDING THAT PETITIONER IS ENTITLED TO PAYMENT
OF JUST COMPENSATION. SHOULD THE HONORABLE COURT AFFIRM THE ENTITLEMENT OF THE PETITIONER TO
II JUST COMPENSATION, THE SAME SHOULD BE PEGGED TO FORTY THOUSAND PESOS (PhP 40,000.00) PER
HECTARE.
THIS DOCTRINE IS A RULE OF EQUITY WHICH MAY BE APPLIED ONLY IN THE ABSENCE OF A LAW. IN THIS CASE,
THERE IS A POSITIVE LAW WHICH MANDATES THE DISTRIBUTION OF THE LAND AS A RESULT OF THE VII
REVOCATION OF THE STOCK DISTRIBUTION PLAN (SDP).
THE HONORABLE COURT, WITH DUE RESPECT, ERRED IN HOLDING THAT LUISITA INDUSTRIAL PARK CORP.
For its part, AMBALA poses the following issues in its Motion for Reconsideration dated July 19, 2011: (LIPCO) AND RIZAL COMMERCIAL BANKING CORPORATION (RCBC) ARE INNOCENT PURCHASERS FOR VALUE.

I In its Motion for Reconsideration dated July 21, 2011, FARM similarly puts forth the following issues:

THE MAJORITY OF THE MEMBERS OF THE HONORABLE COURT, WITH DUE RESPECT, ERRED IN HOLDING THAT I
SECTION 31 OF REPUBLIC ACT 6657 (RA 6657) IS CONSTITUTIONAL.
THE HONORABLE SUPREME COURT SHOULD HAVE STRUCK DOWN SECTION 31 OF [RA 6657] FOR BEING
II UNCONSTITUTIONAL. THE CONSTITUTIONALITY ISSUE THAT WAS RAISED BY THE RESPONDENTS-INTERVENORS
IS THE LIS MOTA OF THE CASE.

II
Set V Corporation Code * Doctrine of Peircing Corpo Veil Cases*Page 23 of 164

THE HONORABLE SUPREME COURT SHOULD NOT HAVE APPLIED THE DOCTRINE OF "OPERATIVE FACT" TO THE I
CASE. THE OPTION GIVEN TO THE FARMERS TO REMAIN AS STOCKHOLDERS OF HACIENDA LUISITA IS
EQUIVALENT TO AN OPTION FOR HACIENDA LUISITA TO RETAIN LAND IN DIRECT VIOLATION OF THE THE HONORABLE COURT, WITH DUE RESPECT, GRAVELY ERRED IN ORDERING THE HOLDING OF A VOTING
COMPREHENSIVE AGRARIAN REFORM LAW. THE DECEPTIVE STOCK DISTRIBUTION OPTION / STOCK OPTION INSTEAD OF TOTALLY REDISTRIBUTING THE SUBJECT LANDS TO [FWBs] in [HLI].
DISTRIBUTION PLAN CANNOT JUSTIFY SUCH RESULT, ESPECIALLY AFTER THE SUPREME COURT HAS AFFIRMED
ITS REVOCATION.
A. THE HOLDING OF A VOTING OPTION HAS NO LEGAL BASIS. THE REVOCATION OF THE [SDP] CARRIES
WITH IT THE REVOCATION OF THE [SDOA].
III

B. GIVING THE [FWBs] THE OPTION TO REMAIN AS STOCKHOLDERS OF HLI WITHOUT MAKING THE
THE HONORABLE SUPREME COURT SHOULD NOT HAVE CONSIDERED [LIPCO] AND [RCBC] AS INNOCENT NECESSARY CHANGES IN THE CORPORATE STRUCTURE WOULD ONLY SUBJECT THEM TO FURTHER
PURCHASERS FOR VALUE IN THE INSTANT CASE. MANIPULATION AND HARDSHIP.

Mallari, et al., on the other hand, advance the following grounds in support of their Motion for Reconsideration C. OTHER VIOLATIONS COMMITTED BY HLI UNDER THE [SDOA] AND PERTINENT LAWS JUSTIFY TOTAL
dated July 21, 2011: LAND REDISTRIBUTION OF HACIENDA LUISITA.

(1) THE HOMELOTS REQUIRED TO BE DISTRIBUTED HAVE ALL BEEN DISTRIBUTED PURSUANT TO THE II
MEMORANDUM OF AGREEMENT. WHAT REMAINS MERELY IS THE RELEASE OF TITLE FROM THE
REGISTER OF DEEDS.
THE HONORABLE COURT, WITH DUE RESPECT, GRAVELY ERRED IN HOLDING THAT THE [RCBC] AND [LIPCO] ARE
INNOCENT PURCHASERS FOR VALUE OF THE 300-HECTARE PROPERTY IN HACIENDA LUISITA THAT WAS SOLD TO
(2) THERE HAS BEEN NO DILUTION OF SHARES. CORPORATE RECORDS WOULD SHOW THAT IF EVER THEM PRIOR TO THE INCEPTION OF THE PRESENT CONTROVERSY.
NOT ALL OF THE 18,804.32 SHARES WERE GIVEN TO THE ACTUAL ORIGINAL FARMWORKER
BENEFICIARY, THE RECIPIENT OF THE DIFFERENCE IS THE NEXT OF KIN OR CHILDREN OF SAID
Ultimately, the issues for Our consideration are the following: (1) applicability of the operative fact doctrine; (2)
ORIGINAL [FWBs]. HENCE, WE RESPECTFULLY SUBMIT THAT SINCE THE SHARES WERE GIVEN TO THE
constitutionality of Sec. 31 of RA 6657 or the Comprehensive Agrarian Reform Law of 1988; (3) coverage of
SAME "FAMILY BENEFICIARY", THIS SHOULD BE DEEMED AS SUBSTANTIAL COMPLIANCE WITH THE
compulsory acquisition; (4) just compensation; (5) sale to third parties; (6) the violations of HLI; and (7) control
PROVISIONS OF SECTION 4 OF DAO 10.
over agricultural lands.

(3) THERE HAS BEEN NO VIOLATION OF THE 3-MONTH PERIOD TO IMPLEMENT THE [SDP] AS PROVIDED
We shall discuss these issues accordingly.
FOR BY SECTION 11 OF DAO 10 AS THIS PROVISION MUST BE READ IN LIGHT OF SECTION 10 OF
EXECUTIVE ORDER NO. 229, THE PERTINENT PORTION OF WHICH READS, "THE APPROVAL BY THE PARC
OF A PLAN FOR SUCH STOCK DISTRIBUTION, AND ITS INITIAL IMPLEMENTATION, SHALL BE DEEMED I. Applicability of the Operative Fact Doctrine
COMPLIANCE WITH THE LAND DISTRIBUTION REQUIREMENT OF THE CARP."
In their motion for partial reconsideration, DAR and PARC argue that the doctrine of operative fact does not
(4) THE VALUATION OF THE LAND CANNOT BE BASED AS OF NOVEMBER 21, 1989, THE DATE OF apply to the instant case since: (1) there is no law or rule which has been invalidated on the ground of
APPROVAL OF THE STOCK DISTRIBUTION OPTION. INSTEAD, WE RESPECTFULLY SUBMIT THAT THE unconstitutionality;4 (2) the doctrine of operative fact is a rule of equity which may be applied only in the
"TIME OF TAKING" FOR VALUATION PURPOSES IS A FACTUAL ISSUE BEST LEFT FOR THE TRIAL COURTS absence of a law, and in this case, they maintain that there is a positive law which mandates the distribution of
TO DECIDE. the land as a result of the revocation of the stock distribution plan (SDP). 5

(5) TO THOSE WHO WILL CHOOSE LAND, THEY MUST RETURN WHAT WAS GIVEN TO THEM UNDER THE Echoing the stance of DAR and PARC, AMBALA submits that the operative fact doctrine should only be made to
SDP. IT WOULD BE UNFAIR IF THEY ARE ALLOWED TO GET THE LAND AND AT THE SAME TIME HOLD ON apply in the extreme case in which equity demands it, which allegedly is not in the instant case. 6 It further
TO THE BENEFITS THEY RECEIVED PURSUANT TO THE SDP IN THE SAME WAY AS THOSE WHO WILL argues that there would be no undue harshness or injury to HLI in case lands are actually distributed to the
CHOOSE TO STAY WITH THE SDO. farmworkers, and that the decision which orders the farmworkers to choose whether to remain as stockholders
of HLI or to opt for land distribution would result in inequity and prejudice to the farmworkers. 7 The foregoing
views are also similarly shared by Rene Galang and AMBALA, through the PILC. 8 In addition, FARM posits that
Lastly, Rene Galang and AMBALA, through the Public Interest Law Center (PILC), submit the following grounds
the option given to the FWBs is equivalent to an option for HLI to retain land in direct violation of RA 6657. 9
in support of their Motion for Reconsideration dated July 22, 2011:
Set V Corporation Code * Doctrine of Peircing Corpo Veil Cases*Page 24 of 164

(a) Operative Fact Doctrine Not Limited to was subsequently declared by the Court, in Taada v. Tuvera, 33 to have no force and effect. Thus, he asserts,
any evidence obtained pursuant thereto is inadmissible in evidence.
Invalid or Unconstitutional Laws
We do not agree. In Taada, the Court addressed the possible effects of its declaration of the invalidity of
Contrary to the stance of respondents, the operative fact doctrine does not only apply to laws subsequently various presidential issuances. Discussing therein how such a declaration might affect acts done on a
declared unconstitutional or unlawful, as it also applies to executive acts subsequently declared as invalid. As presumption of their validity, the Court said:
We have discussed in Our July 5, 2011 Decision:
". . .. In similar situations in the past this Court had taken the pragmatic and realistic course set forth in Chicot
That the operative fact doctrine squarely applies to executive actsin this case, the approval by PARC of the County Drainage District vs. Baxter Bank to wit:
HLI proposal for stock distributionis well-settled in our jurisprudence. In Chavez v. National Housing Authority,
We held: The courts below have proceeded on the theory that the Act of Congress, having been found to be
unconstitutional, was not a law; that it was inoperative, conferring no rights and imposing no duties, and hence
Petitioner postulates that the "operative fact" doctrine is inapplicable to the present case because it is an affording no basis for the challenged decree. . . . It is quite clear, however, that such broad statements as to
equitable doctrine which could not be used to countenance an inequitable result that is contrary to its proper the effect of a determination of unconstitutionality must be taken with qualifications. The actual existence of a
office. statute, prior to [the determination of its invalidity], is an operative fact and may have consequences which
cannot justly be ignored. The past cannot always be erased by a new judicial declaration. The effect of the
subsequent ruling as to invalidity may have to be considered in various aspects with respect to particular
On the other hand, the petitioner Solicitor General argues that the existence of the various agreements
conduct, private and official. Questions of rights claimed to have become vested, of status, of prior
implementing the SMDRP is an operative fact that can no longer be disturbed or simply ignored, citing Rieta v.
determinations deemed to have finality and acted upon accordingly, of public policy in the light of the nature
People of the Philippines.
both of the statute and of its previous application, demand examination. These questions are among the most
difficult of those which have engaged the attention of courts, state and federal, and it is manifest from
The argument of the Solicitor General is meritorious. numerous decisions that an all-inclusive statement of a principle of absolute retroactive invalidity cannot be
justified.
The "operative fact" doctrine is embodied in De Agbayani v. Court of Appeals, wherein it is stated that a
legislative or executive act, prior to its being declared as unconstitutional by the courts, is valid and must be xxx xxx xxx
complied with, thus:
"Similarly, the implementation/ enforcement of presidential decrees prior to their publication in the Official
xxx xxx xxx Gazette is an operative fact which may have consequences which cannot be justly ignored. The past cannot
always be erased by a new judicial declaration . . . that an all-inclusive statement of a principle of absolute
This doctrine was reiterated in the more recent case of City of Makati v. Civil Service Commission, wherein we retroactive invalidity cannot be justified."
ruled that:
The Chicot doctrine cited in Taada advocates that, prior to the nullification of a statute, there is an imperative
Moreover, we certainly cannot nullify the City Government's order of suspension, as we have no reason to do necessity of taking into account its actual existence as an operative fact negating the acceptance of "a
so, much less retroactively apply such nullification to deprive private respondent of a compelling and valid principle of absolute retroactive invalidity." Whatever was done while the legislative or the executive act was in
reason for not filing the leave application. For as we have held, a void act though in law a mere scrap of paper operation should be duly recognized and presumed to be valid in all respects. The ASSO that was issued in
nonetheless confers legitimacy upon past acts or omissions done in reliance thereof. Consequently, the 1979 under General Order No. 60 long before our Decision in Taada and the arrest of petitioner is an
existence of a statute or executive order prior to its being adjudged void is an operative fact to which legal operative fact that can no longer be disturbed or simply ignored. (Citations omitted; emphasis in the original.)
consequences are attached. It would indeed be ghastly unfair to prevent private respondent from relying upon
the order of suspension in lieu of a formal leave application. Bearing in mind that PARC Resolution No. 89-12-2 10an executive actwas declared invalid in the instant case,
the operative fact doctrine is clearly applicable.
The applicability of the operative fact doctrine to executive acts was further explicated by this Court in Rieta v.
People, thus: Nonetheless, the minority is of the persistent view that the applicability of the operative fact doctrine should be
limited to statutes and rules and regulations issued by the executive department that are accorded the same
Petitioner contends that his arrest by virtue of Arrest Search and Seizure Order (ASSO) No. 4754 was invalid, as status as that of a statute or those which are quasi-legislative in nature. Thus, the minority concludes that the
the law upon which it was predicated General Order No. 60, issued by then President Ferdinand E. Marcos phrase "executive act" used in the case of De Agbayani v. Philippine National Bank 11 refers only to acts, orders,
Set V Corporation Code * Doctrine of Peircing Corpo Veil Cases*Page 25 of 164

and rules and regulations that have the force and effect of law. The minority also made mention of the conformity to and in reliance of said decision, which must be respected. It is on this score that the operative
Concurring Opinion of Justice Enrique Fernando in Municipality of Malabang v. Benito, 12 where it was supposedly fact doctrine should be applied to acts and consequences that resulted from the implementation of the PARC
made explicit that the operative fact doctrine applies to executive acts, which are ultimately quasi-legislative in Resolution approving the SDP of HLI.
nature.
More importantly, respondents, and even the minority, failed to clearly explain how the option to remain in HLI
We disagree. For one, neither the De Agbayani case nor the Municipality of Malabang case elaborates what granted to individual farmers would result in inequity and prejudice. We can only surmise that respondents
"executive act" mean. Moreover, while orders, rules and regulations issued by the President or the executive misinterpreted the option as a referendum where all the FWBs will be bound by a majority vote favoring the
branch have fixed definitions and meaning in the Administrative Code and jurisprudence, the phrase "executive retention of all the 6,296 FWBs as HLI stockholders. Respondents are definitely mistaken. The fallo of Our July
act" does not have such specific definition under existing laws. It should be noted that in the cases cited by the 5, 2011 Decision is unequivocal that only those FWBs who signified their desire to remain as HLI stockholders
minority, nowhere can it be found that the term "executive act" is confined to the foregoing. Contrarily, the are entitled to 18,804.32 shares each, while those who opted not to remain as HLI stockholders will be given
term "executive act" is broad enough to encompass decisions of administrative bodies and agencies under the land by DAR. Thus, referendum was not required but only individual options were granted to each FWB whether
executive department which are subsequently revoked by the agency in question or nullified by the Court. or not they will remain in HLI.

A case in point is the concurrent appointment of Magdangal B. Elma (Elma) as Chairman of the Presidential The application of the operative fact doctrine to the FWBs is not iniquitous and prejudicial to their interests but
Commission on Good Government (PCGG) and as Chief Presidential Legal Counsel (CPLC) which was declared is actually beneficial and fair to them. First, they are granted the right to remain in HLI as stockholders and they
unconstitutional by this Court in Public Interest Center, Inc. v. Elma. 13 In said case, this Court ruled that the acquired said shares without paying their value to the corporation. On the other hand, the qualified FWBs are
concurrent appointment of Elma to these offices is in violation of Section 7, par. 2, Article IX-B of the 1987 required to pay the value of the land to the Land Bank of the Philippines (LBP) if land is awarded to them by
Constitution, since these are incompatible offices. Notably, the appointment of Elma as Chairman of the PCGG DAR pursuant to RA 6657. If the qualified FWBs really want agricultural land, then they can simply say no to the
and as CPLC is, without a question, an executive act. Prior to the declaration of unconstitutionality of the said option. And second, if the operative fact doctrine is not applied to them, then the FWBs will be required to
executive act, certain acts or transactions were made in good faith and in reliance of the appointment of Elma return to HLI the 3% production share, the 3% share in the proceeds of the sale of the 500-hectare converted
which cannot just be set aside or invalidated by its subsequent invalidation. land, and the 80.51-hectare Subic-Clark-Tarlac Expressway (SCTEX) lot, the homelots and other benefits
received by the FWBs from HLI. With the application of the operative fact doctrine, said benefits, homelots and
In Tan v. Barrios,14 this Court, in applying the operative fact doctrine, held that despite the invalidity of the the 3% production share and 3% share from the sale of the 500-hectare and SCTEX lots shall be respected with
jurisdiction of the military courts over civilians, certain operative facts must be acknowledged to have existed no obligation to refund or return them. The receipt of these things is an operative fact "that can no longer be
so as not to trample upon the rights of the accused therein. Relevant thereto, in Olaguer v. Military Commission disturbed or simply ignored."
No. 34,15 it was ruled that "military tribunals pertain to the Executive Department of the Government and are
simply instrumentalities of the executive power, provided by the legislature for the President as Commander-in- (b) The Operative Fact Doctrine as Recourse in Equity
Chief to aid him in properly commanding the army and navy and enforcing discipline therein, and utilized under
his orders or those of his authorized military representatives." 16 As mentioned above, respondents contend that the operative fact doctrine is a rule of equity which may be
applied only in the absence of a law, and that in the instant case, there is a positive law which mandates the
Evidently, the operative fact doctrine is not confined to statutes and rules and regulations issued by the distribution of the land as a result of the revocation of the SDP.
executive department that are accorded the same status as that of a statute or those which are quasi-
legislative in nature. Undeniably, the operative fact doctrine is a rule of equity.17 As a complement of legal jurisdiction, equity "seeks
to reach and complete justice where courts of law, through the inflexibility of their rules and want of power to
Even assuming that De Agbayani initially applied the operative fact doctrine only to executive issuances like adapt their judgments to the special circumstances of cases, are incompetent to do so. Equity regards the spirit
orders and rules and regulations, said principle can nonetheless be applied, by analogy, to decisions made by and not the letter, the intent and not the form, the substance rather than the circumstance, as it is variously
the President or the agencies under the executive department. This doctrine, in the interest of justice and expressed by different courts." 18 Remarkably, it is applied only in the absence of statutory law and never in
equity, can be applied liberally and in a broad sense to encompass said decisions of the executive branch. In contravention of said law.19
keeping with the demands of equity, the Court can apply the operative fact doctrine to acts and consequences
that resulted from the reliance not only on a law or executive act which is quasi-legislative in nature but also on In the instant case, respondents argue that the operative fact doctrine should not be applied since there is a
decisions or orders of the executive branch which were later nullified. This Court is not unmindful that such acts positive law, particularly, Sec. 31 of RA 6657, which directs the distribution of the land as a result of the
and consequences must be recognized in the higher interest of justice, equity and fairness. revocation of the SDP. Pertinently, the last paragraph of Sec. 31 of RA 6657 states:

Significantly, a decision made by the President or the administrative agencies has to be complied with because If within two (2) years from the approval of this Act, the land or stock transfer envisioned above is not made or
it has the force and effect of law, springing from the powers of the President under the Constitution and realized or the plan for such stock distribution approved by the PARC within the same period, the agricultural
existing laws. Prior to the nullification or recall of said decision, it may have produced acts and consequences in
Set V Corporation Code * Doctrine of Peircing Corpo Veil Cases*Page 26 of 164

land of the corporate owners or corporation shall be subject to the compulsory coverage of this Act. (Emphasis authorization document, which would logically include a list of the names of the authorizing FWBs, has yet to
supplied.) be submitted to be part of the records."

Markedly, the use of the word "or" under the last paragraph of Sec. 31 of RA 6657 connotes that the law gives II. Constitutionality of Sec. 31, RA 6657
the corporate landowner an "option" to avail of the stock distribution option or to have the SDP approved within
two (2) years from the approval of RA 6657. This interpretation is consistent with the well-established principle FARM insists that the issue of constitutionality of Sec. 31 of RA 6657 is the lis mota of the case, raised at the
in statutory construction that "[t]he word or is a disjunctive term signifying disassociation and independence of earliest opportunity, and not to be considered as moot and academic. 25
one thing from the other things enumerated; it should, as a rule, be construed in the sense in which it ordinarily
implies, as a disjunctive word." 20 In PCI Leasing and Finance, Inc. v. Giraffe-X Creative Imaging, Inc., 21 this Court
This contention is unmeritorious. As We have succinctly discussed in Our July 5, 2011 Decision:
held:

While there is indeed an actual case or controversy, intervenor FARM, composed of a small minority of 27
Evidently, the letter did not make a demand for the payment of the P8,248,657.47 AND the return of the
farmers, has yet to explain its failure to challenge the constitutionality of Sec. 3l of RA 6657, since as early as
equipment; only either one of the two was required. The demand letter was prepared and signed by Atty.
November 21, l989 when PARC approved the SDP of Hacienda Luisita or at least within a reasonable time
Florecita R. Gonzales, presumably petitioners counsel. As such, the use of "or" instead of "and" in the letter
thereafter and why its members received benefits from the SDP without so much of a protest. It was only on
could hardly be treated as a simple typographical error, bearing in mind the nature of the demand, the amount
December 4, 2003 or 14 years after approval of the SDP via PARC Resolution No. 89-12-2 dated November 21,
involved, and the fact that it was made by a lawyer. Certainly Atty. Gonzales would have known that a world of
1989 that said plan and approving resolution were sought to be revoked, but not, to stress, by FARM or any of
difference exists between "and" and "or" in the manner that the word was employed in the letter.
its members, but by petitioner AMBALA. Furthermore, the AMBALA petition did NOT question the
constitutionality of Sec. 31 of RA 6657, but concentrated on the purported flaws and gaps in the subsequent
A rule in statutory construction is that the word "or" is a disjunctive term signifying dissociation and implementation of the SDP. Even the public respondents, as represented by the Solicitor General, did not
independence of one thing from other things enumerated unless the context requires a different question the constitutionality of the provision. On the other hand, FARM, whose 27 members formerly belonged
interpretation.22 to AMBALA, raised the constitutionality of Sec. 31 only on May 3, 2007 when it filed its Supplemental Comment
with the Court. Thus, it took FARM some eighteen (18) years from November 21, 1989 before it challenged the
In its elementary sense, "or", as used in a statute, is a disjunctive article indicating an alternative. It often constitutionality of Sec. 31 of RA 6657 which is quite too late in the day. The FARM members slept on their
connects a series of words or propositions indicating a choice of either. When "or" is used, the various members rights and even accepted benefits from the SDP with nary a complaint on the alleged unconstitutionality of Sec.
of the enumeration are to be taken separately.23 31 upon which the benefits were derived. The Court cannot now be goaded into resolving a constitutional issue
that FARM failed to assail after the lapse of a long period of time and the occurrence of numerous events and
The word "or" is a disjunctive term signifying disassociation and independence of one thing from each of the activities which resulted from the application of an alleged unconstitutional legal provision.
other things enumerated.24 (Emphasis in the original.)
It has been emphasized in a number of cases that the question of constitutionality will not be passed upon by
Given that HLI secured approval of its SDP in November 1989, well within the two-year period reckoned from the Court unless it is properly raised and presented in an appropriate case at the first opportunity. FARM is,
June 1988 when RA 6657 took effect, then HLI did not violate the last paragraph of Sec. 31 of RA 6657. therefore, remiss in belatedly questioning the constitutionality of Sec. 31 of RA 6657. The second requirement
Pertinently, said provision does not bar Us from applying the operative fact doctrine. that the constitutional question should be raised at the earliest possible opportunity is clearly wanting.

Besides, it should be recognized that this Court, in its July 5, 2011 Decision, affirmed the revocation of The last but the most important requisite that the constitutional issue must be the very lis mota of the case
Resolution No. 89-12-2 and ruled for the compulsory coverage of the agricultural lands of Hacienda Luisita in does not likewise obtain. The lis mota aspect is not present, the constitutional issue tendered not being critical
view of HLIs violation of the SDP and DAO 10. By applying the operative fact doctrine, this Court merely gave to the resolution of the case. The unyielding rule has been to avoid, whenever plausible, an issue assailing the
the qualified FWBs the option to remain as stockholders of HLI and ruled that they will retain the homelots and constitutionality of a statute or governmental act. If some other grounds exist by which judgment can be made
other benefits which they received from HLI by virtue of the SDP. without touching the constitutionality of a law, such recourse is favored. Garcia v. Executive Secretary explains
why:

It bears stressing that the application of the operative fact doctrine by the Court in its July 5, 2011 Decision is
favorable to the FWBs because not only were the FWBs allowed to retain the benefits and homelots they Lis Mota the fourth requirement to satisfy before this Court will undertake judicial review means that the
received under the stock distribution scheme, they were also given the option to choose for themselves Court will not pass upon a question of unconstitutionality, although properly presented, if the case can be
whether they want to remain as stockholders of HLI or not. This is in recognition of the fact that despite the disposed of on some other ground, such as the application of the statute or the general law. The petitioner
claims of certain farmer groups that they represent the qualified FWBs in Hacienda Luisita, none of them can must be able to show that the case cannot be legally resolved unless the constitutional question raised is
show that they are duly authorized to speak on their behalf. As We have mentioned, "To date, such determined. This requirement is based on the rule that every law has in its favor the presumption of
Set V Corporation Code * Doctrine of Peircing Corpo Veil Cases*Page 27 of 164

constitutionality; to justify its nullification, there must be a clear and unequivocal breach of the Constitution, Nonetheless, it should be taken into account that this should not prevent the DAR, under its mandate under the
and not one that is doubtful, speculative, or argumentative. agrarian reform law, from subsequently subjecting to agrarian reform other agricultural lands originally held by
Tadeco that were allegedly not transferred to HLI but were supposedly covered by RA 6657.
The lis mota in this case, proceeding from the basic positions originally taken by AMBALA (to which the FARM
members previously belonged) and the Supervisory Group, is the alleged non-compliance by HLI with the DAR, however, contends that the declaration of the area 32 to be awarded to each FWB is too restrictive. It
conditions of the SDP to support a plea for its revocation. And before the Court, the lis mota is whether or not stresses that in agricultural landholdings like Hacienda Luisita, there are roads, irrigation canals, and other
PARC acted in grave abuse of discretion when it ordered the recall of the SDP for such non-compliance and the portions of the land that are considered commonly-owned by farmworkers, and this may necessarily result in
fact that the SDP, as couched and implemented, offends certain constitutional and statutory provisions. To be the decrease of the area size that may be awarded per FWB. 33 DAR also argues that the July 5, 2011 Decision of
sure, any of these key issues may be resolved without plunging into the constitutionality of Sec. 31 of RA 6657. this Court does not give it any leeway in adjusting the area that may be awarded per FWB in case the number
Moreover, looking deeply into the underlying petitions of AMBALA, et al., it is not the said section per se that is of actual qualified FWBs decreases.34
invalid, but rather it is the alleged application of the said provision in the SDP that is flawed.
The argument is meritorious. In order to ensure the proper distribution of the agricultural lands of Hacienda
It may be well to note at this juncture that Sec. 5 of RA 9700, amending Sec. 7 of RA 6657, has all but Luisita per qualified FWB, and considering that matters involving strictly the administrative implementation and
superseded Sec. 31 of RA 6657 vis--vis the stock distribution component of said Sec. 31. In its pertinent part, enforcement of agrarian reform laws are within the jurisdiction of the DAR, 35 it is the latter which shall
Sec. 5 of RA 9700 provides: "[T]hat after June 30, 2009, the modes of acquisition shall be limited to voluntary determine the area with which each qualified FWB will be awarded.
offer to sell and compulsory acquisition." Thus, for all intents and purposes, the stock distribution scheme
under Sec. 31 of RA 6657 is no longer an available option under existing law. The question of whether or not it (a) Conversion of Agricultural Lands
is unconstitutional should be a moot issue. (Citations omitted; emphasis in the original.)

AMBALA insists that the conversion of the agricultural lands violated the conditions of RA 6657 and DAO 10,
Based on the foregoing disquisitions, We maintain that this Court is NOT compelled to rule on the stating that "keeping the land intact and unfragmented is one of the essential conditions of [the] SD[P], RA
constitutionality of Sec. 31 of RA 6657. In this regard, We clarify that this Court, in its July 5, 2011 Decision, 6657 and DAO 10." 36 It asserts that "this provision or conditionality is not mere decoration and is intended to
made no ruling in favor of the constitutionality of Sec. 31 of RA 6657. There was, however, a determination of ensure that the farmers can continue with the tillage of the soil especially since it is the only occupation that
the existence of an apparent grave violation of the Constitution that may justify the resolution of the issue of majority of them knows."37
constitutionality, to which this Court ruled in the negative. Having clarified this matter, all other points raised
by both FARM and AMBALA concerning the constitutionality of RA 6657 deserve scant consideration.
We disagree. As We amply discussed in Our July 5, 2011 Decision:

III. Coverage of Compulsory Acquisition


Contrary to the almost parallel stance of the respondents, keeping Hacienda Luisita unfragmented is also not
among the imperative impositions by the SDP, RA 6657, and DAO 10.
FARM argues that this Court ignored certain material facts when it limited the maximum area to be covered to
4,915.75 hectares, whereas the area that should, at the least, be covered is 6,443 hectares, 26 which is the
The Terminal Report states that the proposed distribution plan submitted in 1989 to the PARC effectively
agricultural land allegedly covered by RA 6657 and previously held by Tarlac Development Corporation
assured the intended stock beneficiaries that the physical integrity of the farm shall remain inviolate.
(Tadeco).27
Accordingly, the Terminal Report and the PARC-assailed resolution would take HLI to task for securing approval
of the conversion to non-agricultural uses of 500 hectares of the hacienda. In not too many words, the Report
We cannot subscribe to this view. Since what is put in issue before the Court is the propriety of the revocation and the resolution view the conversion as an infringement of Sec. 5(a) of DAO 10 which reads: "a. that the
of the SDP, which only involves 4,915.75 has. of agricultural land and not 6,443 has., then We are constrained continued operation of the corporation with its agricultural land intact and unfragmented is viable with
to rule only as regards the 4,915.75 has. of agricultural land. potential for growth and increased profitability."

Moreover, as admitted by FARM itself, this issue was raised for the first time by FARM in its Memorandum dated The PARC is wrong.
September 24, 2010 filed before this Court.28 In this regard, it should be noted that "[a]s a legal recourse, the
special civil action of certiorari is a limited form of review." 29 The certiorari jurisdiction of this Court is narrow in
In the first place, Sec. 5(a)just like the succeeding Sec. 5(b) of DAO 10 on increased income and greater
scope as it is restricted to resolving errors of jurisdiction and grave abuse of discretion, and not errors of
benefits to qualified beneficiariesis but one of the stated criteria to guide PARC in deciding on whether or not
judgment.30 To allow additional issues at this stage of the proceedings is violative of fair play, justice and due
to accept an SDP. Said Sec. 5(a) does not exact from the corporate landowner-applicant the undertaking to
process.31
keep the farm intact and unfragmented ad infinitum. And there is logic to HLIs stated observation that the key
phrase in the provision of Sec. 5(a) is "viability of corporate operations": "[w]hat is thus required is not the
agricultural land remaining intact x x x but the viability of the corporate operations with its agricultural land
Set V Corporation Code * Doctrine of Peircing Corpo Veil Cases*Page 28 of 164

being intact and unfragmented. Corporate operation may be viable even if the corporate agricultural land does benefits, wages and the like, received by the FWBs shall not in any way be reduced or adversely affected,
not remain intact or [un]fragmented." 38 among others.44

It is, of course, anti-climactic to mention that DAR viewed the conversion as not violative of any issuance, let The contentions of respondents are wanting. In the first place, there is no denying that RCBC and LIPCO knew
alone undermining the viability of Hacienda Luisitas operation, as the DAR Secretary approved the land that the converted lands they bought were under the coverage of CARP. Nevertheless, as We have mentioned
conversion applied for and its disposition via his Conversion Order dated August 14, 1996 pursuant to Sec. 65 in Our July 5, 2011 Decision, this does not necessarily mean that both LIPCO and RCBC already acted in bad
of RA 6657 which reads: faith in purchasing the converted lands. As this Court explained:

Sec. 65. Conversion of Lands.After the lapse of five years from its award when the land ceases to be It cannot be claimed that RCBC and LIPCO acted in bad faith in acquiring the lots that were previously covered
economically feasible and sound for agricultural purposes, or the locality has become urbanized and the land by the SDP. Good faith "consists in the possessors belief that the person from whom he received it was the
will have a greater economic value for residential, commercial or industrial purposes, the DAR upon application owner of the same and could convey his title. Good faith requires a well-founded belief that the person from
of the beneficiary or landowner with due notice to the affected parties, and subject to existing laws, may whom title was received was himself the owner of the land, with the right to convey it. There is good faith
authorize the x x x conversion of the land and its dispositions. x x x where there is an honest intention to abstain from taking any unconscientious advantage from another." It is
the opposite of fraud.
Moreover, it is worth noting that the application for conversion had the backing of 5,000 or so FWBs, including
respondents Rene Galang, and Jose Julio Suniga, then leaders of the AMBALA and the Supervisory Group, To be sure, intervenor RCBC and LIPCO knew that the lots they bought were subjected to CARP coverage by
respectively, as evidenced by the Manifesto of Support they signed and which was submitted to the DAR. 39 If at means of a stock distribution plan, as the DAR conversion order was annotated at the back of the titles of the
all, this means that AMBALA should be estopped from questioning the conversion of a portion of Hacienda lots they acquired. However, they are of the honest belief that the subject lots were validly converted to
Luisita, which its leader has fully supported. commercial or industrial purposes and for which said lots were taken out of the CARP coverage subject of PARC
Resolution No. 89-12-2 and, hence, can be legally and validly acquired by them. After all, Sec. 65 of RA 6657
(b) LIPCO and RCBC as Innocent Purchasers for Value explicitly allows conversion and disposition of agricultural lands previously covered by CARP land acquisition
"after the lapse of five (5) years from its award when the land ceases to be economically feasible and sound for
agricultural purposes or the locality has become urbanized and the land will have a greater economic value for
The AMBALA, Rene Galang and the FARM are in accord that Rizal Commercial Banking Corporation (RCBC) and
residential, commercial or industrial purposes." Moreover, DAR notified all the affected parties, more
Luisita Industrial Park Corporation (LIPCO) are not innocent purchasers for value. The AMBALA, in particular,
particularly the FWBs, and gave them the opportunity to comment or oppose the proposed conversion. DAR,
argues that LIPCO, being a wholly-owned subsidiary of HLI, is conclusively presumed to have knowledge of the
after going through the necessary processes, granted the conversion of 500 hectares of Hacienda Luisita
agrarian dispute on the subject land and could not feign ignorance of this fact, especially since they have the
pursuant to its primary jurisdiction under Sec. 50 of RA 6657 to determine and adjudicate agrarian reform
same directors and stockholders.40 This is seconded by Rene Galang and AMBALA, through the PILC, which
matters and its original exclusive jurisdiction over all matters involving the implementation of agrarian reform.
intimate that a look at the General Information Sheets of the companies involved in the transfers of the 300-
The DAR conversion order became final and executory after none of the FWBs interposed an appeal to the CA.
hectare portion of Hacienda Luisita, specifically, Centennary Holdings, Inc. (Centennary), LIPCO and RCBC,
In this factual setting, RCBC and LIPCO purchased the lots in question on their honest and well-founded belief
would readily reveal that their directors are interlocked and connected to Tadeco and HLI. 41 Rene Galang and
that the previous registered owners could legally sell and convey the lots though these were previously subject
AMBALA, through the PILC, also allege that "with the clear-cut involvement of the leadership of all the
of CARP coverage. Ergo, RCBC and LIPCO acted in good faith in acquiring the subject lots. (Emphasis supplied.)
corporations concerned, LIPCO and RCBC cannot feign ignorance that the parcels of land they bought are under
the coverage of the comprehensive agrarian reform program [CARP] and that the conditions of the respective
sales are imbued with public interest where normal property relations in the Civil Law sense do not apply." 42 In the second place, the allegation that the converted lands remain undeveloped is contradicted by the
evidence on record, particularly, Annex "X" of LIPCOs Memorandum dated September 23, 2010, 45 which has
photographs showing that the land has been partly developed. 46 Certainly, it is a general rule that the factual
Avowing that the land subject of conversion still remains undeveloped, Rene Galang and AMBALA, through the
findings of administrative agencies are conclusive and binding on the Court when supported by substantial
PILC, further insist that the condition that "[t]he development of the land should be completed within the period
evidence.47However, this rule admits of certain exceptions, one of which is when the findings of fact are
of five [5] years from the issuance of this Order" was not complied with. AMBALA also argues that since RCBC
premised on the supposed absence of evidence and contradicted by the evidence on record. 48
and LIPCO merely stepped into the shoes of HLI, then they must comply with the conditions imposed in the
conversion order.43
In the third place, by arguing that the companies involved in the transfers of the 300-hectare portion of
Hacienda Luisita have interlocking directors and, thus, knowledge of one may already be imputed upon all the
In addition, FARM avers that among the conditions attached to the conversion order, which RCBC and LIPCO
other companies, AMBALA and Rene Galang, in effect, want this Court to pierce the veil of corporate fiction.
necessarily have knowledge of, are (a) that its approval shall in no way amend, diminish, or alter the
However, piercing the veil of corporate fiction is warranted "only in cases when the separate legal entity is
undertaking and obligations of HLI as contained in the [SDP] approved on November 21, 1989; and (b) that the
used to defeat public convenience, justify wrong, protect fraud, or defend crime, such that in the case of two
Set V Corporation Code * Doctrine of Peircing Corpo Veil Cases*Page 29 of 164

corporations, the law will regard the corporations as merged into one." 49 As succinctly discussed by the Court in Conversion Order can no longer be questioned." Besides, to disregard the Conversion Order through the
Velarde v. Lopez, Inc.:50 revocation of the approval of the SDP would create undue prejudice to LRC, which is not even a party to the
proceedings below, and would be tantamount to deprivation of property without due process of law.
Petitioner argues nevertheless that jurisdiction over the subsidiary is justified by piercing the veil of corporate
fiction. Piercing the veil of corporate fiction is warranted, however, only in cases when the separate legal entity Nonethess, the minority is of the adamant view that since LRC failed to intervene in the instant case and was,
is used to defeat public convenience, justify wrong, protect fraud, or defend crime, such that in the case of two therefore, unable to present evidence supporting its good faith purchase of the 200-hectare converted land,
corporations, the law will regard the corporations as merged into one. The rationale behind piercing a then LRC should be given full opportunity to present its case before the DAR. This minority view is a
corporations identity is to remove the barrier between the corporation from the persons comprising it to thwart contradiction in itself. Given that LRC did not intervene and is, therefore, not a party to the instant case, then it
the fraudulent and illegal schemes of those who use the corporate personality as a shield for undertaking would be incongruous to order them to present evidence before the DAR. Such an order, if issued by this Court,
certain proscribed activities. would not be binding upon the LRC.

In applying the doctrine of piercing the veil of corporate fiction, the following requisites must be established: Moreover, LRC may be considered to have waived its right to participate in the instant petition since it did not
(1) control, not merely majority or complete stock control; (2) such control must have been used by the intervene in the DAR proceedings for the nullification of the PARC Resolution No. 89-12-2 which approved the
defendant to commit fraud or wrong, to perpetuate the violation of a statutory or other positive legal duty, or SDP.
dishonest acts in contravention of plaintiffs legal rights; and (3) the aforesaid control and breach of duty must
proximately cause the injury or unjust loss complained of. (Citations omitted.) (c) Proceeds of the sale of the 500-hectare converted land

Nowhere, however, in the pleadings and other records of the case can it be gathered that respondent has and of the 80.51-hectare land used for the SCTEX
complete control over Sky Vision, not only of finances but of policy and business practice in respect to the
transaction attacked, so that Sky Vision had at the time of the transaction no separate mind, will or existence of
As previously mentioned, We ruled in Our July 5, 2011 Decision that since the Court excluded the 500-hectare
its own. The existence of interlocking directors, corporate officers and shareholders is not enough justification
lot subject of the August 14, 1996 Conversion Order and the 80.51-hectare SCTEX lot acquired by the
to pierce the veil of corporate fiction in the absence of fraud or other public policy considerations.
government from compulsory coverage, then HLI and its subsidiary, Centennary, should be liable to the FWBs
for the price received for said lots. Thus:
Absent any allegation or proof of fraud or other public policy considerations, the existence of interlocking
directors, officers and stockholders is not enough justification to pierce the veil of corporate fiction as in the
There is a claim that, since the sale and transfer of the 500 hectares of land subject of the August 14, 1996
instant case.
Conversion Order and the 80.51-hectare SCTEX lot came after compulsory coverage has taken place, the FWBs
should have their corresponding share of the lands value. There is merit in the claim. Since the SDP approved
And in the fourth place, the fact that this Court, in its July 5, 2011 Decision, ordered the payment of the by PARC Resolution No. 89-12-2 has been nullified, then all the lands subject of the SDP will automatically be
proceeds of the sale of the converted land, and even of the 80.51-hectare land sold to the government, through subject of compulsory coverage under Sec. 31 of RA 6657. Since the Court excluded the 500-hectare lot subject
the Bases Conversion Development Authority, to the qualified FWBs, effectively fulfils the conditions in the of the August 14, 1996 Conversion Order and the 80.51-hectare SCTEX lot acquired by the government from
conversion order, to wit: (1) that its approval shall in no way amend, diminish, or alter the undertaking and the area covered by SDP, then HLI and its subsidiary, Centennary, shall be liable to the FWBs for the price
obligations of HLI as contained in the SDP approved on November 21, 1989; and (2) that the benefits, wages received for said lots. HLI shall be liable for the value received for the sale of the 200-hectare land to LRC in the
and the like, received by the FWBs shall not in any way be reduced or adversely affected, among others. amount of PhP 500,000,000 and the equivalent value of the 12,000,000 shares of its subsidiary, Centennary,
for the 300-hectare lot sold to LIPCO for the consideration of PhP 750,000,000. Likewise, HLI shall be liable for
A view has also been advanced that the 200-hectare lot transferred to Luisita Realty Corporation (LRC) should PhP 80,511,500 as consideration for the sale of the 80.51-hectare SCTEX lot.
be included in the compulsory coverage because the corporation did not intervene.
We, however, note that HLI has allegedly paid 3% of the proceeds of the sale of the 500-hectare land and
We disagree. Since the 200-hectare lot formed part of the SDP that was nullified by PARC Resolution 2005-32- 80.51-hectare SCTEX lot to the FWBs. We also take into account the payment of taxes and expenses relating to
01, this Court is constrained to make a ruling on the rights of LRC over the said lot. Moreover, the 500-hectare the transfer of the land and HLIs statement that most, if not all, of the proceeds were used for legitimate
portion of Hacienda Luisita, of which the 200-hectare portion sold to LRC and the 300-hectare portion corporate purposes. In order to determine once and for all whether or not all the proceeds were properly
subsequently acquired by LIPCO and RCBC were part of, was already the subject of the August 14, 1996 DAR utilized by HLI and its subsidiary, Centennary, DAR will engage the services of a reputable accounting firm to
Conversion Order. By virtue of the said conversion order, the land was already reclassified as be approved by the parties to audit the books of HLI to determine if the proceeds of the sale of the 500-hectare
industrial/commercial land not subject to compulsory coverage. Thus, if We place the 200-hectare lot sold to land and the 80.51-hectare SCTEX lot were actually used for legitimate corporate purposes, titling expenses
LRC under compulsory coverage, this Court would, in effect, be disregarding the DAR Conversion Order, which and in compliance with the August 14, 1996 Conversion Order. The cost of the audit will be shouldered by HLI.
has long attained its finality. And as this Court held in Berboso v. CA, 51 "Once final and executory, the
Set V Corporation Code * Doctrine of Peircing Corpo Veil Cases*Page 30 of 164

If after such audit, it is determined that there remains a balance from the proceeds of the sale, then the Considering that the 500-hectare converted land, as well as the 80.51-hectare SCTEX lot, should have been
balance shall be distributed to the qualified FWBs. included in the compulsory coverage were it not for their conversion and valid transfers, then it is only but
proper that the price received for the sale of these lots should be given to the qualified FWBs. In effect, the
HLI, however, takes exception to the above-mentioned ruling and contends that it is not proper to distribute the proceeds from the sale shall take the place of the lots.
unspent or unused balance of the proceeds of the sale of the 500-hectare converted land and 80.51-hectare
SCTEX lot to the qualified FWBs for the following reasons: (1) the proceeds of the sale belong to the The Court, in its July 5, 2011 Decision, however, takes into account, inter alia, the payment of taxes and
corporation, HLI, as corporate capital and assets in substitution for the portions of its land asset which were expenses relating to the transfer of the land, as well as HLIs statement that most, if not all, of the proceeds
sold to third parties; (2) to distribute the cash sales proceeds of the portions of the land asset to the FWBs, who were used for legitimate corporate purposes. Accordingly, We ordered the deduction of the taxes and expenses
are stockholders of HLI, is to dissolve the corporation and distribute the proceeds as liquidating dividends relating to the transfer of titles to the transferees, and the expenditures incurred by HLI and Centennary for
without even paying the creditors of the corporation; and (3) the doing of said acts would violate the stringent legitimate corporate purposes, among others.
provisions of the Corporation Code and corporate practice. 52
On this note, DAR claims that the "[l]egitimate corporate expenses should not be deducted as there is no basis
Apparently, HLI seeks recourse to the Corporation Code in order to avoid its liability to the FWBs for the price for it, especially since only the auditing to be conducted on the financial records of HLI will reveal the amounts
received for the 500-hectare converted lot and the 80.51-hectare SCTEX lot. However, as We have established to be offset between HLI and the FWBs." 55
in Our July 5, 2011 Decision, the rights, obligations and remedies of the parties in the instant case are primarily
governed by RA 6657 and HLI cannot shield itself from the CARP coverage merely under the convenience of The contention is unmeritorious. The possibility of an offsetting should not prevent Us from deducting the
being a corporate entity. In this regard, it should be underscored that the agricultural lands held by HLI by legitimate corporate expenses incurred by HLI and Centennary. After all, the Court has ordered for a proper
virtue of the SDP are no ordinary assets. These are special assets, because, originally, these should have been auditing "[i]n order to determine once and for all whether or not all the proceeds were properly utilized by HLI
distributed to the FWBs were it not for the approval of the SDP by PARC. Thus, the government cannot renege and its subsidiary, Centennary." In this regard, DAR is tasked to "engage the services of a reputable accounting
on its responsibility over these assets. Likewise, HLI is no ordinary corporation as it was formed and organized firm to be approved by the parties to audit the books of HLI to determine if the proceeds of the sale of the 500-
precisely to make use of these agricultural lands actually intended for distribution to the FWBs. Thus, it cannot hectare land and the 80.51-hectare SCTEX lot were actually used for legitimate corporate purposes, titling
shield itself from the coverage of CARP by invoking the Corporation Code. As explained by the Court: expenses and in compliance with the August 14, 1996 Conversion Order." Also, it should be noted that it is HLI
which shall shoulder the cost of audit to reduce the burden on the part of the FWBs. Concomitantly, the
HLI also parlays the notion that the parties to the SDOA should now look to the Corporation Code, instead of to legitimate corporate expenses incurred by HLI and Centennary, as will be determined by a reputable
RA 6657, in determining their rights, obligations and remedies. The Code, it adds, should be the applicable law accounting firm to be engaged by DAR, shall be among the allowable deductions from the proceeds of the sale
on the disposition of the agricultural land of HLI. of the 500-hectare land and the 80.51-hectare SCTEX lot.

Contrary to the view of HLI, the rights, obligations and remedies of the parties to the SDOA embodying the SDP We, however, find that the 3% production share should not be deducted from the proceeds of the sale of the
are primarily governed by RA 6657. It should abundantly be made clear that HLI was precisely created in order 500-hectare converted land and the 80.51-hectare SCTEX lot. The 3% production share, like the homelots, was
to comply with RA 6657, which the OSG aptly described as the "mother law" of the SDOA and the SDP. 53It is, among the benefits received by the FWBs as farmhands in the agricultural enterprise of HLI and, thus, should
thus, paradoxical for HLI to shield itself from the coverage of CARP by invoking exclusive applicability of the not be taken away from the FWBs.
Corporation Code under the guise of being a corporate entity.
Contrarily, the minority is of the view that as a consequence of the revocation of the SDP, the parties should be
Without in any way minimizing the relevance of the Corporation Code since the FWBs of HLI are also restored to their respective conditions prior to its execution and approval, subject to the application of the
stockholders, its applicability is limited as the rights of the parties arising from the SDP should not be made to principle of set-off or compensation. Such view is patently misplaced.
supplant or circumvent the agrarian reform program.
The law on contracts, i.e. mutual restitution, does not apply to the case at bar. To reiterate, what was actually
Without doubt, the Corporation Code is the general law providing for the formation, organization and regulation revoked by this Court, in its July 5, 2011 Decision, is PARC Resolution No. 89-12-2 approving the SDP. To
of private corporations. On the other hand, RA 6657 is the special law on agrarian reform. As between a elucidate, it was the SDP, not the SDOA, which was presented for approval by Tadeco to DAR. 56 The SDP
general and special law, the latter shall prevailgeneralia specialibus non derogant. 54 Besides, the present explained the mechanics of the stock distribution but did not make any reference nor correlation to the SDOA.
impasse between HLI and the private respondents is not an intra-corporate dispute which necessitates the The pertinent portions of the proposal read:
application of the Corporation Code. What private respondents questioned before the DAR is the proper
implementation of the SDP and HLIs compliance with RA 6657. Evidently, RA 6657 should be the applicable law MECHANICS OF STOCK DISTRIBUTION PLAN
to the instant case. (Emphasis supplied.)
Set V Corporation Code * Doctrine of Peircing Corpo Veil Cases*Page 31 of 164

Under Section 31 of Republic Act No. 6657, a corporation owning agricultural land may distribute among the At the other end of the spectrum, AMBALA alleges that HLI should no longer be paid just compensation for the
qualified beneficiaries such proportion or percentage of its capital stock that the value of the agricultural land agricultural land that will be distributed to the FWBs, since the Manila Regional Trial Court (RTC) already
actually devoted to agricultural activities, bears in relation to the corporations total assets. Conformably with rendered a decision ordering "the Cojuangcos to transfer the control of Hacienda Luisita to the Ministry of
this legal provision, Tarlac Development Corporation hereby submits for approval a stock distribution plan that Agrarian Reform, which will distribute the land to small farmers after compensating the landowners P3.988
envisions the following:57 (Terms and conditions omitted; emphasis supplied) million."62 In the event, however, that this Court will rule that HLI is indeed entitled to compensation, AMBALA
contends that it should be pegged at forty thousand pesos (PhP 40,000) per hectare, since this was the same
xxxx value that Tadeco declared in 1989 to make sure that the farmers will not own the majority of its stocks. 63

The above stock distribution plan is hereby submitted on the basis of all these benefits that the farmworker- Despite the above propositions, We maintain that the date of "taking" is November 21, 1989, the date when
beneficiaries of Hacienda Luisita will receive under its provisions in addition to their regular compensation as PARC approved HLIs SDP per PARC Resolution No. 89-12-2, in view of the fact that this is the time that the
farmhands in the agricultural enterprise and the fringe benefits granted to them by their collective bargaining FWBs were considered to own and possess the agricultural lands in Hacienda Luisita. To be precise, these lands
agreement with management.58 became subject of the agrarian reform coverage through the stock distribution scheme only upon the approval
of the SDP, that is, November 21, 1989. Thus, such approval is akin to a notice of coverage ordinarily issued
under compulsory acquisition. Further, any doubt should be resolved in favor of the FWBs. As this Court held in
Also, PARC Resolution No. 89-12-2 reads as follows:
Perez-Rosario v. CA:64

RESOLUTION APPROVING THE STOCK DISTRIBUTION PLAN OF TARLAC DEVELOPMENT COMPANY/HACIENDA


It is an established social and economic fact that the escalation of poverty is the driving force behind the
LUISITA INCORPORATED (TDC/HLI)
political disturbances that have in the past compromised the peace and security of the people as well as the
continuity of the national order. To subdue these acute disturbances, the legislature over the course of the
NOW THEREFORE, on motion duly seconded, history of the nation passed a series of laws calculated to accelerate agrarian reform, ultimately to raise the
material standards of living and eliminate discontent. Agrarian reform is a perceived solution to social
RESOLVED, as it is hereby resolved, to approve the stock distribution plan of TDC/HLI. instability. The edicts of social justice found in the Constitution and the public policies that underwrite them, the
extraordinary national experience, and the prevailing national consciousness, all command the great
departments of government to tilt the balance in favor of the poor and underprivileged whenever reasonable
UNANIMOUSLY APPROVED.59 (Emphasis supplied)
doubt arises in the interpretation of the law. But annexed to the great and sacred charge of protecting the weak
is the diametric function to put every effort to arrive at an equitable solution for all parties concerned: the jural
Clearly, what was approved by PARC is the SDP and not the SDOA. There is, therefore, no basis for this Court to postulates of social justice cannot shield illegal acts, nor do they sanction false sympathy towards a certain
apply the law on contracts to the revocation of the said PARC Resolution. class, nor yet should they deny justice to the landowner whenever truth and justice happen to be on her side.
In the occupation of the legal questions in all agrarian disputes whose outcomes can significantly affect societal
IV. Just Compensation harmony, the considerations of social advantage must be weighed, an inquiry into the prevailing social
interests is necessary in the adjustment of conflicting demands and expectations of the people, and the social
interdependence of these interests, recognized. (Emphasis supplied.)
In Our July 5, 2011 Decision, We stated that "HLI shall be paid just compensation for the remaining agricultural
land that will be transferred to DAR for land distribution to the FWBs." We also ruled that the date of the
"taking" is November 21, 1989, when PARC approved HLIs SDP per PARC Resolution No. 89-12-2. The minority contends that it is the date of the notice of coverage, that is, January 2, 2006, which is
determinative of the just compensation HLI is entitled to for its expropriated lands. To support its contention, it
cited numerous cases where the time of the taking was reckoned on the date of the issuance of the notice of
In its Motion for Clarification and Partial Reconsideration, HLI disagrees with the foregoing ruling and contends
coverage.
that the "taking" should be reckoned from finality of the Decision of this Court, or at the very least, the
reckoning period may be tacked to January 2, 2006, the date when the Notice of Coverage was issued by the
DAR pursuant to PARC Resolution No. 2006-34-01 recalling/revoking the approval of the SDP. 60 However, a perusal of the cases cited by the minority would reveal that none of them involved the stock
distribution scheme. Thus, said cases do not squarely apply to the instant case. Moreover, it should be noted
that it is precisely because the stock distribution option is a distinctive mechanism under RA 6657 that it
For their part, Mallari, et al. argue that the valuation of the land cannot be based on November 21, 1989, the
cannot be treated similarly with that of compulsory land acquisition as these are two (2) different modalities
date of approval of the SDP. Instead, they aver that the date of "taking" for valuation purposes is a factual issue
under the agrarian reform program. As We have stated in Our July 5, 2011 Decision, RA 6657 "provides two (2)
best left to the determination of the trial courts. 61
alternative modalities, i.e., land or stock transfer, pursuant to either of which the corporate landowner can
comply with CARP."
Set V Corporation Code * Doctrine of Peircing Corpo Veil Cases*Page 32 of 164

In this regard, it should be noted that when HLI submitted the SDP to DAR for approval, it cannot be gainsaid qualified beneficiaries for a period of ten (10) years: Provided, however, That the children or the spouse of the
that the stock distribution scheme is clearly HLIs preferred modality in order to comply with CARP. And when transferor shall have a right to repurchase the land from the government or LBP within a period of two (2)
the SDP was approved, stocks were given to the FWBs in lieu of land distribution. As aptly observed by the years. Due notice of the availability of the land shall be given by the LBP to the Barangay Agrarian Reform
minority itself, "[i]nstead of expropriating lands, what the government took and distributed to the FWBs were Committee (BARC) of the barangay where the land is situated. The Provincial Agrarian Coordinating Committee
shares of stock of petitioner HLI in proportion to the value of the agricultural lands that should have been (PARCCOM), as herein provided, shall, in turn, be given due notice thereof by the BARC.
expropriated and turned over to the FWBs." It cannot, therefore, be denied that upon the approval of the SDP
submitted by HLI, the agricultural lands of Hacienda Luisita became subject of CARP coverage. Evidently, the If the land has not yet been fully paid by the beneficiary, the right to the land may be transferred or conveyed,
approval of the SDP took the place of a notice of coverage issued under compulsory acquisition. with prior approval of the DAR, to any heir of the beneficiary or to any other beneficiary who, as a condition for
such transfer or conveyance, shall cultivate the land himself. Failing compliance herewith, the land shall be
Also, it is surprising that while the minority opines that under the stock distribution option, "title to the property transferred to the LBP which shall give due notice of the availability of the land in the manner specified in the
remains with the corporate landowner, which should presumably be dominated by farmers with majority immediately preceding paragraph.
stockholdings in the corporation," it still insists that the just compensation that should be given to HLI is to be
reckoned on January 2, 2006, the date of the issuance of the notice of coverage, even after it found that the In the event of such transfer to the LBP, the latter shall compensate the beneficiary in one lump sum for the
FWBs did not have the majority stockholdings in HLI contrary to the supposed avowed policy of the law. In amounts the latter has already paid, together with the value of improvements he has made on the land.
effect, what the minority wants is to prejudice the FWBs twice. Given that the FWBs should have had majority (Emphasis supplied.)
stockholdings in HLI but did not, the minority still wants the government to pay higher just compensation to
HLI. Even if it is the government which will pay the just compensation to HLI, this will also affect the FWBs as
To implement the above-quoted provision, inter alia, DAR issued Administrative Order No. 1, Series of 1989
they will be paying higher amortizations to the government if the "taking" will be considered to have taken
(DAO 1) entitled Rules and Procedures Governing Land Transactions. Said Rules set forth the rules on validity of
place only on January 2, 2006.
land transactions, to wit:

The foregoing notwithstanding, it bears stressing that the DAR's land valuation is only preliminary and is not,
II. RULES ON VALIDITY OF LAND TRANSACTIONS
by any means, final and conclusive upon the landowner. The landowner can file an original action with the RTC
acting as a special agrarian court to determine just compensation. The court has the right to review with
finality the determination in the exercise of what is admittedly a judicial function. 65 A. The following transactions are valid:

A view has also been advanced that HLI should pay the qualified FWBs rental for the use and possession of the 1. Those executed by the original landowner in favor of the qualified beneficiary from among those
land up to the time it surrenders possession and control over these lands. What this view fails to consider is the certified by DAR.
fact that the FWBs are also stockholders of HLI prior to the revocation of PARC Resolution No. 89-12-2. Also, the
income earned by the corporation from its possession and use of the land ultimately redounded to the benefit 2. Those in favor of the government, DAR or the Land Bank of the Philippines.
of the FWBs based on its business operations in the form of salaries, benefits voluntarily granted by HLI and
other fringe benefits under their Collective Bargaining Agreement. That being so, there would be unjust
3. Those covering lands retained by the landowner under Section 6 of R.A. 6657 duly certified by the
enrichment on the part of the FWBs if HLI will still be required to pay rent for the use of the land in question.
designated DAR Provincial Agrarian Reform Officer (PARO) as a retention area, executed in favor of
transferees whose total landholdings inclusive of the land to be acquired do not exceed five (5)
V. Sale to Third Parties hectares; subject, however, to the right of pre-emption and/or redemption of tenant/lessee under
Section 11 and 12 of R.A. 3844, as amended.
There is a view that since the agricultural lands in Hacienda Luisita were placed under CARP coverage through
the SDOA scheme on May 11, 1989, then the 10-year period prohibition on the transfer of awarded lands under xxxx
RA 6657 lapsed on May 10, 1999, and, consequently, the qualified FWBs should already be allowed to sell these
lands with respect to their land interests to third parties, including HLI, regardless of whether they have fully
4. Those executed by beneficiaries covering lands acquired under any agrarian reform law in favor of
paid for the lands or not.
the government, DAR, LBP or other qualified beneficiaries certified by DAR.

The proposition is erroneous. Sec. 27 of RA 6657 states:


5. Those executed after ten (10) years from the issuance and registration of the Emancipation Patent
or Certificate of Land Ownership Award.
SEC. 27. Transferability of Awarded Lands. - Lands acquired by beneficiaries under this Act may not be sold,
transferred or conveyed except through hereditary succession, or to the government, or to the LBP, or to other
B. The following transactions are not valid:
Set V Corporation Code * Doctrine of Peircing Corpo Veil Cases*Page 33 of 164

1. Sale, disposition, lease management contract or transfer of possession of private lands executed by that the stock distribution scheme is sanctioned by Sec. 31 of RA 6657, which specifically allows corporations to
the original landowner prior to June 15, 1988, which are registered on or before September 13, 1988, divest a proportion of their capital stock that "the agricultural land, actually devoted to agricultural activities,
or those executed after June 15, 1988, covering an area in excess of the five-hectare retention limit in bears in relation to the companys total assets." On the other hand, no special rules exist under RA 6657
violation of R.A. 6657. concerning the proposed buy-back scheme; hence, the general rules on retention limits should apply.

2. Those covering lands acquired by the beneficiary under R.A. 6657 and executed within ten (10) Further, the position that the qualified FWBs are now free to transact with third parties concerning their land
years from the issuance and registration of an Emancipation Patent or Certificate of Land Ownership interests, regardless of whether they have fully paid for the lands or not, also transgresses the second
Award. paragraph of Sec. 27 of RA 6657, which plainly states that "[i]f the land has not yet been fully paid by the
beneficiary, the right to the land may be transferred or conveyed, with prior approval of the DAR, to any heir of
3. Those executed in favor of a person or persons not qualified to acquire land under R.A. 6657. the beneficiary or to any other beneficiary who, as a condition for such transfer or conveyance, shall cultivate
the land himself. Failing compliance herewith, the land shall be transferred to the LBP x x x." When the words
and phrases in the statute are clear and unequivocal, the law is applied according to its express terms. 68 Verba
4. Sale, transfer, conveyance or change of nature of the land outside of urban centers and city limits
legis non est recedendum, or from the words of a statute there should be no departure. 69
either in whole or in part as of June 15, 1988, when R.A. 6657 took effect, except as provided for under
DAR Administrative Order No. 15, series of 1988.
The minority, however, posits that "[t]o insist that the FWBs rights sleep for a period of ten years is unrealistic,
and may seriously deprive them of real opportunities to capitalize and maximize the victory of direct land
5. Sale, transfer or conveyance by beneficiary of the right to use or any other usufructuary right over
distribution." By insisting that We disregard the ten-year restriction under the law in the case at bar, the
the land he acquired by virtue of being a beneficiary, in order to circumvent the law.
minority, in effect, wants this Court to engage in judicial legislation, which is violative of the principle of
separation of powers.70 The discourse by Ruben E. Agpalo, in his book on statutory construction, is
x x x x (Emphasis supplied.) enlightening:

Without a doubt, under RA 6657 and DAO 1, the awarded lands may only be transferred or conveyed after ten Where the law is clear and unambiguous, it must be taken to mean exactly what it says and the court has no
(10) years from the issuance and registration of the emancipation patent (EP) or certificate of land ownership choice but to see to it that its mandate is obeyed. Where the law is clear and free from doubt or ambiguity,
award (CLOA). Considering that the EPs or CLOAs have not yet been issued to the qualified FWBs in the instant there is no room for construction or interpretation. Thus, where what is not clearly provided in the law is read
case, the 10-year prohibitive period has not even started. Significantly, the reckoning point is the issuance of into the law by construction because it is more logical and wise, it would be to encroach upon legislative
the EP or CLOA, and not the placing of the agricultural lands under CARP coverage. prerogative to define the wisdom of the law, which is judicial legislation. For whether a statute is wise or
expedient is not for the courts to determine. Courts must administer the law, not as they think it ought to be
Moreover, if We maintain the position that the qualified FWBs should be immediately allowed the option to sell but as they find it and without regard to consequences.71 (Emphasis supplied.)
or convey the agricultural lands in Hacienda Luisita, then all efforts at agrarian reform would be rendered
nugatory by this Court, since, at the end of the day, these lands will just be transferred to persons not entitled And as aptly stated by Chief Justice Renato Corona in his Dissenting Opinion in Ang Ladlad LGBT Party v.
to land distribution under CARP. As aptly noted by the late Senator Neptali Gonzales during the Joint COMELEC:72
Congressional Conference Committee on the Comprehensive Agrarian Reform Program Bills:
Regardless of the personal beliefs and biases of its individual members, this Court can only apply and interpret
SEN. GONZALES. My point is, as much as possible let the said lands be distributed under CARP remain with the the Constitution and the laws. Its power is not to create policy but to recognize, review or reverse the policy
beneficiaries and their heirs because that is the lesson that we have to learn from PD No. 27. If you will talk crafted by the political departments if and when a proper case is brought before it. Otherwise, it will tread on
with the Congressmen representing Nueva Ecija, Pampanga and Central Luzon provinces, law or no law, you the dangerous grounds of judicial legislation.
will find out that more than one-third of the original, of the lands distributed under PD 27 are no longer owned,
possessed or being worked by the grantees or the awardees of the same, something which we ought to avoid
Considerably, this Court is left with no other recourse but to respect and apply the law.
under the CARP bill that we are going to enact.66 (Emphasis supplied.)

VI. Grounds for Revocation of the SDP


Worse, by raising that the qualified beneficiaries may sell their interest back to HLI, this smacks of outright
indifference to the provision on retention limits 67 under RA 6657, as this Court, in effect, would be allowing HLI,
the previous landowner, to own more than five (5) hectares of agricultural land, which We cannot countenance. AMBALA and FARM reiterate that improving the economic status of the FWBs is among the legal obligations of
There is a big difference between the ownership of agricultural lands by HLI under the stock distribution HLI under the SDP and is an imperative imposition by RA 6657 and DAO 10. 73 FARM further asserts that "[i]f
scheme and its eventual acquisition of the agricultural lands from the qualified FWBs under the proposed buy- that minimum threshold is not met, why allow [stock distribution option] at all, unless the purpose is not social
back scheme. The rule on retention limits does not apply to the former but only to the latter in view of the fact justice but a political accommodation to the powerful." 74
Set V Corporation Code * Doctrine of Peircing Corpo Veil Cases*Page 34 of 164

Contrary to the assertions of AMBALA and FARM, nowhere in the SDP, RA 6657 and DAO 10 can it be inferred LUISITA, INC. Salaries, Benefits and Credit Privileges (in Thousand Pesos) Since the Stock Option was Approved
that improving the economic status of the FWBs is among the legal obligations of HLI under the SDP or is an by PARC/CARP," detailing what HLI gave their workers from 1989 to 2005. The sum total, as added up by the
imperative imposition by RA 6657 and DAO 10, a violation of which would justify discarding the stock Court, yields the following numbers: Total Direct Cash Out (Salaries/Wages & Cash Benefits) = PhP 2,927,848;
distribution option. As We have painstakingly explained in Our July 5, 2011 Decision: Total Non-Direct Cash Out (Hospital/Medical Benefits) = PhP 303,040. The cash out figures, as stated in the
report, include the cost of homelots; the PhP 150 million or so representing 3% of the gross produce of the
In the Terminal Report adopted by PARC, it is stated that the SDP violates the agrarian reform policy under Sec. hacienda; and the PhP 37.5 million representing 3% from the proceeds of the sale of the 500-hectare converted
2 of RA 6657, as the said plan failed to enhance the dignity and improve the quality of lives of the FWBs lands. While not included in the report, HLI manifests having given the FWBs 3% of the PhP 80 million paid for
through greater productivity of agricultural lands. We disagree. the 80 hectares of land traversed by the SCTEX. On top of these, it is worth remembering that the shares of
stocks were given by HLI to the FWBs for free. Verily, the FWBs have benefited from the SDP.

Sec. 2 of RA 6657 states:


To address urgings that the FWBs be allowed to disengage from the SDP as HLI has not anyway earned profits
through the years, it cannot be over-emphasized that, as a matter of common business sense, no corporation
SECTION 2. Declaration of Principles and Policies.It is the policy of the State to pursue a Comprehensive
could guarantee a profitable run all the time. As has been suggested, one of the key features of an SDP of a
Agrarian Reform Program (CARP). The welfare of the landless farmers and farm workers will receive the highest
corporate landowner is the likelihood of the corporate vehicle not earning, or, worse still, losing money.
consideration to promote social justice and to move the nation towards sound rural development and
industrialization, and the establishment of owner cultivatorship of economic-sized farms as the basis of
Philippine agriculture. The Court is fully aware that one of the criteria under DAO 10 for the PARC to consider the advisability of
approving a stock distribution plan is the likelihood that the plan "would result in increased income and greater
benefits to [qualified beneficiaries] than if the lands were divided and distributed to them individually." But as
To this end, a more equitable distribution and ownership of land, with due regard to the rights of landowners to
aptly noted during the oral arguments, DAO 10 ought to have not, as it cannot, actually exact assurance of
just compensation and to the ecological needs of the nation, shall be undertaken to provide farmers and farm
success on something that is subject to the will of man, the forces of nature or the inherent risky nature of
workers with the opportunity to enhance their dignity and improve the quality of their lives through greater
business.75 Just like in actual land distribution, an SDP cannot guarantee, as indeed the SDOA does not
productivity of agricultural lands.
guarantee, a comfortable life for the FWBs. The Court can take judicial notice of the fact that there were many
instances wherein after a farmworker beneficiary has been awarded with an agricultural land, he just
The agrarian reform program is founded on the right of farmers and regular farm workers, who are landless, to subsequently sells it and is eventually left with nothing in the end.
own directly or collectively the lands they till or, in the case of other farm workers, to receive a share of the
fruits thereof. To this end, the State shall encourage the just distribution of all agricultural lands, subject to the
In all then, the onerous condition of the FWBs economic status, their life of hardship, if that really be the case,
priorities and retention limits set forth in this Act, having taken into account ecological, developmental, and
can hardly be attributed to HLI and its SDP and provide a valid ground for the plans revocation. (Citations
equity considerations, and subject to the payment of just compensation. The State shall respect the right of
omitted; emphasis in the original.)
small landowners and shall provide incentives for voluntary land-sharing.

This Court, despite the above holding, still affirmed the revocation by PARC of its approval of the SDP based on
Paragraph 2 of the above-quoted provision specifically mentions that "a more equitable distribution and
the following grounds: (1) failure of HLI to fully comply with its undertaking to distribute homelots to the FWBs
ownership of land x x x shall be undertaken to provide farmers and farm workers with the opportunity to
under the SDP; (2) distribution of shares of stock to the FWBs based on the number of "man days" or "number
enhance their dignity and improve the quality of their lives through greater productivity of agricultural lands."
of days worked" by the FWB in a years time; and (3) 30-year timeframe for the implementation or distribution
Of note is the term "opportunity" which is defined as a favorable chance or opening offered by circumstances.
of the shares of stock to the FWBs.
Considering this, by no stretch of imagination can said provision be construed as a guarantee in improving the
lives of the FWBs. At best, it merely provides for a possibility or favorable chance of uplifting the economic
status of the FWBs, which may or may not be attained. Just the same, Mallari, et al. posit that the homelots required to be distributed have all been distributed
pursuant to the SDOA, and that what merely remains to be done is the release of title from the Register of
Deeds.76 They further assert that there has been no dilution of shares as the corporate records would show that
Pertinently, improving the economic status of the FWBs is neither among the legal obligations of HLI under the
if ever not all of the 18,804.32 shares were given to the actual original FWB, the recipient of the difference is
SDP nor an imperative imposition by RA 6657 and DAO 10, a violation of which would justify discarding the
the next of kin or children of said original FWB.77 Thus, they submit that since the shares were given to the
stock distribution option. Nothing in that option agreement, law or department order indicates otherwise.
same "family beneficiary," this should be deemed as substantial compliance with the provisions of Sec. 4 of
DAO 10.78 Also, they argue that there has been no violation of the three-month period to implement the SDP as
Significantly, HLI draws particular attention to its having paid its FWBs, during the regime of the SDP (1989- mandated by Sec. 11 of DAO, since this provision must be read in light of Sec. 10 of Executive Order No. 229,
2005), some PhP 3 billion by way of salaries/wages and higher benefits exclusive of free hospital and medical the pertinent portion of which reads, "The approval by the PARC of a plan for such stock distribution, and its
benefits to their immediate family. And attached as Annex "G" to HLIs Memorandum is the certified true report initial implementation, shall be deemed compliance with the land distribution requirement of the CARP." 79
of the finance manager of Jose Cojuangco & Sons Organizations-Tarlac Operations, captioned as "HACIENDA
Set V Corporation Code * Doctrine of Peircing Corpo Veil Cases*Page 35 of 164

Again, the matters raised by Mallari, et al. have been extensively discussed by the Court in its July 5, 2011 Justice Velasco: x x x There is also an allegation that the farmer beneficiaries, the qualified family beneficiaries
Decision. As stated: were not given the 240 square meters each. So, can you also [prove] that the qualified family beneficiaries
were already provided the 240 square meter homelots.
On Titles to Homelots
Atty. Asuncion: We will, your Honor please.
Under RA 6657, the distribution of homelots is required only for corporations or business associations owning or
operating farms which opted for land distribution. Sec. 30 of RA 6657 states: Other than the financial report, however, no other substantial proof showing that all the qualified beneficiaries
have received homelots was submitted by HLI. Hence, this Court is constrained to rule that HLI has not yet fully
SEC. 30. Homelots and Farmlots for Members of Cooperatives.The individual members of the cooperatives or complied with its undertaking to distribute homelots to the FWBs under the SDP.
corporations mentioned in the preceding section shall be provided with homelots and small farmlots for their
family use, to be taken from the land owned by the cooperative or corporation. On "Man Days" and the Mechanics of Stock Distribution

The "preceding section" referred to in the above-quoted provision is as follows: In our review and analysis of par. 3 of the SDOA on the mechanics and timelines of stock distribution, We find
that it violates two (2) provisions of DAO 10. Par. 3 of the SDOA states:
SEC. 29. Farms Owned or Operated by Corporations or Other Business Associations.In the case of farms
owned or operated by corporations or other business associations, the following rules shall be observed by the 3. At the end of each fiscal year, for a period of 30 years, the SECOND PARTY [HLI] shall arrange with the FIRST
PARC. PARTY [TDC] the acquisition and distribution to the THIRD PARTY [FWBs] on the basis of number of days worked
and at no cost to them of one-thirtieth (1/30) of 118,391,976.85 shares of the capital stock of the SECOND
In general, lands shall be distributed directly to the individual worker-beneficiaries. PARTY that are presently owned and held by the FIRST PARTY, until such time as the entire block of
118,391,976.85 shares shall have been completely acquired and distributed to the THIRD PARTY.

In case it is not economically feasible and sound to divide the land, then it shall be owned collectively by the
worker-beneficiaries who shall form a workers cooperative or association which will deal with the corporation Based on the above-quoted provision, the distribution of the shares of stock to the FWBs, albeit not entailing a
or business association. Until a new agreement is entered into by and between the workers cooperative or cash out from them, is contingent on the number of "man days," that is, the number of days that the FWBs
association and the corporation or business association, any agreement existing at the time this Act takes have worked during the year. This formula deviates from Sec. 1 of DAO 10, which decrees the distribution of
effect between the former and the previous landowner shall be respected by both the workers cooperative or equal number of shares to the FWBs as the minimum ratio of shares of stock for purposes of compliance with
association and the corporation or business association. Sec. 31 of RA 6657. As stated in Sec. 4 of DAO 10:

Noticeably, the foregoing provisions do not make reference to corporations which opted for stock distribution Section 4. Stock Distribution Plan.The [SDP] submitted by the corporate landowner-applicant shall provide for
under Sec. 31 of RA 6657. Concomitantly, said corporations are not obliged to provide for it except by the distribution of an equal number of shares of the same class and value, with the same rights and features as
stipulation, as in this case. all other shares, to each of the qualified beneficiaries. This distribution plan in all cases, shall be at least the
minimum ratio for purposes of compliance with Section 31 of R.A. No. 6657.

Under the SDP, HLI undertook to "subdivide and allocate for free and without charge among the qualified
family-beneficiaries x x x residential or homelots of not more than 240 sq. m. each, with each family On top of the minimum ratio provided under Section 3 of this Implementing Guideline, the corporate
beneficiary being assured of receiving and owning a homelot in the barrio or barangay where it actually landowner-applicant may adopt additional stock distribution schemes taking into account factors such as rank,
resides," "within a reasonable time." seniority, salary, position and other circumstances which may be deemed desirable as a matter of sound
company policy.

More than sixteen (16) years have elapsed from the time the SDP was approved by PARC, and yet, it is still the
contention of the FWBs that not all was given the 240-square meter homelots and, of those who were already The above proviso gives two (2) sets or categories of shares of stock which a qualified beneficiary can acquire
given, some still do not have the corresponding titles. from the corporation under the SDP. The first pertains, as earlier explained, to the mandatory minimum ratio of
shares of stock to be distributed to the FWBs in compliance with Sec. 31 of RA 6657. This minimum ratio
contemplates of that "proportion of the capital stock of the corporation that the agricultural land, actually
During the oral arguments, HLI was afforded the chance to refute the foregoing allegation by submitting proof
devoted to agricultural activities, bears in relation to the companys total assets." It is this set of shares of stock
that the FWBs were already given the said homelots:
which, in line with Sec. 4 of DAO 10, is supposed to be allocated "for the distribution of an equal number of
shares of stock of the same class and value, with the same rights and features as all other shares, to each of
the qualified beneficiaries."
Set V Corporation Code * Doctrine of Peircing Corpo Veil Cases*Page 36 of 164

On the other hand, the second set or category of shares partakes of a gratuitous extra grant, meaning that this Justice Abad: Did those new workers give up any right that would have belong to them in 1989 when the land
set or category constitutes an augmentation share/s that the corporate landowner may give under an was supposed to have been placed under CARP?
additional stock distribution scheme, taking into account such variables as rank, seniority, salary, position and
like factors which the management, in the exercise of its sound discretion, may deem desirable. Atty. Dela Merced: If you are talking or referring (interrupted)

Before anything else, it should be stressed that, at the time PARC approved HLIs SDP, HLI Justice Abad: None! You tell me. None. They gave up no rights to land?
recognized 6,296individuals as qualified FWBs. And under the 30-year stock distribution program envisaged
under the plan, FWBs who came in after 1989, new FWBs in fine, may be accommodated, as they appear to
Atty. Dela Merced: They did not do the same thing as we did in 1989, Your Honor.
have in fact been accommodated as evidenced by their receipt of HLI shares.

Justice Abad: No, if they were not workers in 1989 what land did they give up? None, if they become workers
Now then, by providing that the number of shares of the original 1989 FWBs shall depend on the number of
later on.
"man days," HLI violated the afore-quoted rule on stock distribution and effectively deprived the FWBs of equal
shares of stock in the corporation, for, in net effect, these 6,296 qualified FWBs, who theoretically had given up
their rights to the land that could have been distributed to them, suffered a dilution of their due share Atty. Dela Merced: None, Your Honor, I was referring, Your Honor, to the original (interrupted)
entitlement. As has been observed during the oral arguments, HLI has chosen to use the shares earmarked for
farmworkers as reward system chips to water down the shares of the original 6,296 FWBs. Particularly: Justice Abad: So why is it that the rights of those who gave up their lands would be diluted, because the
company has chosen to use the shares as reward system for new workers who come in? It is not that the new
Justice Abad: If the SDOA did not take place, the other thing that would have happened is that there would be workers, in effect, become just workers of the corporation whose stockholders were already fixed. The TADECO
CARP? who has shares there about sixty six percent (66%) and the five thousand four hundred ninety eight (5,498)
farmers at the time of the SDOA? Explain to me. Why, why will you x x x what right or where did you get that
right to use this shares, to water down the shares of those who should have been benefited, and to use it as a
Atty. Dela Merced: Yes, Your Honor.
reward system decided by the company?

Justice Abad: Thats the only point I want to know x x x. Now, but they chose to enter SDOA instead of placing
From the above discourse, it is clear as day that the original 6,296 FWBs, who were qualified beneficiaries at
the land under CARP. And for that reason those who would have gotten their shares of the land actually gave
the time of the approval of the SDP, suffered from watering down of shares. As determined earlier, each
up their rights to this land in place of the shares of the stock, is that correct?
original FWB is entitled to 18,804.32 HLI shares. The original FWBs got less than the guaranteed 18,804.32 HLI
shares per beneficiary, because the acquisition and distribution of the HLI shares were based on "man days" or
Atty. Dela Merced: It would be that way, Your Honor. "number of days worked" by the FWB in a years time. As explained by HLI, a beneficiary needs to work for at
least 37 days in a fiscal year before he or she becomes entitled to HLI shares. If it falls below 37 days, the FWB,
Justice Abad: Right now, also the government, in a way, gave up its right to own the land because that way the unfortunately, does not get any share at year end. The number of HLI shares distributed varies depending on
government takes own [sic] the land and distribute it to the farmers and pay for the land, is that correct? the number of days the FWBs were allowed to work in one year. Worse, HLI hired farmworkers in addition to the
original 6,296 FWBs, such that, as indicated in the Compliance dated August 2, 2010 submitted by HLI to the
Atty. Dela Merced: Yes, Your Honor. Court, the total number of farmworkers of HLI as of said date stood at 10,502. All these farmworkers, which
include the original 6,296 FWBs, were given shares out of the 118,931,976.85 HLI shares representing the
33.296% of the total outstanding capital stock of HLI. Clearly, the minimum individual allocation of each
Justice Abad: And then you gave thirty-three percent (33%) of the shares of HLI to the farmers at that time that
original FWB of 18,804.32 shares was diluted as a result of the use of "man days" and the hiring of additional
numbered x x x those who signed five thousand four hundred ninety eight (5,498) beneficiaries, is that correct?
farmworkers.

Atty. Dela Merced: Yes, Your Honor.


Going into another but related matter, par. 3 of the SDOA expressly providing for a 30-year timeframe for HLI-
to-FWBs stock transfer is an arrangement contrary to what Sec. 11 of DAO 10 prescribes. Said Sec. 11 provides
Justice Abad: But later on, after assigning them their shares, some workers came in from 1989, 1990, 1991, for the implementation of the approved stock distribution plan within three (3) months from receipt by the
1992 and the rest of the years that you gave additional shares who were not in the original list of owners? corporate landowner of the approval of the plan by PARC. In fact, based on the said provision, the transfer of
the shares of stock in the names of the qualified FWBs should be recorded in the stock and transfer books and
Atty. Dela Merced: Yes, Your Honor. must be submitted to the SEC within sixty (60) days from implementation. As stated:
Set V Corporation Code * Doctrine of Peircing Corpo Veil Cases*Page 37 of 164

Section 11. Implementation/Monitoring of Plan.The approved stock distribution plan shall Resolution No. 2005-32-01 and PARC Resolution No. 2006-34-01, revoking the previous approval of the SDP by
be implemented within three (3) months from receipt by the corporate landowner-applicant of the approval PARC.
thereof by the PARC, and the transfer of the shares of stocks in the names of the qualified beneficiaries shall be
recorded in stock and transfer books and submitted to the Securities and Exchange Commission (SEC) within VII. Control over Agricultural Lands
sixty (60) days from the said implementation of the stock distribution plan.

After having discussed and considered the different contentions raised by the parties in their respective
It is evident from the foregoing provision that the implementation, that is, the distribution of the shares of stock motions, We are now left to contend with one crucial issue in the case at bar, that is, control over the
to the FWBs, must be made within three (3) months from receipt by HLI of the approval of the stock distribution agricultural lands by the qualified FWBs.
plan by PARC. While neither of the clashing parties has made a compelling case of the thrust of this provision,
the Court is of the view and so holds that the intent is to compel the corporate landowner to complete, not
Upon a review of the facts and circumstances, We realize that the FWBs will never have control over these
merely initiate, the transfer process of shares within that three-month timeframe. Reinforcing this conclusion is
agricultural lands for as long as they remain as stockholders of HLI. In Our July 5, 2011 Decision, this Court
the 60-day stock transfer recording (with the SEC) requirement reckoned from the implementation of the SDP.
made the following observations:

To the Court, there is a purpose, which is at once discernible as it is practical, for the three-month threshold.
There is, thus, nothing unconstitutional in the formula prescribed by RA 6657. The policy on agrarian reform is
Remove this timeline and the corporate landowner can veritably evade compliance with agrarian reform by
that control over the agricultural land must always be in the hands of the farmers. Then it falls on the shoulders
simply deferring to absurd limits the implementation of the stock distribution scheme.
of DAR and PARC to see to it the farmers should always own majority of the common shares entitled to elect
the members of the board of directors to ensure that the farmers will have a clear majority in the board. Before
The argument is urged that the thirty (30)-year distribution program is justified by the fact that, under Sec. 26 the SDP is approved, strict scrutiny of the proposed SDP must always be undertaken by the DAR and PARC,
of RA 6657, payment by beneficiaries of land distribution under CARP shall be made in thirty (30) annual such that the value of the agricultural land contributed to the corporation must always be more than 50% of
amortizations. To HLI, said section provides a justifying dimension to its 30-year stock distribution program. the total assets of the corporation to ensure that the majority of the members of the board of directors are
composed of the farmers. The PARC composed of the President of the Philippines and cabinet secretaries must
HLIs reliance on Sec. 26 of RA 6657, quoted in part below, is obviously misplaced as the said provision clearly see to it that control over the board of directors rests with the farmers by rejecting the inclusion of non-
deals with land distribution. agricultural assets which will yield the majority in the board of directors to non-farmers. Any deviation,
however, by PARC or DAR from the correct application of the formula prescribed by the second paragraph of
SEC. 26. Payment by Beneficiaries.Lands awarded pursuant to this Act shall be paid for by the beneficiaries to Sec. 31 of RA 6675 does not make said provision constitutionally infirm. Rather, it is the application of said
the LBP in thirty (30) annual amortizations x x x. provision that can be challenged. Ergo, Sec. 31 of RA 6657 does not trench on the constitutional policy of
ensuring control by the farmers. (Emphasis supplied.)

Then, too, the ones obliged to pay the LBP under the said provision are the beneficiaries. On the other hand, in
the instant case, aside from the fact that what is involved is stock distribution, it is the corporate landowner In line with Our finding that control over agricultural lands must always be in the hands of the farmers, We
who has the obligation to distribute the shares of stock among the FWBs. reconsider our ruling that the qualified FWBs should be given an option to remain as stockholders of HLI,
inasmuch as these qualified FWBs will never gain control given the present proportion of shareholdings in HLI.

Evidently, the land transfer beneficiaries are given thirty (30) years within which to pay the cost of the land
thus awarded them to make it less cumbersome for them to pay the government. To be sure, the reason A revisit of HLIs Proposal for Stock Distribution under CARP and the Stock Distribution Option Agreement
underpinning the 30-year accommodation does not apply to corporate landowners in distributing shares of (SDOA) upon which the proposal was based reveals that the total assets of HLI is PhP 590,554,220, while the
stock to the qualified beneficiaries, as the shares may be issued in a much shorter period of time. value of the 4,915.7466 hectares is PhP 196,630,000. Consequently, the share of the farmer-beneficiaries in
the HLI capital stock is 33.296% (196,630,000 divided by 590,554.220); 118,391,976.85 HLI shares represent
33.296%. Thus, even if all the holders of the 118,391,976.85 HLI shares unanimously vote to remain as HLI
Taking into account the above discussion, the revocation of the SDP by PARC should be upheld for violating DAO
stockholders, which is unlikely, control will never be placed in the hands of the farmer-beneficiaries. 1awp+
10. It bears stressing that under Sec. 49 of RA 6657, the PARC and the DAR have the power to issue rules and
+i1 Control, of course, means the majority of 50% plus at least one share of the common shares and other
regulations, substantive or procedural. Being a product of such rule-making power, DAO 10 has the force and
voting shares. Applying the formula to the HLI stockholdings, the number of shares that will constitute the
effect of law and must be duly complied with. The PARC is, therefore, correct in revoking the SDP.
majority is 295,112,101 shares (590,554,220 divided by 2 plus one [1] HLI share). The 118,391,976.85 shares
Consequently, the PARC Resolution No. 89-12-2 dated November 21, l989 approving the HLIs SDP is nullified
subject to the SDP approved by PARC substantially fall short of the 295,112,101 shares needed by the FWBs to
and voided. (Citations omitted; emphasis in the original.)
acquire control over HLI. Hence, control can NEVER be attained by the FWBs. There is even no assurance that
100% of the 118,391,976.85 shares issued to the FWBs will all be voted in favor of staying in HLI, taking into
Based on the foregoing ruling, the contentions of Mallari, et al. are either not supported by the evidence on account the previous referendum among the farmers where said shares were not voted unanimously in favor of
record or are utterly misplaced. There is, therefore, no basis for the Court to reverse its ruling affirming PARC
Set V Corporation Code * Doctrine of Peircing Corpo Veil Cases*Page 38 of 164

retaining the SDP. In light of the foregoing consideration, the option to remain in HLI granted to the individual DAR shall segregate from the HLI agricultural land with an area of 4,915.75 hectares subject of PARCs SDP-
FWBs will have to be recalled and revoked. approving Resolution No. 89-12-2 the 500-hectare lot subject of the August 14, l996 Conversion Order and the
80.51-hectare lot sold to, or acquired by, the government as part of the SCTEX complex. After the segregation
Moreover, bearing in mind that with the revocation of the approval of the SDP, HLI will no longer be operating process, as indicated, is done, the remaining area shall be turned over to DAR for immediate land distribution
under SDP and will only be treated as an ordinary private corporation; the FWBs who remain as stockholders of to the original 6,296 FWBs or their successors-in-interest which will be identified by the DAR. The 4,206 non-
HLI will be treated as ordinary stockholders and will no longer be under the protective mantle of RA 6657. qualified FWBs are not entitled to any share in the land to be distributed by DAR.1wphi1

In addition to the foregoing, in view of the operative fact doctrine, all the benefits and homelots 80 received by HLI is directed to pay the original 6,296 FWBs the consideration of PhP 500,000,000 received by it from Luisita
all the FWBs shall be respected with no obligation to refund or return them, since, as We have mentioned in our Realty, Inc. for the sale to the latter of 200 hectares out of the 500 hectares covered by the August 14, 1996
July 5, 2011 Decision, "the benefits x x x were received by the FWBs as farmhands in the agricultural enterprise Conversion Order, the consideration of PhP 750,000,000 received by its owned subsidiary, Centennary
of HLI and other fringe benefits were granted to them pursuant to the existing collective bargaining agreement Holdings, Inc., for the sale of the remaining 300 hectares of the aforementioned 500-hectare lot to Luisita
with Tadeco." Industrial Park Corporation, and the price of PhP 80,511,500 paid by the government through the Bases
Conversion Development Authority for the sale of the 80.51-hectare lot used for the construction of the SCTEX
road network. From the total amount of PhP 1,330,511,500 (PhP 500,000,000 + PhP 750,000,000 + PhP
One last point, the HLI land shall be distributed only to the 6,296 original FWBs. The remaining 4,206 FWBs are
80,511,500 = PhP 1,330,511,500) shall be deducted the 3% of the proceeds of said transfers that were paid to
not entitled to any portion of the HLI land, because the rights to said land were vested only in the 6,296 original
the FWBs, the taxes and expenses relating to the transfer of titles to the transferees, and the expenditures
FWBs pursuant to Sec. 22 of RA 6657.
incurred by HLI and Centennary Holdings, Inc. for legitimate corporate purposes. For this purpose, DAR is
ordered to engage the services of a reputable accounting firm approved by the parties to audit the books of HLI
In this regard, DAR shall verify the identities of the 6,296 original FWBs, consistent with its administrative and Centennary Holdings, Inc. to determine if the PhP 1,330,511,500 proceeds of the sale of the three (3)
prerogative to identify and select the agrarian reform beneficiaries under RA 6657. 81 aforementioned lots were actually used or spent for legitimate corporate purposes. Any unspent or unused
balance and any disallowed expenditures as determined by the audit shall be distributed to the 6,296 original
WHEREFORE, the Motion for Partial Reconsideration dated July 20, 2011 filed by public respondents Presidential FWBs.
Agrarian Reform Council and Department of Agrarian Reform, the Motion for Reconsideration dated July 19,
2011 filed by private respondent Alyansa ng mga Manggagawang Bukid sa Hacienda Luisita, the Motion for HLI is entitled to just compensation for the agricultural land that will be transferred to DAR to be reckoned from
Reconsideration dated July 21, 2011 filed by respondent-intervenor Farmworkers Agrarian Reform Movement, November 21, 1989 which is the date of issuance of PARC Resolution No. 89-12-2. DAR and LBP are ordered to
Inc., and the Motion for Reconsideration dated July 22, 2011 filed by private respondents Rene Galang and determine the compensation due to HLI.
AMBALA are PARTIALLY GRANTED with respect to the option granted to the original farmworker-beneficiaries of
Hacienda Luisita to remain with Hacienda Luisita, Inc., which is hereby RECALLED and SET ASIDE. The Motion
DAR shall submit a compliance report after six (6) months from finality of this judgment. It shall also submit,
for Clarification and Partial Reconsideration dated July 21, 2011 filed by petitioner HLI and the Motion for
after submission of the compliance report, quarterly reports on the execution of this judgment within the first
Reconsideration dated July 21, 2011 filed by private respondents Noel Mallari, Julio Suniga, Supervisory Group
15 days after the end of each quarter, until fully implemented.The temporary restraining order is lifted.SO
of Hacienda Luisita, Inc. and Windsor Andaya are DENIED.
ORDERED.

The fallo of the Courts July 5, 2011 Decision is hereby amended and shall read:
CASE Digest

PARC Resolution No. 2005-32-01 dated December 22, 2005 and Resolution No. 2006-34-01 dated May 3, 2006,
Facts:
placing the lands subject of HLIs SDP under compulsory coverage on mandated land acquisition scheme of the
CARP, are hereby AFFIRMED with the following modifications: The SC en banc voted 11-0 dismissing the petition filed by HLI Affirm with modifications the
resolutions of the Presidential Agrarian Reform Council (PARC for brevity) revoking Hacienda Luisita Inc. (HLI for
brevity) Stock Distribution Plan (SDP) and placing the subject land in HL under compulsory coverage of the
All salaries, benefits, the 3% of the gross sales of the production of the agricultural lands, the 3% share in the
CARP of the government.
proceeds of the sale of the 500-hectare converted land and the 80.51-hectare SCTEX lot and the homelots
already received by the 10,502 FWBs composed of 6,296 original FWBs and the 4,206 non-qualified FWBs shall Thereafter, the SC voting 6-5 averred that there are operative facts that occurred in the premises. The
be respected with no obligation to refund or return them. The 6,296 original FWBs shall forfeit and relinquish SC thereat declared that the revocation of the SDP shall, by application of the operative fact principle, give the
their rights over the HLI shares of stock issued to them in favor of HLI. The HLI Corporate Secretary shall cancel 5296 qualified Farmworkers Beneficiaries (FWBs for brevity) to choose whether they want to remain as HLI
the shares issued to the said FWBs and transfer them to HLI in the stocks and transfer book, which transfers stockholders or choose actual land distribution. Considering the premises, DAR immediately scheduled a
shall be exempt from taxes, fees and charges. The 4,206 non-qualified FWBs shall remain as stockholders of meeting regarding the effects of their choice and therefrom proceeded to secret voting of their choice.
HLI.
The parties, thereafter, filed their respective Motion for Reconsideration regarding the SCs decision.
Set V Corporation Code * Doctrine of Peircing Corpo Veil Cases*Page 39 of 164

CORPORATION (PNB-MADECOR), and MEGA PRIME REALTY AND HOLDINGS CORPORATION (MEGA
Issue: PRIME), Respondents.

1) Whether or not operative fact doctrine is applicable in the said case.


x - - - - - - - - - - - - - - - - - - - - - - -x
2) Whether or not Sec. 31 of R.A. 6657 unconstitutional.
3) Whether or not the 10-year period prohibition on the transfer of awarded lands under RA 6657 lapsed on May
10, 1999, since Hacienda Luisita were placed under CARP coverage through the SDOA scheme on May 11, G.R. No. 170705 March 17, 2009
1989, and thus the qualified FWBs should now be allowed to sell their land interests in Hacienda Luisita to third
parties, whether they have fully paid for the lands or not?
4) Whether or not qualified FWBs shall be entitled to the option of remaining as stockholder be reconsidered. PHILIPPINE NATIONAL BANK, Petitioner, vs.
PANTRANCO EMPLOYEES ASSOCIATION, INC. (PEA-PTGWO), PANTRANCO RETRENCHED EMPLOYEES
Ruling: ASSOCIATION (PANREA) AND PANTRANCO ASSOCIATION OF CONCERNED EMPLOYEES (PACE), ET AL.,
PHILIPPINE NATIONAL BANK-MANAGEMENT DEVELOPMENT CORPORATION (PNB-MADECOR), and
1) Operative Fact Doctrine is applicable to the instant case. The court ruled that the doctrine is not limited only to MEGA PRIME REALTY HOLDINGS, INC., Respondents.
invalid or unconstitutional law but also to decisions made by the president or the administrative agencies that
have the force and effect of laws, especially if the said decisions produced acts and consequences that must be
respected. That the implementation of PARC resolution approving SDP of HLI manifested such right and benefits DECISION
favorable to the FWBs;

2) The SC said that the constitutionality of Sec. 31 of R.A. 6657 is not the lis mota of the case and it was not raised NACHURA, J.:
at the earliest opportunity and did not rule on the constitutionality of the law;
Before us are two consolidated petitions assailing the Court of Appeals (CA) Decision 1 dated June 3, 2005 and
3) The SC ruled that it has not yet lapsed on May 10, 1999, and qualified FWBs are not allowed to sell their land
its Resolution2 dated December 7, 2005 in CA-G.R. SP No. 80599.
interest in HL to third parties; That the start of the counting of the prohibitive period shall be ten years from the
issuance and registration of the Emancipation Patent (EP for brevity) or Certificate of Land Ownership Award
(CLOA for brevity), and considering that the EPs and CLOAs have not yet been issued, the prohibitive period has In G.R. No. 170689, the Pantranco Employees Association (PEA) and Pantranco Retrenched Employees
not started yet. Association (PANREA) pray that the CA decision be set aside and a new one be entered, declaring the Philippine
National Bank (PNB) and PNB Management and Development Corporation (PNB-Madecor) jointly and solidarily
4) The SC ruled in the affirmative, giving qualified FWBs the option to remain as stockholder
liable for the P722,727,150.22 National Labor Relations Commission (NLRC) judgment in favor of the Pantranco
YES, the ruling in the July 5, 2011 Decision that the qualified FWBs be given an option to remain as North Express, Inc. (PNEI) employees;3 while in G.R. No. 170705, PNB prays that the auction sale of the
stockholders of HLI should be reconsidered. Pantranco properties be declared null and void.4

[The Court reconsidered its earlier decision that the qualified FWBs should be given an option to remain as
The facts of the case, as found by the CA, 5 and established in Republic of the Phils. v. NLRC, 6 Pantranco North
stockholders of HLI, inasmuch as these qualified FWBs will never gain control [over the subject lands] given the
present proportion of shareholdings in HLI. The Court noted that the share of the FWBs in the HLI capital stock Express, Inc. v. NLRC,7 and PNB MADECOR v. Uy,8 follow:
is [just] 33.296%. Thus, even if all the holders of this 33.296% unanimously vote to remain as HLI stockholders,
which is unlikely, control will never be in the hands of the FWBs. Control means the majority of [sic] 50% plus The Gonzales family owned two corporations, namely, the PNEI and Macris Realty Corporation (Macris). PNEI
at least one share of the common shares and other voting shares. Applying the formula to the HLI
provided transportation services to the public, and had its bus terminal at the corner of Quezon and Roosevelt
stockholdings, the number of shares that will constitute the majority is 295,112,101 shares (590,554,220 total
HLI capital shares divided by 2 plus one [1] HLI share). The 118,391,976.85 shares subject to the SDP Avenues in Quezon City. The terminal stood on four valuable pieces of real estate (known as Pantranco
approved by PARC substantially fall short of the 295,112,101 shares needed by the FWBs to acquire control properties) registered under the name of Macris. 9 The Gonzales family later incurred huge financial losses
over HLI.] despite attempts of rehabilitation and loan infusion. In March 1975, their creditors took over the management
of PNEI and Macris. By 1978, full ownership was transferred to one of their creditors, the National Investment
The SC PARTIALLY GRANTED the motions for reconsideration of respondents PARC, et al., The 6,296 original Development Corporation (NIDC), a subsidiary of the PNB.
FWBs shall forfeit and relinquish their rights over the HLI shares of stock issued to them in favor of HLI. The HLI
Corporate Secretary shall cancel the shares issued to the said FWBs and transfer them to HLI in the stocks and
transfer book. The 4,206 non-qualified FWBs shall remain as stockholders of HLI. Macris was later renamed as the National Realty Development Corporation (Naredeco) and eventually merged
THIRD DIVISION G.R. No. 170689 March 17, 2009 with the National Warehousing Corporation (Nawaco) to form the new PNB subsidiary, the PNB-Madecor.
PANTRANCO EMPLOYEES ASSOCIATION (PEA-PTGWO) and PANTRANCO RETRENCHED EMPLOYEES
ASSOCIATION (PANREA), Petitioners,
vs. In 1985, NIDC sold PNEI to North Express Transport, Inc. (NETI), a company owned by Gregorio Araneta III. In
NATIONAL LABOR RELATIONS COMMISSION (NLRC), PANTRANCO NORTH EXPRESS, INC. (PNEI), 1986, PNEI was among the several companies placed under sequestration by the Presidential Commission on
PHILIPPINE NATIONAL BANK (PNB), PHILIPPINE NATIONAL BANK-MANAGEMENT AND DEVELOPMENT Good Government (PCGG) shortly after the historic events in EDSA. In January 1988, PCGG lifted the
Set V Corporation Code * Doctrine of Peircing Corpo Veil Cases*Page 40 of 164

sequestration order to pave the way for the sale of PNEI back to the private sector through the Asset The Motion to Quash of PNB Madecor and Mega Prime Holdings, Inc. is hereby PARTIALLY GRANTED insofar as
Privatization Trust (APT). APT thus took over the management of PNEI. the amount of the writ exceeds P7,884,000.00.

In 1992, PNEI applied with the Securities and Exchange Commission (SEC) for suspension of payments. A The Motion for Recomputation and Examination of Judgment Awards is hereby DENIED for want of merit.
management committee was thereafter created which recommended to the SEC the sale of the company
through privatization. As a cost-saving measure, the committee likewise suggested the retrenchment of several The Motion to Expunge from the Records claimants/complainants Opposition dated August 3, 2002 is hereby
PNEI employees. Eventually, PNEI ceased its operation. Along with the cessation of business came the various DENIED for lack of merit.SO ORDERED.18
labor claims commenced by the former employees of PNEI where the latter obtained favorable decisions.

On appeal to the NLRC, the same was denied and the Labor Arbiters disposition was affirmed. 19 Specifically,
On July 5, 2002, the Labor Arbiter issued the Sixth Alias Writ of Execution 10 commanding the NLRC Sheriffs to the NLRC concluded as follows:
levy on the assets of PNEI in order to satisfy the P722,727,150.22 due its former employees, as full and final
satisfaction of the judgment awards in the labor cases. The sheriffs were likewise instructed to proceed against
(1) PNB-Madecor and Mega Prime contended that it would be impossible for them to comply with the
PNB, PNB-Madecor and Mega Prime.11 In implementing the writ, the sheriffs levied upon the four valuable
requirement of the labor arbiter to pay to the PNEI employees the amount of P7.8 million as a
pieces of real estate located at the corner of Quezon and Roosevelt Avenues, on which the former Pantranco
condition to the lifting of the levy on the properties, since the credit was already garnished by Gerardo
Bus Terminal stood. These properties were covered by Transfer Certificate of Title (TCT) Nos. 87881-87884,
Uy and other creditors of PNEI. The NLRC found no evidence that Uy had satisfied his judgment from
registered under the name of PNB-Madecor.12 Subsequently, Notice of Sale of the foregoing real properties was
the promissory note, and opined that even if the credit was in custodia legis, the claim of the PNEI
published in the newspaper and the sale was set on July 31, 2002. Having been notified of the auction sale,
employees should enjoy preference under the Labor Code.
motions to quash the writ were separately filed by PNB-Madecor and Mega Prime, and PNB. They likewise filed
their Third-Party Claims.13 PNB-Madecor anchored its motion on its right as the registered owner of the
Pantranco properties, and Mega Prime as the successor-in-interest. For its part, PNB sought the nullification of (2) The PNEI employees contested the finding that PNB-Madecor was indebted to the PNEI for
the writ on the ground that it was not a party to the labor case. 14 In its Third-Party Claim, PNB alleged that PNB- only P7.8 million without considering the accrual of interest. But the NLRC said that there was no
Madecor was indebted to the former and that the Pantranco properties evidence that demand was made as a basis for reckoning interest.

would answer for such debt. As such, the scheduled auction sale of the aforesaid properties was not legally in (3) The PNEI employees further argued that the labor arbiter may not properly conclude from a
order.15 decision of Judge Demetrio Macapagal Jr. of the RTC of Quezon City that PNB-Madecor was the owner
of the properties as his decision was reconsidered by the next presiding judge, nor from a decision of
the Supreme Court that PNEI was a mere lessee of the properties, the fact being that the transfer of
On September 10, 2002, the Labor Arbiter declared that the subject Pantranco properties were owned by PNB-
the properties to PNB-Madecor was done to avoid satisfaction of the claims of the employees with the
Madecor. It being a corporation with a distinct and separate personality, its assets could not answer for the
NLRC and that as a result of a civil case filed by Mega Prime, the subsequent sale of the properties by
liabilities of PNEI. Considering, however, that PNB-Madecor executed a promissory note in favor of PNEI
PNB to Mega Prime was rescinded. The NLRC pointed out that while the Macapagal decision was set
forP7,884,000.00, the writ of execution to the extent of the said amount was concerned was considered valid. 16
aside by Judge Bruselas and hence, his findings could not be invoked by the labor arbiter, the titles of
PNB-Madecor are conclusive and there is no evidence that PNEI had ever been an owner. The Supreme
PNBs third-party claim to nullify the writ on the ground that it has an interest in the Pantranco properties Court had observed in its decision that PNEI owed back rentals of P8.7 million to PNB-Madecor.
being a creditor of PNB-Madecor, on the other hand, was denied because it only had an inchoate interest in
the properties.17
(4) The PNEI employees faulted the labor arbiter for not finding that PNEI, PNB, PNB-Madecor and
Mega Prime were all jointly and severally liable for their claims. The NLRC underscored the fact that
The dispositive portion of the Labor Arbiters September 10, 2002 Resolution is quoted hereunder: PNEI and Macris were subsidiaries of NIDC and had passed through and were under the Asset
Privatization Trust (APT) when the labor claims accrued. The labor arbiter was correct in not granting
WHEREFORE, the Third Party Claim of PNB Madecor and/or Mega Prime Holdings, Inc. is hereby GRANTED and PNBs third-party claim because at the time the causes of action accrued, the PNEI was managed by a
concomitantly the levies made by the sheriffs of the NLRC on the properties of PNB Madecor should be as it management committee appointed by the PNB as the new owner of PNRI (sic) and Macris through a
(sic) is hereby LIFTED subject to the payment by PNB Madecor to the complainants the amount deed of assignment or transfer of ownership. The NLRC says at length that the same is not true with
of P7,884,000.00. PNB-Madecor which is now the registered owner of the properties. 20

The Motion to Quash and Third Party Claim of PNB is hereby DENIED. The parties separate motions for reconsideration were likewise denied. 21 Thereafter, the matter was elevated
to the CA by PANREA, PEA-PTGWO and the Pantranco Association of Concerned Employees. The latter group,
however, later withdrew its petition. The former employees petition was docketed as CA-G.R. SP No. 80599.
Set V Corporation Code * Doctrine of Peircing Corpo Veil Cases*Page 41 of 164

PNB-Madecor and Mega Prime likewise filed their separate petition before the CA which was docketed as CA- PROMISSORY NOTE IN FAVOR OF PNEI) IS NOT IN ORDER AS THE SAID PROPERTY IS NOT OWNED BY PNEI.
G.R. SP No. 80737, but the same was dismissed.22 FURTHER, THE SAID PROMISSORY NOTE HAD ALREADY BEEN GARNISHED IN FAVOR OF GERARDO C. UY WHICH
LED TO THREE (3) PROPERTIES UNDER THE NAME OF PNB-MADECOR, NAMELY TCT NOS. 87881, 87882 AND
In view of the P7,884,000.00 debt of PNB-Madecor to PNEI, on June 23, 2004, an auction sale was conducted 87883, BEING LEVIED AND SOLD ON EXECUTION IN THE "PNB-MADECOR VS. UY" CASE (363 SCRA 128 [2001])
over the Pantranco properties to satisfy the claim of the PNEI employees, wherein CPAR Realty was adjudged as AND "GERARDO C. UY VS. PNEI" (CIVIL CASE NO. 95-72685, RTC MANILA, BRANCH 38). 32
the highest bidder.23
PNB insists that the Pantranco properties could no longer be levied upon because the promissory note for which
On June 3, 2005, the CA rendered the assailed decision affirming the NLRC resolutions. the Labor Arbiter held PNB-Madecor liable to PNEI, and in turn to the latters former employees, had already
been satisfied in favor of Gerardo C. Uy. It added that the properties were in fact awarded to the highest bidder.
Besides, says PNB, the subject properties were not owned by PNEI, hence, the execution sale thereof was not
The appellate court pointed out that PNB, PNB-Madecor and Mega Prime are corporations with personalities
validly effected.33
separate and distinct from PNEI. As such, there being no cogent reason to pierce the veil of corporate fiction,
the separate personalities of the above corporations should be maintained. The CA added that the Pantranco
properties were never owned by PNEI; rather, their titles were registered under the name of PNB-Madecor. If Both petitions must fail.
PNB and PNB-Madecor could not answer for the liabilities of PNEI, with more reason should Mega Prime not be
held liable being a mere successor-in-interest of PNB-Madecor. G.R. No. 170689

Unsatisfied, PEA-PTGWO and PANREA filed their motion for reconsideration; 24 while PNB filed its Partial Motion Stripped of the non-essentials, the sole issue for resolution raised by the former PNEI employees is whether
for Reconsideration.25 PNB pointed out that PNB-Madecor was made to answer for P7,884,000.00 to the PNEI they can attach the properties (specifically the Pantranco properties) of PNB, PNB-Madecor and Mega Prime to
employees by virtue of the promissory note it (PNB-Madecor) earlier executed in favor of PNEI. PNB, however, satisfy their unpaid labor claims against PNEI.
questioned the June 23, 2004 auction sale as the P7.8 million debt had already been satisfied pursuant to this
Courts decision in PNB MADECOR v. Uy.26 We answer in the negative.

Both motions were denied by the appellate court.27 First, the subject property is not owned by the judgment debtor, that is, PNEI. Nowhere in the records was it
shown that PNEI owned the Pantranco properties. Petitioners, in fact, never alleged in any of their pleadings the
In two separate petitions, PNB and the former PNEI employees come up to this Court assailing the CA decision fact of such ownership. What was established, instead, in PNB MADECOR v. Uy 34 and PNB v. Mega Prime Realty
and resolution. The former PNEI employees raise the lone error, thus: and Holdings Corporation/Mega Prime Realty and Holdings Corporation v. PNB 35 was that the properties were
owned by Macris, the predecessor of PNB-Madecor. Hence, they cannot be pursued against by the creditors of
The Honorable Court of Appeals palpably departed from the established rules and jurisprudence in ruling that PNEI.
private respondents Pantranco North Express, Inc. (PNEI), Philippine National Bank (PNB), Philippine National
Bank Management and Development Corporation (PNB-MADECOR), Mega Prime Realty and Holdings, Inc. (Mega We would like to stress the settled rule that the power of the court in executing judgments extends only to
Prime) are not jointly and severally answerable to the P722,727,150.22 Million NLRC money judgment awards properties unquestionably belonging to the judgment debtor alone. 36 To be sure, one mans goods shall not be
in favor of the 4,000 individual members of the Petitioners.28 sold for another mans debts.37 A sheriff is not authorized to attach or levy on property not belonging to the
judgment debtor, and even incurs liability if he wrongfully levies upon the property of a third person. 38
They claim that PNB, through PNB-Madecor, directly benefited from the operation of PNEI and had complete
control over the funds of PNEI. Hence, they are solidarily answerable with PNEI for the unpaid money claims of Second, PNB, PNB-Madecor and Mega Prime are corporations with personalities separate and distinct from that
the employees.29 Citing A.C. Ransom Labor Union-CCLU v. NLRC,30 the employees insist that where the of PNEI. PNB is sought to be held liable because it acquired PNEI through NIDC at the time when PNEI was
employer corporation ceases to exist and is no longer able to satisfy the judgment awards in favor of its suffering financial reverses. PNB-Madecor is being made to answer for petitioners labor claims as the owner of
employees, the owner of the employer corporation should be made jointly and severally liable. 31 They added the subject Pantranco properties and as a subsidiary of PNB. Mega Prime is also included for having acquired
that malice or bad faith need not be proven to make the owners liable. PNBs shares over PNB-Madecor.

On the other hand, PNB anchors its petition on this sole assignment of error, viz.: The general rule is that a corporation has a personality separate and distinct from those of its stockholders and
other corporations to which it may be connected. 39 This is a fiction created by law for convenience and to
THE AUCTION SALE OF THE PROPERTY COVERED BY TCT NO. 87884 INTENDED TO PARTIALLY SATISFY THE prevent injustice.40 Obviously, PNB, PNB-Madecor, Mega Prime, and PNEI are corporations with their own
CLAIMS OF FORMER WORKERS OF PNEI IN THE AMOUNT OFP7,884,000.00 (THE AMOUNT OF PNB-MADECORS personalities. The "separate personalities" of the first three corporations had been recognized by this Court in
PNB v. Mega Prime Realty and Holdings Corporation/Mega Prime Realty and Holdings Corporation v.
Set V Corporation Code * Doctrine of Peircing Corpo Veil Cases*Page 42 of 164

PNB41 where we stated that PNB was only a stockholder of PNB-Madecor which later sold its shares to Mega except when acting as employer. It was clarified in Carag and McLeod that Article 212(e) of the Labor Code, by
Prime; and that PNB-Madecor was the owner of the Pantranco properties. Moreover, these corporations are itself, does not make a corporate officer personally liable for the debts of the corporation. It added that the
registered as separate entities and, absent any valid reason, we maintain their separate identities and we governing law on personal liability of directors or officers for debts of the corporation is still Section 31 52 of the
cannot treat them as one. Corporation Code.

Neither can we merge the personality of PNEI with PNB simply because the latter acquired the former. Settled is More importantly, as aptly observed by this Court in AC Ransom, it appears that Ransom, foreseeing the
the rule that where one corporation sells or otherwise transfers all its assets to another corporation for value, possibility or probability of payment of backwages to its employees, organized Rosario to replace Ransom, with
the latter is not, by that fact alone, liable for the debts and liabilities of the transferor. 42 the latter to be eventually phased out if the strikers win their case. The execution could not be implemented
against Ransom because of the disposition posthaste of its leviable assets evidently in order to evade its just
Lastly, while we recognize that there are peculiar circumstances or valid grounds that may exist to warrant the and due obligations.53 Hence, the Court sustained the piercing of the corporate veil and made the officers of
piercing of the corporate veil, 43 none applies in the present case whether between PNB and PNEI; or PNB and Ransom personally liable for the debts of the latter.
PNB-Madecor.
Clearly, what can be inferred from the earlier cases is that the doctrine of piercing the corporate veil applies
Under the doctrine of "piercing the veil of corporate fiction," the court looks at the corporation as a mere only in three (3) basic areas, namely: 1) defeat of public convenience as when the corporate fiction is used as a
collection of individuals or an aggregation of persons undertaking business as a group, disregarding the vehicle for the evasion of an existing obligation; 2) fraud cases or when the corporate entity is used to justify a
separate juridical personality of the corporation unifying the group. 44 Another formulation of this doctrine is that wrong, protect fraud, or defend a crime; or 3) alter ego cases, where a corporation is merely a farce since it is a
when two business enterprises are owned, conducted and controlled by the same parties, both law and equity mere alter ego or business conduit of a person, or where the corporation is so organized and controlled and its
will, when necessary to protect the rights of third parties, disregard the legal fiction that two corporations are affairs are so conducted as to make it merely an instrumentality, agency, conduit or adjunct of another
distinct entities and treat them as identical or as one and the same. 45 corporation.54 In the absence of malice, bad faith, or a specific provision of law making a corporate officer
liable, such corporate officer cannot be made personally liable for corporate liabilities. 55

Whether the separate personality of the corporation should be pierced hinges on obtaining facts appropriately
pleaded or proved. However, any piercing of the corporate veil has to be done with caution, albeit the Court will Applying the foregoing doctrine to the instant case, we quote with approval the CA disposition in this wise:
not hesitate to disregard the corporate veil when it is misused or when necessary in the interest of justice. After
all, the concept of corporate entity was not meant to promote unfair objectives. 46 It would not be enough, then, for the petitioners in this case, the PNEI employees, to rest on their laurels with
evidence that PNB was the owner of PNEI. Apart from proving ownership, it is necessary to show facts that will
As between PNB and PNEI, petitioners want us to disregard their separate personalities, and insist that because justify us to pierce the veil of corporate fiction and hold PNB liable for the debts of PNEI. The burden
the company, PNEI, has already ceased operations and there is no other way by which the judgment in favor of undoubtedly falls on the petitioners to prove their affirmative allegations. In line with the basic jurisprudential
the employees can be satisfied, corporate officers can be held jointly and severally liable with the company. principles we have explored, they must show that PNB was using PNEI as a mere adjunct or instrumentality or
Petitioners rely on the pronouncement of this Court in A.C. Ransom Labor Union-CCLU v. NLRC 47 and subsequent has exploited or misused the corporate privilege of PNEI.
cases.48
We do not see how the burden has been met. Lacking proof of a nexus apart from mere ownership, the
This reliance fails to persuade. We find the aforesaid decisions inapplicable to the instant case. petitioners have not provided us with the legal basis to reach the assets of corporations separate and distinct
from PNEI.56

For one, in the said cases, the persons made liable after the companys cessation of operations were the
officers and agents of the corporation. The rationale is that, since the corporation is an artificial person, it must Assuming, for the sake of argument, that PNB may be held liable for the debts of PNEI, petitioners still cannot
have an officer who can be presumed to be the employer, being the person acting in the interest of the proceed against the Pantranco properties, the same being owned by PNB-Madecor, notwithstanding the fact
employer. The corporation, only in the technical sense, is the employer. 49 In the instant case, what is being that PNB-Madecor was a subsidiary of PNB. The general rule remains that PNB-Madecor has a personality
made liable is another corporation (PNB) which acquired the debtor corporation (PNEI). separate and distinct from PNB. The mere fact that a corporation owns all of the stocks of another corporation,
taken alone, is not sufficient to justify their being treated as one entity. If used to perform legitimate functions,
a subsidiarys separate existence shall be respected, and the liability of the parent corporation as well as the
Moreover, in the recent cases Carag v. National Labor Relations Commission 50 and McLeod v. National Labor
subsidiary will be confined to those arising in their respective businesses. 57
Relations Commission,51 the Court explained the doctrine laid down in AC Ransom relative to the personal
liability of the officers and agents of the employer for the debts of the latter. In AC Ransom, the Court imputed
liability to the officers of the corporation on the strength of the definition of an employer in Article 212(c) (now In PNB v. Ritratto Group, Inc.,58 we outlined the circumstances which are useful in the determination of whether
Article 212[e]) of the Labor Code. Under the said provision, employer includes any person acting in the interest a subsidiary is but a mere instrumentality of the parent-corporation, to wit:
of an employer, directly or indirectly, but does not include any labor organization or any of its officers or agents
Set V Corporation Code * Doctrine of Peircing Corpo Veil Cases*Page 43 of 164

1. The parent corporation owns all or most of the capital stock of the subsidiary; It has been repeatedly stated that the Pantranco properties which were the subject of execution sale were
owned by Macris and later, the PNB-Madecor. They were never owned by PNEI or PNB. Following our earlier
2. The parent and subsidiary corporations have common directors or officers; discussion on the separate personalities of the different corporations involved in the instant case, the only
entity which has the right and interest to question the execution sale and the eventual right to annul the same,
if any, is PNB-Madecor or its successor-in-interest. Settled is the rule that proceedings in court must be
3. The parent corporation finances the subsidiary;
instituted by the real party in interest.

4. The parent corporation subscribes to all the capital stock of the subsidiary or otherwise causes its
A real party in interest is the party who stands to be benefited or injured by the judgment in the suit, or the
incorporation;
party entitled to the avails of the suit. 59 "Interest" within the meaning of the rule means material interest, an
interest in issue and to be affected by the decree, as distinguished from mere interest in the question involved,
5. The subsidiary has grossly inadequate capital; or a mere incidental interest. 60 The interest of the party must also be personal and not one based on a desire to
vindicate the constitutional right of some third and unrelated party. 61 Real interest, on the other hand, means a
6. The parent corporation pays the salaries and other expenses or losses of the subsidiary; present substantial interest, as distinguished from a mere expectancy or a future, contingent, subordinate, or
consequential interest.62

7. The subsidiary has substantially no business except with the parent corporation or no assets except
those conveyed to or by the parent corporation; Specifically, in proceedings to set aside an execution sale, the real party in interest is the person who has an
interest either in the property sold or the proceeds thereof. Conversely, one who is not interested or is not
injured by the execution sale cannot question its validity. 63
8. In the papers of the parent corporation or in the statements of its officers, the subsidiary is
described as a department or division of the parent corporation, or its business or financial
responsibility is referred to as the parent corporations own; In justifying its claim against the Pantranco properties, PNB alleges that Mega Prime, the buyer of its entire
stockholdings in PNB-Madecor was indebted to it (PNB). Considering that said indebtedness remains unpaid,
PNB insists that it has an interest over PNB-Madecor and Mega Primes assets.
9. The parent corporation uses the property of the subsidiary as its own;

Again, the contention is bereft of merit. While PNB has an apparent interest in Mega Primes assets being the
10. The directors or executives of the subsidiary do not act independently in the interest of the
creditor of the latter for a substantial amount, its interest remains inchoate and has not yet ripened into a
subsidiary, but take their orders from the parent corporation;
present substantial interest, which would give it the standing to maintain an action involving the subject
properties. As aptly observed by the Labor Arbiter, PNB only has an inchoate right to the properties of Mega
11. The formal legal requirements of the subsidiary are not observed. Prime in case the latter would not be able to pay its indebtedness. This is especially true in the instant case, as
the debt being claimed by PNB is secured by the accessory contract of pledge of the entire stockholdings of
None of the foregoing circumstances is present in the instant case. Thus, piercing of PNB-Madecors corporate Mega Prime to PNB-Madecor.64
veil is not warranted. Being a mere successor-in-interest of PNB-Madecor, with more reason should no liability
attach to Mega Prime. The Court further notes that the Pantranco properties (or a portion thereof ) were sold on execution to satisfy
the unpaid obligation of PNB-Madecor to PNEI. PNB-Madecor was thus made liable to the former PNEI
employees as the judgment debtor of PNEI. It has long been established in PNB-Madecor v. Uy and other similar
cases that PNB-Madecor had an unpaid obligation to PNEI amounting to more or less P7 million which could be
validly pursued by the creditors of the latter. Again, this strengthens the proper parties right to question the
G.R. No. 170705
validity of the execution sale, definitely not PNB.

Besides, the issue of whether PNB has a substantial interest over the Pantranco properties has already been
laid to rest by the Labor Arbiter. 65 It is noteworthy that in its Resolution dated September 10, 2002, the Labor
In its petition before this Court, PNB seeks the annulment of the June 23, 2004 execution sale of the Pantranco Arbiter denied PNBs Third-Party Claim primarily because PNB only has an inchoate right over the Pantranco
properties on the ground that the judgment debtor (PNEI) never owned said lots. It likewise contends that the properties.66Such conclusion was later affirmed by the NLRC in its Resolution dated June 30,
levy and the eventual sale on execution of the subject properties was null and void as the promissory note on 2003.67 Notwithstanding said conclusion, PNB did not elevate the matter to the CA via a petition for review.
which PNB-Madecor was made liable had already been satisfied. Hence it is presumed to be satisfied with the adjudication therein. 68 That decision of the NLRC has become final
as against PNB and can no longer be reviewed, much less reversed, by this Court. 69 This is in accord with the
doctrine that a party who has not appealed cannot obtain from the appellate court any affirmative relief other
Set V Corporation Code * Doctrine of Peircing Corpo Veil Cases*Page 44 of 164

than the ones granted in the appealed decision.70WHEREFORE, premises considered, the petitions are hereby On April 26, 2000, the CTA rendered a Decision dismissing the petition for review for lack of merit. 5
DENIED for lack of merit.SO ORDERED.
The CTA sustained petitioners contention that pursuant to Sec. 4 of RA 7432, the 20% sales discounts
SECOND DIVISION G.R. No. 151413 February 13, 2008 petitioner extended to qualified senior citizens in 1995 should be treated as tax credit and not as deductions
from the gross sales as erroneously interpreted in RR 2-94. The CTA reiterated its consistent holdings that RR 2-
CAGAYAN VALLEY DRUG CORPORATION, petitioner, vs. COMMISSIONER OF INTERNAL 94 is an invalid administrative interpretation of the law it purports to implement as it contravenes and does not
REVENUE, respondent. conform to the standards RA 7432 prescribes.

DECISION Notwithstanding petitioners entitlement to a tax credit from the 20% sales discounts it extended to qualified
senior citizens in 1995, the CTA nonetheless dismissed petitioners action for refund or tax credit on account of
petitioners net loss in 1995. First, the CTA rejected the refund as it is clear that RA 7432 only grants the 20%
VELASCO, JR., J.:
sales discounts extended to qualified senior citizens as tax credit and not as tax refund. Second, in rejecting
the tax credit, the CTA reasoned that while petitioner may be qualified for a tax credit, it cannot be so extended
The Case to petitioner on account of its net loss in 1995.

This petition for review under Rule 45 of the Rules of Court seeks the recall of the August 31, 2000 The CTA ratiocinated that on matters of tax credit claim, the government applies the amount determined to be
Resolution1 of the Court of Appeals (CA) in CA-G.R. SP No. 59778, which dismissed petitioner Cagayan Valley reimbursable after proper verification against any sum that may be due and collectible from the taxpayer.
Drug Corporations petition for review of the April 26, 2000 Decision 2 of the Court of Tax Appeals (CTA) in C.T.A. However, if no tax has been paid or if no amount is due and collectible from the taxpayer, then a tax credit is
Case No. 5581 on the ground of defective verification and certification against forum shopping. unavailing. Moreover, it held that before allowing recovery for claims for a refund or tax credit, it must first be
established that there was an actual collection and receipt by the government of the tax sought to be
The Facts recovered. In the instant case, the CTA found that petitioner did not pay any tax by virtue of its net loss position
in 1995.

Petitioner, a corporation duly organized and existing under Philippine laws, is a duly licensed retailer of
medicine and other pharmaceutical products. It operates two drugstores, one in Tuguegarao, Cagayan, and the Petitioners Motion for Reconsideration was likewise denied through the appellate tax courts June 30, 2000
other in Roxas, Isabela, under the name and style of "Mercury Drug." Resolution.6

Petitioner alleged that in 1995, it granted 20% sales discounts to qualified senior citizens on purchases of The Ruling of the Court of Appeals
medicine pursuant to Republic Act No. (RA) 74323 and its implementing rules and regulations.
Aggrieved, petitioner elevated the matter before the CA, docketed as CA-G.R. SP No. 59778. On August 31,
In compliance with Revenue Regulation No. (RR) 2-94, petitioner treated the 20% sales discounts granted to 2000, the CA issued the assailed Resolution7 dismissing the petition on procedural grounds. The CA held that
qualified senior citizens in 1995 as deductions from the gross sales in order to arrive at the net sales, instead of the person who signed the verification and certification of absence of forum shopping, a certain Jacinto J.
treating them as tax credit as provided by Section 4 of RA 7432. Concepcion, President of petitioner, failed to adduce proof that he was duly authorized by the board of directors
to do so.

On December 27, 1996, however, petitioner filed with the Bureau of Internal Revenue (BIR) a claim for tax
refund/tax credit of the full amount of the 20% sales discount it granted to senior citizens for the year 1995, As far as the CA was concerned, the main issue was whether or not the verification and certification of non-
allegedly totaling to PhP 123,083 in accordance with Sec. 4 of RA 7432. forum shopping signed by the President of petitioner is sufficient compliance with Secs. 4 and 5, Rule 7 of the
1997 Rules of Civil Procedure.

The BIRs inaction on petitioners claim for refund/tax credit compelled petitioner to file on March 18, 1998 a
petition for review before the CTA docketed as C.T.A. Case No. 5581 in order to forestall the two-year The verification and certification in question reads:
prescriptive period provided under Sec. 230 4 of the 1977 Tax Code, as amended. Thereafter, on March 31,
2000, petitioner amended its petition for review. I, JACINTO J. CONCEPCION, of legal age with office address at 2 nd Floor, Mercury Drug Corporation, No.
7 Mercury Ave, Bagumbayan, Quezon City, under oath, hereby state that:
The Ruling of the Court of Tax Appeals
Set V Corporation Code * Doctrine of Peircing Corpo Veil Cases*Page 45 of 164

1. I am the President of Cagayan Valley Drug Corporation, Petitioner in the above-entitled case and am solely exercise any corporate power pertaining to the corporation without authority from the board of directors.
duly authorized to sign this Verification and Certification of Absence of Forum Shopping by the Board This has been our constant holding in cases instituted by a corporation.
of Director.
In a slew of cases, however, we have recognized the authority of some corporate officers to sign the
xxxx verification and certification against forum shopping. In Mactan-Cebu International Airport Authority v. CA, we
recognized the authority of a general manager or acting general manager to sign the verification and
The CA found no sufficient proof to show that Concepcion was duly authorized by the Board of Directors of certificate against forum shopping;9 in Pfizer v. Galan, we upheld the validity of a verification signed by an
petitioner. The appellate court anchored its disposition on our ruling in Premium Marble Resources, Inc. v. Court "employment specialist" who had not even presented any proof of her authority to represent the
of Appeals (Premium), that "[i]n the absence of an authority from the Board of Directors, no person, not even company;10 in Novelty Philippines, Inc., v. CA, we ruled that a personnel officer who signed the petition but did
the officers of the corporation, can validly bind the corporation." 8 not attach the authority from the company is authorized to sign the verification and non-forum shopping
certificate;11 and in Lepanto Consolidated Mining Company v. WMC Resources International Pty. Ltd. (Lepanto),
we ruled that the Chairperson of the Board and President of the Company can sign the verification and
Hence, we have this petition.
certificate against non-forum shopping even without the submission of the boards authorization. 12

The Issues
In sum, we have held that the following officials or employees of the company can sign the verification and
certification without need of a board resolution: (1) the Chairperson of the Board of Directors, (2) the President
Petitioner raises two issues: first, whether petitioners president can sign the subject verification and of a corporation, (3) the General Manager or Acting General Manager, (4) Personnel Officer, and (5) an
certification sans the approval of its Board of Directors. And second, whether the CTA committed reversible Employment Specialist in a labor case.
error in denying and dismissing petitioners action for refund or tax credit in C.T.A. Case No. 5581.
While the above cases do not provide a complete listing of authorized signatories to the verification and
The Courts Ruling certification required by the rules, the determination of the sufficiency of the authority was done on a case to
case basis. The rationale applied in the foregoing cases is to justify the authority of corporate officers or
The petition is meritorious. representatives of the corporation to sign the verification or certificate against forum shopping, being "in a
position to verify the truthfulness and correctness of the allegations in the petition." 13

Premium not applicable


Authority from board of directors required

As regards the first issue, we find the CA to have erroneously relied on Premium. In said case, the issue tackled
was not on whether the president of Premium Marble Resources, Inc. was authorized to sign the verification and In Philippine Airlines v. Flight Attendants and Stewards Association of the Philippines , we ruled that only
certification against forum shopping, but rather on which of the two sets of officers, both claiming to be the individuals vested with authority by a valid board resolution may sign the certificate of non-forum shopping on
legal board of directors of Premium, have the authority to file the suit for and in behalf of the company. The behalf of a corporation. The action can be dismissed if the certification was submitted unaccompanied by proof
factual antecedents and issues in Premium are not on all fours with the instant case and is, therefore, not of the signatorys authority.14 We believe that appending the board resolution to the complaint or petition is the
applicable. better procedure to obviate any question on the authority of the signatory to the verification and certification.
The required submission of the board resolution is grounded on the basic precept that corporate powers are
exercised by the board of directors,15 and not solely by an officer of the corporation. Hence, the power to sue
With respect to an individual litigant, there is no question that litigants must sign the sworn verification and
and be sued in any court or quasi-judicial tribunal is necessarily lodged with the said board.
certification unless they execute a power of attorney authorizing another person to sign it. With respect to a
juridical person, Sec. 4, Rule 7 on verification and Sec. 5, Rule 7 on certification against forum shopping are
silent as to who the authorized signatory should be. Said rules do not indicate if the submission of a board There is substantial compliance with Rule 7, Secs. 4 and 5
resolution authorizing the officer or representative is necessary.
In the case at bar, we so hold that petitioner substantially complied with Secs. 4 and 5, Rule 7 of the 1997
Corporate powers exercised through board of directors Revised Rules on Civil Procedure. First, the requisite board resolution has been submitted albeit belatedly by
petitioner. Second, we apply our ruling in Lepanto with the rationale that the President of petitioner is in a
position to verify the truthfulness and correctness of the allegations in the petition. Third, the President of
It must be borne in mind that Sec. 23, in relation to Sec. 25 of the Corporation Code, clearly enunciates that all
petitioner has signed the complaint before the CTA at the inception of this judicial claim for refund or tax credit.
corporate powers are exercised, all business conducted, and all properties controlled by the board of directors.
A corporation has a separate and distinct personality from its directors and officers and can only exercise its
corporate powers through the board of directors. Thus, it is clear that an individual corporate officer cannot
Set V Corporation Code * Doctrine of Peircing Corpo Veil Cases*Page 46 of 164

Consequently, the petition in CA-G.R. SP No. 59778 ought to be reinstated. However, in view of the enactment Although this tax credit benefit is available, it need not be used by losing ventures, since there is no
of RA 9282 which made the decisions of the CTA appealable to this Court, we will directly resolve the second tax liability that calls for its application. Neither can it be reduced to nil by the quick yet callow stroke
issue which is a purely legal one. of an administrative pen, simply because no reduction of taxes can instantly be effected. By its nature,
the tax credit may still be deducted from a future, not a present, tax liability, without which it does not
Petitioner entitled to tax credit have any use. x x x

The pith of the dispute between petitioner and respondent is whether petitioner is entitled to a tax refund or xxxx
tax credit of 20% sales discount granted to senior citizens under RA 7432 or whether the discount should be
treated as a deduction from gross income. While a tax liability is essential to the availment or use of any tax credit, prior tax payments are not.
On the contrary, for the existence or grant solely of such credit, neither a tax liability nor a prior tax
This issue is not new, as the Court has resolved several cases involving the very same issue. In Commissioner payment is needed. The Tax Code is in fact replete with provisions granting or allowing tax credits,
of Internal Revenue v. Central Luzon Drug Corporation (Central Luzon),16 we held that private drug companies even though no taxes have been previously paid.19
are entitled to a tax credit for the 20% sales discounts they granted to qualified senior citizens under RA 7432
and nullified Secs. 2.i and 4 of RR 2-94. In Bicolandia Drug Corporation (formerly Elmas Drug Corporation) v. It is thus clear that petitioner is entitled to a tax credit for the full 20% sales discounts it extended to qualified
Commissioner of Internal Revenue,17 we ruled that petitioner therein is entitled to a tax credit of the "cost" or senior citizens for taxable year 1995. Considering that the CTA has not disallowed the PhP 123,083 sales
the full 20% sales discounts it granted pursuant to RA 7432. In the related case of Commissioner of Internal discounts petitioner claimed before the BIR and CTA, we are constrained to grant them as tax credit in favor of
Revenue v. Bicolandia Drug Corporation,18 we likewise ruled that respondent drug company was entitled to a petitioner.
tax credit, and we struck down RR 2-94 to be null and void for failing to conform with the law it sought to
implement. Consequently, petitioners appeal before the CA in CA-G.R. SP No. 59778 must be granted, and, necessarily, the
April 26, 2000 CTA Decision in C.T.A. Case No. 5581 reversed and set aside.
A perusal of the April 26, 2000 CTA Decision shows that the appellate tax court correctly ruled that the 20%
sales discounts petitioner granted to qualified senior citizens should be deducted from petitioners income tax WHEREFORE, the petition is GRANTED. The August 31, 2000 CA Resolution in CA-G.R. SP No. 59778
due and not from petitioners gross sales as erroneously provided in RR 2-94. However, the CTA erred in isANNULLED AND SET ASIDE. The April 26, 2000 CTA Decision in C.T.A. Case No. 5581 dismissing petitioners
denying the tax credit to petitioner on the ground that petitioner had suffered net loss in 1995, and ruling that claim for tax credit is accordingly REVERSED AND SET ASIDE. The Commissioner of Internal Revenue
the tax credit is unavailing. isORDERED to issue a Tax Credit Certificate in the name of petitioner in the amount of PhP 123,083. No
costs.SO ORDERED
Net loss in a taxable year does not preclude grant of tax credit
THIRD DIVISION G.R. No. 155056-57 October 19, 2007
It is true that petitioner did not pay any tax in 1995 since it suffered a net loss for that taxable year. This fact,
however, without more, does not preclude petitioner from availing of its statutory right to a tax credit for the THE HEIRS OF THE LATE PANFILO V. PAJARILLO, Petitioners, vs.
20% sales discounts it granted to qualified senior citizens. The law then applicable on this point is clear and THE HON. COURT OF APPEALS, NATIONAL LABOR RELATIONS COMMISSION and SAMAHAN NG MGA
without any qualification. Sec. 4 (a) of RA 7432 pertinently provides: MANGGAGAWA NG PANFILO V. PAJARILLO, ALFREDO HOYOHOY, HERMINIO CASTILLO, BERNARDO
ROCO, RODOLFO TORRES, JULIAN JORVINA, LOURDES ROCO, FLORITA YAPOC, MARLON ALDANA,
Sec. 4. Privileges for the Senior citizens.The senior citizens shall be entitled to the following: PARALUMAN ULANG, TOLENTINO SANHI, JOHNNY SORIANO, ANDRES CALAQUE, ROBERTO LAVAREZ,
FRANCISCO MORALES, SALVACION PERINA, ANTONIO ABALA, ROMEO SALONGA, AUGUR M.
a) the grant of twenty percent (20%) discount from all establishments relative to utilization of MANIPOL, BIENVENIDA TEQUIL, MARIO ELEP, ALADINO LATIGO, BERNARDINE BANSAL, PEDRO DE
transportation services, hotels and similar lodging establishments, restaurants and recreation centers BAGUIO, RICARDO CALICA, LAURA CO, VICENTE RECANA, ELENA TOLLEDO, ALFREDO PLAZA, SR.,
and purchase of medicines anywhere in the country: Provided, That private establishments may claim HERMINIO BALDONO, FELIPE YAPOC, ARISTON NIPA, and ALFONSO C. BALDOMAR, Respondents.
the cost as tax credit. (Emphasis ours.)
DECISION
The fact that petitioner suffered a net loss in 1995 will not make the tax credit due to petitioner unavailable.
This is the core issue resolved in Central Luzon, where we ruled that the net loss for a taxable year does not bar CHICO-NAZARIO, J.:
the grant of the tax credit to a taxpayer pursuant to RA 7432 and that prior tax payments are not required for
such grant. We explained:
Set V Corporation Code * Doctrine of Peircing Corpo Veil Cases*Page 47 of 164

In this Petition for Review on Certiorari under Rule 45 of the Rules of Court, 1 petitioners, heirs of Panfilo V. and PANFILO V. PAJARILLO, as its General Manager/Operator." This was docketed as NLRC Case No. 00-01-
Pajarillo, seek to set aside the Decision, 2 and Resolution,3 dated 12 March 2002 and 28 August 2002, 00331-88.11
respectively, of the Court of Appeals in CA-G.R. SP No. 54330 and CA-G.R. SP No. 54331, reversing the two Per
Curiam Orders dated 28 October 1996 and 10 January 1997, 4 of the National Labor Relations Commission Notifications and summons with respect to NLRC/NCR Case No. 00-08-03013-87 were addressed and sent to
(NLRC) in NLRC NCR Cases No. 08-03013-87 and 01-00331-88. "PANFILO V. PAJARILLO, President/Manager, Panfilo V. Pajarillo Liner, Pasig Line St., Sta. Ana, Manila" on 31
August 1987. The Registry Return Receipt dated 4 September 1987 was addressed to Panfilo V. Pajarillo, and a
Stripped of the non-essentials, the facts are as follows: signature therein appears on top of the signature of the name of the addressee. 12 With regard to NLRC Case No.
00-01-00331-88, notifications and summonses were addressed and sent to "THE PRESIDENT/MANAGER, PVP
Panfilo V. Pajarillo (Panfilo) was the owner and operator of several buses plying certain routes in Metro Manila. Liner Inc. and Panfilo V. Pajarillo, 2175 Zamora Street, Sta. Ana, Manila" on 25 January 1988. The Registry
He used the name "PVP Liner" in his buses. Private respondents were employed as drivers, conductors and Return Receipt dated 4 February 1988 was addressed to "PVP Liner Inc." and was signed by a certain "Irene G.
conductresses by Panfilo. Pajarillo" as the addressees agent.13

During their employment with Panfilo, private respondents worked at least four times a week or for an average Panfilo denied the charges in the complaints. He maintained that private respondents were not dismissed from
of fifteen working days per month. They were required to observe a work schedule starting from 4:00 in the work on account of their union activities; that private respondents and several of their co-employees either
morning up to 10:00 in the evening on a straight time basis. Private respondent drivers were paid a daily resigned or were separated from work, or simply abandoned their employment long before the respondent
commission of 10%, while private respondent conductors and conductresses received a daily commission of union was organized and registered with the DOLE; that the private respondents are not entitled to ECOLA and
7%. In sum, each of the private respondents earned an average daily commission of about P150.00 a day. They 13th month pay because they received wages above the minimum provided by law; that the private
were not given emergency cost of living allowance (ECOLA), 13th month pay, legal holiday pay and service respondents are not entitled to overtime and legal holiday pay because these are already included in their daily
incentive leave pay.5 commissions; that the private respondents are not entitled to five days incentive leave pay because they work
only four days a week; that no deductions were made in the daily commissions of the private respondents; that
the private respondents voluntarily and directly paid certain individuals for barangay protection and for the
The following were deducted from the private respondents daily commissions: (a) costs of washing the
cleaning of the assigned buses; that he had no participation in these activities/arrangements; that the private
assigned buses; (b) terminal fees; (c) fees for sweeping the assigned buses; (d) fees paid to the barangay
respondents were not dismissed from work; and that the private respondents either abandoned their jobs or
tanod at bus terminals; and (e) rental fees for the use of stereo in the assigned buses. Any employee who
voluntarily resigned from work.14
refused such deductions were either barred from working or dismissed from work. 6

Upon motion of Panfilo, the complaints in NLRC/NCR Case No. 00-08-03013-87 and NLRC Case No. 00-01-
Thereafter, private respondents and several co-employees formed a union called "SAMAHAN NG MGA
00331-88 were consolidated.15 On 29 January 1991, Panfilo died.16
MANGGAGAWA NG PANFILO V. PAJARILLO" (respondent union). The Department of Labor and Employment
(DOLE) issued a Certificate of Registration in favor of the respondent union. 7
After hearing and submission by both parties of their respective position papers and memoranda, Labor Arbiter
Manuel P. Asuncion (Arbiter Asuncion) rendered a Decision 17 dated 28 December 1992, dismissing the
Upon learning of the formation of respondent union, Panfilo and his children ordered some of the private
consolidated complaints for lack of merit. Thus:
respondents to sign a document affirming their trust and confidence in Panfilo and denying any irregularities on
his part. Other private respondents were directed to sign a blank document which turned out to be a
resignation letter. Private respondents refused to sign the said documents, hence, they were barred from IN THE LIGHT OF ALL THE FOREGOING CONSIDERATIONS, the complaint should be as it is hereby dismissed for
working or were dismissed without hearing and notice. Panfilo and his children and relatives also formed a lack of merit.
company union where they acted as its directors and officers.8
Respondent union appealed to the NLRC. On 18 June 1996, the NLRC reversed the decision of Arbiter Asuncion
On 25 August 1987, respondent union and several employees filed a Complaint for unfair labor practice and and ordered the reinstatement of, and payment of backwages, ECOLA, 13th month pay, legal holiday pay and
illegal deduction before the Labor Arbiter with "Panfilo V. Pajarillo Liner" as party-respondent. This was docketed service incentive leave pay to, private respondents. 18 The dispositive portion of the NLRC decision reads:
as NLRC/NCR Case No. 00-08-03013-87.9 On 28 September 1987, the respondent union filed an Amended
Complaint alleging this time not only unfair labor practice and illegal deduction but also illegal dismissal. 10 Wherefore, the appealed decision is hereby set aside. Accordingly, judgment is hereby rendered directing:

On 20 January 1988, respondent union and several employees filed another Complaint for violation of labor (1) The respondent, PVP Liner, Inc. to reinstate to their former positions, without loss of seniority rights
standard laws claiming non-payment of (1) ECOLA, (2) 13th month pay, (3) overtime pay, (4) legal holiday pay, and other benefits, the following complainants: Alfredo [Hoyohoy], Bernardo Roco, Rodolfo Torres,
(5) premium pay, and (6) service incentive leave. The party-respondents in this complaint were "PVP LINER INC. Julian Jorvina, Florita Yapoc, Marlon Aldana, Paraluman Ulang, Tolentino Sanhi, Johnny Soriano, Andres
Calaque, Roberto Lavarez, Francisco Morales, Salvacion Perina, Antonio Abala, Alfonso Baldomar, Jr.,
Set V Corporation Code * Doctrine of Peircing Corpo Veil Cases*Page 48 of 164

Romeo Salonga, Augur Manipol, Bienvenida Tequil, Mario Elep, Aladino Latigo, Bernardine Bansal, On 12 March 2002, the Court of Appeals rendered a Decision granting the respondent unions petition and
Pedro de Baguio, Ricardo Calica, Laura Co, Vicente Recana, Elena Tolledo, Alfredo Plaza, Sr., Herminio nullifying the Orders dated 28 October 1996 and 10 January 1997 of the NLRC. It also reinstated the Decision
Baldono, Felioe Yapoc, Ariston Nipa and Herminia Castillo and to pay them their backwages dated 18 June 1986 of the NLRC.22 The appellate court decreed:
corresponding to a period of three (3) years without qualifications and deductions;
WHEREFORE, premises considered, the PETITION FOR CERTIORARI is hereby GRANTED. Accordingly, the Order
(2) The same respondent PVP Liner, Inc. to pay amounts to be computed in a hearing called for said dated October 28 1996 and January 10, 1997 of the NLRC are hereby NULLIFIED and its Decision dated 18 June
purpose by the Arbitration Branch of Origin, the aforesaid complainants their claims for emergency 1986 be REINSTATED.
cost of living allowance (ECOLA), 13th month pay, legal holiday pay and service incentive leave
benefits subject to the three-year prescriptive period provided under Article 291 of the Labor Code, as Panfilos counsel filed a motion for reconsideration of the said decision but this was denied by the appellate
amended; court in its Resolution dated 28 August 2002. 23

(3) The dismissal of the claims on alleged illegal deductions of the respondents for lack of merits; and Herein petitioners, as heirs of Panfilo, filed the instant petition before this Court assigning the following errors:

(4) The dismissal of the case of Lourdes Roco due to prescription. I.

All other claims of the complainants and the respondents are likewise DISMISSED, for being without merit. THE HONORABLE COURT OF APPEALS SERIOUSLY ERRED IN ARRIVING AT THE CONCLUSION THAT PVP
LINER INC. WAS PROPERLY MISPLEADED, WHICH IS A NON-EXISTING CORPORATION.
The Arbitration Branch of Origin is hereby directed to enforce this decision.
II.
Panfilos counsel filed a motion for reconsideration which was partially granted by the NLRC in its Order dated
28 October 1996, to wit: THE HONORABLE COURT OF APPEALS SERIOUSLY ERRED IN NOT CONSIDERING THAT THERE WAS NO
PROPER AND EFFECTIVE SERVICE OF SUMMONS.
Dictated, however, by the imperatives of due process, we find it more judicious to just remand this case for
further hearing on key questions of: III.

1) whether or not PVP Liner Inc. was properly impleaded as party respondent in the consolidated cases THE HONORABLE COURT OF APPEALS SERIOUSLY ERRED IN PIERCING THE VEIL OF CORPORATE ENTITY
below; OF PVP PAJARILLO LINER INC.

2) whether or not summons was properly served on said corporation below; and IV.

3) whether or not the subject cases can be considered as principally money claims which have to be THE HONORABLE COURT OF APPEALS SERIOUSLY ERRED IN REINSTATING THE ORDER OF THE NLRC
litigated in intestate/testate proceedings involving the estate of the late Panfilo V. Pajarillo. DATED JUNE 18, 1996, WHICH DECLARED THAT PRIVATE RESPONDENTS WERE ILLEGALLY DISMISSED. 24

WHEREFORE, our decision dated June 18, 1996 is hereby set aside. Let this case be remanded to the NCR Anent the first issue, petitioners alleged that the Decision dated 18 June 1996 of the NLRC, ordered PVP Liner
Arbitration Branch for further hearing on the questions above-mentioned. 19 Inc. to reinstate private respondents and pay their backwages, ECOLA, 13th month pay, legal holiday pay and
service incentive leave pay; that there was no such entity as PVP Liner Inc. organized and existing in the
Respondent union filed a motion for reconsideration of the above-stated Order, but this was denied by the Philippines; that it was not possible for Arbiter Asuncion and the NLRC to acquire jurisdiction over a non-existing
NLRC in its Order dated 10 January 1997. 20 Thus, respondent union filed a Petition for Certiorari under Rule 65 company; that there can never be a service of summons or notice to a non-existent entity; that the true
before this Court. Pursuant, however, to our ruling in St. Martin Funeral Home v. National Labor Relations employer of private respondents was Panfilo as the sole proprietor/operator of passenger buses doing business
Commission,21we remanded the petition to the Court of Appeals for proper disposition. under the tradename, PVP Liner, and not PVP Liner Inc. which was non-existent; that Panfilo never used PVP
Liner Inc. as his tradename; that the present operator of PVP Liner buses is P.V. PAJARILLO LINER, a corporation
duly registered with the Securities and Exchange Commission; that at the time the instant case was filed before
Arbiter Asuncion in 1987, the latter did not have jurisdiction over P.V. PAJARILLO LINER because it was organized
and duly registered only on 22 January 1990; that P.V. PAJARILLO LINER has a separate and distinct personality
Set V Corporation Code * Doctrine of Peircing Corpo Veil Cases*Page 49 of 164

from Panfilo as the sole operator of PVP Liner buses; that, therefore, P.V. PAJARILLO LINER cannot be made a President/Manager of PVP Liner Inc., the latter being non-existent; and that Irene was not an officer of P.V.
party or impleaded in the present case; that the amended complaint in NLRC/NCR Case No. 00-08-03013-87 Pajarillo Liner.30
impleaded as party-respondent "PANFILO V. PAJARILLO LINER and PANFILO V. PAJARILLO, as operator and
responsible officer"; that PVP Liner Inc. was not impleaded in the instant case; and that no summons was ever Sections 4 and 5 of Rule IV of the Revised Rules of Procedure of the NLRC provides the rule for the service of
served on PVP Liner Inc. in NLRC/NCR Case No. 00-08-03013-87.25 summonses and notices in NLRC cases, viz:

The contentions are bereft of merit. Sec. 4. Service of notices and resolutions. a) Notices or summons and copies of orders, resolutions or
decisions shall be served personally by the bailiff or the duly authorized public officer or by registered mail on
In the Complaint dated 20 January 1988, PVP Liner Inc. and Panfilo were impleaded as party-respondents, thus: the parties to the case within five (5) days from receipt thereof by the serving officer.

That respondent PVP Liner, Inc., is a private business entity, engaged in transportation of passengers, duly Sec. 5. Proof and completeness of service. The return is prima facie proof of the facts indicated therein.
organized and existing pursuant to law and for this purpose maintains its principal office at 2175, Zamora Service by registered mail is complete upon receipt by the addressee or his agent. 31
Street, Sta. Ana, Manila; while individual respondent [Panfilo] is the General Manager/Operator and may be
served with summons, notices and other processes at the aforementioned principal office. 26 Records show that Irene received the summons for NLRC Case No. 00-01-00331-88 on 4 February 1988 in
behalf of PVP Liner Inc. These summonses were addressed and sent to "THE PRESIDENT/MANAGER, PVP Liner
Panfilo did not question in his position paper or in his motion for consolidation of the complaints the foregoing Inc. and Panfilo V. Pajarillo, 2175 Zamora Street, Sta. Ana, Manila" on 25 January 1988. The Registry Return
allegations. Neither did he assail the inclusion of PVP Liner Inc. as party-respondent in respondent unions Receipt dated 4 February 1988 was addressed to "PVP Liner Inc." and was signed by Irene as the addressees
position paper dated 6 June 1988. agent.32Abel, one of the heirs of Panfilo and the Operations Manager of PVP Liner Inc., testified during the
hearing before Arbiter Asuncion that Irene was one of the secretaries of PVP Liner Inc. 33 Hence, there was a
In Panfilos position paper as well as in the records of the proceedings before Arbiter Asuncion, there is nothing valid service of summons.
that shows that Panfilo challenged the jurisdiction of Arbiter Asuncion over PVP Liner Inc. When Arbiter
Asuncion decided in favor of Panfilo, the latter said nothing about the inclusion of PVP Liner Inc. as party Regarding the third issue, petitioners posited that P.V. Pajarillo Liner Inc. is an independent corporation and
respondent and the lack of jurisdiction of Arbiter Asuncion over the same. It was only when the NLRC rendered cannot be considered as an adjunct or extension of Panfilo as the sole operator of PVP Liner buses; and that at
a Decision adverse to Panfilo that the latter alleged the non-existence of PVP Liner Inc. and the fact that Arbiter the time P.V. Pajarillo Liner Inc. was established, it had no liability or obligation which it tried to shield or
Asuncion and the NLRC had no jurisdiction over it. circumvent.34

Petitioners are now precluded from questioning the inclusion of PVP Liner Inc. as party-respondent as well as It is a fundamental principle of corporation law that a corporation is an entity separate and distinct from its
the jurisdiction of Arbiter Asuncion and the NLRC over them under the principle of estoppel. It is settled that the stockholders and from other corporations to which it may be connected. However, this separate and distinct
active participation of a party against whom the action was brought, coupled with his failure to object to the personality of a corporation is merely a fiction created by law for convenience and to promote justice. Hence,
jurisdiction of the court or quasi-judicial body where the action is pending, is tantamount to an invocation of when the notion of separate juridical personality is used to defeat public convenience, justify wrong, protect
that jurisdiction and a willingness to abide by the resolution of the case and will bar said party from later on fraud or defend crime, or is used as a device to defeat labor laws, this separate personality of the corporation
impugning the court or bodys jurisdiction. 27 This Court has time and again frowned upon the undesirable may be disregarded or the veil of the corporate fiction pierced. This is true likewise when the corporation is
practice of a party submitting his case for decision and then accepting the judgment only if favorable, and merely an adjunct, a business conduit or an alter ego of another corporation. The corporate mask may be lifted
attacking it for lack of jurisdiction when adverse. 28 and the corporate veil may be pierced when a corporation is but the alter ego of a person or another
corporation.35
It is apparent that Panfilo V. Pajarillo Liner and PVP Liner Inc. are one and the same entity belonging to one and
the same person, Panfilo. When PVP Liner Inc. and Panfilo V. Pajarillo Liner were impleaded as party- It is apparent that Panfilo started his transportation business as the sole owner and operator of passenger
respondents, it was Panfilo, through counsel, who answered the complaints and filed the position papers, buses utilizing the name PVP Liner for his buses. After being charged by respondent union of unfair labor
motions for reconsideration and appeals. It was also Panfilo, through counsel, who participated in the hearings practice, illegal deductions, illegal dismissal and violation of labor standard laws, Panfilo transformed his
and proceedings. In fact, Abel Pajarillo (Abel), son of Panfilo, testified before Arbiter Asuncion that he was the transportation business into a family corporation, namely, P.V. Pajarillo Liner Inc. He and petitioners were the
operations manager of PVP Liner Inc.29 Further, both Panfilo and PVP Liner Inc. were charged jointly and incorporators, stockholders and officers therein. P.V. Pajarillo Inc. and the sole proprietorship of Panfilo have the
severally in the aforesaid complaints. same business address. P.V. Pajarillo Inc. also uses the name "PVP Liner" in its buses. Further, the license to
operate or franchise of the sole proprietorship was merely transferred to P.V. Pajarillo Liner Inc. The testimony
Apropos the second issue, petitioners alleged that the notices and summons were received by a certain Irene of Abel during the hearing before Arbiter Asuncion is revealing, thus:
G. Pajarillo (Irene) for and in behalf of the PVP Liner Inc.; that Irene was neither and could not have been the
Set V Corporation Code * Doctrine of Peircing Corpo Veil Cases*Page 50 of 164

Q: Mr. Pajarillo, when did you start assuming the functions of operations manager of PVP Liner? It is clear from the foregoing that P.V. Pajarillo Liner Inc. was a mere continuation and successor of the sole
proprietorship of Panfilo. It is also quite obvious that Panfilo transformed his sole proprietorship into a family
A: Seven years from now, sometime in the year 1984 or 1985, sir. corporation in a surreptitious attempt to evade the charges of respondent union. Given these considerations,
Panfilo and P.V. Pajarillo Liner Inc. should be treated as one and the same person for purposes of liability. 37

Q: Do you have any written appointment as Operations Manager?


Finally, petitioners averred that no unfair labor practice was committed, and that private respondents were not
illegally dismissed from work.
A: No, sir.

In its Decision dated 18 June 1996, the NLRC made an exhaustive discussion of the allegations and evidence of
Q: I noticed that your surname is Pajarillo you are one way or another related to Mr. Panfilo V. Pajarillo,
both parties as regards unfair labor practice and illegal dismissal. It concluded that private respondents,
is that correct?
officers and members of respondent union were dismissed by reason of their union activities and that there
was no compliance with substantial and procedural due process in terminating their services. It also held that
Witness: the private respondents who were not members of the respondent union were also dismissed without just or
valid cause, and that they were denied due process. These factual findings and conclusions were supported by
A: I am the son of Panfilo Pajarillo, sir. substantial evidence comprised of affidavits, sworn statements, testimonies of witnesses during hearings
before Arbiter Asuncion, and other documentary evidence. These findings were sustained by the Court of
Appeals.
Q: In so far as PVP Liner is concerned and being the operations manager, are you aware if it is a single
proprietor or a corporation?
The rule is that findings of fact of quasi-judicial agencies like the NLRC are accorded by this Court not only
respect but even finality if they are supported by substantial evidence, or that amount of relevant evidence
A: At the start it was a single proprietorship, lately, it has become a family corporation.
which a reasonable mind might accept as adequate to justify a conclusion. 38 We find no compelling reason to
deviate from such findings of the NLRC as affirmed by the Court of Appeals.
Atty. Flores, Jr. (to witness)

Consequently, the private respondents are entitled to reinstatement, backwages and other privileges and
Q: When you became the Operations Manager of PVP Liner, is it a single proprietor or a family benefits under Article 279 of the Labor Code. Separation pay may be given in lieu of reinstatement if the
Corporation? employee concerned occupies a position of trust and confidence. In the case at bar, however, the private
respondents, as former bus drivers, conductors and conductresses of petitioners, do not hold the position of
A: It was a single proprietorship. trust and confidence.39

Q: Mr. Witness, since PVP Liner is a transportation business it has a license to operate these buses? Nonetheless, it appears from the records that some of the private respondents, namely, Augur Manipol, Rodolfo
Torres, Ricardo Calica, Paraluman Ulang, Edith Chua, Alfredo Hoyohoy, Johnny Soriano, Bernardo Roco, Tolentino
Sanhi, Salvacion Perina, Pedro L. de Baguio, Ariston Nipa, Felipe Yapoc, Laura Co, Bienvenida Tequil, Roberto
A: Yes, there is, sir.
Lavarez, Francisco Morales and Herminio Castillo, had executed a Quitclaim/Release discharging petitioners
"from any and all claims by way of unpaid wages, separation pay, overtime pay, differential pay, ECOLA, 13th
Atty. Flores, Jr. (to witness) month pay, holiday pay, service incentive leave pay or otherwise." 40

Q: In whose name was it registered? Generally, deeds of release, waivers, or quitclaims cannot bar employees from demanding benefits to which
they are legally entitled or from contesting the legality of their dismissal, since quitclaims are looked upon with
A: Before it was with my father Panfilo V. Pajarillo, sir. disfavor and are frowned upon as contrary to public policy. Where, however, the person making the waiver has
done so voluntarily, with a full understanding thereof, and the consideration for the quitclaim is credible and
Q: Do I understand that the licensing of this transportation company was transferred to another reasonable, the transaction must be recognized as being a valid and binding undertaking. 41
person?
There is no showing that the executions of these quitclaims were tainted with deceit or coercion. On the
A: It was never transferred to another person, except now, that it has been transferred to a contrary, each of the private respondents Sinumpaang Salaysay, which accompanied the quitclaims, evinces
corporation.36 voluntariness and full understanding of the execution and consequence of the quitclaim. In their said
Set V Corporation Code * Doctrine of Peircing Corpo Veil Cases*Page 51 of 164

Sinumpaang Salaysay, the private respondents stated that their lawyer had extensively explained to them the on Non-Forum Shopping was defective because co-petitioner Peter C. Maligro was not a signatory thereto, as
computation and the actual amount of consideration they would receive; that they were not forced or tricked reiterated in its subsequent Resolution of July 16, 2002,2 denying the petitioners' motion for reconsideration.
by their lawyer in accepting the same; and that they already received the amount of consideration. 42
The facts:
Further, the considerations received by the private respondents were credible and reasonable because they
were not grossly disproportionate to the computation by the NLRC of the amount of backwages and other Petitioner Petron Corporation (Petron), a corporation duly organized and existing under the laws of the
money claims.43 Philippines, is engaged in the refining, sale and distribution of petroleum and other related products, while its
co-petitioner Peter C. Maligro was the former Visayas Operations Assistant Manager of Petron's Visayas-
Given these circumstances, the quitclaims should be considered as binding on the private respondents who Mindanao District Office at Lahug, Cebu City.
executed them. It is settled that a legitimate waiver which represents a voluntary and reasonable settlement of
a workers claim should be respected as the law between the parties. 44 Accordingly, the private respondents On May 15, 1990, Petron, through its Cebu District Office, hired the herein private respondent Chito S. Mantos,
who made such quitclaims are already precluded from claiming reinstatement, backwages, ECOLA, 13 TH month an Industrial Engineer, as a managerial, professional and technical employee with initial designation as a Bulk
pay, legal holiday pay, service incentive leave pay, and other monetary claims. Plant Engineering Trainee. He attained regular employment status on November 15, 1990 and was later on
designated as a Bulk Plant Relief Supervisor, remaining as such for the next five years while being assigned to
With regard to the other private respondents who did not execute such quitclaims, they are entitled to the different plants and offices of Petron within the Visayas area.
reinstatement, backwages, ECOLA, 13 TH month pay, legal holiday pay and service incentive leave pay in
accordance with the computation of the NLRC. It was while assigned at Petron's Cebu District Office with petitioner Peter Maligro as his immediate superior,
when Mantos, thru a Notice of Disciplinary Action dated October 29, 1996, 3 a copy of which was received by
WHEREFORE, the petition is hereby DENIED. The Decision and Resolution dated 12 March 2002 and 28 August him on November 18, 1996,4 was suspended for 30 days from November 1 to 30, 1996 for violating company
2002, respectively, of the Court of Appeals in CA-G.R. SP No. 54330 and CA-G.R. SP No. 54331, are hereby rules and regulations regarding Absence Without Leave (AWOL), not having reported for work during the period
AFFIRMED with the following MODIFICATIONS: (1) Private respondents Augur Manipol, Rodolfo M. Torres, Ricardo August 5 to 27, 1996.
Calica, Paraluman Ulang, Edith Chua, Alfredo Hoyohoy, Johnny Soriano, Bernardo Roco, Tolentino Sanhi,
Salvacion Perina, Pedro L. de Baguio, Ariston Nipa, Felipe Yapoc, Laura Co, Bienvenida Tequil, Roberto Lavarez, Subsequently, in a notice Termination of Services bearing date November 20, 1996 5 and received by him on
Francisco Morales and Herminio Castillo are hereby precluded from claiming reinstatement, backwages, ECOLA, November 25, 1996,6 Mantos' services were altogether terminated effective December 1, 1996, by reason of
13TH month pay, legal holiday pay and service incentive leave pay by reason of their respective quitclaims; (2) his continued absences from August 28, 1996 onwards, as well as for Insubordination/Discourtesy for making
Petitioners are hereby ordered to reinstate private respondents Julian Jorvina, Florita Yapoc, Marlon Aldana, false accusations against his superior.
Andres Calaque, Antonio Abala, Alfonso Baldomar, Romeo Salonga, Mario Elep, Aladino Latigo, Bernardine
Bansal, Vicente Recana, Elena Tolledo and Alfredo Plaza, Sr., and to pay these respondents backwages from the
Meanwhile, on November 8, 1996, contending that he has been constructively dismissed as of August 5, 1996,
time of their dismissal up to the finality of this Decision. Petitioners are also ordered to pay the foregoing
Mantos filed with the National Labor Relations Commission, Regional Arbitration Branch (NLRC-RAB), Cebu City,
private respondents ECOLA, 13TH month pay, legal holiday pay and service incentive leave pay in accordance
a complaint for illegal dismissal and other monetary claims against Petron and/or Peter C. Maligro. The case
with the computation of the NLRC. Costs against petitioners.SO ORDERED.
was docketed as NLRC RAB-VII Case No. 11-1439-96.

SECOND DIVISION G.R. No. 154532 October 27, 2006


In his complaint, Mantos made the following allegations:

PETRON CORPORATION AND PETER C. MALIGRO, petitioners, vs. NATIONAL LABOR RELATIONS
xxx He had an unblemished record in his service with [Petron]. Intrigues and professional jealousies,
COMMISSION AND CHITO S. MANTOS, respondents.
however, have prevailed over the work atmosphere in [Petron]. This became more particularly true in
regard to his close relationship with Jaime "Boy" Tamayo, then the VISMIN Operations Manager who
DECISION later left the company to migrate to Canada. His closeness to Tamayo has caused problems with his
relationship with Peter Maligro, Visayas Operations Assistant Manager, who has been after his neck for
GARCIA, J.: sometime. Maligro's hatred on him became evident when he was assigned to Nasipit Bulk Plant at
Nasipit, Agusan del Norte for two (2) months or so. He was deprived of his usual P1,000.00 a day per
Assailed and sought to be set aside in this petition for review under Rule 45 of the Rules of Court is diem. He was also deprived of the usual facilities such as the service vehicle and the use and access
theResolution dated November 26, 20011 of the Court of Appeals (CA) in CA-G.R. SP No. 67702, dismissing the to lighterage services.
petition for certiorari thereat filed by the herein petitioners on the ground that the Verification and Certification
Set V Corporation Code * Doctrine of Peircing Corpo Veil Cases*Page 52 of 164

Because of the tremendous work pressure, he availed and was granted a vacation leave in March
1996. Before he reported back to work he was summoned to the office of Peter Paul Shotwell. There,
he was advised by [Petron's] officers to resign from [Petron] as they were instructed by superiors that b. Cash equivalent of certificate of stocks - P 66,600.00
he should quit as they no longer liked him. Failing to convince him he was later offered to avail of
[Petron's] early retirement program dubbed as "Manpower Reduction Program" or MRP. Thereafter he
was advised to avail of his remaining vacation leave while they process his MRP papers. After his
vacation, he was no longer allowed to report back at his assignment at Mactan Aviation Facilities but Total P159,120.00
directly to Maligro at the Cebu District Office. While being designated as Operations Engineer, he was
assigned only menial tasks such as recopying errands, digging up files, drafting and redrafting
memoranda and other mere clerical works. On August 5, 1996, Maligro bad-mouthed him in the
presence of his co-employees for alleged dissatisfaction of his work as a mere clerk. What [Petron and
Maligro] have done to him amounts to constructive dismissal. Hence, his complaint. 7 (Words in Minus - P 56, 191.59
brackets supplied.)

For their part, Petron and Maligro averred that Mantos was dismissed for just and valid causes effective
December 1, 1996, asserting that: Net Award P102, 928.41

xxx complainant [Mantos] incurred absences without leave (AWOL) on August 5 to 27, 1996 inclusive.
He failed to comply with the instruction of a superior for him to report for work at the Cebu City 9
SO ORDERED.
District office and to submit a formal explanation of his AWOL. From August 28, 1996, up to the filing
of respondents' position paper, complainant has not reported for work but continued to receive the
salary for the months of August, September and October 2, 1996. An investigation was conducted on Explains the Labor Arbiter in his decision:
September 2, 1996 but complainant failed to appear. Instead he sent two (2) letters thru his counsel
accusing respondent Maligro of certain acts humiliating and prejudicing him. After a series of hearings, It is an established fact that for his absences from August 5 to August 27, 1996, complainant was
[Petron's] Investigation Committee in a report and recommendation of November 19, 1996, imposed the penalty of suspension for thirty (30) days from November 1 to 30, 1996 per the letter of
recommended that after a 30-day suspension, complainant should be subjected to a more severe respondent Maligro to complainant dated October 29, 1996 (Annex "D"). From respondents' Annex "6"
penalty. Hence, they deny complainant's claims. 8 which is a memorandum of November 19, 1996 containing the report of the Investigation Committee it
is shown therein that the summons in this case was received by respondents on November 14, 1996.
In a decision dated June 30, 1998, Labor Arbiter Dominador A. Almirante declared Mantos to have been The following day, November 15, 1996, the Committee met to determine the factual basis of the
constructively dismissed but ruled that only Petron could be held liable to him for separation pay in lieu of charges of absence without leave and insubordination against complainant. The Committee was
reinstatement and the cash equivalent of his certificate of stocks, less his personal accountabilities. More convened seven (7) days after the filing of the complaint herein on November 8, 1996.
specifically, the decision dispositively states:
We find that the foregoing factual milieu militates badly against the cause for the respondents. It
WHEREFORE, foregoing premises considered, judgment is hereby rendered ordering the respondent appears that the Investigation Committee was belatedly constituted as an afterthought after the
Petron Corporation VISMIN District Office to pay complainant the amount of One Hundred Two respondents received the summons in this case. For his AWOL, complainant was already sufficiently
thousand Nine Hundred Twenty-Eight Pesos and 41/100 (P102,928.41) representing the separation pay penalized by suspension for thirty (30) days, the maximum penalty authorized by law. In fact,
for his six (6) years of service at P15,420.00 a month, the cash equivalent of his certificate of stocks complainant was still serving his suspension when the Committee was convened and issued the
minus his outstanding account, computed as follows: memorandum of November 19, 1996 recommending his dismissal for AWOL and insubordination. The
insubordination aspect stemmed from complainant's accusation in his complaint for constructive
dismissal and withholding of his stock certificates. The imposition of the penalty of dismissal smacks
of a desire to get even for complainant's filing of a complaint against the respondents. Anyway, the
a. Separation Pay: penalty of dismissal was too harshly and [d]isproportionately imposed on the complainant considering
P15,420.00 x 6 years - P 92, 520.00 his length of service.

Furthermore, there is in an (sic) unrebutted evidence for the complainant that earlier while being
assigned directly under respondent Maligro at the Cebu District Office, with the designation as
Set V Corporation Code * Doctrine of Peircing Corpo Veil Cases*Page 53 of 164

Operations Engineer, he was assigned only menial tasks like recopying errands, digging up files, Complainant is likewise entitled to ten percent (10%) of the total awards by way of attorney's fees.
drafting and redrafting memoranda and other clerical works.
The foregoing liabilities are solidary against respondents Petron Corporation and Peter C. Maligro. SO
We find that respondents' act was tantamount to constructive dismissal xxx Under such ORDERED.11
circumstances, the continuance of complainant's employment with respondent corporation has been
rendered impossible, unreasonable and unlikely. There exists also a demotion in rank.

xxx xxx xxx Justifying its decision, the NLRC explained that Mantos failed to prove that he had to quit his job on August 5,
1996 because his continued employment was rendered impossible, unbearable and unlikely. On the other hand,
We find therefore that complainant was illegally dismissed from the service. He should have been Petron and Maligro did not observe the requisite procedural due process considering that (1) the alleged Notice
reinstated to his former position without loss of seniority rights. We find however, that the filing of this of Violation of Company Rules and Regulations dated August 27, 1996 which preceded the suspension of
complaint has spawned strained relationship between the parties. Hence, reinstatement is no longer Mantos was not received by the latter; and (2) no separate notice for the two new charges of Absence Without
practical and feasible. Instead complainant should be awarded his separation pay equivalent to one Leave (AWOL) starting August 28, 1996 and Insubordination/Discourtesy for making false accusations against
(1) month pay per year of service. He is not however entitled to backwages. He is not completely free his superior, were sent to Mantos prior to the Notice of Termination dated November 20, 1996 based on the
from blame in his separation from the service. He committed absences without leave. xxx report/recommendation dated November 19, 1996 of the Investigation Committee. Furthermore, the
Commission noted that on the day after Petron and Maligro received the summons with respect to Mantos'
xxx xxx xxx complaint with the NLRC-RAB, the Investigation Committee was immediately convened regarding Mantos'
continued absences beginning August 28, 1996 with Maligro himself being a member of said committee.

Complainant is also entitled to the cash equivalent of his certificate of stocks admitted in respondent's
Exhibit "7" to be P66,600.00. From the total award shall be deducted the amount of P56,191.59 With their motion for reconsideration having been denied by the NLRC in its Resolution of August 31,
complainant's outstanding account to respondent. 2001,12 the petitioners elevated the case via certiorari to the CA in CA-G.R. SP No. 67702.

The rest of the claims are hereby ordered dismissed for lack of merit not having been substantiated by As stated at the threshold hereof, the CA, in its assailed Resolution of November 26, 2001, outrightly dismissed
clear and convincing evidence. Respondent Peter C. Maligro is hereby absolved from any liability the petition for being defective in form because only petitioner Petron signed the verification and certification
hereof there being no showing that he acted in bad faith and in excess of his authority in dealing with on non-forum shopping without its co-petitioner Peter Maligro likewise signing the same.
the complainant.10
Their motion for reconsideration having been denied by the CA in its second impugned Resolution of July 16,
Both dissatisfied, the parties questioned the aforementioned Labor Arbiter's decision: Petron and Maligro, by 2002, the petitioners are now with us via the present recourse on the following grounds:13
way of an appeal to the NLRC at Cebu City, accompanied by a P102, 928.41 surety bond in favor of Mantos;
and the latter, by a motion for reconsideration which the NLRC eventually treated as an appeal. A. THE COURT OF APPEALS ERRED IN DISMISSING PETITIONERS' PETITION FOR CERTIORARI ON THE GROUND
THAT THE SAME FAILED TO COMPLY WITH THE RULE ON CERTIFICATION ON NON-FORUM SHOPPING
On July 31, 2000, the NLRC reversed the findings of the Labor Arbiter regarding Mantos' constructive dismissal CONSIDERING THAT:
as of November 1, 1996 and considered him to have been illegally dismissed only on December 1, 1996. In the
same decision, the NLRC adjudged Maligro solidarily liable with Petron, and accordingly modified the Labor 1. THERE WAS SUBSTANTIAL COMPLIANCE BY PETITIONERS WITH THE REQUIREMENTS ON
Arbiter's decision as follows: CERTIFICATION OF NON-FORUM SHOPPING.

WHEREFORE, the questioned Decision is MODIFIED in that complainant was illegally suspended from 2. THERE WAS A REASONABLE CAUSE FOR PETITIONER MALIGRO'S FAILURE TO ATTACH A
November 1-30, 1996 and was ILLEGALLY DISMISSED on December 1, 1996, accordingly and as VERIFICATION/CERTIFICATION OF NON-FORUM SHOPPING.
discussed, he should be paid separation pay based on his one month salary (P15,420.00) per year of
service computed until the month of promulgation (July, 2000) of this Decision. In addition,
complainant is entitled to full backwages from November 1, 1996 until July, 2000.

B. THE OUTRIGHT DISMISSAL OF THE PETITION BY THE COURT OF APPEALS WOULD DEFEAT SUBSTANTIAL
The finding below of cash equivalent of certificate of stocks in the amount of P66,600.00 is deleted. JUSTICE CONSIDERING THAT THE NLRC COMMITTED GRAVE ABUSE OF DISCRETION AMOUNTING TO LACK OR
The accountability of complainant in the amount of P56,191.59 shall be deleted from his total awards. EXCESS OF JURISDICTION IN FINDING THAT:
Set V Corporation Code * Doctrine of Peircing Corpo Veil Cases*Page 54 of 164

1. PRIVATE RESPONDENT'S COMPLAINT FOR ILLEGAL DISMISSAL WAS NOT FILED AS A MALICIOUS It must be remembered that the petitioners in CA-G.R. SP No. 67702 are Petron and its operations assistant
SCHEME AGAINST PETITIONERS, DESPITE OVERWHELMING EVIDENCE ON RECORD. manager, Peter Maligro. Evidently, Maligro was included in the complaint filed by Mantos in NLRC RAB-VII Case
No. 11-1439-96 in Maligro's capacity as Petron's corporate officer. Maligro has no separate and distinct
2. PETITIONERS DISMISSED PRIVATE RESPONDENT MANTOS WITHOUT OBSERVING THE REQUISITE personality from that of Petron, undoubtedly the direct employer of Mantos against which any award in the
PROCEDURAL DUE PROCESS BECAUSE PETITIONERS ALLEGEDLY DID NOT PROVE THAT MANTOS latter's favor is enforceable. With Petron being the real party-in interest in that case and not Maligro, the latter's
RECEIVED THE NOTICE OF VIOLATION OF COMPANY RULES DATED 27 AUGUST 1996 AS WELL AS THE failure to equally sign the verification and certification on non-forum shopping should not have merited the CA's
TWO TELEGRAMS REQUIRING MANTOS TO REPORT FOR WORK, CONTRARY TO SUBSTANTIAL EVIDENCE outright dismissal of the certiorari petition in CA-G.R. SP No. 67702.
ON RECORD.
In outrightly dismissing the petition, the CA relied on Loquias v. Office of the Ombudsman.17 The appellate
3. THAT PETITIONERS DISMISSED MANTOS WITHOUT OBSERVING THE REQUISITE PROCEDURAL DUE court's reliance on that case is misplaced. For, in the subsequent case of Micro Sales Operation
PROCESS BECAUSE PETITIONERS ALLEGEDLY DID NOT SEND A NOTICE OF VIOLATION OF COMPANY Network and Willy Bendol v. NLRC, et. al., 18 wherein the CA based its dismissal of the therein similarly
RULES TO PRIVATE RESPONDENT FOR THE OFFENSES THAT HE COMMITTED FOR THE SECOND TIME, defective petition for certiorari on the strength of Loquias, this Court ruled:
DESPITE CONTRARY EVIDENCE ON RECORD.
The Court of Appeals relied on Loquias v. Office of the Ombudsman, which held that a certification on
4. THAT PETITIONERS DID NOT SHOW HOW THE INVESTIGATION COMMITTEE THAT INVESTIGATED non-forum shopping signed by only one of two or more petitioners is defective, unless he was duly
MANTOS' VIOLATIONS OF COMPANY RULES WAS CREATED AND THAT THE SAME WAS BIASED AGAINST authorized by his co-petitioner. However, the said ruling applies when the co-parties are being sued in
MANTOS MERELY BECAUSE ITS CHAIRMAN WAS MANTOS' SUPERIOR, DESPITE CONTRARY EVIDENCE their individual capacities. Note that the petitioners in Loquias are the mayor, vice-mayor, and three
ON RECORD. members of the municipal board of San Miguel, Zamboanga del Sur. The said co-parties were charged
with violation of Republic Act No. 3019 15 in their various capacities.

5. THAT PETITIONER PETER C. MALIGRO IS SOLIDARILY LIABLE WITH PETITIONER PETRON


CORPORATION FOR THE LATTER'S ALLEGED LIABILITY TO MANTOS NOTWITHSTANDING THE ABSENCE In the instant case, the petitioners are the company and its operations manager, Willy Bendol. The
OF EVIDENCE INDICATING THAT MALIGRO ACTED WITH BAD FAITH AGAINST MANTOS. latter was impleaded simply because he was a co-respondent in the illegal dismissal complaint. He has
no interest in this case separate and distinct from the company, which was the direct employer of
private respondents. Any award of reinstatement, backwages, and attorney's fees in favor of private
6. THAT PRIVATE RESPONDENT IS ENTITLED TO AWARD OF FULL BACKWAGES FROM 1 NOVEMBER 1996
respondents will be enforced against the company as the real party in interest in an illegal dismissal
UNTIL JULY 2000 AND TO THE OTHER MONETARY AWARDS MADE BY THE NLRC.
case. Petitioner Bendol is clearly a mere nominal party in the case. His failure to sign the verification
and certification on non-forum shopping is not a ground for the dismissal of the petition. The appellate
In his Comment,14 the private respondent avers, among others, that the petitioners' petition for certiorari in CA- court erred in dismissing outright petitioners' special civil action for certiorari solely on that ground .
G.R. SP No. 67702 cannot alter the factual findings of the Labor Arbiter as affirmed by the NLRC. He argues that (Emphasis supplied.)
the sole office of a writ of certiorari is to correct jurisdictional errors including grave abuse of discretion
amounting to lack or excess of jurisdiction, and does not include correction of the NLRC's evaluation of the
In any event, considering that Maligro derives his standing or personality in the case from Petron, the
evidence, whose factual findings are generally accorded not only great respect but even finality.
certification on non-forum shopping executed and signed only by the corporation benefited Maligro such that
the attachment of said certification to the petition in CA-G.R. SP No. 67702 should be deemed substantial
The petition is partly meritorious. compliance with the rule on certification on non-forum shopping.

Concededly, the fact that only Petron, minus its co-petitioner Peter C. Maligro, executed and signed the We have, therefore, opted to give due course to the present petition. And realizing that a remand of this case to
Verification and Certification on Non-Forum Shopping, 15 attached to the petition for certiorari in CA-G.R. SP No. the CA would only entail further delay in the proceedings, we deemed it prudent to resolve the controversy to
67702, is a cause for the dismissal of that petition, conformably with Section 5, Rule 7 of the Rules of Court finally put it to a rest.
which expressly requires that the certification against forum shopping must have to be certified under oath by
"the plaintiff or principal party," and failure to comply therewith shall cause the dismissal of the action. 16
In the review of NLRC decisions through the special civil action of certiorari, resolution is confined only to issues
of jurisdiction and grave abuse of discretion on the part of the labor tribunal. The Court refrains from reviewing
Be that as it may, we hold that the CA erred in outrightly dismissing CA-G.R. SP No. 67702 solely on the ground factual assessments of lower courts and agencies exercising adjudicative functions, such as the NLRC. 19
that therein co-petitioner Peter Maligro failed to equally sign the verification and certification on non-forum
shopping.
Here, however, we are constrained to make a review of the records and a re-examination of the questioned
NLRC findings to arrive at a complete, just and proper determination of the case.
Set V Corporation Code * Doctrine of Peircing Corpo Veil Cases*Page 55 of 164

Essentially, the issue posed is the validity of private respondent's dismissal. On the other hand, while respondent has indeed been absent from August 28, 1996, the penalty of dismissal
therefor is too harsh considering that all the while, he deemed himself to have been already dismissed as early
The validity of an employee's dismissal hinges on the satisfaction of two substantive requirements, to wit: (1) as August 5, 1996. Besides, private respondent has already been penalized with suspension for his
the employee was accorded due process, basic of which are the opportunity to be heard and to defend himself; unauthorized absences, which notice of suspension he only received on November 18, 1996.
and (2) the dismissal must be for any of the causes provided for in Article 282 of the Labor Code. 20
Likewise, the petitioners failed to prove that they complied with the requisites of procedural due process in
The illegality of the act of dismissal constitutes discharge without just cause, while the illegality in the manner dismissing private respondent.
of dismissal is dismissal without due process. 21
It is horn-book law that an employee sought to be dismissed must be served two (2) written notices before
Here, private respondent was successively charged with two (2) sets of offenses and separately penalized for termination of employment: a notice to apprise the employee of the particular acts or omissions for which his
each set. dismissal is sought; and the subsequent notice to inform him of the employer's decision to discharge him from
the service.26 The procedure is mandatory and non-observance thereof renders the dismissal illegal and void. 27

The first set of infractions consisted of private respondent's being AWOL from August 5 to 27, 1996 and
Insubordination/Discourtesy as set forth in the Notice of Violation of Company Rules and Regulations dated Here, while the private respondent received the Notice of Disciplinary Action dated October 29, 1996 informing
August 27, 1996,22 for which he was penalized with suspension for 30 days effective November 1 to 30, 1996 him of his suspension, and the Memorandum dated November 20, 1996 terminating his services, he did not
but only for the charge of being AWOL. The second set, as contained in the Notice of Violation of Company receive any prior notice[s] apprising him of the particular acts for which his suspension and/or termination were
Rules and Regulations (EM 300) dated November 12, 1996 23 consisted also of being AWOL, this time beginning being sought.
August 28, 1996, and Insubordination/Discourtesy for making false accusations against his superior, for which
he wasdismissed effective December 1, 1996. As rightly found by the NLRC, the private respondent was not given the following notices, to wit: (1) the Notice
of Violation of Company Rules and Regulations dated August 27, 1996 on his AWOL from August 5 to 27, 1996
Private respondent did not report for work starting August 5, 1996 due to his belief that he has already been and Insubordination/Discourtesy with notice of an investigation on September 2, 1996; and (2) the Notice dated
dismissed as of said date. But since he failed to prove his allegation of clear acts of harassment and November 12, 1996 on the second set of charges of AWOL starting August 28, 1996 and
humiliation, which had allegedly become so unbearable as to leave him with no choice but to forego his Insubordination/Discourtesy for allegedly making false accusations against his superior with notice of the
continued employment, we uphold the legality of his suspension due to his unauthorized absences from August investigation on November 15, 1996.
5 to 27, 1996.
As borne by the records, it was only in their motion for reconsideration of the NLRC decision that the petitioners
With respect to respondent's dismissal, however, we find the same unjustified. proffered the delivery records of a private courier to show that the aforementioned notices, as well as two
alleged telegrams requiring the private respondent to report for work, 28 were in fact sent to the latter. But, a
perusal of said delivery records does not bear the petitioners' claim. For, apart from the private respondent's
Under paragraph (a), Article 282 of the Labor Code, 24 an employer may terminate the services of an employee
full name, Chito S. Mantos, being written in block letters on the said delivery records, there is no other way of
for his willful disobedience of the employer's lawful orders in connection with his work.
knowing whether it was really him who received the notices or that another person could have received the
same in his behalf.29Verily, said delivery records do not substantially show respondent's receipt of the notices in
Verily, the employer's rules, instructions or commands, in order to be a ground for discharge on the score of question.
disobedience, must be reasonable and lawful, must be known to the employee, and must pertain to the duties
for which his services were engaged.25
Given the above, we cannot give credence to petitioners' claim that as early as August 27, 1996, the date of
the notice allegedly sent to the respondent informing him of the first set of offenses, the latter already knew
From the foregoing, it is clear that the factual basis for the petitioners' charge of insubordination against the that a committee was going to investigate him for infractions of company rules and regulations in connection
private respondent, i.e., making false accusations against his superior cannot constitute a just cause for with thesecond set and that he was invited to attend the investigating committee's scheduled hearing.
dismissal. The so-called accusations are embodied in the complaint filed by the private respondent in NLRC
RAB-VII Case No. 11-1439-96, in which complaint he believed himself to have been constructively dismissed as
We, therefore, lend concurrence to the common findings of both the NLRC and the Labor Arbiter that the
of August 5, 1996. By no stretch of imagination can the filing of such complaint constitute insubordination. If,
committee which investigated the alleged second set of offenses and which eventually led to the committee's
as asserted by the private respondent, he had been constructively dismissed as of August 5, 1996, such
recommendation for his dismissal was created only on November 15, 1996 or a day at the heels of the
assertion could not have risen to the level of false accusation against his superior.
petitioners' receipt on November 14, 1996 of the summons issued in NLRC RAB-VII Case No. 11-1439-96.
Set V Corporation Code * Doctrine of Peircing Corpo Veil Cases*Page 56 of 164

With the reality that no notice of any investigation was timely served on the private respondent, the latter's Besides, the fact that Maligro himself was the committee chairman is not itself sufficient to impute bad faith on
filing of his complaint for illegal dismissal in NLRC RAB-VII Case No. 11-1439-96 on November 8, 1996 could not his part or attribute bias against him. It is undisputed that Maligro was private respondent's superior, being
be said to have been made to preempt the investigation regarding his alleged offenses as he was yet unaware Petron's Operations Assistant Manager for Visayas and Mindanao. It is thus logical for him to be part of the
of any such investigation. Moreover, as the NLRC rightly observed: committee that will investigate private respondent's alleged infractions of company rules and regulations. As
well, the committee was composed of three other Petron officers as members, and nowhere is there any
We note from the records that although complainant quit working starting August 5, 1996 because he showing that Maligro, as committee chairman, influenced the other committee members to side against the
felt he was "constructively dismissed" he did not file outright the present complaint. Instead, he wrote private respondent.
respondent Maligro on October 18, 1996, thru counsel asking an explanation why no case for illegal
dismissal with damages would be filed against respondents. When he therefore finally filed the present In any event, it must be stressed that private respondent's allegation of bad faith on the part of Maligro was not
case on Novemeber 8, 1996, that showed his lingering belief that he was constructively dismissed established in this case. We quote the NLRC's finding in this regard:
although from the viewpoint of respondents, he was already penalized with "grave suspension" for his
AWOL from August 5-27, 1996. In short, the filing of the complaint was not a "malicious scheme" on Whether he really caught the ire of his immediate supervisor (respondent Maligro) in view of his
the part of the complainant contrary to the contention of respondents. 30 alleged closeness to the previous one who migrated to Canada, and whether or not he was assigned
to menial clerical jobs when his designation was that of Operations Engineer, were not clearly
Petitioners' failure to comply with the two-notice requirement as shown above, let alone the lack of just cause established by complainant.35
for terminating the services of private respondent, rendered the latter's dismissal illegal.
Lastly, as to the award of backwages, we refer to Article 279 of the Labor Code (as amended by Section 34 of
In fine, we rule and so hold that the NLRC did not gravely abuse its discretion in declaring the illegality of R.A. 6715) which provides that an employee who is unjustly dismissed from work is entitled to reinstatement
private respondent's dismissal. without loss of seniority rights and other privileges, and to the payment of his full backwages, inclusive of
allowances, and other benefits or their monetary equivalent computed from the time his compensation was
We are, however, with the petitioners in their submission that the NLRC erred in holding petitioner Peter Maligro withheld from him (which, as a rule, is from the time of his illegal dismissal) up to the time of his actual
jointly and severally liable with petitioner Petron for the money claims of the private respondent. reinstatement. Similarly, under R.A. 6715,36 employees who are illegally dismissed are entitled to full
backwages, inclusive of allowances and other benefits or their monetary equivalent, computed from the time
their actual compensation was withheld from them up to the time of their actual reinstatement but if
Settled is the rule in this jurisdiction that a corporation is invested by law with a legal personality separate and
reinstatement is no longer possible, the backwages shall be computed from the time of their illegal termination
distinct from those acting for and in its behalf and, in general, from the people comprising it. 31 Thus, obligations
up to the finality of the decision.37
incurred by corporate officers acting as corporate agents are not theirs but the direct accountabilities of the
corporation they represent.32 True, solidary liabilities may at times be incurred by corporate officers, but only
when exceptional circumstances so warrant. 33 For instance, in labor cases, corporate directors and officers may Since the circumstances obtaining in this case do not warrant private respondent's reinstatement in the light of
be held solidarily liable with the corporation for the termination of employment if done with malice or in bad the antagonism generated by this litigation which must have caused a severe strain in the parties' employer-
faith.34 employee relationship, an award of separation pay in lieu of reinstatement, equivalent to one month pay for
every year of service, in addition to full backwages, allowances, and other benefits or the monetary equivalent
thereof, is in order. The award of attorney's fees is sanctioned by law and must be upheld.
In the present case, the apparent basis for the NLRC in holding petitioner Maligro solidarily liable with Petron
were its findings that (1) the Investigation Committee was created a day after the summons in NLRC RAB-VII
Case No. 11-1439-96 was received, with Maligro no less being the chairman thereof; and (2) the basis for the WHEREFORE, the assailed Resolution of the Court of Appeals is SET ASIDE, and the NLRC decision dated July 31,
charge of insubordination was the private respondent's alleged making of false accusations against Maligro. 2000 is AFFIRMED with the MODIFICATION that (1) private respondent Chito S. Mantos is awarded separation
pay equivalent to one month pay for every year of service and full backwages, other privileges and benefits or
to the monetary equivalent thereof, computed from the date of his illegal dismissal on December 1, 1996 until
Those findings, however, cannot justify a finding of personal liability on the part of Maligro inasmuch as said
the finality of this decision; and (2) petitioner Peter C. Maligro is ABSOLVED from any liability adjudged against
findings do not point to Maligro's extreme personal hatred and animosity with the respondent. It cannot,
co-petitioner Petron Corporation.Costs against the petitioners.SO ORDERED.
therefore, be said that Maligro was motivated by malice and bad faith in connection with private respondent's
dismissal from the service.
SECOND DIVISION G.R. No. 149237 June 11, 2006

If at all, what said findings show are the illegality itself of private respondent's dismissal, the lack of just cause
therefor and the non-observance of procedural due process. Verily, the creation of the investigation committee CHINA BANKING CORPORATION, petitioner, vs. DYNE-SEM ELECTRONICS
and said committee's consideration of the insubordination charge against the private respondent, were merely CORPORATION, respondent.
aimed to cover up the illegal dismissal or to give it a semblance of legality.
Set V Corporation Code * Doctrine of Peircing Corpo Veil Cases*Page 57 of 164

DECISION 5.2 [n]ot one of the directors of [respondent] is, or has ever been, a director, officer, or stockholder of
Dynetics, Inc.;
CORONA, J.:
5.3 [t]he various facilities, machineries and equipment being used by [respondent] in its business
On June 19 and 26, 1985, Dynetics, Inc. (Dynetics) and Elpidio O. Lim borrowed a total of P8,939,000 from operations were legitimately and validly acquired, under arms-length transactions, from various
petitioner China Banking Corporation. The loan was evidenced by six promissory notes. 1 corporations which had become absolute owners thereof at the time of said transactions; these were
not just "taken over" nor "acquired from Dynetics" by [respondent], contrary to what plaintiff falsely
and maliciously alleges;
The borrowers failed to pay when the obligations became due. Petitioner consequently instituted a complaint
for sum of money2 on June 25, 1987 against them. The complaint sought payment of the unpaid promissory
notes plus interest and penalties. 5.4 [respondent] acquired most of its present machineries and equipment as second-hand items to
keep costs down;

Summons was not served on Dynetics, however, because it had already closed down. Lim, on the other hand,
filed his answer on December 15, 1987 denying that "he promised to pay [the obligations] jointly and severally 5.5 [t]he present plant site is under lease from Food Terminal, Inc., a government-controlled
to [petitioner]."3 corporation, and is located inside the FTI Complex in Taguig, Metro Manila, where a number of other
firms organized in 1986 and also engaged in the same or similar business have likewise established
their factories; practical convenience, and nothing else, was behind [respondents] choice of plant site;
On January 7, 1988, the case was scheduled for pre-trial with respect to Lim. The case against Dynetics was
archived.
5.6 [respondent] operates its own bonded warehouse under authority from the Bureau of Customs
which has the sole and absolute prerogative to authorize and assign customs bonded warehouses;
On September 23, 1988, an amended complaint4 was filed by petitioner impleading respondent Dyne-Sem
again, practical convenience played its role here since the warehouse in question was virtually lying
Electronics Corporation (Dyne-Sem) and its stockholders Vicente Chuidian, Antonio Garcia and Jacob Ratinoff.
idle and unused when said Bureau decided to assign it to [respondent] in June 1986. 6
According to petitioner, respondent was formed and organized to be Dynetics alter ego as established by the
following circumstances:
On February 28, 1989, the trial court issued an order archiving the case as to Chuidian, Garcia and Ratinoff
since summons had remained unserved.
Dynetics, Inc. and respondent are both engaged in the same line of business of manufacturing,
producing, assembling, processing, importing, exporting, buying, distributing, marketing and testing
integrated circuits and semiconductor devices; After hearing, the court a quo rendered a decision on December 27, 1991 which read:

[t]he principal office and factory site of Dynetics, Inc. located at Avocado Road, FTI Complex, Taguig, xxx [T]he Court rules that Dyne-Sem Electronics Corporation is not an alter ego of Dynetics, Inc. Thus,
Metro Manila, were used by respondent as its principal office and factory site; Dyne-Sem Electronics Corporation is not liable under the promissory notes.

[r]espondent acquired some of the machineries and equipment of Dynetics, Inc. from banks which xxx xxx xxx
acquired the same through foreclosure;
WHEREFORE, judgment is hereby rendered ordering Dynetics, Inc. and Elpidio O. Lim, jointly and
[r]espondent retained some of the officers of Dynetics, Inc. 5 severally, to pay plaintiff.

xxx xxx xxx xxx xxx xxx

On December 28, 1988, respondent filed its answer, alleging that: Anent the complaint against Dyne-Sem and the latters counterclaim, both are hereby dismissed,
without costs.SO ORDERED.7

5.1 [t]he incorporators as well as present stockholders of [respondent] are totally different from those
of Dynetics, Inc., and not one of them has ever been a stockholder or officer of the latter; From this adverse decision, petitioner appealed to the Court of Appeals 8 but the appellate court dismissed the
appeal and affirmed the trial courts decision.9 It found that respondent was indeed not an alter ego of
Dynetics. The two corporations had different articles of incorporation. Contrary to petitioners claim, no merger
Set V Corporation Code * Doctrine of Peircing Corpo Veil Cases*Page 58 of 164

or absorption took place between the two. What transpired was a mere sale of the assets of Dynetics to legitimate issues; or when the corporation is merely an adjunct, a business conduit or an alter ego of
respondent. The appellate court denied petitioners motion for reconsideration. 10 another corporation or where the corporation is so organized and controlled and its affairs are so
conducted as to make it merely an instrumentality, agency, conduit or adjunct of another corporation;
Hence, this petition for review11 with the following assigned errors: or when the corporation is used as a cloak or cover for fraud or illegality, or to work injustice, or where
necessary to achieve equity or for the protection of the creditors. In such cases, the corporation will be
considered as a mere association of persons. The liability will directly attach to the stockholders or to
VI.
the other corporation.

Issues
To disregard the separate juridical personality of a corporation, the wrongdoing must be proven clearly and
convincingly.18
What is the quantum of evidence needed for the trial court to determine if the veil of corporat[e]
fiction should be pierced?
In this case, petitioner failed to prove that Dyne-Sem was organized and controlled, and its affairs conducted, in
a manner that made it merely an instrumentality, agency, conduit or adjunct of Dynetics, or that it was
[W]hether or not the Regional Trial Court of Manila Branch 15 in its Decision dated December 27, 1991 established to defraud Dynetics creditors, including petitioner.
and the Court of Appeals in its Decision dated February 28, 2001 and Resolution dated July 27, 2001,
which affirmed en toto [Branch 15, Manila Regional Trial Courts decision,] have ruled in accordance
The similarity of business of the two corporations did not warrant a conclusion that respondent was but a
with law and/or applicable [jurisprudence] to the extent that the Doctrine of Piercing the Veil of
conduit of Dynetics. As we held in Umali v. Court of Appeals,19 "the mere fact that the businesses of two or
Corporat[e] Fiction is not applicable in the case at bar? 12
more corporations are interrelated is not a justification for disregarding their separate personalities, absent
sufficient showing that the corporate entity was purposely used as a shield to defraud creditors and third
We find no merit in the petition. persons of their rights."

The question of whether one corporation is merely an alter ego of another is purely one of fact. So is the Likewise, respondents acquisition of some of the machineries and equipment of Dynetics was not proof that
question of whether a corporation is a paper company, a sham or subterfuge or whether petitioner adduced the respondent was formed to defraud petitioner. As the Court of Appeals found, no merger 20 took place between
requisite quantum of evidence warranting the piercing of the veil of respondents corporate entity. This Court is Dynetics and respondent Dyne-Sem. What took place was a sale of the assets 21 of the former to the latter.
not a trier of facts. Findings of fact of the Court of Appeals, affirming those of the trial court, are final and Merger is legally distinct from a sale of assets. 22 Thus, where one corporation sells or otherwise transfers all its
conclusive. The jurisdiction of this Court in a petition for review on certiorari is limited to reviewing only errors assets to another corporation for value, the latter is not, by that fact alone, liable for the debts and liabilities of
of law, not of fact, unless it is shown, inter alia, that: (a) the conclusion is grounded entirely on speculations, the transferor.
surmises and conjectures; (b) the inference is manifestly mistaken, absurd and impossible; (c) there is grave
abuse of discretion; (d) the judgment is based on a misapplication of facts; (e) the findings of fact of the trial
Petitioner itself admits that respondent acquired the machineries and equipment not directly from Dynetics but
court and the appellate court are contradicted by the evidence on record and (f) the Court of Appeals went
from the various corporations which successfully bidded for them in an auction sale. The contracts of sale
beyond the issues of the case and its findings are contrary to the admissions of both parties. 13
executed between the winning bidders and respondent showed that the assets were sold for considerable
amounts.23 The Court of Appeals thus correctly ruled that the assets were not "diverted" to respondent as an
We have reviewed the records and found that the factual findings of the trial and appellate courts and alter ego of Dynetics.24 The machineries and equipment were transferred and disposed of by the winning
consequently their conclusions were supported by the evidence on record. bidders in their capacity as owners. The sales were therefore valid and the transfers of the properties to
respondent legal and not in any way in contravention of petitioners rights as Dynetics creditor.
The general rule is that a corporation has a personality separate and distinct from that of its stockholders and
other corporations to which it may be connected. 14 This is a fiction created by law for convenience and to Finally, it may be true that respondent later hired Dynetics former Vice-President Luvinia Maglaya and
prevent injustice.15 Assistant Corporate Counsel Virgilio Gesmundo. From this, however, we cannot conclude that respondent was
an alter ego of Dynetics. In fact, even the overlapping of incorporators and stockholders of two or more
Nevertheless, being a mere fiction of law, peculiar situations or valid grounds may exist to warrant the corporations will not necessarily lead to such inference and justify the piercing of the veil of corporate
disregard of its independent being and the piercing of the corporate veil. 16 In Martinez v. Court of Appeals,17 we fiction.25 Much more has to be proven.
held:
Premises considered, no factual and legal basis exists to hold respondent Dyne-Sem liable for the obligations of
The veil of separate corporate personality may be lifted when such personality is used to defeat public Dynetics to petitioner.WHEREFORE, the petition is hereby DENIED.The assailed Court of Appeals decision and
convenience, justify wrong, protect fraud or defend crime; or used as a shield to confuse the resolution in CA-G.R. CV No. 40672 are hereby AFFIRMED.Costs against petitioner.SO ORDERED.
Set V Corporation Code * Doctrine of Peircing Corpo Veil Cases*Page 59 of 164

FIRST DIVISIONG.R. No. 130998 August 10, 2001 damages or as the reasonable value of the services he rendered to petitioners, and further claimed twenty
percent (20%) of the amount recoverable as attorney's fees and the costs of suit.
MARUBENI CORPORATION, RYOICHI TANAKA, RYOHEI KIMURA and SHOICHI ONE, petitioners,
vs. FELIX LIRAG, respondent. In their answer, petitioners denied the consultancy agreement. Petitioner Ryohei Kimura did not have the
authority to enter into such agreement in behalf of Marubeni. Only Mr. Morihiko Maruyama, the general
PARDO, J.: manager, upon issuance of a special power of attorney by the principal office in Tokyo, Japan, could enter into
any contract in behalf of the corporation. Mr. Maruyama did not discuss with respondent Lirag any of the
matters alleged in the complaint, nor agreed to the payment of commission. Moreover, Marubeni did not
The case is an appeal via certiorari to annul and set aside the decision 1 of the Court of Appeals finding
participate in the bidding for the Bureau of Post project, nor benefited from the supposed project. Thus,
petitioners Ryoichi Tanaka, Ryohei Kimura and Shoichi One, as officers of petitioner Marubeni Corporation,
petitioners moved for the dismissal of the complaint.
jointly and severally liable with the corporation for the commission claimed by respondent Felix Lirag in the
amount of six million (P6,000,000.00) pesos arising from an oral consultancy agreement.
Petitioner Shoichi One submitted a separate answer raising similar arguments.

Petitioner Marubeni Corporation (hereafter, Marubeni) is a foreign corporation organized and existing under the
laws of Japan. It was doing business in the Philippines through its duly licensed, wholly owned subsidiary, With regard to petitioner Ryohei Kimura, the trial court did not acquire jurisdiction over his person because he
Marubeni Philippines Corporation. Petitioners Ryoichi Tanaka, Ryohei Kimura and Shoichi One were officers of was recalled to the principal office in Tokyo, Japan before the complaint and the summons could be served on
Marubeni assigned to its Philippine branch.2 him.

On January 27, 1989, respondent Felix Lirag filed with the Regional Trial Court, Makati a complaint 3 for specific During the pre-trial conferences held on September 18 and October 16, 1989 and on January 24, March 15 and
performance and damages claiming that petitioners owed him the sum of P6,000,000.00 representing May 17, 1990, no amicable settlement was reached. Trial on the merits ensued.
commission pursuant to an oral consultancy agreement with Marubeni. Lirag claimed that on February 2, 1987,
petitioner Ryohei Kimura hired his consultancy group for the purpose of obtaining government contracts of On April 29, 1993, the trial court promulgated a decision and ruled that respondent is entitled to a commission.
various projects. Petitioner Kimura authorized him to work on the following projects: (1) National Telephone Respondent was led to believe that there existed an oral consultancy agreement. Hence, he performed his part
Project, (2) Regional Telecommunications Project; (3) Cargo Handling Equipment; (4) Maritime Communications; of the agreement and helped petitioners get the project. The dispositive portion of the decision reads:
(5) Philippine National Railways Depot; and (6) Bureau of Posts (Phase II). 4 Petitioners promised to pay him six
percent (6%) consultancy fee based on the total costs of the projects obtained. "WHEREFORE, defendants are ordered, jointly and severally, to pay to the plaintiff: (1) the amount of
P6,000,000.00, with interest at the legal rate (12% per annum) from January 10, 1989 until fully paid;
The consultancy agreement was not reduced into writing because of the mutual trust between Marubeni and (2) 20% of this amount to serve as reimbursement of plaintiff's attorney's fees; and (3) to pay the cost
the Lirag family.5 Their close business and personal relationship dates back to 1960, when respondent's family of the suit.
was engaged in the textile fabric manufacturing business, in which Marubeni supplied the needed machinery,
equipment, spare parts and raw materials.6 "SO ORDERED.

In compliance with the agreement, respondent Lirag made representations with various government officials, "Makati, Metro Manila, April 29, 1993.
arranged for meetings and conferences, relayed pertinent information as well as submitted feasibility studies
and project proposals, including pertinent documents required by petitioners. As petitioners had been
impressed with respondent's performance, six (6) additional projects were given to his group under the same "[Original Signed]
undertaking.7 "SALVADOR P. DE GUZMAN, Jr.
"Pairing Judge"9

One of the projects handled by respondent Lirag, the Bureau of Post project, amounting to P100,000,000.00
was awarded to the "Marubeni-Sanritsu tandem." 8 Despite respondent's repeated formal verbal demands for On May 26, 1993, petitioners interposed an appeal from the decision to the Court of Appeals. 10
payment of the agreed consultancy fee, petitioners did not pay. In response to the first demand letter,
petitioners promised to reply within fifteen (15) days, but they did not do so. After due proceedings, on October 9, 1997, the Court of Appeals promulgated a decision affirming the decision
of the trial court. The Court of Appeals ruled that preponderance of evidence favored the existence of a
Pursuant to the consultancy agreement, respondent claimed a commission of six percent (6%) of the total consultancy agreement between the parties. It upheld the factual findings of the trial court, thus:
contract price, or a total of P6,000,000.00, or in the alternative, that he be paid the same amount by way of
Set V Corporation Code * Doctrine of Peircing Corpo Veil Cases*Page 60 of 164

"Plaintiff's evidence details the efforts he exerted after having been extended an appointment by As a general rule, factual findings of the Court of Appeals are conclusive on the parties and are not reviewed by
Marubeni as its consultant. He tendered a thanksgiving dinner for the defendants at the Nandau the Supreme Court and they carry even more weight when the Court of Appeals affirmed the factual findings
Restaurant; he and Napoleon Rama visited Marubeni's Morihiko Maruyama in the latter's office during of the trial court. It is not the function of the Supreme Court to weigh anew the evidence passed upon by the
which they discussed the BOP II project. He arranged several conferences between the Marubeni Court of Appeals.19 Moreover, only questions of law may be raised before the Supreme Court in a petition for
officials and Postmaster General Angelito Banayo. In one meeting which took place in the office of Mr. review under Rule 45 of the Revised Rules of Court.20
Banayo at Liwasang Bonifacio, a Mr. Ida, the General Manager of Sanritsu, was conspicuously present.
Mr. Banayo testified that Mr. Ida told him that Sanritsu was representing Marubeni in the BOP II project However, the rule is subject to exceptions, 21 such as when the conclusion is grounded on speculation, surmises,
(tsn., 6/11/90, pp. 15-17; 5/15/91, pp. 10-12). At least thirty (30) conferences between plaintiff and or conjectures,22 as in the instant case.
defendants took place at the Marubeni offices, lasting at least two hours each meeting. Eventually, the
bid was awarded by the Bureau of Post to Sanritsu. Aware that Sanritsu represented Marubeni, and in
An assiduous scrutiny of the testimonial and documentary evidence extant leads us to the conclusion that the
fact Marubeni assigned Sanritsu to enter its bid, plaintiff sent his bill for his services to the defendants
evidence could not support a solid conclusion that a consultancy agreement, oral or written, was agreed
in a letter dated April 20, 1988. This was followed by a letter dated September 26, 1990 of plaintiff's
between petitioners and respondent. Respondent attempted to fortify his own testimony by presenting several
counsel. This time Mr. Tanaka asked for 15 days within which to contact their Head Office to seek
corroborative witnesses. However, what was apparent in the testimonies of these witnesses was the fact that
instructions."11
they learned about the existence of the consultancy agreement only because that was what respondent told
them.23
The Court of Appeals relied on the doctrine of admission by silence 12 in upholding the existence of a
consultancy agreement, noting that petitioner Tanaka's reaction to respondent's September 26, 1988 demand
In civil cases, he who alleges a fact has the burden of proving it; a mere allegation is not evidence. 24 He must
letter was not consistent with their claim that there was no consultancy agreement. On the contrary, it lent
establish his cause by a preponderance of evidence, 25 which respondent failed to establish in the instant case.
credence to respondent's claim that they had an existing consultancy agreement. Petitioner Tanaka's response
dated October 13, 1988 to the demand letter of September 26, 1988 reads:
Assuming for the sake of argument that an oral consultancy agreement has been perfected between the
parties, respondent Lirag could not still claim fees on the project that has not been awarded to Marubeni.
"Referring to your letter dated September 26, 1988, we are pleased to inform you that the issue is
currently being reviewed by us and we would like to reply to you within fifteen (15) days." 13
If respondent's contentions were to be taken as truth, he would be entitled to 6% consulting fee based on the
total cost of the projects obtained,26 or on success basis. 27 However, even respondent admitted that the Bureau
The Court of Appeals observed that if indeed there were no consultancy agreement, it would have been easy
of Post project was not awarded to Marubeni, but to Sanritsu. 28 Marubeni did not even join the bidding for the
for petitioners to simply deny respondent's claim. Yet, they did not do so. The conglomeration of these
Bureau of Post project.
circumstances bolstered the existence of the oral consultancy agreement. The dispositive portion of the
decision reads:
Respondent could not claim from Sanritsu because of the absence of any agreement between him and the
latter. When asked to clarify whether he has an existing consultancy agreement with Sanritsu, respondent
"WHEREFORE, the decision appealed from is hereby AFFIRMED." 14
answered in the negative, thus:

"COURT:

Hence, this appeal.15


One clarificatory question

In this appeal, petitioners raise the following issues: (1) whether or not there was a consultancy agreement
Do you have any consultancy service contract with Marubeni/San Ritsu do you have?
between petitioners and respondent; and corollary to this, (2) whether or not respondent is entitled to receive a
commission if there was, in fact, a consultancy agreement. 16
A: No, sir. I have only Consultancy Agreement on verbal basis with Marubeni." 29
We find the appeal meritorious.
Hence, how could he be entitled to the 6% commission, when it was not his client who won in the bidding?
In deciding this appeal, we rely on the rule that a party who has the burden of proof in a civil case must
establish his case by a preponderance of evidence. 17 When the evidence of the parties is in equipoise, or when Respondent tried to justify his commission of roughly about P6,000,000.00 in the guise that Marubeni and
there is a doubt as to where the preponderance of evidence lies, the party with the burden of proof fails and Sanritsu are sister corporations, thereby implying the need to pierce the veil of corporate fiction. Respondent
the petition must thus be denied.18
Set V Corporation Code * Doctrine of Peircing Corpo Veil Cases*Page 61 of 164

claimed that Marubeni as the supplier and real contractor of the project hired and sub-contracted the project to to hire consultants, it would then give a power of attorney to its general manager in Manila authorizing the
Sanritsu. latter to enter into such agreement.

We believe that this line of reasoning is too far-fetched. Not because two foreign companies came from the In the instant case, the parties did not reach the second stage as the headquarters in Tokyo, Japan did not see
same country and closely worked together on certain projects would the conclusion arise that one was the it fit to hire a consultant as they decided not to participate in the bidding. Hence, no consultancy agreement
conduit of the other, thus piercing the veil of corporate fiction. was perfected, whether oral or written. There was no absolute acceptance of respondent's offer of consultancy
services.
To disregard the separate juridical personality of a corporation, the wrongdoing must be clearly and
convincingly established. It cannot be presumed. The separate personality of the corporation may be Assuming arguendo that the petitioner accepted respondent's offer of consultancy services, we could not give
disregarded only when the corporation is used as a cloak or cover for fraud or illegality, or to work injustice, or legal imprimatur to the agreement. The service rendered by respondent contemplated the exploitation of
where necessary for the protection of creditors. 30 We could not just rely on respondent's testimony regarding personal influence and solicitation on a public officer.
the existence of the "Marubeni-Sanritsu tandem" to justify his claim for payment of commission. This
conclusion is too conjectural to be believed. Respondent said that petitioners sought out his services because they "needed somebody who can help them
'penetrate' and establish goodwill" with the government.34 Petitioners found it difficult to arrange a meeting
Aside from the self-serving testimony of respondent regarding the existence of a close working relationship with Postmaster General Angelito Banayo because of petitioners' reputation of engaging in questionable
between Marubeni and Sanritsu, there was nothing that would support the conclusion that Sanritsu was an transactions.35 Suddenly, through the intervention of respondent, the postmaster general became accessible to
agent of Marubeni. Mr. Lito Banayo, whom respondent presented to corroborate his testimony on this particular petitioners. This became possible because of respondent's close personal relationship with the postmaster
issue said, thus: general, his trusted and long-time friend.36 Respondent testified, to wit:

"ATTY. VALERO "Q: In other words you are saying that Marubeni and San Ritsu representatives had a conference
with the Post Master General Banayo in connection with this Project?
My question is do you know for a fact whether the impression you have about Japanese
Trading Firm working through Agents was the relationship between Marubeni and San Ritsu "A: Yes and I was the one who made the arrangement." 37
when Mr. Ida said that they were working together?
In another instance, respondent said, thus:
"A: I did not know for a fact because I did not see any contract between Marubeni and San Ritsu
presented to me."31 "WITNESS:

Contrary to the trial court's finding that petitioners led respondent to believe that they hired respondent's What we have done by that . . . first, Mr. Banayo went to Tokyo and when he was in Tokyo we
services as consultant, the evidence proved otherwise. Petitioner Shoichi One, one of the officers of Marubeni were able to arrange the Marubeni representative in Tokyo to meet and talk with Mr. Banayo
Phils., testified that at the onset, Marubeni Phils. informed respondent that it had no authority to commit to in Tokyo . . .
anything, as it all depended on the decision of the principal headquarters in Tokyo, Japan. However, respondent
Lirag insisted on providing assistance to Marubeni to get coveted government contracts because Marubeni
"COURT:
might encounter difficulties due to discrimination from the government. 32 Despite such knowledge, respondent
said that "it's alright" with him as he "believes Marubeni was an old time friend so he wanted to work for those
projects."33 Hence, how could petitioners be guilty of misleading respondent on the acceptance of the latter's Mr. . . . ?
offer of consultancy service?
"A. . . . Banayo, the Post Master General and representatives of Marubeni in Tokyo this was
With regard to the Court of Appeal's ratiocination that petitioner Tanaka's response dated October 13, 1988 to done because of my intervention." 38
the demand letter of September 26, 1988, amounted to an implied admission of the consultancy agreement,
the records showed that, to the contrary, this fact strengthened petitioners' allegation that Marubeni Phils.
lacked the requisite authority to enter into any binding agreement.

Any agreement entered into because of the actual or supposed influence which the party has, engaging him to
As explained by petitioner Shoichi One, Marubeni Phils. could enter into a consultancy agreement only after influence executive officials in the discharge of their duties, which contemplates the use of personal influence
submitting a recommendation to the principal headquarters in Tokyo, Japan. If the office in Tokyo, Japan agrees
Set V Corporation Code * Doctrine of Peircing Corpo Veil Cases*Page 62 of 164

and solicitation rather than an appeal to the judgment of the official on the merits of the object sought is On 26 August 1968, owing to Cardale's failure to settle its mortgage obligation, Gutierrez filed a complaint for
contrary to public policy. 39 Consequently, the agreement, assuming that the parties agreed to the consultancy, rescission of the contract with the Quezon City Regional Trial Court (RTC), which was docketed as Civil Case No.
is null and void as against public policy.40 Therefore, it is unenforceable before a court of justice.41 Q-12366.2 On 20 October 1969, during the pendency of the rescission case, Gutierrez died and was substituted
by her executrix, respondent Rita C. Mejia (Mejia). In 1971, plaintiff's presentation of evidence was terminated.
In light of the foregoing, we rule that the preponderance of evidence established no consultancy agreement However, Cardale, which was represented by petitioner Adalia B. Francisco (Francisco) in her capacity as Vice-
between petitioners and respondent from which the latter could anchor his claim for a six percent (6%) President and Treasurer of Cardale, lost interest in proceeding with the presentation of its evidence and the
consultancy fee on a project that was not awarded to petitioners.WHEREFORE, the petition is GRANTED. The case lapsed into inactive status for a period of about fourteen years.
decision of the Court of Appeals 42 is hereby SET ASIDE. Civil Case No. 89-3037 filed before the Regional Trial
Court, Branch 143, Makati City is hereby DISMISSED.No costs.SO ORDERED. In the meantime, the mortgaged parcels of land covered by TCT Nos. 7532 and 7533 became delinquent in the
payment of real estate taxes in the amount of P102,300.00, while the other mortgaged property covered by
THIRD DIVISION G.R. No. 141617 August 14, 2001 TCT No. 7531 became delinquent in the amount of P89,231.37, which culminated in their levy and auction sale
on 1 and 12 September 1983, in satisfaction of the tax arrears. The highest bidder for the three parcels of land
was petitioner Merryland Development Corporation (Merryland), whose President and majority stockholder is
ADALIA B. FRANCISCO and MERRYLAND DEVELOPMENT CORPORATION, petitioners, vs.
Francisco. A memorandum based upon the certificate of sale was then made upon the original copies of TCT
RITA C. MEJIA, as Executrix of Testate Estate of ANDREA CORDOVA VDA. DE GUTERREZ, respondent.
Nos. 7531 to 7533.

GONZAGA-REYES,J.:
On 13 August 1984, before the expiration of the one year redemption period, Mejia filed a Motion for Decision
with the trial court. The hearing of said motion was deferred, however, due to a Motion for Postponement filed
by Cardale through Francisco, who signed the motion in her capacity as "officer-in-charge," claiming that
Cardale needed time to hire new counsel. However, Francisco did not mention the tax delinquencies and sale in
In this petition for review by certiorari, petitioners pray for the setting aside of the Decision of the Court favor of Merryland. Subsequently, the redemption period expired and Merryland, acting through Francisco, filed
Appeals promulgated on 13 April 1999 and its 15 December 1999 Resolution in CA-G.R. CV No. 19281. petitions for consolidation of title,3 which culminated in the issuance of certain orders 4 decreeing the
cancellation of Cardales' TCT Nos. 7531 to 7533 and the issuance of new transfer certificates of title "free from
any encumbrance or third-party claim whatsoever" in favor of Merryland. Pursuant to such orders, the Register
As culled from the decisions of the lower courts and the pleadings of the parties, the factual background of this
of Deeds of Caloocan City issued new transfer certificates of title in the name of Merryland which did not bear a
case is as set out herein:
memorandum of the mortgage liens in favor of Gutierrez.

Andrea Cordova Vda. de Gutierrez (Gutierrez) was the registered owner of a parcel of land in Camarin, Caloocan
Thereafter, sometime in June 1985, Francisco filed in Civil Case No. Q-12366 an undated Manifestation to the
City known as Lot 861 of the Tala Estate. The land had an aggregate area of twenty-five (25) hectares and was
effect that the properties subject of the mortgage and covered by TCT Nos. 7531 to 7533 had been levied upon
covered by Transfer Certificate of Title (TCT) No. 5779 of the Registry of Deeds of Caloocan City. The property
by the local government of Caloocan City and sold at a tax delinquency sale. Francisco further claimed that the
was later subdivided into five lots with an area of five hectares each and pursuant thereto, TCT No. 5779 was
delinquency sale had rendered the issues in Civil Case No. Q-12366 moot and academic. Agreeing with
cancelled and five new transfer certificates of title were issued in the name of Gutierrez, namely TCT No. 7123
Francisco, the trial court dismissed the case, explaining that since the properties mortgaged to Cardale had
covering Lot 861-A, TCT No. 7124 covering Lot 861-B, TCT No. 7125 covering Lot 861-C, TCT No. 7126 covering
been transferred to Merryland which was not a party to the case for rescission, it would be more appropriate for
Lot 861-D and TCT No. 7127 covering Lot 861-E.
the parties to resolve their controversy in another action.

On 21 December 1964, Gutierrez and Cardale Financing and Realty Corporation (Cardale) executed a Deed of
On 14 January 1987, Mejia, in her capacity as executrix of the Estate of Gutierrez, filed with the RTC of Quezon
Sale with Mortgage relating to the lots covered by TCT Nos. 7124, 7125, 7126 and 7127, for the consideration
City a complaint for damages with prayer for preliminary attachment against Francisco, Merryland and the
of P800,000.00. Upon the execution of the deed, Cardale paid Gutierrez P171,000.00. It was agreed that the
Register of Deeds of Caloocan City. The case was docketed as Civil Case No. Q-49766. On 15 April 1988, the
balance of P629,000.00 would be paid in several installments within five years from the date of the deed, at an
trial court rendered a decision 5 in favor of the defendants, dismissing the complaint for damages filed by Mejia.
interest of nine percent per annum "based on the successive unpaid principal balances." Thereafter, the titles
It was held that plaintiff Mejia, as executrix of Gutierrez's estate, failed to establish by clear and convincing
of Gutierrez were cancelled and in lieu thereof TCT Nos. 7531 to 7534 were issued in favor of Cardale.
evidence her allegations that Francisco controlled Cardale and Merryland and that she had employed fraud by
intentionally causing Cardale to default in its payment of real property taxes on the mortgaged properties so
To secure payment of the balance of the purchase price, Cardale constituted a mortgage on three of the four that Merryland could purchase the same by means of a tax delinquency sale. Moreover, according to the trial
parcels of land covered by TCT Nos. 7531, 7532 and 7533, encompassing fifteen hectares of land. 1 The court, the failure to recover the property subject of the Deed of Sale with Mortgage was due to Mejia's failure to
encumbrance was annotated upon the certificates of title and the owner's duplicate certificates. The owner's actively pursue the action for rescission (Civil Case No. 12366), allowing the case to drag on for eighteen years.
duplicates were retained by Gutierrez. Thus, it ruled that
Set V Corporation Code * Doctrine of Peircing Corpo Veil Cases*Page 63 of 164

xxx xxx xxx . . . Had the plaintiff not slept on her rights and had it not been for her failure to perform her commensurate
duty to pursue vigorously her case against Cardale Financing and Realty Corporation in said Civil Case No.
The act of not paying or failing to pay taxes due the government by the defendant Adalia B. Francisco, as 12366, she could have easily known said non-payment of realty taxes on the said properties by said Cardale
treasurer of Cardale Financing and Realty Corporation do not, per se, constitute perpetration of fraud or an Financing and Realty Corporation, or, at least the auction sales that followed, and from which she could have
illegal act. It do [sic] not also constitute an act of evasion of an existing obligation (to plaintiff) if there is no redeemed said properties within the one year period provided by law, or, have availed of remedies at the time
clear showing that such an act of non-payment of taxes was deliberately made despite its (Cardale's) solvency to protect the interest of the testate estate of the late Andrea Cordova Vda. de Gutierrez.
and capability to pay. There is no evidence showing that Cardale Financing and Realty Corporation was
financially capable of paying said taxes at the time. xxx xxx xxx

"There are times when the corporate fiction will be disregarded: (1) where all the members or The dispositive portion of the trial court's decision states
stockholders commit illegal act; (2) where the corporation is used as dummy to commit fraud or
wrong; (3) where the corporation is an agency for a parent corporation; and (4) where the stock of a WHEREFORE, in view of all the foregoing consideration, the court hereby renders judgment in favor of
corporation is owned by one person." (I, Fletcher, 58, 59, 61 and 63). None of the foregoing reasons the defendants Register of Deeds of Caloocan City, Merryland Development Corporation and Adalia B.
can be applied to the incidents in this case: (1) there appears no illegal act committed by the Francisco, and against plaintiff Rita C. Mejia, as Executrix of the Testate Estate of Andrea Cordova Vda.
stockholders of defendant Merryland Development Corporation and Cardale Financing and Realty De Gutierrez, and hereby orders:
Corporation; (2) the incidents proven by evidence of the plaintiff as well as that of the defendants do
not show that either or both corporations were used as dummies by defendant Adalia B. Francisco to
1. That this case for damages be dismissed, at the same time, plaintiffs motion for reconsideration
commit fraud or wrong. To be used as [a] dummy, there has to be a showing that the dummy
dated September 23, 1987 is denied;
corporation is controlled by the person using it. The evidence of plaintiff failed to prove that defendant
Adalia B. Francisco has controlling interest in either or both corporations. On the other hand, the
evidence of defendants clearly show that defendant Francisco has no control over either of the two 2. Plaintiff pay the defendants Merryland Development Corporation and the Register of Deeds the sum
corporations; (3) none of the two corporations appears to be an agency for a parent (the other) of P20,000.00, and another sum of P20,000.00 to the defendant Adalia B. Francisco, as and for
corporation; and (4) the stock of either of the two corporation [sic] is not owned by one person attorney's fees and litigation expenses, and pay the costs of the proceedings.SO ORDERED.
(defendant Adalia B. Francisco). Except for defendant Adalia B. Francisco, the incorporators and
stockholders of one corporation are different from the other. The Court of Appeals,6 in its decision7 promulgated on 13 April 1999, reversed the trial court, holding that the
corporate veil of Cardale and Merryland must be pierced in order to hold Francisco and Merryland solidarily
xxx xxx xxx liable since these two corporations were used as dummies by Francisco, who employed fraud in allowing
Cardale to default on the realty taxes for the properties mortgaged to Gutierrez so that Merryland could acquire
the same free from all liens and encumbrances in the tax delinquency sale and, as a consequence thereof,
The said case (Civil Case No. 12366) remained pending for almost 18 years before the then Court of First
frustrating Gutierrez's rights as a mortgagee over the subject properties. Thus, the Court of Appeals premised
Instance, now the Regional Trial Court. Even if the trial of the said case became protracted on account of the
its findings of fraud on the following circumstances
retirement and/or promotion of the presiding judge, as well as the transfer of the case from one sala to another,
and as claimed by the plaintiff "that the defendant lost interest", (which allegation is unusual, so to speak), the
court believe [sic] that it would not have taken that long to dispose [of] said case had plaintiff not slept on her xxx xxx xxx
rights, and her duty and obligation to see to it that the case is always set for hearing so that it may be
adjudicated [at] the earliest possible time. This duty pertains to both parties, but plaintiff should have been . . . Appellee Francisco knew that Cardale of which she was vice-president and treasurer had an
more assertive, as it was her obligation, similar to the obligation of plaintiff relative to the service of summons outstanding obligation to Gutierrez for the unpaid balance of the real properties covered by TCT Nos.
in other cases. The fact that Cardale Financing and Realty Corporation did not perform its obligation as 7531 to 7533, which Cardale purchased from Gutierrez which account, as of December 1988, already
provided in the said "Deed of Sale with Mortgage" (Exhibit"A") is very clear. Likewise, the fact that Andrea amounted to P4,414,271.43 (Exh. K, pp. 39-44, record); she also knew that Gutierrez had a mortgage
Cordova, the contracting party, represented by the plaintiff in this case, did not also perform her duties and/or lien on the said properties to secure payment of the aforesaid obligation; she likewise knew that the
obligation provided in the said contract is also clear. This could have been the reason why the plaintiff in said said mortgaged properties were under litigation in Civil Case No. Q-12366 which was an action filed by
case (Exhibit "E") slept on her rights and allowed the same to remain pending for almost 18 years. However, Gutierrez against Cardale for rescission of the sale and/or recovery of said properties (Exh. E). Despite
and irrespective of any other reason behind the same, the court believes that plaintiff, indeed, is the one to such knowledge, appellee Francisco did not inform Gutierrez's Estate or the Executrix (herein
blame for the failure of the testate estate of the late Andrea Cordova Vda. de Gutierrez to recover the money or appellant) as well as the trial court that the mortgaged properties had incurred tax delinquencies, and
property due it on the basis of Exhibit "A". that Final Notices dated July 9, 1982 had been sent by the City Treasurer of Caloocan demanding
payment of such tax arrears within ten (10) days from receipt thereof (Exhs. J & J-1, pp. 37-38, record).
xxx xxx xxx Both notices which were addressed to
Set V Corporation Code * Doctrine of Peircing Corpo Veil Cases*Page 64 of 164

Cardale Financing & Realty Corporation c/o Merryland Development Corporation August 13, 1984 a Motion for Decision in the aforesaid case, appellee Francisco moved to defer
consideration of appellant's Motion on the pretext that defendant Cardale needed time to employ
and sent to appellee Francisco's address at 83 Katipunan Road, White Plains, Quezon City, gave warning that if another counsel. Significantly, in her aforesaid Motion for Postponement dated August 16, 1984 which
the taxes were not paid within the aforesaid period, the properties would be sold at public auction to satisfy the appellee Francisco personally signed as Officer-in-Charge of Cardale, she also did not disclose the fact
tax delinquencies. that the properties subject matter of the case had long been sold at a tax delinquency sale and
acquired by her other corporation Merryland.

To reiterate, notwithstanding receipt of the aforesaid notices, appellee Francisco did not inform the Estate of
Gutierrez or her executrix about the tax delinquencies and of the impending auction sale of the said properties. And as if what she had already accomplished were not enough fraudulence, appellee Francisco, acting
Even a modicum of good faith and fair play should have encouraged appellee Francisco to at least advise in behalf of Merryland, caused the issuance of new transfer certificates of title in the name of
Gutierrez's Estate through her executrix (herein appellant) and the trial court which was hearing the complaint Merryland, which did not anymore bear the mortgage lien in favor of Gutierrez. In the meantime, to
for rescission and recovery of said properties of such fact, so that the Estate of Gutierrez, which had a real further avoid payment of the mortgage indebtedness owing to Gutierrez's estate, Cardale corporation
interest on the properties as mortgagee and as plaintiff in the rescission and recovery suit, could at least take was dissolved. Finally, to put the properties beyond the reach of the mortgagee, Gutierrez's estate,
steps to forestall the auction sale and thereby preserve the properties and protect its interests thereon. And not Merryland caused the subdivision of such properties, which were subsequently sold on installment
only did appellee Francisco allow the auction sale to take place, but she used her other corporation (Merryland) basis.
in participating in the auction sale and in acquiring the very properties which her first corporation (Cardale) had
mortgaged to Gutierrez. Again, appellee Francisco did not thereafter inform the Estate of Gutierrez or its In its petition for certiorari, petitioners argue that there is no law requiring the mortgagor to inform the
executrix (herein appellant) about the auction sale, thus precluding the Estate from exercising its right of mortgagee of the tax delinquencies, if any, of the mortgaged properties. Moreover, petitioners claim that
redemption. And it was only after the expiration of the redemption period that appellee Francisco filed a Cardale's failure to pay the realty taxes, per se, does not constitute fraud since it was not proven that Cardale
Manifestation in Civil Case No. Q-12366 (Exh. 1, p. 36, record), in which she disclosed for the first time to the was capable of paying the taxes' Petitioners also contend that if Mejia, as executrix of Gutierrez's estate, was
trial court and appellant that the properties subject of the case and on which Gutierrez or her Estate had a not remiss in her duty to pursue Civil Case No. 12366, she could have easily learned of the non-payment of
mortgage lien, had been sold in a tax delinquency sale. And in order to further conceal her deceptive realty taxes on the subject properties and of the auction sale that followed and thus, have redeemed the
maneuver, appellee Francisco did not divulge in her aforesaid Manifestation that it was her other corporation properties or availed of some other remedy to conserve the estate of Gutierrez. In addition, Mejia could have
(Merryland) that acquired the properties in the auction sale. annotated a notice oflis pendens on the titles of the mortgaged properties, but she failed to do so. It is the
stand of petitioners that respondent has not adduced any proof that Francisco controlled both Cardale and
We are not impressed by appellee's submission that no evidence was adduced to prove that Cardale Merryland and that she used these two corporations to perpetuate a fraud upon Gutierrez or her estate.
had the capacity to pay the tax arrears and therefore she or Cardale may not be faulted for the tax Petitioners maintain that the "evidence shows that, apart form the meager share of petitioner Francisco, the
delinquency sale of the properties in question. Appellee Francisco's bad faith or deception did not stockholdings of both corporations comprise other shareholders, and the stockholders of either of them, aside
necessarily lie in Cardale's or her failure to settle the tax deliquencies in question, but in not disclosing from petitioner Francisco, are composed of different persons." As to Civil Case No. 12366, petitioners insist that
to Gutierrez's estate or its executrix (herein appellant) which had a mortgage lien on said properties the decision of the trial court in that case constitutesres judicata to the instant case.8
the tax delinquencies and the impending auction sale of the encumbered properties.
It is dicta in corporation law that a corporation is a juridical person with a separate and distinct personality from
Appellee Francisco's deception is further shown by her concealment of the tax delinquency sale of the mat of the stockholders or members who compose it 9 However, when the legal fiction of the separate corporate
properties from the estate or its executrix, thus preventing the latter from availing of the right of personality is abused, such as when the same is used for fraudulent or wrongful ends, the courts have not
redemption of said properties. That appellee Francisco divulged the auction sale of the properties only hesitated to pierce the corporate veil. One of the earliest formulations of this doctrine of piercing the corporate
after such redemption period had lapsed clearly betrays her intention to keep Gutierrez's Estate or its veil was made in the American case ofUnited States v. Milwaukee Refrigerator Transit Co.10
Executrix from availing of such right. And as the evidence would further show, appellee Francisco had
a hand in securing for Merryland consolidation of its ownership of the properties and in seeing to it If any general rule can be laid down, in the present state of authority, it is that a corporation will be
that Merryland's torrens certificates for the properties were free from liens and encumbrances. All looked upon as a legal entity as a general rule, and until sufficient reason to the contrary appears; but,
these appellee Francisco did even as she was fully aware that Gutierrez or her estate had a valid and when the notion of legal entity is used to defeat public convenience, justify wrong, protect fraud, or
subsisting mortgage lien on the said properties. defend crime, the law will regard the corporation as an association of persons.

It is likewise worthy of note that early on appellee Francisco had testified in the action for rescission of Since then a good number of cases have firmly implanted this doctrine in Philippine jurisprudence. 11 One such
sale and recovery of possession and ownership of the properties which Gutierrez filed against Cardale case isUmali v. Court of Appeals12 wherein the Court declared that
(Civil Case No. Q-12366) in her capacity as defendant Cardale's vice-president and treasurer. But then,
for no plausible reason whatsoever, she lost interest in continuing with the presentation of evidence Under the doctrine of piercing the veil of corporate entity, when valid grounds therefore exist, the
for defendant Cardale. And then, when appellant Mejia as executrix of Gutierrez's Estate filed on legal fiction that a corporation is an entity with a juridical personality separate and distinct from its
Set V Corporation Code * Doctrine of Peircing Corpo Veil Cases*Page 65 of 164

members or stockholders may be disregarded. In such cases, the corporation will be considered as a made no attempts to redeem the mortgaged property during this time. Moreover, instead of informing Mejia or
mere association of persons. The members or stockholders of the corporation will be considered as the the trial court in Q-12366 about the tax sale, the records show that Francisco filed a Motion for
corporation, that is, liability will attach directly to the officers and stockholders. The doctrine applies Postponement27 in behalf of Cardale even signing the motion in her capacity as "officer-in-charge" which
when the corporate fiction is used to defeat public convenience, justify wrong, protect fraud, or defend worked to defer the hearing of Mejia's Motion for Decision. No mention was made by Francisco of the tax sale in
crime, or when it is made as a shield to confuse the legitimate issues, or where a corporation is the the motion for postponement. Only after the redemption period had expired did Francisco decide to reveal what
merealter ego or business conduit of a person, or where the corporation is so organized and controlled had transpired by filing a Manifestation stating that the properties subject of the mortgage in favor of Gutierrez
and its affairs are so conducted as to make it merely an instrumentality, agency, conduit or adjunct of had been sold at a tax delinquency sale; however, Francisco failed to mention that it was Merryland that
another corporation. acquired the properties since she was probably afraid that if she did so the court would see behind her
fraudulent scheme. In this regard, it is also significant to note that it was Francisco herself who filed the
With specific regard to corporate officers, the general rule is that the officer cannot be held personally liable petitions for consolidation of title and who helped secure for Merryland titles over the subject properties "free
with the corporation, whether civilly or otherwise, for the consequences of his acts, if he acted for and in behalf from any encumbrance or third-party claim whatsoever."
of the corporation, within the scope of his authority and in good faith. In such cases, the officer's acts are
properly attributed to the corporation.13 However, if it is proven that the officer has used the corporate fiction to It is exceedingly apparent to the Court that the totality of Francisco's actions clearly betray an intention to
defraud a third party, 14 or that he has acted negligently, maliciously or in bad faith, 15 then the corporate veil conceal the tax delinquencies, levy and public auction of the subject properties from the estate of Gutierrez
shall be lifted and he shall be held personally liable for the particular corporate obligation involved. and the trial court in Civil Case No. Q-12366 until after the expiration of the redemption period when the
remotest possibility for the recovery of the properties would be extinguished. 28 Consequently, Francisco had
The Court, after an assiduous study of this case, is convinced that the totality of the circumstances effectively deprived the estate of Gutierrez of its rights as mortgagee over the three parcels of land which were
appertaining conduce to the inevitable conclusion that petitioner Francisco acted in bad faith. The events sold to Cardale. If Francisco was acting in good faith, then she should have disclosed the status of the
leading up to the loss by the Gutierrez estate of its mortgage security attest to this. It has been established mortgaged properties to the trial court in Civil Case No. Q-12366 especially after Mejia had filed a Motion for
that Cardale failed to comply with its obligation to pay the balance of the purchase price for the four parcels of Decision, in response to which she filed a motion for postponement wherein she could easily have mentioned
land it bought from Gutierrez covered by TCT Nos. 7531 to 7534, which obligation was secured by a mortgage the tax sale since this action directly affected such properties which were the subject of both the sale and
upon the lands covered by TCT Nos. 7531, 7532 and 7533. This prompted Gutierrez to file an action for mortgage.
rescission of the Deed of Sale with Mortgage (Civil Case No. Q-12366), but the case dragged on for about
fourteen years when Cardale, as represented by Francisco, who was Vice-President and Treasurer of the That Merryland acquired the property at the public auction only serves to shed more light upon Francisco's
same,16 lost interest in completing its presentation of evidence. fraudulent purposes. Based on the findings of the Court of Appeals, Francisco is the controlling stockholder and
President of Merryland.29 Thus, aside from the instrumental role she played as an officer of Cardale, in evading
Even before 1984 when Mejia, in her capacity as executrix of Gutierrez's estate, filed a Motion for Decision with that corporation's legitimate obligations to Gutierrez, it appears that Francisco's actions were also oriented
the trial court, there is no question that Francisco knew that the properties subject of the mortgage had towards securing advantages for another corporation in which she had a substantial interest. We cannot agree,
become tax delinquent. In fact, as treasurer of Cardale, Francisco herself was the officer charged with the however, with the Court of Appeals' decision to hold Merryland solidarily liable with Francisco. The only act
responsibility of paying the realty taxes on the corporation's properties. This was admitted by the trial court in imputable to Merryland in relation to the mortgaged properties is that it purchased the same and this by itself
its decision.17 In addition, notices dated 9 July 1982 from the City Treasurer of Caloocan demanding payment of is not a fraudulent or wrongful act. No evidence has been adduced to establish that Merryland was a mere alter
the tax arrears on the subject properties and giving warning that if the realty taxes were not paid within the ego or business conduit of Francisco. Time and again it has been reiterated that mere ownership by a single
given period then such properties would be sold at public auction to satisfy the tax delinquencies were sent stockholder or by another corporation of all or nearly all of the capital stock of a corporation is not of itself
directly to Francisco's address in White Plains, Quezon City. 18 Thus, as early as 1982, Francisco could have sufficient ground for disregarding the separate corporate personality. 30 Neither has it been alleged or proven
informed the Gutierrez estate or the trial court in Civil Case No. Q-12366 of the tax arrears and of the notice that Merryland is so organized and controlled and its affairs are so conducted as to make it merely an
from the City Treasurer so that the estate could have taken the necessary steps to prevent the auction sale and instrumentality, agency, conduit or adjunct of Cardale. 31 Even assuming that the businesses of Cardale and
to protect its interests in the mortgaged properties, but she did no such thing. Finally, in 1983, the properties Merryland are interrelated, this alone is not justification for disregarding their separate personalities, absent
were levied upon and sold at public auction wherein Merryland a corporation where Francisco is a any showing that Merryland was purposely used as a shield to defraud creditors and third persons of their
stockholder19 and concurrently acts as President and director 20 was the highest bidder. rights.32 Thus, Merryland's separate juridical personality must be upheld.

When Mejia filed the Motion for Decision in Civil Case No. Q-12366, 21 the period for redeeming the properties Based on a statement of account submitted by Mejia, the Court of Appeals awarded P4,314,271.43 in favor of
subject of the tax sale had not yet expired. 22 Under the Realty Property Tax Code, 23 pursuant to which the tax the estate of Gutierrez which represents the unpaid balance of the purchase price in the amount of
levy and sale were prosecuted, 24 both the delinquent taxpayer and in his absence, any person holding a lien or P629,000.00 with an interest rate of nine percent (9% ) per annum, in accordance with the agreement of the
claim over the property shall have the right to redeem the property within one year from the date of parties under the Deed of Sale with Mortgage, 33 as of December 1988.34 Therefore, in addition to the amount
registration of the sale.25 However, if these persons fail to redeem the property within the time provided, then awarded by the appellate court, Francisco should pay the estate of Gutierrez interest on the unpaid balance of
the purchaser acquires the property "free from any encumbrance or third party claim whatsoever." 26 Cardale
Set V Corporation Code * Doctrine of Peircing Corpo Veil Cases*Page 66 of 164

the purchase price (in the amount of P629,000.00) at the rate of nine percent (9%) per annum computed from "In its complaint, the plaintiff [herein respondent] alleged that it is a partnership duly organized, existing, and
January, 1989 until fully satisfied. operating under the laws of the Philippines, with office and principal place of business at Nos. 794-812 Del
Monte [A]venue, Quezon City, while the defendant [herein petitioner] Philippine National Bank (herein referred
Finally, contrary to petitioner's assertions, we agree with the Court of Appeals that the decision of the trial court to as PNB), is a semi-government corporation duly organized, existing and operating under the laws of the
in Civil Case No. Q-12366 does not constituteres judicata insofar as the present case is concerned because the Philippines, with office and principal place of business at Escolta Street, Sta. Cruz, Manila; whereas, the other
decision in the first case was not a judgment on the merits. Rather, it was merely based upon the premise that defendant, the National Sugar Development Corporation (NASUDECO in brief), is also a semi-government
since Cardale had been dissolved and the property acquired by another corporation, the action for rescission corporation and the sugar arm of the PNB, with office and principal place of business at the 2nd Floor,
would not prosper. As a matter of fact, it was even expressly stated by the trial court that the parties should Sampaguita Building, Cubao, Quezon City; and the defendant Pampanga Sugar Mills (PASUMIL in short), is a
ventilate their issues in another action. corporation organized, existing and operating under the 1975 laws of the Philippines, and had its business
office before 1975 at Del Carmen, Floridablanca, Pampanga; that the plaintiff is engaged in the business of
general construction for the repairs and/or construction of different kinds of machineries and buildings; that on
WHEREFORE, the 13 April 1999 Decision of the Court of Appeals is hereby accordingly MODIFIED so as to hold
August 26, 1975, the defendant PNB acquired the assets of the defendant PASUMIL that were earlier foreclosed
ADALIA FRANCISCO solely liable to the estate of Gutierrez for the amount of P4,314,271.43 and for interest on
by the Development Bank of the Philippines (DBP) under LOI No. 311; that the defendant PNB organized the
the unpaid balance of the purchase price (in the amount of P629,000.00) at the rate of nine percent (9%) per
defendant NASUDECO in September, 1975, to take ownership and possession of the assets and ultimately to
annum computed from January, 1989 until fully satisfied. MERRYLAND is hereby absolved from all liability.SO
nationalize and consolidate its interest in other PNB controlled sugar mills; that prior to October 29, 1971, the
ORdeRED.
defendant PASUMIL engaged the services of plaintiff for electrical rewinding and repair, most of which were
partially paid by the defendant PASUMIL, leaving several unpaid accounts with the plaintiff; that finally, on
. October 29, 1971, the plaintiff and the defendant PASUMIL entered into a contract for the plaintiff to perform
the following, to wit
THIRD DIVISION G.R. No. 142936 April 17, 2002
(a) Construction of one (1) power house building;
PHILIPPINE NATIONAL BANK & NATIONAL SUGAR DEVELOPMENT CORPORATION, petitioners, vs.
ANDRADA ELECTRIC & ENGINEERING COMPANY, respondent. (b) Construction of three (3) reinforced concrete foundation for three (3) units 350 KW diesel engine
generating set[s];
PANGANIBAN, J.:
(c) Construction of three (3) reinforced concrete foundation for the 5,000 KW and 1,250 KW turbo
Basic is the rule that a corporation has a legal personality distinct and separate from the persons and entities generator sets;
owning it. The corporate veil may be lifted only if it has been used to shield fraud, defend crime, justify a
wrong, defeat public convenience, insulate bad faith or perpetuate injustice. Thus, the mere fact that the (d) Complete overhauling and reconditioning tests sum for three (3) 350 KW diesel engine generating
Philippine National Bank (PNB) acquired ownership or management of some assets of the Pampanga Sugar Mill set[s];
(PASUMIL), which had earlier been foreclosed and purchased at the resulting public auction by the
Development Bank of the Philippines (DBP), will not make PNB liable for the PASUMILs contractual debts to
(e) Installation of turbine and diesel generating sets including transformer, switchboard, electrical
respondent.
wirings and pipe provided those stated units are completely supplied with their accessories;

Statement of the Case


(f) Relocating of 2,400 V transmission line, demolition of all existing concrete foundation and drainage
canals, excavation, and earth fillings all for the total amount of P543,500.00 as evidenced by a
Before us is a Petition for Review assailing the April 17, 2000 Decision 1 of the Court of Appeals (CA) in CA-GR contract, [a] xerox copy of which is hereto attached as Annex A and made an integral part of this
CV No. 57610. The decretal portion of the challenged Decision reads as follows: complaint;

"WHEREFORE, the judgment appealed from is hereby AFFIRMED." 2 that aside from the work contract mentioned-above, the defendant PASUMIL required the plaintiff to perform
extra work, and provide electrical equipment and spare parts, such as:
The Facts
(a) Supply of electrical devices;
The factual antecedents of the case are summarized by the Court of Appeals as follows:
(b) Extra mechanical works;
Set V Corporation Code * Doctrine of Peircing Corpo Veil Cases*Page 67 of 164

(c) Extra fabrication works; "The motion to dismiss was by the court a quo denied in its Order of November 27, 1980; in the same order,
that court directed the defendants to file their answer to the complaint within 15 days.
(d) Supply of materials and consumable items;
"In their answer, the defendant NASUDECO reiterated the grounds of its motion to dismiss, to wit:
(e) Electrical shop repair;
That the complaint does not state a sufficient cause of action against the defendant NASUDECO
(f) Supply of parts and related works for turbine generator; because: (a) NASUDECO is not x x x privy to the various electrical construction jobs being sued upon
by the plaintiff under the present complaint; (b) the taking over by NASUDECO of the assets of
defendant PASUMIL was solely for the purpose of reconditioning the sugar central of defendant
(g) Supply of electrical equipment for machinery;
PASUMIL pursuant to martial law powers of the President under the Constitution; (c) nothing in the LOI
No. 189-A (as well as in LOI No. 311) authorized or commanded the PNB or its subsidiary corporation,
(h) Supply of diesel engine parts and other related works including fabrication of parts. the NASUDECO, to assume the corporate obligations of PASUMIL as that being involved in the present
case; and, (d) all that was mentioned by the said letter of instruction insofar as the PASUMIL liabilities
that out of the total obligation of P777,263.80, the defendant PASUMIL had paid only P250,000.00, leaving an [were] concerned [was] for the PNB, or its subsidiary corporation the NASUDECO, to make a study of,
unpaid balance, as of June 27, 1973, amounting to P527,263.80, as shown in the Certification of the chief and submit [a] recommendation on the problems concerning the same.
accountant of the PNB, a machine copy of which is appended as Annex C of the complaint; that out of said
unpaid balance of P527,263.80, the defendant PASUMIL made a partial payment to the plaintiff of P14,000.00, "By way of counterclaim, the NASUDECO averred that by reason of the filing by the plaintiff of the present suit,
in broken amounts, covering the period from January 5, 1974 up to May 23, 1974, leaving an unpaid balance of which it [labeled] as unfounded or baseless, the defendant NASUDECO was constrained to litigate and incur
P513,263.80; that the defendant PASUMIL and the defendant PNB, and now the defendant NASUDECO, failed litigation expenses in the amount of P50,000.00, which plaintiff should be sentenced to pay. Accordingly,
and refused to pay the plaintiff their just, valid and demandable obligation; that the President of the NASUDECO NASUDECO prayed that the complaint be dismissed and on its counterclaim, that the plaintiff be condemned to
is also the Vice-President of the PNB, and this official holds office at the 10th Floor of the PNB, Escolta, Manila, pay P50,000.00 in concept of attorneys fees as well as exemplary damages.
and plaintiff besought this official to pay the outstanding obligation of the defendant PASUMIL, inasmuch as the
defendant PNB and NASUDECO now owned and possessed the assets of the defendant PASUMIL, and these
"In its answer, the defendant PNB likewise reiterated the grounds of its motion to dismiss, namely: (1) the
defendants all benefited from the works, and the electrical, as well as the engineering and repairs, performed
complaint states no cause of action against the defendant PNB; (2) that PNB is not a party to the contract
by the plaintiff; that because of the failure and refusal of the defendants to pay their just, valid, and
alleged in par. 6 of the complaint and that the alleged services rendered by the plaintiff to the defendant
demandable obligations, plaintiff suffered actual damages in the total amount of P513,263.80; and that in order
PASUMIL upon which plaintiffs suit is erected, was rendered long before PNB took possession of the assets of
to recover these sums, the plaintiff was compelled to engage the professional services of counsel, to whom the
the defendant PASUMIL under LOI No. 189-A; (3) that the PNB take-over of the assets of the defendant PASUMIL
plaintiff agreed to pay a sum equivalent to 25% of the amount of the obligation due by way of attorneys fees.
under LOI 189-A was solely for the purpose of reconditioning the sugar central so that PASUMIL may resume its
Accordingly, the plaintiff prayed that judgment be rendered against the defendants PNB, NASUDECO, and
operations in time for the 1974-75 milling season, and that nothing in the said LOI No. 189-A, as well as in LOI
PASUMIL, jointly and severally to wit:
No. 311, authorized or directed PNB to assume the corporate obligation/s of PASUMIL, let alone that for which
the present action is brought; (4) that PNBs management and operation under LOI No. 311 did not refer to any
(1) Sentencing the defendants to pay the plaintiffs the sum of P513,263.80, with annual interest of asset of PASUMIL which the PNB had to acquire and thereafter [manage], but only to those which were
14% from the time the obligation falls due and demandable; foreclosed by the DBP and were in turn redeemed by the PNB from the DBP; (5) that conformably to LOI No.
311, on August 15, 1975, the PNB and the Development Bank of the Philippines (DBP) entered into a
(2) Condemning the defendants to pay attorneys fees amounting to 25% of the amount claim; Redemption Agreement whereby DBP sold, transferred and conveyed in favor of the PNB, by way of
redemption, all its (DBP) rights and interest in and over the foreclosed real and/or personal properties of
PASUMIL, as shown in Annex C which is made an integral part of the answer; (6) that again, conformably with
(3) Ordering the defendants to pay the costs of the suit.
LOI No. 311, PNB pursuant to a Deed of Assignment dated October 21, 1975, conveyed, transferred, and
assigned for valuable consideration, in favor of NASUDECO, a distinct and independent corporation, all its (PNB)
"The defendants PNB and NASUDECO filed a joint motion to dismiss the complaint chiefly on the ground that rights and interest in and under the above Redemption Agreement. This is shown in Annex D which is also
the complaint failed to state sufficient allegations to establish a cause of action against both defendants, made an integral part of the answer; [7] that as a consequence of the said Deed of Assignment, PNB on
inasmuch as there is lack or want of privity of contract between the plaintiff and the two defendants, the PNB October 21, 1975 ceased to managed and operate the above-mentioned assets of PASUMIL, which function was
and NASUDECO, said defendants citing Article 1311 of the New Civil Code, and the case law ruling in Salonga v. now actually transferred to NASUDECO. In other words, so asserted PNB, the complaint as to PNB, had become
Warner Barnes & Co., 88 Phil. 125; and Manila Port Service, et al. v. Court of Appeals, et al., 20 SCRA 1214. moot and academic because of the execution of the said Deed of Assignment; [8] that moreover, LOI No. 311
did not authorize or direct PNB to assume the corporate obligations of PASUMIL, including the alleged obligation
upon which this present suit was brought; and [9] that, at most, what was granted to PNB in this respect was
Set V Corporation Code * Doctrine of Peircing Corpo Veil Cases*Page 68 of 164

the authority to make a study of and submit recommendation on the problems concerning the claims of Issues
PASUMIL creditors, under sub-par. 5 LOI No. 311.
In their Memorandum, petitioners raise the following errors for the Courts consideration:
"In its counterclaim, the PNB averred that it was unnecessarily constrained to litigate and to incur expenses in
this case, hence it is entitled to claim attorneys fees in the amount of at least P50,000.00. Accordingly, PNB "I
prayed that the complaint be dismissed; and that on its counterclaim, that the plaintiff be sentenced to pay
defendant PNB the sum of P50,000.00 as attorneys fees, aside from exemplary damages in such amount that
The Court of Appeals gravely erred in law in holding the herein petitioners liable for the unpaid
the court may seem just and equitable in the premises.
corporate debts of PASUMIL, a corporation whose corporate existence has not been legally
extinguished or terminated, simply because of petitioners[] take-over of the management and
"Summons by publication was made via the Philippines Daily Express, a newspaper with editorial office at 371 operation of PASUMIL pursuant to the mandates of LOI No. 189-A, as amended by LOI No. 311.
Bonifacio Drive, Port Area, Manila, against the defendant PASUMIL, which was thereafter declared in default as
shown in the August 7, 1981 Order issued by the Trial Court.
"II

"After due proceedings, the Trial Court rendered judgment, the decretal portion of which reads:
The Court of Appeals gravely erred in law in not applying [to] the case at bench the ruling enunciated
in Edward J. Nell Co. v. Pacific Farms, 15 SCRA 415." 6
WHEREFORE, judgment is hereby rendered in favor of plaintiff and against the defendant Corporation,
Philippine National Bank (PNB) NATIONAL SUGAR DEVELOPMENT CORPORATION (NASUDECO) and
Succinctly put, the aforesaid errors boil down to the principal issue of whether PNB is liable for the unpaid debts
PAMPANGA SUGAR MILLS (PASUMIL), ordering the latter to pay jointly and severally the former the
of PASUMIL to respondent.
following:

This Courts Ruling


1. The sum of P513,623.80 plus interest thereon at the rate of 14% per annum as claimed
from September 25, 1980 until fully paid;
The Petition is meritorious.
2. The sum of P102,724.76 as attorneys fees; and,
Main Issue:
3. Costs. SO ORDERED.
Liability for Corporate Debts
Manila, Philippines, September 4, 1986.
As a general rule, questions of fact may not be raised in a petition for review under Rule 45 of the Rules of
Court.7 To this rule, however, there are some exceptions enumerated in Fuentes v. Court of Appeals.8 After a
careful scrutiny of the records and the pleadings submitted by the parties, we find that the lower courts
'(SGD) ERNESTO S. TENGCO misappreciated the evidence presented.9 Overlooked by the CA were certain relevant facts that would justify a
Judge"3 conclusion different from that reached in the assailed Decision. 10

Petitioners posit that they should not be held liable for the corporate debts of PASUMIL, because their takeover
of the latters foreclosed assets did not make them assignees. On the other hand, respondent asserts that
Ruling of the Court of Appeals petitioners and PASUMIL should be treated as one entity and, as such, jointly and severally held liable for
PASUMILs unpaid obligation.1wphi1.nt
Affirming the trial court, the CA held that it was offensive to the basic tenets of justice and equity for a
corporation to take over and operate the business of another corporation, while disavowing or repudiating any As a rule, a corporation that purchases the assets of another will not be liable for the debts of the selling
responsibility, obligation or liability arising therefrom.4 corporation, provided the former acted in good faith and paid adequate consideration for such assets, except
when any of the following circumstances is present: (1) where the purchaser expressly or impliedly agrees to
Hence, this Petition.5 assume the debts, (2) where the transaction amounts to a consolidation or merger of the corporations, (3)
Set V Corporation Code * Doctrine of Peircing Corpo Veil Cases*Page 69 of 164

where the purchasing corporation is merely a continuation of the selling corporation, and (4) where the Being the party that asked for the piercing of the corporate veil, respondent had the burden of presenting clear
transaction is fraudulently entered into in order to escape liability for those debts. 11 and convincing evidence to justify the setting aside of the separate corporate personality rule. 34 However, it
utterly failed to discharge this burden; 35 it failed to establish by competent evidence that petitioners separate
Piercing the Corporate corporate veil had been used to conceal fraud, illegality or inequity. 36

Veil Not Warranted While we agree with respondents claim that the assets of the National Sugar Development Corporation
(NASUDECO) can be easily traced to PASUMIL, 37 we are not convinced that the transfer of the latters assets to
petitioners was fraudulently entered into in order to escape liability for its debt to respondent. 38
A corporation is an artificial being created by operation of law. It possesses the right of succession and such
powers, attributes, and properties expressly authorized by law or incident to its existence. 12 It has a personality
separate and distinct from the persons composing it, as well as from any other legal entity to which it may be A careful review of the records reveals that DBP foreclosed the mortgage executed by PASUMIL and acquired
related.13 This is basic. the assets as the highest bidder at the public auction conducted. 39 The bank was justified in foreclosing the
mortgage, because the PASUMIL account had incurred arrearages of more than 20 percent of the total
outstanding obligation.40 Thus, DBP had not only a right, but also a duty under the law to foreclose the subject
Equally well-settled is the principle that the corporate mask may be removed or the corporate veil pierced
properties.41
when the corporation is just an alter ego of a person or of another corporation. 14 For reasons of public policy
and in the interest of justice, the corporate veil will justifiably be impaled 15 only when it becomes a shield for
fraud, illegality or inequity committed against third persons.16 Pursuant to LOI No. 189-A42 as amended by LOI No. 311,43 PNB acquired PASUMILs assets that DBP had
foreclosed and purchased in the normal course. Petitioner bank was likewise tasked to manage temporarily the
operation of such assets either by itself or through a subsidiary corporation. 44
Hence, any application of the doctrine of piercing the corporate veil should be done with caution. 17 A court
should be mindful of the milieu where it is to be applied.18 It must be certain that the corporate fiction was
misused to such an extent that injustice, fraud, or crime was committed against another, in disregard of its PNB, as the second mortgagee, redeemed from DBP the foreclosed PASUMIL assets pursuant to Section 6 of Act
rights.19 The wrongdoing must be clearly and convincingly established; it cannot be presumed. 20 Otherwise, an No. 3135.45 These assets were later conveyed to PNB for a consideration, the terms of which were embodied in
injustice that was never unintended may result from an erroneous application. 21 the Redemption Agreement.46 PNB, as successor-in-interest, stepped into the shoes of DBP as PASUMILs
creditor.47 By way of a Deed of Assignment,48 PNB then transferred to NASUDECO all its rights under the
Redemption Agreement.
This Court has pierced the corporate veil to ward off a judgment credit, 22 to avoid inclusion of corporate assets
as part of the estate of the decedent, 23 to escape liability arising from a debt, 24 or to perpetuate fraud and/or
confuse legitimate issues25 either to promote or to shield unfair objectives26 or to cover up an otherwise blatant In Development Bank of the Philippines v. Court of Appeals,49 we had the occasion to resolve a similar issue. We
violation of the prohibition against forum-shopping. 27 Only in these and similar instances may the veil be ruled that PNB, DBP and their transferees were not liable for Marinduque Minings unpaid obligations to
pierced and disregarded.28 Remington Industrial Sales Corporation (Remington) after the two banks had foreclosed the assets of
Marinduque Mining. We likewise held that Remington failed to discharge its burden of proving bad faith on the
part of Marinduque Mining to justify the piercing of the corporate veil.
The question of whether a corporation is a mere alter ego is one of fact. 29 Piercing the veil of corporate fiction
may be allowed only if the following elements concur: (1) control -- not mere stock control, but complete
domination -- not only of finances, but of policy and business practice in respect to the transaction attacked, In the instant case, the CA erred in affirming the trial courts lifting of the corporate mask. 50 The CA did not
must have been such that the corporate entity as to this transaction had at the time no separate mind, will or point to any fact evidencing bad faith on the part of PNB and its transferee. 51 The corporate fiction was not
existence of its own; (2) such control must have been used by the defendant to commit a fraud or a wrong to used to defeat public convenience, justify a wrong, protect fraud or defend crime. 52 None of the foregoing
perpetuate the violation of a statutory or other positive legal duty, or a dishonest and an unjust act in exceptions was shown to exist in the present case. 53 On the contrary, the lifting of the corporate veil would
contravention of plaintiffs legal right; and (3) the said control and breach of duty must have proximately result in manifest injustice. This we cannot allow.
caused the injury or unjust loss complained of. 30
No Merger or Consolidation
We believe that the absence of the foregoing elements in the present case precludes the piercing of the
corporate veil. First, other than the fact that petitioners acquired the assets of PASUMIL, there is no showing Respondent further claims that petitioners should be held liable for the unpaid obligations of PASUMIL by virtue
that their control over it warrants the disregard of corporate personalities. 31 Second, there is no evidence that of LOI Nos. 189-A and 311, which expressly authorized PASUMIL and PNB to merge or consolidate. On the other
their juridical personality was used to commit a fraud or to do a wrong; or that the separate corporate entity hand, petitioners contend that their takeover of the operations of PASUMIL did not involve any corporate
was farcically used as a mere alter ego, business conduit or instrumentality of another entity or person. 32 Third, merger or consolidation, because the latter had never lost its separate identity as a corporation.
respondent was not defrauded or injured when petitioners acquired the assets of PASUMIL. 33
Set V Corporation Code * Doctrine of Peircing Corpo Veil Cases*Page 70 of 164

A consolidation is the union of two or more existing entities to form a new entity called the consolidated On 14 January 1592, Labor Arbiter Felipe T. Garduque II denied the motion to dismiss holding that the allegation
corporation. A merger, on the other hand, is a union whereby one or more existing corporations are absorbed of lack of employer-employee relationship between Capulso and AZCOR was not clearly established. Thereafter,
by another corporation that survives and continues the combined business. 54 the Labor Arbiter ordered that hearings be conducted for the presentation of evidence by both parties.

The merger, however, does not become effective upon the mere agreement of the constituent The evidence presented by Capulso showed that he worked for AZCOR as ceramics worker for more than two
corporations.55Since a merger or consolidation involves fundamental changes in the corporation, as well as in (2) years starting from 3 April 1989 to 1 June 1991 receiving a daily wage of P118.00 plus other benefits such
the rights of stockholders and creditors, there must be an express provision of law authorizing them. 56 For a as vacation and sick leaves. From April to September 1989 the amount of P50.00 was deducted from his salary
valid merger or consolidation, the approval by the Securities and Exchange Commission (SEC) of the articles of without informing him of the reason therefor.
merger or consolidation is required.57 These articles must likewise be duly approved by a majority of the
respective stockholders of the constituent corporations.58 In the second week of February 1991, upon his doctor's recommendation, Capulso verbally requested to go on
sick leave due to bronchial asthma. It appeared that his illness was, directly caused by his job as ceramics
In the case at bar, we hold that there is no merger or consolidation with respect to PASUMIL and PNB. The worker where, for lack of the prescribed occupational safety gadgets, he inhaled and absorbed harmful ceramic
procedure prescribed under Title IX of the Corporation Code 59 was not followed. dusts. His supervisor, Ms. Emily Apolinaria, approved his request. Later, on 1 June 1991, Capulso went back to
petitioner AZCOR to resume his work after recuperating from his illness. He was not allowed to do so by his
In fact, PASUMILs corporate existence, as correctly found by the CA, had not been legally extinguished or supervisors who informed him that only the owner, Arturo Zuluaga, could allow him to continue in his job. He
terminated.60 Further, prior to PNBs acquisition of the foreclosed assets, PASUMIL had previously made partial returned five (5) times to AZCOR but when it became apparent that he would not be reinstated, he immediately
payments to respondent for the formers obligation in the amount of P777,263.80. As of June 27, 1973, filed the instant complaint for illegal dismissal. 2
PASUMIL had paid P250,000 to respondent and, from January 5, 1974 to May 23, 1974, another P14,000.
Capulso presented the following documentary evidence in support of his claim: (a) His affidavit and testimony
Neither did petitioner expressly or impliedly agree to assume the debt of PASUMIL to respondent. 61 LOI No. 11 to prove that he was terminated without just cause and without due process; 3 (b) Identification card issued by
explicitly provides that PNB shall study and submit recommendations on the claims of PASUMILs AZCOR which he continued to use even after his supposed employment by Filipinas Paso; 4 (c) Certification of
creditors.62Clearly, the corporate separateness between PASUMIL and PNB remains, despite respondents SSS premium payments; 5 (d) SSS Member Assistance Form wherein he stated that he worked with AZCOR from
insistence to the contrary.63WHEREFORE, the Petition is hereby GRANTED and the assailed Decision SET ASIDE. March 1989 to April 1991; 6 (e) Certification of Employee Contribution with SSS; 7 and, (f) Payslips issued by
No pronouncement as to costs.SO ORDERED. AZCOR. 8

SECOND DIVISION G.R. No. 117963 February 11, 1999 On the other hand, petitioners alleged that Capulso was a former employee of AZCOR who resigned on 28
February 1990 as evidenced by a letter of resignation and joined Filipinas Paso on 1 March 1990 as shown by a
contract of employment; in February 1991 Capulso allegedly informed his supervisor, Ms. Emilia Apolinaria, that
AZCOR MANUFACTURING INC., FILIPINAS PASO and/or ARTURO ZULUAGA/Owner, petitioners,
he intended to go on terminal leave because he was not feeling well; on 1 March 1991 he submitted a letter of
vs. NATIONAL LABOR RELATIONS COMMISSION (NLRC) AND CANDIDO CAPULSO, respondents.
resignation addressed to the President of Filipinas Paso, Manuel Montilla; and, in the early part of June 1991
Capulso tried to apply for work again with Filipinas Paso but there was no vacancy.
BELLOSILLO, J.:

Petitioners submitted the following documentary evidence: (a) Sworn Statement of Ms. Emilia Apolinaria and
AZCOR MANUFACTURING, INC., Filipinas Paso and Arturo Zuluaga instituted this petition for certiorari under her actual testimony to prove that respondent indeed resigned voluntarily from AZCOR to transfer to Filipinas
Rule 65 of the Rules of Court to assail, for having been rendered with grave abuse of discretion amounting to Paso, and thereafter, from Filipinas Paso hug to failing health; 9 (b) Contract of Employment between Filipinas
lack or excess of jurisdiction, the Decision of the National Labor Relations Commission which reversed the Paso and respondent which took effect 1 March 1991; 10 (c) Letter of resignation of respondent from AZCOR
decision of the Labor Arbiter dismissing the complaint of respondent Candido Capulso against petitioners. 1 dated 28 February 1990, to take effect on the same date; 11 (d) Undated letter of resignation of respondent
addressed to Filipinas Paso to take effect 1 March 1991; 12 (e) BIR Form No. W-4 filed 6 June 1990; 13 (f)
Candido Capuslo file with the Labor Arbiter a complaint for constructive illegal dismissal and illegal deduction of Individual Income Tax Return of respondent for 1990; 14 and, (g) BIR Form 1701-B which was an alphabetical list
P50.00 per day for the period April to September 1989. Petitioners Azcor Manufacturing, Inc. (AZCOR) and of employees of Filipinas Paso for the year ending 31 December 1990. 15
Arturo Zuluaga who were respondents before the Labor Arbiter (Filipinas Paso was not yet a party then in that
case) moved to dismiss the complaint on the ground that there was no employer-employee relationship On 29 December 1992 the Labor Arbiter rendered a decision dismissing the complaint for illegal dismissal for
between AZCOR and herein respondent Capulso; .that the latter became an employee of Filipinas Paso effective lack of merit, but ordered AZCOR and/or Arturo Zuluaga to refund to Capulso the sum of P200.00 representing
1 March 1996 but voluntarily resigned there from a year after, Capulso later amended his complaint by the amount illegally deducted from his salary.
impleading Filipinas Paso as additional respondent before the Labor Arbiter.
Set V Corporation Code * Doctrine of Peircing Corpo Veil Cases*Page 71 of 164

On appeal by Capulso, docketed as NLRC CA No. 004476-93 (NLRC NCR 00-09-05271-91), "Capulso v. Azcor As a rule the original and exclusive jurisdiction to review a decision or resolution of respondent NLRC in a
Manufacturing Inc., Filipinas Paso and/or Arturo Zuluaga/owner," the NLRC modified the Labor Arbiter's decision petition for certiorari under Rule 65 of the Rules of Court does not include a correction of its evaluation of the
by: (a) declaring the dismissal of Capulso as illegal for lack of just and valid cause; (b) ordering petitioners to evidence but is confined to issues of jurisdiction or grave abuse of discretion. The NLRC factual findings, if
reinstate Capulso to his former or equivalent position without loss of Seniority rights and without diminution of supported by substantial evidence, are entitled to great respect and even finality, unless petitioner is able to
benefits, and, (c) ordering petitioners to jointly and solidarily pay Capulso his backwages computed from the show that it simply and arbitrarily disregarded the evidence before it or had misappreciated the evidence to
time of his dismissal up to the date of his actual reinstatement. The NLRC held in part- such an extent as to compel a contrary conclusion if such evidence had been properly appreciated. 16 We find
no cogent reason to disturb the findings of the NLCR.
. . . . the contract of employment (Exh. 2, p. 187, Rollo) issued to complainant indicates that the work
to be done during the period was contracted with Filipinas Paso. The said contract was signed by, the Petitioners insist that Capulso was not really dismissed but he voluntarily resigned from AZCOR and Filipinas
Personnel Officer of Ascor Manufacturing Inc. Likewise, the contract period is for six (6) months, which Paso, and that there was nothing illegal or unusual in the letters of resignation he executed.
establishes a presumption that the said contract could pass either as to cover the probationary period,
or job contracting, the completion of which automatically terminates employment, whichever will work We disagree. To constitute a resignation, it must be unconditional and with the intent to operate as such. There
to respondent's advantage should the case be filed. However, appellant continued working with must be an intention to relinquish a portion of the term of office accompanied by an act of relinquishment. 17 In
respondent after the lapse of the contract and until the alleged termination of employment of the instant case, the fact that Capulso signified his desire to resume his work when he went back to petitioner
appellant. AZCOR after recuperating from his illness, and actively pursued his case for illegal dismissal before the labor
courts when he was refused admission by his employer, negated any intention on his part to relinquish his job
Secondly, the two resignation letters allegedly executed by appellant are exactly worded, which only at AZCOR.
shows that the same work were prepared by respondents-appellees plus after the fact that
complainant denied having executed and signed the same. Moreover, a closer look at the subject resignation letters readily reveals the following: (a) the resignation letter
allegedly tendered by Capulso to Filipinas Paso was identically worded with that supposedly addressed by him
. . . . the letter of resignation (Exh. "3", p. 188, Rollo) supposed to have been executed by to AZCOR; (b) both were pre-drafted with blank spaces filled up with the purported dates of effectivity of his
complainant-appellant shows that he resigned from Ascor Mfg., Inc. on February 28, 1990 while Exhibit resignation; and, (c) it was written in English, a language which Capulso was not conversant with considering
"2", page 187, Rollo, which was the contract of Employment issued to Candido Capulso by the his low level of education. No other plausible explanation can be drawn from these circumstances than that the
personnel officer of Ascor Mfg., Inc. shows that appellant was being hired from March 1, 1990 to subject letters of resignation were prepared by a person or persons other than Capulso. And the fact that he
August 31, 1990 by respondent Ascor Mfg., Inc. to do jobs for Filipinas Paso; A run-around of events categorically disowned the signatures therein and denied having executed them clearly indicates that the
and dates. resignation letters were drafted, without his consent and participation.

The events that transpired clearly show that there was no interruption in the service of complainant Even assuming for the sake of argument that the signatures were, genuine, we still cannot give credence to
with Ascor Mfg., Inc. from April 13 1989 up to June 1, 1991 when complainant was unceremoniously those letters in the absence of any showing that Capulso was aware that what he was signing then were in fact
dismissed. resignation letters or that he fully understood the contents thereof. Having introduced those resignation letters
in evidence, it was incumbent upon petitioners to prove clearly and convincingly their genuineness and due
Considering that Ascor Mfg., Inc. and Filipinas Paso orchestrated the events that appeared to be in execution, especially considering the serious doubts an their authenticity. Petitioners miserably failed in this
order with the alleged execution of resignation letters which was disputed by complainant and respect.
confirmed spurious as explained above, likewise overwhelmingly show the bad faith of respondents in
the treatment of their employees. The Labor Arbiter held that Capulso's repudiation of the signatures affixed in the letters of resignation was
weakened by the fact that he filed the case only after almost four (4) months from the date of his dismissal. But
Petitioners' motion for reconsideration was denied by the NLRC through its Resolution of 14 October 1994; it should be noted that private respondent still wanted his job and thus, understandably, refrained from filing
hence, the instant-petition. Meanwhile, during the pendency of the case before this Court, Capulso succumbed the illegal dismissal case against his employer so as not to jeopardize his chances of continuing with his
to asthma and heart disease. employment. True enough, when it became apparent that he was no longer welcome at AZCOR he immediately
instituted the instant case.

The issue to be resolved is whether the NLRC committed grave abuse of discretion in declaring that private
respondent Capulso was illegally dismissed and in holding petitioners jointly and solidarily liable to Capulso for In addition, an action for reinstatement by reason of illegal dismissal is one based on an injury which may be
back wages. brought within four (4) years from the time of dismissal pursuant to Art. 1146 of the Civil Code. Hence,
Capulso's case which was filed after a measly delay of four (4) months should not be treated with skepticism or
cynicism. By law and settled jurisprudence, he has four (4) years to file his complaint for illegal dismissal. A
delay of merely four (4) months in instituting an illegal dismissal case is more than sufficient compliance with
Set V Corporation Code * Doctrine of Peircing Corpo Veil Cases*Page 72 of 164

the prescriptive period. It may betray an unlettered man's lack of awareness of his rights as a lowly worker but, Paso. 22 After causing much confusion, petitioners have the temerity to use as evidence the ignorance of
certainly, he must not be penalized for his tarrying. Capulso in identifying his true employer. It is evident from the foregoing discussion that Capulso was led into
believing that while he was working with Filipinas Paso, his real employer was AZCOR. Petitioners never dealt
In illegal dismissal cases like the present one, the onus of proving that the dismissal of the employee was for a with him openly and in good faith, nor was he informed of the developments within the company, i.e., his
valid and authorized cause rests on the employer 18 and failure to discharge the same would mean that the alleged transfer to Filipinas Paso and the closure of AZCOR's manufacturing operations beginning 1 March
dismissal is not justified and therefore illegal. 19 Petitioners failed in this regard. 1990. 23 Understandably, he sued AZCOR alone and was constrained to implead Filipinas Paso as additional
respondent only when it became apparent that the latter also appeared to be his employer.

Petitioners also contend that they could not be held jointly and severally liable to Capulso for back wages since
AZCOR and Filipinas Paso are separate and distinct corporations with different corporate personalities; and, the In fine, we see in the totality of the evidence a veiled attempt by petitioners to deprive Capulso of what he had
mere fact that the businesses of these corporations are interrelated and both owned and controlled by a single earned through hard labor by taking advantage of his low level of education and confusing. him as to who
stockholder are not sufficient grounds to disregard their separate corporate entities. really was his true employer - such a callous and despicable treatment of a worker who had rendered faithful
service to their company.

We are not persuaded. The doctrine that a corporation is a legal entity or a person in law distinct from the
persons composing it is merely a legal fiction for purposes of convenience and to subserve the ends of justice. However, considering that private respondent died during the pendency of the case before this Court,
This fiction cannot be extended to a point beyond its reason and policy. 20 Where, as in this case, the corporate reinstatement is no longer feasible. In lieu thereof, separation pay shall be awarded. With respect to the
fiction was used as a means to perpetrate a social injustice or as a vehicle to evade obligations or confuse the amount of back wages, it shall be computed from the time of private respondent's illegal dismissal up to the
legitimate issues, it would be discarded and the two (2) corporations would be merged as one, the first being time of his death.WHEREFORE, the petition is DISMISSED. The NLRC Decision of 12 September 1994 is
merely considered as the instrumentality, agency, conduit or adjunct of the other. 21 MODIFIED. Petitioners AZCOR MANUFACTURING, INC., FILIPINAS PASO and ARTURO ZULUAGA are ORDERED to
pay, jointly and solidarily, the heirs of private respondent Candido Capulso the amounts representing his back
wages, inclusive of allowances and other benefits, and separation pay to, be computed in accordance with
In this particular case, there was much confusion as to the identity of Capulso's employer - whether it was
law.SO ORDERED.
AZCOR or Filipinas Paso; but, for sure, it was petitioners' own making, as shown by the following: First, Capulso
had no knowledge that he was already working under petitioner Filipinas Paso since he contained to retain his
AZCOR Identification card; Second, his payslips contained the name of AZCOR giving the impression that FIRST DIVISION G.R. No. L-30822 July 31, 1975
AZCOR was paying his salary; Third, he was paid the same salary and he performed the same kind of job, in the
same work area, in the same location, using the same tools and under the same supervisor; Fourth, there was EDUARDO CLAPAROLS, ROMULO AGSAM and/or CLAPAROLS STEEL AND NAIL PLANT, petitioners,
no gap in his employment as he continued to work from the time he was hired up to the last day of his vs.
work; Fifth, the casting department of AZCOR where Capulso was working was abolished when he, together COURT OF INDUSTRIAL RELATIONS, ALLIED WORKERS' ASSOCIATION and/or DEMETRIO GARLITOS,
with six (6) others, transferred to Filipinas Paso; and Sixth, the employment contract was signed by an AZCOR ALFREDO ONGSUCO, JORGE SEMILLANO, SALVADOR DOROTEO, ROSENDO ESPINOSA, LUDOVICO
personnel officer, which showed that Capulso was being hired from 1 March 1990 to 31 August 1990 by AZCOR BALOPENOS, ASER AMANCIO, MAXIMO QUIOYO, GAUDENCIO QUIOYO, and IGNACIO
to do jobs for Filipinas Paso. The employment contract provided in part: QUIOYO,respondents.

The contract is for a specific job contract only and shall be effective for the period covered, unless MAKASIAR, J.:
sooner terminated when the job contract is completed earlier or withdrawn by client, or when the
employee is dismissed for just and lawful causes provided by law and the company's rules and A petition for certiorari to set aside the order of respondent Court of Industrial Relations dated May 30, 1969
regulations, in which case the employment contract will automatically terminate. directing petitioners to pay back wages and bonuses to private respondents as well as its resolution of July 5,
1969 denying the motion for reconsideration of said order in Case No. 32-ULP-Iloilo entitled "Allied Workers'
As correctly observed by the NLRC, the contract was only for six (6) months, which could pass either as a Association, et. al., versus Eduardo Claparols, et. al.."
probationary period or a job contracting, the completion of which automatically terminated the employment.
Observe further, however, that respondent continued working even after the lapse of the period in the contract It appears that on August 6, 1957, a complaint for unfair labor practice was filed by herein private respondent
- for whom it was not clear. It may be asked: Was the six (6)-month period probationary in nature, in which Allied Workers' Association, respondent Demetrio Garlitos and ten (10) respondent workers against herein
case, after the lapse of the period he became a regular employee of Filipinas Paso? Or was the period job- petitioners on account of the dismissal of respondent workers from petitioner Claparols Steel and Nail Plant.
contracting in character, in which case, after the period he was deemed to have come back to AZCOR?

On September 16, 1963, respondent Court rendered its decision finding "Mr. Claparols guilty of union busting
Interestingly, petitioners likewise argue that it was grave abuse of discretion for the NLRC to hold them and" of having "dismissed said complainants because of their union activities," and ordering respondents "(1)
solidarily, liable to Capulso when the latter himself testified that he was not even an employee of Filipinas To cease and desist from committing unfair labor practices against their employees and laborers; (2) To
Set V Corporation Code * Doctrine of Peircing Corpo Veil Cases*Page 73 of 164

reinstate said complainants to their former or equivalent jobs, as soon as possible, with back wages from the Claparols Steel Corporation stopped operations on December 7, 1962, three (3) computations are
date of their dismissal up to their actual reinstatement" (p. 12, Decision; p. 27, rec.). presented herein for the consideration of this Honorable Court (p. 2, Report of Examiner, p. 29, rec.).

A motion to reconsider the above decision was filed by herein petitioners, which respondent Court, sitting en On January 23, 1965, petitioners filed an opposition alleging that under the circumstances presently engulfing
banc, denied in a resolution dated January 27, 1964. the company, petitioner Claparols could not personally reinstate respondent workers; that assuming the
workers are entitled to back wages, the same should only be limited to three months pursuant to the court
On March 30, 1964, counsel for herein respondent workers (complainants in the ULP case) filed a motion for ruling in the case of Sta. Cecilia Sawmills vs. CIR (L-19273-74, February 20, 1964); and that since Claparols
execution of respondent Court's September 16, 1963 decision. Steel Corporation ceased to operate on December 7, 1962, re-employment of respondent workers cannot go
beyond December 7, 1962.

On May 14, 1964, respondent Court, in its order of September 16, 1963, granted execution and directed herein
petitioners A reply to petitioner's opposition was filed by respondent workers, alleging among others, that Claparols Steel
and Nail Plant and Claparols Steel and Nail Corporation are one and the same corporation controlled by
petitioner Claparols, with the latter corporation succeeding the former.
to reinstate the above complainants to their former or equivalent jobs within five (5) days after receipt
of a copy of this order. In order to implement the award of back wages, the Chief of the Examining
Division or any of his assistants is hereby directed to proceed to the office of the respondents at On November 28, 1966, after conducting a series of hearings on the report of the examiner, respondent Court
Matab-ang, Talisay, Negros Occidental, and examine its payrolls and other pertinent records and issued an order, the dispositive portion of which reads:
compute the back wages of the complainants in accordance with the decision dated September 16,
1963, and, upon termination, to submit his report as soon as possible for further disposition (p. 7, Brief WHEREFORE, the Report of the. Examiner filed on January 15, 1965, is hereby approved subject to the
for Respondents, p. 113, rec.). foregoing findings and dispositions. Consequently, the Corporation Auditing Examiner is directed to
recompute the back wages of complainants Demetrio Garlitos and Alfredo Ongsuco on the basis of
which was reiterated by respondent Court in a subsequent order dated November 10, 1964 (pp. 7-8, Brief for P200.00 and P270.00 a month, respectively; to compute those of complainant Ignacio Quioyo as
Respondents, p. 113, rec.). aforesaid; to compute the deductible earnings of complainants Ongsuco, Jorge Semillano and Garlitos,
as found in the body of this order; and to compute the bonuses of each and every complainant, except
Honorato Quioyo. Thereafter, as soon as possible, the Examiner should submit a report in compliance
On December 14, 1964, respondent workers were accompanied by the Chief of Police of Talisay, Negros
herewith of the Court's further disposition (p. 24, Brief for Respondents, p. 113, rec.).
Occidental to the compound of herein petitioner company to report for reinstatement per order of the court.
Respondent workers were, however, refused reinstatement by company accountant Francisco Cusi for he had
no order from plant owner Eduardo Claparols nor from his lawyer Atty. Plaridel Katalbas, to reinstate respondent On December 7, 1966, a motion for reconsideration was filed by petitioner, assailing respondent Court's ruling
workers. that (1) the ruling in the case of Sta. Cecilia Sawmills Inc. CIR, et. al, does not apply in the case at bar; and (2)
that bonus should be included in the recoverable wages.

Again, on December 15, 1964, respondent workers were accompanied by a police officer to the company
compound, but then, they were again refused reinstatement by Cusi on the same ground. On December 14, 1966, a counter-opposition was filed by private respondents alleging that petitioners' motion
for reconsideration was pro forma, it not making express reference to the testimony or documentary evidence
or to the provision of law alleged to be contrary to such findings or conclusions of respondent Court.
On January 15, 1965, the CIR Chief Examiner Submitted his report containing three computations, to wit:

On February 8, 1967, respondent Court of Industrial Relations dismissed petitioners' motion for reconsideration
The first computation covers the period February 1, 1957 to October 31, 1964. The second is up to and
for being pro forma.
including December 7, 1962, when the corporation stopped operations, while the third is only up to
June 30, 1957 when the Claparols Steel and Nail Plant ceased to operate (Annex B, Petition for Review
on Certiorari, p. 14, Brief for appellees, p. 113, rec.). Whereupon, petitioners filed a petition for certiorari with this COURT in G.R. No. L-27272 to set aside the
November 28, 1966 order of respondent Court, as well as its February 8, 1967 resolution. Petitioners assigned
therein as errors of law the very same assignment of errors it raises in the present case, to wit:
with the explanation that:

I
6. Since the records of the Claparols Steel Corporation show that it was established on July 1, 1957
succeeding the Claparols Steel and Nail Plant which ceased operations on June 30, 1957, and that the
Set V Corporation Code * Doctrine of Peircing Corpo Veil Cases*Page 74 of 164

THE RESPONDENT COURT ERRED AND/OR ACTED WITH GRAVE ABUSE OF DISCRETION, AMOUNTING d. For every dependent in college 5% of monthly salary
TO LACK OF JURISDICTION, IN HOLDING IN THE ORDER UNDER REVIEW THAT BONUSES SHOULD BE
PAID TO THE RESPONDENT WORKERS DESPITE THE FACT THAT THE SAME WAS NOT ADJUDICATED IN xxx xxx xxx
ITS ORIGINAL DECISION.

7. The computed ... bonuses after deducting the earnings elsewhere of Messrs. Ongsuco, Garlitos and
II Semillano are as follows:

THE RESPONDENT COURT ERRED AND/OR ACTED WITH GRAVE ABUSE OF DISCRETION, AMOUNTING Name x x x Bonuses x x x
TO LACK OF JURISDICTION, IN NOT APPLYING THE DOCTRINE LAID DOWN BY THIS HONORABLE
TRIBUNAL IN THE CASE OF "STA. CECILIA SAWMILLS, INC. VS. C.I.R., ET. AL.," G.R. No.
1. Alfredo Ongsuco P1,620.00
L-19273-74, PROMULGATED ON FEBRUARY 29, 1964 (pp. 10-11, rec.).
2. Demetrio Garlitos 1,200.00
3. Ignacio Quioyo 455.23
4. Aser Abancio 461.00
5. Ludovico Belopeos 752.05
On April 27, 1967, the Supreme Court denied petitioners' petition for certiorari (p. 77, rec. of L-27272), which 6. Salvador Doroteo 714.70
was reiterated on May 19, 1967 (p. 27, Respondent's Brief, p. 113, rec.; p. 81, rec. of L-27272). 7. Rosendo Espinosa 1,075.40
8. Gaudencio Quioyo 1,167.92
On May 3, 1967, private respondents moved to have the workers' back wages properly recomputed. A motion 9. Jorge Semillano 1,212.08
to the same end was reiterated by private respondents on June 14, 1967. 10. Maximo Quioyo 449.41
Total P9,107.79(Pp. 30-31, Respondent's Brief, p. 113, rec.)

On July 13, 1967, respondent Court directed a recomputation of the back wages of respondent workers in
accordance with its order dated November 28, 1966. The said order in part reads: On April 16, 1968, petitioners filed their opposition to the report of the Examiner dated March 21, 1968 on
grounds already rejected by respondent Court in its order dated November 28, 1966, and by the Supreme Court
also in its ruling in G.R. No. L-27272.
WHEREFORE, the Chief Auditing Examiner of the Court or any of his assistants, is hereby
directed to recompute the back wages of the workers involved in this case in accordance with
the Order of November 28, 1966 within 20 days from receipt of a copy of this Order (p. 28, On May 4, 1968, a rejoinder to petitioners' opposition was filed by private respondents, alleging among others
Brief for Respondents, p. 113, rec.). "that the grounds of petitioners' opposition were the same grounds raised by them before and passed upon by
respondent Court and this Honorable Tribunal; that this order of November 28, 1966 which passed upon these
issues became final and executory on June 3, 1967 from the Honorable Supreme Court. (Order of respondent
Then on March 21, 1968, the Chief Examiner came out with his report, the disputed portion of which (regarding
Court dated July 13, 1967). [p. 32, Brief for Respondents, p. 113, rec.].
bonuses) reads:

On July 26, 1968, private respondents filed their motion for approval of the Report of the Examiner submitted
xxx xxx xxx
on March 21, 1968, alleging, among others, that petitioners, in their opposition, did not actually dispute the
data elicited by the Chief Examiner but rather harped on grounds which, as already stated, had already been
4. The yearly bonuses of the employees and laborers of respondent corporation are given on the turned down by the Supreme Court.
following basis:
On October 19, 1968, herein private respondents filed their "Constancia", submitting the case for resolution of
Basic Additional: respondent Court of Industrial Relations.

a. For every dependent 1% of monthly salary On May 30, 1969, respondent Court issued an order, subject of the present appeal, the dispositive portion of
which reads:
b. For every dependent in elementary grade 2% of monthly salary
WHEREFORE, there being no proof offered to substantiate respondent Eduardo Claparols' opposition,
c. For every dependent in high school 3% of monthly salary the Examiner's Report should be, and it is hereby, APPROVED. Consequently, pursuant to the decision
Set V Corporation Code * Doctrine of Peircing Corpo Veil Cases*Page 75 of 164

dated September 16, 1963, respondent ... (petitioners herein) are hereby directed to pay the without any condition imposed for its payment ... then it is part of the wage. (Emphasis
respective back wages and bonuses of the complainants (respondents herein) ... (p. 35, Brief for supplied).1wph1.t
Respondents; p. 113, rec.; emphasis supplied).1wph1.t
In Altomonte vs. Philippine American Drug Co. (106 Phil. 137), the Supreme Court held that an employee is not
On June 7, 1969, petitioners filed a motion for reconsideration on practically the same grounds previously entitled to bonus where there is no showing that it had been granted by the employer to its employees
raised by them. periodically or regularly as to become part of their wages or salaries. The clear implication is that bonus is
recoverable as part of the wage or salary where the employer regularly or periodically gives it to employees.
On June 30, 1969, respondents filed an opposition to petitioners' motion for reconsideration, with the following
allegations: American jurisprudence equally regards bonuses as part of compensation or recoverable wages.

1. The issues raised, namely, whether bonuses should be included in the award for back wages had Thus, it was held that "... it follows that in determining the regular rate of pay, a bonus which in fact constitutes
already been resolved by respondent court in its orders dated November 28, 1966, and December 7, PART OF AN EMPLOYEE'S compensation, rather than a true gift or gratuity, has to be taken into consideration ."
1966, and in the Resolution of the Honorable Supreme Court in G.R. No. L-27272 dated April 26, 1967 (48 Am. Jur. 2d, Labor and Labor Relations, No. 1555, citing the cases of Triple "AAA" Co. vs. Wirtz and Haber
and May 19, 1967, and the same is already a settled and final issue. vs. Americana Corporation; Emphasis supplied). It was further held that "... the regular rate includes incentive
bonuses paid to the employees in addition to the guaranteed base rates regardless of any contract provision to
2. Petitioners' motion for reconsideration is merely a rehash of previous arguments, effete and the contrary and even though such bonuses could not be determined or paid until such time after the pay day "
unrejuvenated, pro forma, and intended merely to delay the proceedings. (48 Am. Jur. 2d, Labor and Labor Relations, No. 1555, citing the case of Walling vs. Harnischfeger Corp., 325 US
427, 89 L Ed 1711, 65 S Ct. 1246; Emphasis supplied).1wph1.t

As correctly contended by private respondents, the present petition is barred by Our resolutions of April 26,
1967 and May 19, 1967 in G.R. No. L-27272 (Eduardo Claparols, et. al. vs. CIR, et. al.) [pp. 77-83, rec. of L- Petitioners in the present case do not dispute that as a matter of tradition, the company has been doling out
27272], dismissing said case, wherein said petitioners invoked the applicability of the doctrine in Sta. Cecilia bonuses to employees. In fact, the company balance sheets for the years 1956 to 1962 contained bonus and
Sawmills, Inc. vs. CIR, et. al. (L-19273-74, Feb. 29, 1964, 10 SCRA 433) and impugned the illegality of the order pension computations which were never repudiated or questioned by petitioners. As such, bonus for a given
of respondent Court dated November 28, 1966 directing the computation and payment of the bonuses, aside year earmarked as a matter of tradition for distribution to employees has formed part of their recoverable
from back wages on the ground that these bonuses were not included in the decision of September 16, 1963, wages from the company. Moreover, with greater reason, should recovery of bonuses as part of back wages be
which had long become final. observed in the present case since the company, in the light of the very admission of company accountant
Francisco Cusi, distributes bonuses to its employees even if the company has suffered losses. Specifically,
petitioner company has done this in 1962 (t.s.n., p. 149, Sept. 20, 1965).
The aforesaid resolutions in G.R. No. L-27272 constitute the law of the instant case, wherein herein petitioners
raised again practically the same issues invoked in the abovementioned case. The denial of the petition in G.R.
No. L-27272 suffices to warrant the denial of the present petition; and We need not go any further. Since bonuses are part of back wages of private respondents, the order of May 30, 1969, directing the payment
of their bonuses, did not amend the decision of September 16, 1963 of respondent Court directing payment of
their wages, which has long become final and executory, in the same way that the previous order of May 14,
However, without lending a sympathetic ear to the obvious desire of herein petitioners of this Court to re-
1964 granting execution of said decision of September 16, 1963 also directed the computation of the wages to
examine which would be an exercise in futility the final ruling in G.R. No. L-27272, which as above-stated
be paid to private respondents as decreed by the decision of September 16, 1963. All the orders of May 30,
is the law of the instant case, but solely to remind herein petitioners, We reiterate the governing principles.
1969, November 28, 1966 and May 14, 1964 merely implement the already final and executory decision of
September 16, 1963.
WE uniformly held that "a bonus is not a demandable and enforceable obligation, except when it is a part of
the wage or salary compensation" (Philippine Education Co. vs. CIR and the Union of Philippine Co. Employees
Petitioners insist that We adopt the ruling in the Sta. Cecilia Sawmills case wherein the recoverable back wages
[NLU], 92 Phil. 381; Ansay, et. al. vs. National Development Co., et. al., 107 Phil. 998, 999; Emphasis supplied).
were limited to only three (3) months; because as in the Sta. Cecilia Sawmills case, the Claparols Steel and Nail
Plant ceased operations due to enormous business reverses.
In Atok Big Wedge Mining Co. vs. Atok Big Wedge Mutual Benefit Association (92 Phil. 754), this Court, thru
Justice Labrador, held:
Respondent Court's findings that indeed the Claparols Steel and Nail Plant, which ceased operation of June 30,
1957, was SUCCEEDED by the Claparols Steel Corporation effective the next day, July 1, 1957 up to December
Whether or not bonus forms part of wages depends upon the condition or circumstance for its 7, 1962, when the latter finally ceased to operate, were not disputed by petitioners. It is very clear that the
payment. If it is an additional compensation WHICH THE EMPLOYER PROMISED AND AGREED to give latter corporation was a continuation and successor of the first entity, and its emergence was skillfully timed to
avoid the financial liability that already attached to its predecessor, the Claparols Steel and Nail Plant. Both
Set V Corporation Code * Doctrine of Peircing Corpo Veil Cases*Page 76 of 164

predecessors and successor were owned and controlled by the petitioner Eduardo Claparols and there was no IN VIEW OF THE FOREGOING, we find no legal basis to support the assessment in question against
break in the succession and continuity of the same business. This "avoiding-the-liability" scheme is very patent, petitioner. If at all, the assessment should have been directed against JACKBILT, the manufacturer.
considering that 90% of the subscribed shares of stocks of the Claparols Steel Corporation (the second Accordingly, the decision appealed from is reversed, and the surety bond filed to guarantee payment
corporation) was owned by respondent (herein petitioner) Claparols himself, and all the assets of the dissolved of said assessment is ordered cancelled. No pronouncement as to costs.
Claparols Steel and Nail Plant were turned over to the emerging Claparols Steel Corporation.
Norton and Harrison is a corporation organized in 1911, (1) to buy and sell at wholesale and retail, all kinds of
It is very obvious that the second corporation seeks the protective shield of a corporate fiction whose veil in the goods, wares, and merchandise; (2) to act as agents of manufacturers in the United States and foreign
present case could, and should, be pierced as it was deliberately and maliciously designed to evade its countries; and (3) to carry on and conduct a general wholesale and retail mercantile establishment in the
financial obligation to its employees. Philippines. Jackbilt is, likewise, a corporation organized on February 16, 1948 primarily for the purpose of
making, producing and manufacturing concrete blocks. Under date of July 27, 1948. Norton and Jackbilt entered
It is well remembering that in Yutivo & Sons Hardware Company vs. Court of Tax Appeals (L-13203, Jan. 28, into an agreement whereby Norton was made the sole and exclusive distributor of concrete blocks
1961, 1 SCRA 160), We held that when the notion of legal entity is used to defeat public convenience, justify manufactured by Jackbilt. Pursuant to this agreement, whenever an order for concrete blocks was received by
wrong, protect fraud, or defend crime, the law will regard the corporation as an association or persons, or, in the Norton & Harrison Co. from a customer, the order was transmitted to Jackbilt which delivered the
the case of two corporations, will merge them into one. merchandise direct to the customer. Payment for the goods is, however, made to Norton, which in turn pays
Jackbilt the amount charged the customer less a certain amount, as its compensation or profit. To exemplify the
sales procedures adopted by the Norton and Jackbilt, the following may be cited. In the case of the sale of 420
In Liddel & Company, Inc. vs. Collector of Internal Revenue (L-9687, June 30, 1961, 2 SCRA 632), this Court
pieces of concrete blocks to the American Builders on April 1, 1952, the purchaser paid to Norton the sum of
likewise held that where a corporation is a dummy and serves no business purpose and is intended only as a
P189.00 the purchase price. Out of this amount Norton paid Jackbilt P168.00, the difference obviously being its
blind, the corporate fiction may be ignored.
compensation. As per records of Jackbilt, the transaction was considered a sale to Norton. It was under this
procedure that the sale of concrete blocks manufactured by Jackbilt was conducted until May 1, 1953, when the
In Commissioner of Internal Revenue vs. Norton and Harrison Company (L-17618, Aug. 31, 1964, 11 SCRA 714), agency agreement was terminated and a management agreement between the parties was entered into. The
We ruled that where a corporation is merely an adjunct, business conduit or alter ego of another corporation, management agreement provided that Norton would sell concrete blocks for Jackbilt, for a fixed monthly fee of
the fiction of separate and distinct corporate entities should be disregarded. P2,000.00, which was later increased to P5,000.00.

To the same uniform effect are the decisions in the cases of Republic vs. Razon (L-17462, May 29, 1967, 20 During the existence of the distribution or agency agreement, or on June 10, 1949, Norton & Harrison acquired
SCRA 234) and A.D. Santos, Inc. vs. Vasquez (L-23586, March 20, 1968, 22 SCRA 1156). by purchase all the outstanding shares of stock of Jackbilt. Apparently, due to this transaction, the
Commissioner of Internal Revenue, after conducting an investigation, assessed the respondent Norton &
WE agree with respondent Court of Industrial Relations, therefore, that the amount of back wages recoverable Harrison for deficiency sales tax and surcharges in the amount of P32,662.90, making as basis thereof the sales
by respondent workers from petitioners should be the amount accruing up to December 7, 1962 when the of Norton to the Public. In other words, the Commissioner considered the sale of Norton to the public as
Claparols Steel Corporation ceased operations.WHEREFORE, PETITION IS HEREBY DENIED WITH TREBLE COSTS the original sale and not the transaction from Jackbilt. The period covered by the assessment was from July 1,
AGAINST PETITIONERS TO BE PAID BY THEIR COUNSEL. 1949 to May 31, 1953. As Norton and Harrison did not conform with the assessment, the matter was brought to
the Court of Tax Appeals.

EN BANC G.R. No. L-17618 August 31, 1964


The Commissioner of Internal Revenue contends that since Jackbilt was owned and controlled by Norton &
Harrison, the corporate personality of the former (Jackbilt) should be disregarded for sales tax purposes, and
COMMISSIONER OF INTERNAL REVENUE, petitioner, vs. NORTON and HARRISON
the sale of Jackbilt blocks by petitioner to the public must be considered as the original sales from which the
COMPANY, respondent.
sales tax should be computed. The Norton & Harrison Company contended otherwise that is, the transaction
subject to tax is the sale from Jackbilt to Norton.
.

Wherefore, the parties respectfully pray that the foregoing stipulation of facts be admitted and approved by
PAREDES, J.: this Honorable Court, without prejudice to the parties adducing other evidence to prove their case not covered
by this stipulation of facts. 1wph1.t
This is an appeal interposed by the Commissioner of Internal Revenue against the following judgment of the
Court of Tax Appeals: The majority of the Tax Court, in relieving Norton & Harrison of liability under the assessment, made the
following observations:
Set V Corporation Code * Doctrine of Peircing Corpo Veil Cases*Page 77 of 164

The law applicable to the case is Section 186 of the National Internal Revenue Code which imposes a identities of the two companies should be disregarded. Among these circumstances, which we find not
percentage tax of 7% on every original sale of goods, wares or merchandise, such tax to be based on successfully refuted by appellee Norton are: (a) Norton and Harrison owned all the outstanding stocks of
the gross selling price of such goods, wares or merchandise. The term "original sale" has been defined Jackbilt; of the 15,000 authorized shares of Jackbilt on March 31, 1958, 14,993 shares belonged to Norton and
as the first sale by every manufacturer, producer or importer. (Sec. 5, Com. Act No. 503.) Subsequent Harrison and one each to seven others; (b) Norton constituted Jackbilt's board of directors in such a way as to
sales by persons other than the manufacturer, producer or importer are not subject to the sales tax. enable it to actually direct and manage the other's affairs by making the same officers of the board for both
companies. For instance, James E. Norton is the President, Treasurer, Director and Stockholder of Norton. He
If JACKBILT actually sold concrete blocks manufactured by it to petitioner under the distributorship or also occupies the same positions in Jackbilt corporation, the only change being, in the Jackbilt, he is merely a
agency agreement of July 27, 1948, such sales constituted the original sales which are taxable under nominal stockholder. The same is true with Mr. Jordan, F. M. Domingo, Mr. Mantaring, Gilbert Golden and
Section 186 of the Revenue Code, while the sales made to the public by petitioner are subsequent Gerardo Garcia, while they are merely employees of the North they are Directors and nominal stockholders of
sales which are not taxable. But it appears to us that there was no such sale by JACKBILT to petitioner. the Jackbilt (c) Norton financed the operations of the Jackbilt, and this is shown by the fact that the loans
Petitioner merely acted as agent for JACKBILT in the marketing of its products. This is shown by the obtained from the RFC and Bank of America were used in the expansion program of Jackbilt, to pay advances
fact that petitioner merely accepted orders from the public for the purchase of JACKBILT blocks. The for the purchase of equipment, materials rations and salaries of employees of Jackbilt and other sundry
purchase orders were transmitted to JACKBILT which delivered the blocks to the purchaser directly. expenses. There was no limit to the advances given to Jackbilt so much so that as of May 31, 1956, the unpaid
There was no instance in which the blocks ordered by the purchasers were delivered to the petitioner. advances amounted to P757,652.45, which were not paid in cash by Jackbilt, but was offset by shares of stock
Petitioner never purchased concrete blocks from JACKBILT so that it never acquired ownership of such issued to Norton, the absolute and sole owner of Jackbilt; (d) Norton treats Jackbilt employees as its own.
concrete blocks. This being so, petitioner could not have sold JACKBILT blocks for its own account. It Evidence shows that Norton paid the salaries of Jackbilt employees and gave the same privileges as Norton
did so merely as agent of JACKBILT. The distributorship agreement of July 27, 1948, is denominated by employees, an indication that Jackbilt employees were also Norton's employees. Furthermore service rendered
the parties themselves as an "agency for marketing" JACKBILT products. ... . in any one of the two companies were taken into account for purposes of promotion; (e) Compensation given to
board members of Jackbilt, indicate that Jackbilt is merely a department of Norton. The income tax return of
Norton for 1954 shows that as President and Treasurer of Norton and Jackbilt, he received from Norton
xxx xxx xxx
P56,929.95, but received from Jackbilt the measly amount of P150.00, a circumstance which points out that
remuneration of purported officials of Jackbilt are deemed included in the salaries they received from Norton.
Therefore, the taxable selling price of JACKBILT blocks under the aforesaid agreement is the price The same is true in the case of Eduardo Garcia, an employee of Norton but a member of the Board of Jackbilt.
charged to the public and not the amount billed by JACKBILT to petitioner. The deficiency sales tax His Income tax return for 1956 reveals that he received from Norton in salaries and bonuses P4,220.00, but
should have been assessed against JACKBILT and not against petitioner which merely acted as the received from Jackbilt, by way of entertainment, representation, travelling and transportation allowances
former's agent. P3,000.00. However, in the withholding statement (Exh. 28-A), it was shown that the total of P4,200.00 and
P3,000.00 (P7,220.00) was received by Garcia from Norton, thus portraying the oneness of the two companies.
xxx xxx xxx The Income Tax Returns of Albert Golden and Dioscoro Ramos both employees of Norton but board members of
Jackbilt, also disclose the game method of payment of compensation and allowances. The offices of Norton and
Jackbilt are located in the same compound. Payments were effected by Norton of accounts for Jackbilt and vice
Presiding Judge Nable of the same Court expressed a partial dissent, stating:
versa. Payments were also made to Norton of accounts due or payable to Jackbilt and vice versa.

Upon the aforestated circumstances, which disclose Norton's control over and direction of Jackbilt's
Norton and Harrison, while not denying the presence of the set up stated above, tried to explain that the
affairs, the corporate personality of Jackbilt should be disregarded, and the transactions between
control over the affairs of Jackbilt was not made in order to evade payment of taxes; that the loans obtained by
these two corporations relative to the concrete blocks should be ignored in determining the
it which were given to Jackbilt, were necessary for the expansion of its business in the manufacture of concrete
percentage tax for which Norton is liable. Consequently, the percentage tax should be computed on
blocks, which would ultimately benefit both corporations; that the transactions and practices just mentioned,
the basis of the sales of Jackbilt blocks to the public.
are not unusual and extraordinary, but pursued in the regular course of business and trade; that there could be
no confusion in the present set up of the two corporations, because they have separate Boards, their cash
The majority opinion is now before Us on appeal by the Commissioner of Internal Revenue, on four (4) assigned assets are entirely and strictly separate; cashiers and official receipts and bank accounts are distinct and
errors, all of which pose the following propositions: (1) whether the acquisition of all the stocks of the Jackbilt different; they have separate income tax returns, separate balance sheets and profit and loss statements.
by the Norton & Harrison Co., merged the two corporations into a single corporation; (2) whether the basis of These explanations notwithstanding an over-all appraisal of the circumstances presented by the facts of the
the computation of the deficiency sales tax should be the sale of the blocks to the public and not to Norton. case, yields to the conclusion that the Jackbilt is merely an adjunct, business conduit or alter ego, of Norton and
Harrison and that the fiction of corporate entities, separate and distinct from each, should be disregarded. This
It has been settled that the ownership of all the stocks of a corporation by another corporation does not is a case where the doctrine of piercing the veil of corporate fiction, should be made to apply. In the case
necessarily breed an identity of corporate interest between the two companies and be considered as a of Liddell & Co. Inc. v. Coll. of Int. Rev., supra, it was held:
sufficient ground for disregarding the distinct personalities (Liddell & Co., Inc. v. Coll. of Int. Rev. L-9687, June
30, 1961). However, in the case at bar, we find sufficient grounds to support the theory that the separate
Set V Corporation Code * Doctrine of Peircing Corpo Veil Cases*Page 78 of 164

There are quite a series of conspicuous circumstances that militates against the separate and distinct basis and their returns are accurate, we would have the following result: Jackbilt declared a taxable net income
personality of Liddell Motors Inc., from Liddell & Co. We notice that the bulk of the business of Liddell of P161,202.31 in which the income tax due was computed at P37,137.00 (Exh. 8); whereas Norton declared as
& Co. was channel Red through Liddell Motors, Inc. On the other hand, Liddell Motors Inc. pursued no taxable, a net income of P120,101.59, on which the income tax due was computed at P25,628.00. The total of
activities except to secure cars, trucks, and spare parts from Liddell & Co., Inc. and then sell them to these liabilities is P50,764.84. On the other hand, if the net taxable earnings of both corporations are
the general public. These sales of vehicles by Liddell & Co, to Liddell Motors. Inc. for the most part combined, during the same taxable year, the tax due on their total which is P281,303.90 would be P70,764.00.
were shown to have taken place on the same day that Liddell Motors, Inc. sold such vehicles to the So that, even on the question of income tax alone, it would be to the advantages of Norton that the
public. We may even say that the cars and trucks merely touched the hands of Liddell Motors, Inc. as a corporations should be regarded as separate entities.WHEREFORE, the decision appealed from should be as it
matter of formality. is hereby reversed and another entered making the appellee Norton & Harrison liable for the deficiency sales
taxes assessed against it by the appellant Commissioner of Internal Revenue, plus 25% surcharge thereon.
xxx xxx xxx Costs against appellee Norton & Harrison.

Accordingly, the mere fact that Liddell & Co. and Liddell Motors, Inc. are corporations owned and FIRST DIVISION G.R. No. 108734 May 29, 1996
controlled by Frank Liddell directly or indirectly is not by itself sufficient to justify the disregard of the
separate corporate identity of one from the other. There is however, in this instant case, a peculiar CONCEPT BUILDERS, INC., petitioner, vs.
sequence of the organization and activities of Liddell Motors, Inc. THE NATIONAL LABOR RELATIONS COMMISSION, (First Division); and Norberto Marabe; Rodolfo
Raquel, Cristobal Riego, Manuel Gillego, Palcronio Giducos, Pedro Aboigar, Norberto Comendador,
As opined in the case of Gregory v. Helvering "the legal right of a tax payer to decrease the amount of Rogelio Salut, Emilio Garcia, Jr., Mariano Rio, Paulina Basea, Alfredo Albera, Paquito Salut,
what otherwise would be his taxes, or altogether avoid them, by means which the law permits, cannot Domingo Guarino, Romeo Galve, Dominador Sabina, Felipe Radiana, Gavino Sualibio, Moreno
be doubted". But as held in another case, "where a corporation is a dummy, is unreal or a sham and Escares, Ferdinand Torres, Felipe Basilan, and Ruben Robalos, respondents.
serves no business purpose and is intended only as a blind, the corporate form may be ignored for the
law cannot countenance a form that is bald and a mischievous fictions". HERMOSISIMA, JR., J.:p

... a taxpayer may gain advantage of doing business thru a corporation if he pleases, but the revenue
officers in proper cases, may disregard the separate corporate entity where it serves but as a shield
for tax evasion and treat the person who actually may take benefits of the transactions as the person The corporate mask may be lifted and the corporate veil may be pierced when a corporation is just but the
accordingly taxable. alter ego of a person or of another corporation. Where badges of fraud exist; where public convenience is
defeated; where a wrong is sought to be justified thereby, the corporate fiction or the notion of legal entity
... to allow a taxpayer to deny tax liability on the ground that the sales were made through another should come to naught. The law in these instances will regard the corporation as a mere association of persons
and distinct corporation when it is proved that the latter is virtually owned by the former or that they and, in case of two corporations, merge them into one.
are practically one and the same is to sanction a circumvention of our tax laws. (and cases cited
therein.) Thus, where a sister corporation is used as a shield to evade a corporation's subsidiary liability for damages,
the corporation may not be heard to say that it has a personality separate and distinct from the other
In the case of Yutivo Sons Hardware Co. v. Court of Tax Appeals, L-13203, Jan. 28, 1961, this Court made a corporation. The piercing of the corporate veil comes into play.
similar ruling where the circumstances of unity of corporate identities have been shown and which are identical
to those obtaining in the case under consideration. Therein, this Court said: This special civil action ostensibly raises the question of whether the National Labor Relations Commission
committed grave abuse of discretion when it issued a "break-open order" to the sheriff to be enforced against
We are, however, inclined to agree with the court below that SM was actually owned and controlled by personal property found in the premises of petitioner's sister company.
petitioner as to make it a mere subsidiary or branch of the latter created for the purpose of selling the
vehicles at retail (here concrete blocks) ... . Petitioner Concept Builders, Inc., a domestic corporation, with principal office at 355 Maysan Road, Valenzuela,
Metro Manila, is engaged in the construction business. Private respondents were employed by said company as
It may not be amiss to state in this connection, the advantages to Norton in maintaining a semblance of laborers, carpenters and riggers.
separate entities. If the income of Norton should be considered separate from the income of Jackbilt, then each
would declare such earning separately for income tax purposes and thus pay lesser income tax. The combined On November, 1981, private respondents were served individual written notices of termination of employment
taxable Norton-Jackbilt income would subject Norton to a higher tax. Based upon the 1954-1955 income tax by petitioner, effective on November 30, 1981. It was stated in the individual notices that their contracts of
return of Norton and Jackbilt (Exhs. 7 & 8), and assuming that both of them are operating on the same fiscal employment had expired and the project in which they were hired had been completed.
Set V Corporation Code * Doctrine of Peircing Corpo Veil Cases*Page 79 of 164

Public respondent found it to be, the fact, however, that at the time of the termination of private respondent's The said special sheriff recommended that a "break-open order" be issued to enable him to enter petitioner's
employment, the project in which they were hired had not yet been finished and completed. Petitioner had to premises so that he could proceed with the public auction sale of the aforesaid personal properties on
engage the services of sub-contractors whose workers performed the functions of private respondents. November 7, 1989.

Aggrieved, private respondents filed a complaint for illegal dismissal, unfair labor practice and non-payment of On November 6, 1989, a certain Dennis Cuyegkeng filed a third-party claim with the Labor Arbiter alleging that
their legal holiday pay, overtime pay and thirteenth-month pay against petitioner. the properties sought to be levied upon by the sheriff were owned by Hydro (Phils.), Inc. (HPPI) of which he is
the Vice-President.
On December 19, 1984, the Labor Arbiter rendered judgment 1 ordering petitioner to reinstate private
respondents and to pay them back wages equivalent to one year or three hundred working days. On November 23, 1989, private respondents filed a "Motion for Issuance of a Break-Open Order," alleging that
HPPI and petitioner corporation were owned by the same incorporator/stockholders. They also alleged that
On November 27, 1985, the National Labor Relations Commission (NLRC) dismissed the motion for petitioner temporarily suspended its business operations in order to evade its legal obligations to them and
reconsideration filed by petitioner on the ground that the said decision had already become final and that private respondents were willing to post an indemnity bond to answer for any damages which petitioner
executory. 2 and HPPI may suffer because of the issuance of the break-open order.

On October 16, 1986, the NLRC Research and Information Department made the finding that private In support of their claim against HPPI, private respondents presented duly certified copies of the General
respondents' back wages amounted to P199,800.00. 3 Informations Sheet, dated May 15, 1987, submitted by petitioner to the Securities Exchange Commission (SEC)
and the General Information Sheet, dated May 25, 1987, submitted by HPPI to the Securities and Exchange
Commission.
On October 29, 1986, the Labor Arbiter issued a writ of execution directing the sheriff to execute the Decision,
dated December 19, 1984. The writ was partially satisfied through garnishment of sums from petitioner's
debtor, the Metropolitan Waterworks and Sewerage Authority, in the amount of P81,385.34. Said amount was The General Information Sheet submitted by the petitioner revealed the following:
turned over to the cashier of the NLRC.
1. Breakdown of Subscribed Capital
On February 1, 1989, an Alias Writ of Execution was issued by the Labor Arbiter directing the sheriff to collect
from herein petitioner the sum of P117,414.76, representing the balance of the judgment award, and to Name of Stockholder Amount Subscribed
reinstate private respondents to their former positions.
HPPI P 6,999,500.00
On July 13, 1989, the sheriff issued a report stating that he tried to serve the alias writ of execution on
petitioner through the security guard on duty but the service was refused on the ground that petitioner no Antonio W. Lim 2,900,000.00
longer occupied the premises.

Dennis S. Cuyegkeng 300.00


On September 26, 1986, upon motion of private respondents, the Labor Arbiter issued a second alias writ of
execution.
Elisa C. Lim 100,000.00

The said writ had not been enforced by the special sheriff because, as stated in his progress report, dated
Teodulo R. Dino 100.00
November 2, 1989:

Virgilio O. Casino 100.00


1. All the employees inside petitioner's premises at 355 Maysan Road, Valenzuela, Metro Manila, claimed that
they were employees of Hydro Pipes Philippines, Inc. (HPPI) and not by respondent;
2. Board of Directors
2. Levy was made upon personal properties he found in the premises;
Antonio W. Lim Chairman
4
3. Security guards with high-powered guns prevented him from removing the properties he had levied upon.
Dennis S. Cuyegkeng Member
Set V Corporation Code * Doctrine of Peircing Corpo Veil Cases*Page 80 of 164

Elisa C. Lim Member Antonio W. Lim Chairman

Teodulo R. Dino Member Elisa C. Lim Member

Virgilio O. Casino Member Dennis S. Cuyegkeng Member

3. Corporate Officers Virgilio O. Casino Member

Antonio W. Lim President Teodulo R. Dino Member

Dennis S. Cuyegkeng Assistant to the President 3. Corporate Officers

Elisa O. Lim Treasurer Antonio W. Lim President

Virgilio O. Casino Corporate Secretary Dennis S. Cuyegkeng Assistant to the President

4. Principal Office Elisa C. Lim Treasurer

355 Maysan Road Virgilio O. Casino Corporate Secretary

Valenzuela, Metro Manila. 5 4. Principal Office

On the other hand, the General Information Sheet of HPPI revealed the following: 355 Maysan Road, Valenzuela, Metro Manila. 6

1. Breakdown of Subscribed Capital On February 1, 1990, HPPI filed an Opposition to private respondents' motion for issuance of a break-open
order, contending that HPPI is a corporation which is separate and distinct from petitioner. HPPI also alleged
Name of Stockholder Amount Subscribed that the two corporations are engaged in two different kinds of businesses, i.e., HPPI is a manufacturing firm
while petitioner was then engaged in construction.

Antonio W. Lim P 400,000.00


On March 2, 1990, the Labor Arbiter issued an Order which denied private respondents' motion for break-open
order.
Elisa C. Lim 57,700.00

Private respondents then appealed to the NLRC. On April 23, 1992, the NLRC set aside the order of the Labor
AWL Trading 455,000.00
Arbiter, issued a break-open order and directed private respondents to file a bond. Thereafter, it directed the
sheriff to proceed with the auction sale of the properties already levied upon. It dismissed the third-party claim
Dennis S. Cuyegkeng 40,100.00 for lack of merit.

Teodulo R. Dino 100.00 Petitioner moved for reconsideration but the motion was denied by the NLRC in a Resolution, dated December
3, 1992.
Virgilio O. Casino 100.00
Hence, the resort to the present petition.
2. Board of Directors
Set V Corporation Code * Doctrine of Peircing Corpo Veil Cases*Page 81 of 164

Petitioner alleges that the NLRC committed grave abuse of discretion when it ordered the execution of its 1. Control, not mere majority or complete stock control, but complete domination, not only of finances
decision despite a third-party claim on the levied property. Petitioner further contends, that the doctrine of but of policy and business practice in respect to the transaction attacked so that the corporate entity
piercing the corporate veil should not have been applied, in this case, in the absence of any showing that it as to this transaction had at the time no separate mind, will or existence of its own;
created HPPI in order to evade its liability to private respondents. It also contends that HPPI is engaged in the
manufacture and sale of steel, concrete and iron pipes, a business which is distinct and separate from 2. Such control must have been used by the defendant to commit fraud or wrong, to perpetuate the
petitioner's construction business. Hence, it is of no consequence that petitioner and HPPI shared the same violation of a statutory or other positive legal duty or dishonest and unjust act in contravention of
premises, the same President and the same set of officers and subscribers. 7 plaintiff's legal rights; and

We find petitioner's contention to be unmeritorious. 3. The aforesaid control and breach of duty must proximately cause the injury or unjust loss
complained of.
It is a fundamental principle of corporation law that a corporation is an entity separate and distinct from its
stockholders and from other corporations to which it may be connected. 8 But, this separate and distinct The absence of any one of these elements prevents "piercing the corporate veil." In applying the
personality of a corporation is merely a fiction created by law for convenience and to promote justice. 9 So, "instrumentality" or "alter ego" doctrine, the courts are concerned with reality and not form, with how
when the notion of separate juridical personality is used to defeat public convenience, justify wrong, protect the corporation operated and the individual defendant's relationship to that operation. 14
fraud or defend crime, or is used as a device to defeat the labor laws, 10 this separate personality of the
corporation may be disregarded or the veil of corporate fiction pierced. 11 This is true likewise when the
Thus the question of whether a corporation is a mere alter ego, a mere sheet or paper corporation, a sham or a
corporation is merely an adjunct, a business conduit or an alter ego of another corporation. 12
subterfuge is purely one of fact. 15

The conditions under which the juridical entity may be disregarded vary according to the peculiar facts and
In this case, the NLRC noted that, while petitioner claimed that it ceased its business operations on April 29,
circumstances of each case. No hard and fast rule can be accurately laid down, but certainly, there are some
1986, it filed an Information Sheet with the Securities and Exchange Commission on May 15, 1987, stating that
probative factors of identity that will justify the application of the doctrine of piercing the corporate veil, to wit:
its office address is at 355 Maysan Road, Valenzuela, Metro Manila. On the other hand, HPPI, the third-party
claimant, submitted on the same day, a similar information sheet stating that its office address is at 355
1. Stock ownership by one or common ownership of both corporations. Maysan Road, Valenzuela, Metro Manila.

2. Identity of directors and officers. Furthermore, the NLRC stated that:

3. The manner of keeping corporate books and records. Both information sheets were filed by the same Virgilio O. Casio as the corporate secretary of both
corporations. It would also not be amiss to note that both corporations had the same president,
4. Methods of conducting the business. 13 thesame board of directors, the same corporate officers, and substantially the same subscribers.

The SEC en banc explained the "instrumentality rule" which the courts have applied in disregarding the From the foregoing, it appears that, among other things, the respondent (herein petitioner) and the
separate juridical personality of corporations as follows: third-party claimant shared the same address and/or premises. Under this circumstances, (sic) it
cannot be said that the property levied upon by the sheriff were not of respondents. 16
Where one corporation is so organized and controlled and its affairs are conducted so that it is, in fact,
a mere instrumentality or adjunct of the other, the fiction of the corporate entity of the Clearly, petitioner ceased its business operations in order to evade the payment to private respondents of back
"instrumentality" may be disregarded. The control necessary to invoke the rule is not majority or even wages and to bar their reinstatement to their former positions. HPPI is obviously a business conduit of
complete stock control but such domination of instances, policies and practices that the controlled petitioner corporation and its emergence was skillfully orchestrated to avoid the financial liability that already
corporation has, so to speak, no separate mind, will or existence of its own, and is but a conduit for its attached to petitioner corporation.
principal. It must be kept in mind that the control must be shown to have been exercised at the time
the acts complained of took place. Moreover, the control and breach of duty must proximately cause The facts in this case are analogous to Claparols v. Court of Industrial Relations, 17 where we had the occasion
the injury or unjust loss for which the complaint is made. to rule:

The test in determining the applicability of the doctrine of piercing the veil of corporate fiction is as follows: Respondent court's findings that indeed the Claparols Steel and Nail Plant, which ceased operation of
June 30, 1957, was SUCCEEDED by the Claparols Steel Corporation effective the next day, July 1,
Set V Corporation Code * Doctrine of Peircing Corpo Veil Cases*Page 82 of 164

1957, up to December 7, 1962, when the latter finally ceased to operate, were not disputed by COMPLEX ELECTRONICS CORPORATION, petitioner, vs.NATIONAL LABOR RELATIONS COMMISSION,
petitioner. It is very clear that the latter corporation was a continuation and successor of the first COMPLEX ELECTRONICS EMPLOYEES ASSOCIATION (CEEA), represented by Union President,
entity . . . . Both predecessors and successor were owned and controlled by petitioner Eduardo CECILIA TALAVERA, respondents.
Claparols and there was no break in the succession and continuity of the same business. This
"avoiding-the-liability" scheme is very patent, considering that 90% of the subscribed shares of stock
of the Claparols Steel Corporation (the second corporation) was owned by respondent . . . Claparols
himself, and all the assets of the dissolved Claparols Steel and Nail plant were turned over to the
KAPUNAN, J.:
emerging Claparols Steel Corporation.

These consolidated cases filed by Complex Electronics Employees Association (G.R. No. 121315) and Complex
It is very obvious that the second corporation seeks the protective shield of a corporate fiction whose
Electronics Corporation (G.R. No. 122136) assail the Decision of the NLRC dated March 10, 1995 which set aside
veil in the present case could, and should, be pierced as it was deliberately and maliciously designed
the Decision of the Labor Arbiter dated April 30, 1993.
to evade its financial obligation to its employees.

The antecedents of the present petitions are as follows:


In view of the failure of the sheriff, in the case at bar, to effect a levy upon the property subject of the
execution, private respondents had no other recourse but to apply for a break-open order after the third-party
claim of HPPI was dismissed for lack of merit by the NLRC. This is in consonance with Section 3, Rule VII of the Complex Electronics Corporation (Complex) was engaged in the manufacture of electronic products. It was
NLRC Manual of Execution of Judgment which provides that: actually a subcontractor of electronic products where its customers gave their job orders, sent their own
materials and consigned their equipment to it. The customers were foreign-based companies with different
product lines and specifications requiring the employment of workers with specific skills for each product line.
Should the losing party, his agent or representative, refuse or prohibit the Sheriff or his representative
Thus, there was the AMS Line for the Adaptive Micro System, Inc., the Heril Line for Heril Co., Ltd., the Lite-On
entry to the place where the property subject of execution is located or kept, the judgment creditor
Line for the Lite-On Philippines Electronics Co., etc.
may apply to the Commission or Labor Arbiter concerned for a break-open order.

The rank and file workers of Complex were organized into a union known as the Complex Electronics Employees
Furthermore, our perusal of the records shows that the twin requirements of due notice and hearing were
Association, herein referred to as the Union.
complied with. Petitioner and the third-party claimant were given the opportunity to submit evidence in support
of their claim.
On March 4, 1992, Complex received a facsimile message from Lite-On Philippines Electronics Co., requiring it
to lower its price by 10%. The full text reads as follows:
Hence, the NLRC did not commit any grave abuse of discretion when it affirmed the break-open order issued by
the Labor Arbiter.Finally, we do not find any reason to disturb the rule that factual findings of quasi-judicial
agencies supported by substantial evidence are binding on this Court and are entitled to great respect, in the This is to inform your office that Taiwan required you to reduce your assembly cost since it is higher by
absence of showing of grave abuse of a discretion. 18WHEREFORE, the petition is DISMISSED and the assailed 50% and no longer competitive with that of mainland China. It is further instructed that Complex Price
resolutions of the NLRC, dated April 23, 1992 and December 3, 1992, are AFFIRMED.SO ORDERED. be patterned with that of other sources, which is 10% lower.

1
FIRST DIVISION Please consider and give us your revised rates soon.

G.R. No. 121315 July 19, 1999 Consequently, on March 9, 1992, a meeting was held between Complex and the personnel of the Lite-On
Production Line. Complex informed its Lite-On personnel that such request of lowering their selling price by
10% was not feasible as they were already incurring losses at the present prices of their products. Under such
COMPLEX ELECTRONICS EMPLOYEES ASSOCIATION (CEEA) represented by its union president
circumstances, Complex regretfully informed the employees that it was left with no alternative but to close
CECILIA TALAVERA, GEORGE ARSOLA, MARIO DIAGO AND SOCORRO BONCAYAO, petitioners, vs.
down the operations of the Lite-On Line. The company, however, promised that:
THE NATIONAL LABOR RELATIONS COMMISSION, COMPLEX ELECTRONICS CORPORATION, IONICS
CIRCUIT, INC., LAWRENCE QUA, REMEDIOS DE JESUS, MANUEL GONZAGA, ROMY DELA ROSA,
TERESITA ANDINO, ARMAN CABACUNGAN, GERRY GABANA, EUSEBIA MARANAN and BERNADETH 1) Complex will follow the law by giving the people to be retrenched the necessary 1 month notice.
GACAD, respondents. Hence, retrenchment will not take place until after 1 month from March 09, 1992.

G.R. No. 122136 July 19, 1999


Set V Corporation Code * Doctrine of Peircing Corpo Veil Cases*Page 83 of 164

2) The Company will try to prolong the work for as many people as possible for as long as it can by exercise of management prerogatives. Likewise, the transfer of the machinery, equipment and materials from
looking for job slots for them in another line if workload so allows and if their skills are compatible with Complex was the decision of the owners who were common customers of Complex and Ionics.
the line requirement.
On April 30, 1993, the Labor Arbiter rendered a decision the dispositive portion of which reads:
3) The company will give the employees to be retrenched a retrenchment pay as provided for by
law i.e. half a month for every year of service in accordance with Article 283 of the Labor Code of WHEREFORE, all the foregoing premises being considered, judgment is hereby rendered ordering the
Philippines. 2 respondent Complex Electronics Corporation and/or Ionics Circuit Incorporated and/or Lawrence Qua,
to reinstate the 531 above-listed employees to their former position with all the rights, privileges and
The Union, on the other hand, pushed for a retrenchment pay equivalent to one (1) month salary for every year benefits appertaining thereto, and to pay said complainants-employees the aggregate backwages
of service, which Complex refused. amounting P26,949,891.80 as of April 6, 1993 and to such further backwages until their actual
reinstatement. In the event reinstatement is no longer feasible for reasons not attributable to the
On March 13, 1992, Complex filed a notice of closure of the Lite-On Line with the Department of Labor and complainants, said respondents are also liable to pay complainants-employees their separation pay to
Employment (DOLE) and the retrenchment of the ninety-seven (97) affected employees. 3 be computed at the rate of one (1) month pay for every year of service, a fraction of at least six (6)
months to be considered as one whole year.

On March 25, 1993, the Union filed a notice of strike with the National Conciliation and Mediation Board
(NCMB).1wphi1.nt Further, the aforenamed three (3) respondents are hereby ordered to pay jointly and solidarily the
complainants-employees an aggregate moral damages in the amount of P1,062,000.00 and exemplary
damages in the aggregate sum of P531,000.00.
Two days thereafter, or on March 27, 1993, the Union conducted a strike vote which resulted in a "yes" vote.

And finally, said respondents are ordered to pay attorney's fees equivalent to ten percent (10%) of
In the evening of April 6, 1992, the machinery, equipment and materials being used for production at Complex
whatever has been adjudicated herein in favor of the complainants.
were pulled-out from the company premises and transferred to the premises of Ionics Circuit, Inc. (Ionics) at
Cabuyao, Laguna. The following day, a total closure of company operation was effected at Complex.
The charge of slowdown strike filed by respondent Complex against the union is hereby dismissed for
lack of merit.SO ORDERED. 5
A complaint was, thereafter, filed with the Labor Arbitration Branch of the NLRC for unfair labor practice, illegal
closure/illegal lockout, money claims for vacation leave, sick leave, unpaid wages, 13th month pay, damages
and attorney's fees. The Union alleged that the pull-out of the machinery, equipment and materials from the Separate appeals were filed by Complex, Ionics and Lawrence Qua before the respondent NLRC which rendered
company premises, which resulted to the sudden closure of the company was in violation of Section 3 and 8, the questioned decision on March 10, 1995, the decretal portion of which states:
Rule XIII, Book V of the Labor Code of the Philippines 4 and the existing CBA. Ionics was impleaded as a party
defendant because the officers and management personnel of Complex were also holding office at Ionics with WHEREFORE, premises considered, the assailed decision is hereby ordered vacated and set aside, and
Lawrence Qua as the President of both companies. a new one entered ordering respondent Complex Electronics Corporation to pay 531 complainants
equivalent to one month pay in lieu of notice and separation pay equivalent to one month pay for
Complex, on the other hand, averred that since the time the Union filed its notice of strike, there was a every year of service and a fraction of six months considered as one whole year.
significant decline in the quantity and quality of the products in all of the production lines. The delivery
schedules were not met prompting the customers to lodge complaints against them. Fearful that the Respondents Ionics Circuit Incorporated and Lawrence Qua are hereby ordered excluded as parties
machinery, equipment and materials would be rendered inoperative and unproductive due to the impending solidarily liable with Complex Electronics Corporation.
strike of the workers, the customers ordered their pull-out and transfer to Ionics. Thus, Complex was compelled
to cease operations. The award of moral damages is likewise deleted for lack of merit.

Ionics contended that it was an entity separate and distinct from Complex and had been in existence since July Respondent Complex, however, is hereby ordered to pay attorney's fees equivalent to ten (10%)
5, 1984 or eight (8) years before the labor dispute arose at Complex. Like Complex, it was also engaged in the percent of the total amount of award granted the complainants.
semi-conductor business where the machinery, equipment and materials were consigned to them by their
customers. While admitting that Lawrence Qua, the President of Complex was also the President of Ionics, the 6
SO ORDERED.
latter denied having Qua as their owner since he had no recorded subscription of P1,200,00.00 in Ionics as
claimed by the Union. Ionics further argued that the hiring of some displaced workers of Complex was an
Set V Corporation Code * Doctrine of Peircing Corpo Veil Cases*Page 84 of 164

Complex, Ionics and the Union filed their motions for reconsideration of the above decision which were denied On December 23, 1996, the Union filed a motion for consolidation of G.R. No. 122136 with G.R. No.
by the respondent NLRC in an Order dated July 11, 1995. 7 121315. 10The motion was granted by this Court in a Resolution dated June 23, 1997. 11

Hence these petitions. On November 10, 1997, the Union presented additional documentary evidence which consisted of a newspaper
clipping in the Manila Bulletin, dated August 18, 1997 bearing the picture of Lawrence Qua with the following
In G.R. No. 121315, petitioner Complex Electronics Employees Association asseverates that the respondent inscription:
NLRC erred when it:
RECERTIFICATION. The Cabuyao (Laguna) operation of Ionic Circuits, Inc. consisting of plants 2, 3, 4
I and 5 was recertified to ISO 9002 as electronics contract manufacturer by the TUV, a rating firm with
headquarters in Munich, Germany. Lawrence Qua, Ionics president and chief executive officer, holds
the plaque of recertification presented by Gunther Theisz (3rd from left), regional manager of TUV
SET ASIDE THE DECISION DATED APRIL 30, 1993 ISSUED BY THE HON. LABOR ARBITER JOSE DE VERA.
Products Services Asia during ceremonies held at Sta. Elena Golf Club. This is the first of its kind in the
country that four plants were certified at the same time. 12
II
The Union claimed that the said clipping showed that both corporations, Ionics and Complex are one and the
EXCLUDED PRIVATE RESPONDENTS IONICS CIRCUITS, INCORPORATED AND LAWRENCE QUA AS PARTIES same.
SOLIDARILY LIABLE WITH COMPLEX ELECTRONICS CORPORATION.
In answer to this allegation, Ionics explained that the photo which appeared at the Manila Bulletin issue of
III August 18, 1997 pertained only to respondent Ionics' recertification of ISO 9002. There was no mention about
Complex Electronics Corporation. Ionics claimed that a mere photo is insufficient to conclude that Ionics and
FOUND THAT COMPLEX ELECTRONICS CORPORATION WAS NOT GUILTY OF ILLEGAL CLOSURE AND ILLEGAL Complex are one and the same. 13
DISMISSAL OF THE PETITIONERS.
We shall first delve on the issues raised by the petitioner Union.
IV
The Union anchors its position on the fact that Lawrence Qua is both the president of Complex and Ionics and
REMOVED THE AWARD FOR BACKWAGES, REINSTATEMENT AND DAMAGES IN THE DECISION DATED APRIL 30, that both companies have the same set of Board of Directors. It claims that business has not ceased at
1993 ISSUED BY THE HON. LABOR ARBITER JOSE DE VERA. 8 Complex but was merely transferred to Ionics, a runaway shop. To prove that Ionics was just a runaway shop,
petitioner asserts that out of the 80,000 shares comprising the increased capital stock of Ionics, it was Complex
that owns majority of said shares with P1,200,000.00 as its capital subscription and P448,000.00 as its paid up
On the other hand, in G.R. No. 122136, petitioner Complex Electronics Corporation raised the following issues,
investment, compared to P800,000.00 subscription and P324,560.00 paid-up owing to the other stockholders,
to wit:
combined. Thus, according to the Union, there is a clear ground to pierce the veil of corporate fiction.

I
The Union further posits that there was an illegal lockout/illegal dismissal considering that as of March 11,
1992, the company had a gross sales of P61,967,559 from a capitalization of P1,500,000.00. It even ranked
PUBLIC RESPONDENT NLRC ACTED IN GRAVE ABUSE OF DISCRETION AMOUNTING TO LACK OF OR IN EXCESS OF number thirty among the top fifty corporations in Muntinlupa. Complex, therefore, cannot claim that it was
JURISDICTION IN PROMULGATING ITS DECISION AND ORDER DATED 10 MARCH 1995, AND 11 JULY 1995, losing in its business which necessitated its closure.
RESPECTIVELY, THE SAME BEING IN CONTRAVENTION OF THE EXPRESS MANDATE OF THE LAW GOVERNING THE
PAYMENT OF ONE MONTH PAY IN LIEUOF NOTICE, SEPARATION PAY AND ATTORNEY'S FEES.
With regards to Lawrence Qua, petitioner maintains that he should be made personally liable to the Union since
he was the principal player in the closure of the company, not to mention the clandestine and surreptitious
II manner in which such closure was carried out, without regard to their right to due process.

THERE IS NO APPEAL, NOR ANY PLAIN, SPEEDY AND ADEQUATE REMEDY IN THE ORDINARY COURSE OF The Union's contentions are untenable.
LAW. 9
Set V Corporation Code * Doctrine of Peircing Corpo Veil Cases*Page 85 of 164

A "runaway shop" is defined as an industrial plant moved by its owners from one location to another to escape As to the additional documentary evidence which consisted of a newspaper clipping filed by petitioner Union,
union labor regulations or state laws, but the term is also used to describe a plant removed to a new location in we agree with respondent Ionics that the photo/newspaper clipping itself does not prove that Ionics and
order to discriminate against employees at the old plant because of their union Complex are one and the same entity. The photo/newspaper clipping merely showed that some plants of Ionics
activities. 14 It is one wherein the employer moves its business to another location or it temporarily closes its were recertified to ISO 9002 and does not show that there is a relation between Complex and Ionics except for
business for anti-union purposes. 15 A "runaway shop" in this sense, is a relocation motivated by anti- the fact that Lawrence Qua was also the president of Ionics. However, as we have stated above, the mere fact
union animus rather than for business reasons. In this case, however, Ionics was not set up merely for the that both of the corporations have the same president is not in itself sufficient to pierce the veil of corporate
purpose of transferring the business of Complex. At the time the labor dispute arose at Complex, Ionics was fiction of the two corporations.
already existing as an independent company. As earlier mentioned, it has been in existence since July 5, 1984.
It cannot, therefore, be said that the temporary closure in Complex and its subsequent transfer of business to We, likewise, disagree with the Union that there was in this case an illegal lockout/illegal dismissal. Lockout is
Ionics was for anti-union purposes. The Union failed to show that the primary reason for the closure of the the temporary refusal of employer to furnish work as a result of an industrial or labor dispute. 21 It may be
establishment was due to the union activities of the employees. manifested by the employer's act of excluding employees who are union members. 22 In the present case, there
was a complete cessation of the business operations at Complex not because of the labor dispute. It should be
The mere fact that one or more corporations are owned or controlled by the same or single stockholder is not a recalled that, before the labor dispute, Complex had already informed the employees that they would be
sufficient ground for disregarding separate corporate personalities. Thus, in Indophil Textile Mill Workers Union closing the Lite-On Line. The employees, however, demanded for a separation pay equivalent to one (1) month
vs.Calica, 16 we ruled that: salary for every year of service which Complex refused to give. When Complex filed a notice of closure of its
Lite-On Line, the employees filed a notice of strike which greatly alarmed the customers of Complex and this
[I]n the case at bar, petitioner seeks to pierce the veil of corporate entity of Acrylic, alleging that the led to the pull-out of their equipment, machinery and materials from Complex. Thus, without the much needed
creation of the corporation is a devise to evade the application of the CBA between petitioner Union equipment, Complex was unable to continue its business. It was left with no other choice except to shut down
and private respondent company. While we do not discount the possibility of the similarities of the the entire business. The closure, therefore, was not motivated by the union activities of the employees, but
businesses of private respondent and Acrylic, neither are we inclined to apply the doctrine invoked by rather by necessity since it can no longer engage in production without the much needed materials, equipment
petitioner in granting the relief sought. The fact that the businesses of private respondent and Acrylic and machinery. We quote with approval the findings of the respondent NLRC on this matter:
are related, that some of the employees of the private respondent are the same persons manning and
providing for auxiliary services to the units of Acrylic, and that the physical plants, offices and facilities At first glance after reading the decision a quo, it would seem that the closure of respondent's
are situated in the same compound, it is our considered opinion that these facts are not sufficient to operation is not justified. However, a deeper examination of the records along with the evidence,
justify the piercing of the corporate veil of Acrylic. would show that the closure, although it was done abruptly as there was no compliance with the 30-
day prior notice requirement, said closure was not intended to circumvent the provisions of the Labor
Likewise, in Del Rosario vs. National Labor Relations Commission, 17 the Court stated that substantial identity of Code on termination of employment. The closure of operation by Complex on April 7, 1992 was not
the incorporators of two corporations does not necessarily imply that there was fraud committed to justify without valid reasons. Customers of respondent alarmed by the pending labor dispute and the
piercing the veil of corporate fiction. imminent strike to be foisted by the union, as shown by their strike vote, directed respondent Complex
to pull-out its equipment, machinery and materials to other safe bonded warehouse. Respondent
18
being mere consignees of the equipment, machinery and materials were without any recourse but to
In the recent case of Santos vs. National Labor Relations Commission, we also ruled that:
oblige the customers' directive. The pull-out was effected on April 6, 1992. We can see here that
Complex's action, standing alone, will not result in illegal closure that would cause the illegal dismissal
The basic rule is still that which can be deduced from the Court's pronouncement in Sunio vs.National of the complainant workers. Hence, the Labor Arbiter's conclusion that since there were only two (2) of
Labor Relations Commission, thus: respondent's customers who have expressed pull-out of business from respondent Complex while
most of the customer's have not and, therefore, it is not justified to close operation cannot be upheld.
. . . . . Mere ownership by a single stockholder or by another corporation of all or nearly all of the The determination to cease operation is a prerogative of management that is usually not interfered
capital stock of a corporation is not of itself sufficient ground for disregarding the separate corporate with by the State as no employer can be required to continue operating at a loss simply to maintain
personality. the workers in employment. That would be taking of property without due process of law which the
employer has the right to resist. (Columbia Development Corp. vs. Minister of Labor and Employment,
146 SCRA 42).
Ionics may be engaged in the same business as that of Complex, but this fact alone is not enough reason to
pierce the veil of corporate fiction of the corporation. Well-settled is the rule that a corporation has a
personality separate and distinct from that of its officers and stockholders. This fiction of corporate entity can As to the claim of petitioner Union that Complex was gaining profit, the financial statements for the years 1990,
only be disregarded in certain cases such as when it is used to defeat public convenience, justify wrong, protect 1991 and 1992 issued by the auditing and accounting firm Sycip, Gorres and Velayo readily show that Complex
fraud, or defend crime. 19 To disregard said separate juridical personality of a corporation, the wrongdoing must was indeed continuously experiencing deficit and losses. 23 Nonetheless, whether or not Complex was incurring
be clearly and convincingly established. 20
Set V Corporation Code * Doctrine of Peircing Corpo Veil Cases*Page 86 of 164

great losses, it still one of the management's prerogative to close down its business as long as it is done in benefits due the complainants who will be retrenched at the closure of the Lite-On Line, gave a wrong
good faith. Thus, inCatatista et al., vs. NLRC and Victorias Milling Co., Inc. 24 we ruled: signal to customers of Complex, which consequently resulted in the loss of employment of not only a
few but to all the of the workers. It may be worth saying that the right to strike should only be a
In any case, Article 283 of the Labor Code is clear that an employer may close or cease his business remedy of last resort and must not be used as a show of force against the employer. 27
operations or undertaking even if he is not suffering from serious business losses or financial reverses,
as long as he pays his employees their termination pay in the amount corresponding to their length of We shall now go to the issues raised by Complex in G.R. No. 122136.
service. It would indeed, be stretching the intent and spirit of the law if we were to unjustly interfere in
management's prerogative to close or cease its business operations just because said business Complex claims that the respondent NLRC erred in ordering them to pay the Union one (1) month pay as
operations or undertaking is not suffering from any loss. indemnity for failure to give notice to its employees at least thirty (30) days before such closure since it was
quite clear that the employees were notified of the impending closure of the Lite-On Line as early as March 9,
Going now to the issue of personal liability of Lawrence Qua, it is settled that in the absence of malice or bad 1992. Moreover, the abrupt cessation of operations was brought about by the sudden pull-out of the customers
faith, a stockholder or an officer of a corporation cannot be made personally liable for corporate liabilities. 25 In which rendered it impossible for Complex to observe the required thirty (30) days notice.
the present case, while it may be true that the equipment, materials and machinery were pulled-out of
Complex and transferred to Ionics during the night, their action was sufficiently explained by Lawrence Qua in Art. 283 of the Labor Code provides that:
his Comment to the petition filed by the Union. We quote:

Art. 283. Closure of establishment and reduction of personnel. The employer may also terminate
the employment of any employee due to the installation of labor saving devices, redundancy,
retrenchment to prevent losses or the closing or cessation of operation of the establishment or
The fact that the pull-out of the machinery, equipment and materials was effected during nighttime is undertaking unless the closing is for the purpose of circumventing the provisions of this Title, by
not per se an indicia of bad faith on the part of respondent Qua since he had no other recourse, and serving a written notice on the workers and the Ministry of Labor and Employment at least
the same was dictated by the prevailing mood of unrest as the laborers were already vandalizing the one (1)month before the intended date thereof . . . . (Emphasis ours.)
equipment, bent on picketing the company premises and threats to lock out the company officers
were being made. Such acts of respondent Qua were, in fact, made pursuant to the demands of The purpose of the notice requirement is to enable the proper authorities to determine after hearing whether
Complex's customers who were already alarmed by the pending labor dispute and imminent strike to such closure is being done in good faith, i.e., for bona fide business reasons, or whether, to the contrary, the
be stage by the laborers, to have their equipment, machinery and materials pull out of Complex. As closure is being resorted to as a means of evading compliance with the just obligations of the employer to the
such, these acts were merely done pursuant to his official functions and were not, in any way, made employees affected. 28
with evident bad faith. 26

While the law acknowledges the management prerogative of closing the business, it does not, however, allow
We perceive no intention on the part of Lawrence Qua and the other officers of Complex to defraud the the business establishment to disregard the requirements of the law. The case of Magnolia Dairy Products
employees and the Union. They were compelled to act upon the instructions of their customers who were the v. NLRC 29is quite emphatic about this:
real owners of the equipment, materials and machinery. The prevailing labor unrest permeating within the
premises of Complex left the officers with no other choice but to pull them out of Complex at night to prevent
The law authorizes an employer, like the herein petitioners, to terminate the employment of any
their destruction. Thus, we see no reason to declare Lawrence Qua personally liable to the Union.
employee due to the installation of labor saving devices. The installation of these devices is a
management prerogative, and the courts will not interfere with its exercise in the absence of abuse of
Anent the award of damages, we are inclined to agree with the NLRC that there is no basis for such award. We discretion, arbitrariness, or maliciousness on the part of management, as in this case. Nonetheless,
again quote the respondent NLRC with favor: this did not excuse petitioner from complying with the required written notice to the employee and to
the Department of Labor and Employment (DOLE) at least one month before the intended date of
By and large, we cannot hold respondents guilty of unfair labor practice as found by the Labor Arbiter termination. This procedure enables an employee to contest the reality or good faith character of the
since the closure of operation of Complex was not established by strong evidence that the purpose of asserted ground for the termination of his services before the DOLE.
said closure was to interfere with the employees' right to self-organization and collective bargaining.
As very clearly established, the closure was triggered by the customers' pull-out of their equipment, The failure of petitioner to serve the written notice to private respondent and to the DOLE, however,
machinery and materials, who were alarmed by the pending labor dispute and the imminent strike by does not ipso facto make private respondent's termination from service illegal so as to entitle her to
the union, and as a protection to their interest pulled-out of business from Complex who had no reinstatement and payment of backwages. If at all, her termination from service is merely defective
recourse but to cease operation to prevent further losses. The indiscretion committed by the Union in because it was not tainted with bad faith or arbitrariness and was due to a valid cause.
filing the notice of strike, which to our mind is not the proper remedy to question the amount of
Set V Corporation Code * Doctrine of Peircing Corpo Veil Cases*Page 87 of 164

The well settled rule is that the employer shall be sanctioned for non-compliance with the EN BANC A.C. No. 2797 October 4, 2002
requirements of, or for failure to observe due process in terminating from service its employee.
InWenphil Corp. v. NLRC, we sanctioned the employer for this failure by ordering it to indemnify the ROSAURA P. CORDON, complainant, vs. JESUS BALICANTA, respondent.
employee the amount of P1,000.00. Similarly, we imposed the same amount as indemnification
inRubberworld (Phils.), Inc. v. NLRC, and, Aurelio v. NLRC and Alhambra Industries, Inc. v. NLRC.
RESOLUTION
Subsequently, the sum of P5,000.00 was awarded to an employee in Worldwide
Papermills, Inc. v.NLRC, and P2,000.00 in Sebuguero, et al., v. NLRC, et al. Recently, the sum of
P5,000.00 was again imposed as indemnify against the employer. We see no valid and cogent reason PER CURIAM:
why petitioner should not be likewise sanctioned for its failure to serve the mandatory written notice.
Under the attendant facts, we find the amount of P5,000.00, to be just and reasonable. On August 21, 1985, herein complainant Rosaura Cordon filed with this Court a complaint for disbarment,
docketed as Administrative Case No. 2797, against Atty. Jesus Balicanta. After respondents comment to the
We, therefore, find no grave abuse of discretion on the part of the NLRC in ordering Complex to pay one (1) complaint and complainants reply thereto, this Court, on March 29, 1995 referred the matter to the Integrated
month salary by way of indemnity. It must be borne in mind that what is at stake is the means of livelihood of Bar of the Philippines (IBP, for brevity) for investigation, report and recommendation within 90 days from
the workers so they are at least entitled to be formally informed of the management decisions regarding their notice. Commissioner George Briones of the IBP Commission on Bar Discipline was initially tasked to investigate
employment. 30 the case. Commissioner Briones was later on replaced by Commissioner Renato Cunanan. Complainant filed a
supplemental complaint which was duly admitted and, as agreed upon, the parties filed their respective
position papers.
Complex, likewise, maintains that it is not liable for the payment for the payment of separation pay since
Article 283 of the Labor Code awards separation pay only in cases of closure not due to serious business
reversals. In this case, the closure of Complex was brought about by the losses being suffered by the Based on her complaint, supplemental complaint, reply and position paper, the complainant alleged the
corporation. following facts:

We disagree. When her husband Felixberto C. Jaldon died, herein complainant Rosaura Cordon and her daughter Rosemarie
inherited the properties left by the said decedent. All in all, complainant and her daughter inherited 21 parcels
of land located in Zamboanga City. The lawyer who helped her settle the estate of her late husband was
Art. 283 further provides:
respondent Jesus Balicanta.

. . . . In case of termination due to the installation of labor saving devices or redundancy, the worker
Sometime in the early part of 1981, respondent enticed complainant and her daughter to organize a
affected thereby shall be entitled to a separation pay equivalent to at least his one (1) month pay or to
corporation that would develop the said real properties into a high-scale commercial complex with a beautiful
at least one (1) month pay for every year of service, whichever is higher. In case of retrenchment to
penthouse for complainant. Relying on these apparently sincere proposals, complainant and her daughter
prevent losses and in case of cessation of operations of establishment or undertaking not due to
assigned 19 parcels of land to Rosaura Enterprises, Incorporated, a newly-formed and duly registered
serious business losses or financial reverses, the separation pay shall be equivalent to one (1) month
corporation in which they assumed majority ownership. The subject parcels of land were then registered in the
pay or at least one-half (1/2) month pay for every year of service, whichever is higher. A fraction of at
name of the corporation.
least six (6) months shall be considered one (1) whole year.

Thereafter, respondent single-handedly ran the affairs of the corporation in his capacity as Chairman of the
It is settled that in case of closures or cessation of operation of business establishments not due to serious
Board, President, General Manager and Treasurer. The respondent also made complainant sign a document
business losses or financial reverses, 31 the employees are always given separation benefits.
which turned out to be a voting trust agreement. Respondent likewise succeeded in making complainant sign a
special power of attorney to sell and mortgage some of the parcels of land she inherited from her deceased
In the instant case, notwithstanding the financial losses suffered by Complex, such was, however, not the main husband. She later discovered that respondent transferred the titles of the properties to a certain Tion Suy Ong
reason for its closure. Complex admitted in its petition that the main reason for the cessation of the operations who became the new registered owner thereof. Respondent never accounted for the proceeds of said transfers.
was the pull-out of the materials, equipment and machinery from the premises of the corporation as dictated
by its customers. It was actually still capable of continuing the business but opted to close down to prevent
In 1981, respondent, using a spurious board resolution, contracted a loan from the Land Bank of the Philippines
further losses. Under the facts and circumstances of the case, we find no grave abuse of discretion on the part
(LBP, for brevity) in the amount of Two Million Two Hundred Twenty Pesos (P2,220,000) using as collateral 9 of
of the public respondent in awarding the employees one (1) month pay for every year of service as termination
the real properties that the complainant and her daughter contributed to the corporation. The respondent
pay.1wphi1.ntWHEREFORE, premises considered, the assailed decision of the NLRC is AFFIRMED.SO
ostensibly intended to use the money to construct the Baliwasan Commercial Center (BCC, for brevity).
ORDERED.
Complainant later on found out that the structure was made of poor materials such as sawali, coco lumber and
bamboo which could not have cost the corporation anything close to the amount of the loan secured.
Set V Corporation Code * Doctrine of Peircing Corpo Veil Cases*Page 88 of 164

For four years from the time the debt was contracted, respondent failed to pay even a single installment. As a Complainant likewise accused respondent of circulating rumors among her friends and relatives that she had
result, the LBP, in a letter dated May 22, 1985, informed respondent that the past due amortizations and become insane to prevent them from believing whatever complainant said. According to complainant,
interest had already accumulated to Seven Hundred Twenty-nine Thousand Five Hundred Three Pesos and respondent proposed that she legally separate from her present husband so that the latter would not inherit
Twenty-five Centavos (P729,503.25). The LBP made a demand on respondent for payment for the tenth time. from her and that respondent be adopted as her son.
Meanwhile, when the BCC commenced its operations, respondent started to earn revenues from the rentals of
BCCs tenants. On October 28, 1987, the LBP foreclosed on the 9 mortgaged properties due to non-payment of For his defense, respondent, in his comment and position paper, denied employing deceit and machination in
the loan. convincing complainant and her daughter to assign their real properties to the corporation; that they freely and
voluntary executed the deeds of assignment and the voting trust agreement that they signed; that he did not
Respondent did not exert any effort to redeem the foreclosed properties. Worse, he sold the corporations right single-handedly manage the corporation as evidenced by certifications of the officers and directors of the
to redeem the mortgaged properties to a certain Hadji Mahmud Jammang through a fake board resolution corporation; that he did not use spurious board resolutions authorizing him to contract a loan or sell the
dated January 14, 1989 which clothed himself with the authority to do so. Complainant and her daughter, the properties assigned by the complainant and her daughter; that complainant and her daughter should be the
majority stockholders, were never informed of the alleged meeting held on that date. Again, respondent never ones who should render an accounting of the records and revenues inasmuch as, since 1984 up to the present,
accounted for the proceeds of the sale of the right to redeem. Respondent also sold to Jammang a parcel of the part-time corporate book-keeper, with the connivance of the complainant and her daughter, had custody of
land belonging to complainant and her daughter which was contiguous to the foreclosed properties and the corporate records; that complainant and her daughter sabotaged the operation of BCC when they illegally
evidenced by Transfer Certificate of Title No. 62807. He never accounted for the proceeds of the sale. took control of it in 1986; that he never pocketed any of the proceeds of the properties contributed by the
complainant and her daughter; that the demolition of the ancestral home followed legal procedures; that
Sometime in 1983, complainants daughter, Rosemarie, discovered that their ancestral home had been complainant was never detained in Culianan but she freely and voluntarily lived with the family of P03 Joel
demolished and that her mother, herein complainant, was being detained in a small nipa shack in a place Constantino as evidenced by complainants own letter denying she was kidnapped; and that the instant
called Culianan. Through the help of Atty. Linda Lim, Rosemarie was able to locate her mother. Rosemarie later disbarment case should be dismissed for being premature, considering the pendency of cases before the SEC
learned that respondent took complainant away from her house on the pretext that said ancestral home was and the Regional Trial Court of Zamboanga involving him and complainant.
going to be remodeled and painted. But respondent demolished the ancestral home and sold the lot to Tion Suy
Ong, using another spurious board resolution designated as Board Resolution No. 1, series of 1992. The Based on the pleadings and position papers submitted by the parties, Commissioner Renato Cunanan, in his
resolution contained the minutes of an alleged organizational meeting of the directors of the corporation and report1 dated July 1, 1999, recommended respondents disbarment based on the following findings:
was signed by Alexander Wee, Angel Fernando, Erwin Fernando and Gabriel Solivar. Complainant and her
daughter did not know how these persons became stockholders and directors of the corporation. Respondent "A. The complainant, Rosaura Jaldon-Cordon and her daughter, Rosemarie were stockholders of a
again did not account for the proceeds of the sale. corporation, together with respondent, named Rosaura Enterprises, Inc.

Complainant and her daughter made several demands on respondent for the delivery of the real properties "Per the Articles of Incorporation marked as Annex A of Complainants Position Paper, complainants
they allegedly assigned to the corporation, for an accounting of the proceeds of the LBP loan and as well as the subscription consists of 55% of the outstanding capital stock while her daughters consists of 18%,
properties sold, and for the rentals earned by BCC. But the demands remained unheeded. Hence, complainant giving them a total of 73%. Respondents holdings consist of 24% while three other incorporators,
and her daughter, in a letter dated June 4, 1985, terminated the services of respondent as their lawyer and Rosauro L. Alvarez, Vicente T. Maalac and Darhan S. Graciano each held 1% of the capital stock of the
repeated their demands for accounting and turn-over of the corporate funds, and the return of the 19 titles that corporation.
respondent transferred to the corporation. They also threatened him with legal action in a letter dated August
3, 1985.
"B. On April 5, 1981, complainant and her daughter Rosemarie Jaldon executed two Deeds of Transfer
and Assignment conveying and transferring to the corporation 19 parcels of land in exchange for
Soon after, complainant found out from the Securities and Exchange Commission (SEC, for brevity) that shares of stock in the corporation.
Rosaura Enterprises, Inc., due to respondents refusal and neglect, failed to submit the corporations annual
financial statements for 1981, 1982 and 1983; SEC General Information Sheets for 1982, 1983 and 1984;
"x x x xxx xxx
Minutes of Annual Meetings for 1982, 1983 and 1984; and Minutes of Annual Meetings of Directors for 1982,
1983 and 1984.
"C. Both Deeds of Assignment particularly page 3 thereof indicate that respondent accepted said
assignment of properties and titles in behalf of the corporation as Treasurer. The deeds were signed on
Complainant also discovered that respondent collected rental payments from the tenants of BCC and issued
April 5, 1981.
handwritten receipts which he signed, not as an officer of the corporation but as the attorney-at-law of
complainant. Respondent also used the tennis court of BCC to dry his palay and did not keep the buildings in a
satisfactory state, so much so that the divisions were losing plywood and other materials to thieves. "x x x xxx xxx
Set V Corporation Code * Doctrine of Peircing Corpo Veil Cases*Page 89 of 164

"Together, therefore, complainant and her daughter owned 1,711 shares of the 1,750 shares "The subject attachment however reveals that only the following persons signed their conformity to
comprising the authorized capital stock of the corporation of 97% thereof. the said resolution: respondent Balicanta who owned 109 shares, Vicente Maalac (1 share), Daihan
Graciano (1 share).
"No increase in capitalization was applied for by the corporation.
"Complainants who collectively held a total of 1,711 shares out of the 1,750 outstanding capital stock
"F. Respondent claims in his Comment, his Answer and his Position Paper that on April 4, 1981 he was of the corporation were not represented in the purported stockholders meeting authorizing the
elected as Chairman and Director and on April 5, 1981 he was elected President of the corporation. mortgage of the subject properties.
Respondents own Annexes marked as G and G-1 of his Comment show that on April 4, 1981 he was
not only elected as Chairman and Director as he claims but as Director, Board Chairman and "The 2/3 vote required by law was therefore not complied with yet respondent proceeded to mortgage
President. The purported minutes was only signed by respondent and an acting Secretary by the the subject 9 parcels of land by the corporation.
name of Vicente Maalac.
"J. Respondent further relies on Annex J of his Comment, purportedly the minutes of a special
meeting of the Board of Directors authorizing him to obtain a loan and mortgage the properties of the
corporation dated August 29, 1981. This claim is baseless. The required ratification of 2/3 by the
"Said Annex does not show who was elected Treasurer. stockholders of records was not met. Again, respondent attempts to mislead the Commission and
Court.

"K. Further, the constitution of the Board is dubious. The alleged minutes of the organizational meeting
of the stockholders electing the members of the Board, have not been duly signed by the stockholders
"Respondents Annex H and H-1 shows that in the alleged organizational meeting of the directors on
as shown in respondents annex G which was purportedly the organizational meeting of the
April 5, 1981 a certain Farnacio Bucoy was elected Treasurer. Bucoys name does not appear as an
stockholders.
incorporator nor a stockholder anywhere in the documents submitted.

"L. Also, Annex J of respondents Comment which purportedly authorized him to obtain a loan and to
"The purported minutes of the organizational meeting of the directors was signed only by respondent
mortgage the 9 parcels of land was only signed by himself and a secretary.
Balicanta and a Secretary named Verisimo Martin.

"M. In said Annex 'J' of respondents Comment he stated that complainant Rosaura Cordon was on
"G. Since respondent was elected as Director, Chairman and President on April 4, 1981 as
leave by virtue of a voting trust agreement allegedly executed by complainant in his favor covering
respondents own Annexes G to G-1 would show, then complainants claim that respondent was
all her shares of stock. The claim is baseless. The voting trust referred to by respondent (annex D of
likewise acting as Treasurer of two corporations bear truth and credence as respondent signed and
his Comment), even if it were assumed to be valid, covered only 266 shares of complainants yet she
accepted the titles to 19 parcels of land ceded by the complainant and her daughter, as Treasurer on
owned a total of 1,039 shares after she and her daughter ceded in favor of the corporation 19 parcels
April 5, 1981 after he was already purportedly elected as Chairman, President and Director.
of land.

"H. Respondent misleads the Commission into believing that all the directors signed the minutes
"Being a former lawyer to complainant, respondent should have ensured that her interest was
marked as Exhibit H to H-1 by stating that the same was duly signed by all the Board of Directors
safeguarded. Yet, complainant was apparently and deliberately left our (sic) on the pretext that, she
when the document itself shows that only he and one Verisimo Martin signed the same.
had executed a voting trust agreement in favor of respondent.

"He also claims that all the stockholders signed the minutes of organizational meeting marked as
"It is suspicious that complainant was made to sign a voting trust agreement on 21 August 1981 and
Annexes G and G-1 of his Comment yet the same shows that only the acting Chairman and acting
immediately thereafter, the resolutions authorizing respondent to obtain a loan and to mortgage the 9
Secretary signed.
parcels of land were passed and approved.

"I. Respondent claims that the Board or its representative was authorized by the stockholders
"N. It is also highly irregular for respondent who is a lawyer, to allow a situation to happen where, with
comprising 2/3 of the outstanding capital stock, as required by law, to mortgage the parcels of land
the exclusion of complainant as director the result was that there remained only 4 members of the
belonging to the corporation, which were all assigned to the corporation by complainant and her
Board,.
daughter, by virtue of Annex I and I-1: attached to his Comment.
Set V Corporation Code * Doctrine of Peircing Corpo Veil Cases*Page 90 of 164

"O. Respondents own pleadings submitted to the Commission contradict each other. "Respondents purported minutes of stockholders meeting (Exhs. 15 and 17) do not reflect this.

"1. For instance, while in his Comment respondent DENIES that he employed deceit and machination "There was no explanation whatsoever from respondent on how complainant and her daughter lost
in convincing the complainant and her daughter to sign the articles of incorporation of Rosaura their 97% control holding in the corporation.
Enterprises and in ceding to the corporation 19 parcels of land in Zamboanga City, because they
freely, intelligently and voluntarily signed the same, yet, in his Position Paper, respondent took "3. As a further contradiction in respondents pleadings, we note that in paragraph 2.7.C of his
another stance. Comment he said that only recently, this year, 1985, the complainant and her aforenamed daughter
examined said voluminous supporting receipts/documents which had previously been examined by
"In paragraphs 1.1 and 1.2 of his Position Paper which was submitted 12 years later, respondent the Land Bank for loan releases, during which occasion respondent suggested to them that the
claimed that it was actually the idea of Atty. Rosaura L. Alvarez that a corporation be put up to corporation will have to hire a full-time book-keeper to put in order said voluminous supporting
incorporate the estate of the late Felixberto D. Jaldon. receipts/documents, to which they adversely reacted due to lack of corporate money to pay for said
book-keeper. But in respondents Position Paper par. 6.3 he stated that:
"2. Likewise, respondent claimed that complainant and her daughter were not directors, hence they
were not notified of meetings, in paragraph 2-6 (c) of his Comment he blamed the other stockholders Anyway, it is not the respondent but rather the complainant who should render a detailed accounting
and directors for the corporations inability to comply with the Land Banks demands saying that they to the corporation of the corporate records as well as corporate revenues/income precisely
have consistently failed since 1982 to convene (1.) for the annual stockholders meetings and (i.i) for becausesince 1994 to the present:
the monthly board meeting.
(a). The corporate part-time book-keeper Edilberto Benedicto, with the indispensable connivance and
"His own pleadings claim that he had been the Chairman/President since 1981 to the present. If (sic) instigation of the complainant and her daughter, among others, has custody of the corporate records,
so, it was his duty to convene the stockholders and the directors for meetings. xxx

"Respondent appeared able to convene the stockholders and directors when he needed to make a "4. In other contradictory stance, respondent claims in par. 7.3 of his position paper that complainant
loan of p2.2 million; when he sold the corporations right of redemption over the foreclosed properties and her daughter sabotaged the BCC operations of the corporation by illegally taking over actual
of the corporation to Jammang, when he sold one parcel of land covered by TCT 62,807 to Jammang in control and supervision thereof sometime in 1986, xxx
addition to the 9 parcels of land which were foreclosed, and when he sold the complainants ancestral
home covered by TCT No. 72,004. "Yet respondents own exhibits in his position paper particularly Exhibit 15 and 16 where the subject of
the foreclosed properties of the corporation comprising the Baliwasan Commercial Center (BCC) was
"It is thus strange why respondent claims that the corporation could not do anything to save the taken up, complainant and her daughter were not even present nor were they the subject of the
corporations properties from being foreclosed because the stockholders and directors did not discussion, belying respondents claim that the complainant and her daughter illegally took actual
convene. control of BCC.

"This assertion of respondent is clearly evident of dishonest, deceitful and immoral conduct especially "5. On the matter of the receipts issued by respondent evidencing payment to him of rentals by
because, in all his acts constituting conveyances of corporate property, respondent used minutes of lessees of the corporation, attached to the complaint as Annexes H to H-17, respondent claims that
stockholders and directors meetings signed only by him and a secretary or signed by him and the receipts are temporary in nature and that subsequently regular corporate receipts were issued. On
persons who were not incorporators much less stockholders. their face however the receipts clearly appear to be official receipts, printed and numbered duly
signed by the respondent bearing his printed name.
"It is worthy of note that in respondents Exhibits 15, 16, 17 and 18 of his position paper, there were 7
new stockholders and complainant appeared to have only 266 shares to her name while her daughter "It is difficult to believe that a lawyer of respondent stature would issue official receipts to lessees if
Rosemarie had no shares at all. Respondent did not present any proof of conveyance of shares by he only meant to issue temporary ones.
complainant and her daughter.
"6. With regard to respondents claim that the complainant consented to the sale of her ancestral
"It is further worth noting that complainants voting trust (annex D of respondents Comment) where home, covered by TCT No. T-72,004 to one Tion Suy Ong for which he attached as Exhibit 22 to his
she allegedly entrusted 266 shares to respondent on August 21, 1981 had only a validity of 5 years. Position Paper the minutes of an annual meeting of the stockholders, it behooves this Commission why
Thus, she should have had her entire holdings of 1,283 shares back in her name in August 1986. complainants signature had to be accompanied by her thumb mark. Furthermore, complainants
signature appears unstable and shaky. This Office is thus persuaded to believe complainants
Set V Corporation Code * Doctrine of Peircing Corpo Veil Cases*Page 91 of 164

allegation in paragraph 3b of her position paper that since September 1992 up to March 1993 she was "It is therefore our unpleasant duty to recommend that respondent, having committed acts in violation
being detained by one PO# (sic) Joel Constantino and his wife under instructions from respondent of the Canons of Professional Responsibility, thereby causing a great disservice to the profession, be
Balicanta. meted the ultimate sanction of disbarment."2

"This conclusion is supported by a letter from respondent dated March 1993, Annex H of On September 30, 1999, while Commissioner Cunanans recommendation for respondents disbarment was
complainants position paper, where respondent ordered Police Officer Constantino to allow Atty. pending review before Executive Vice-President and Northern Luzon Governor Teofilo Pilando, respondent filed a
Linda Lim and Rosemarie Jaldon to talk to Tita Rosing. motion requesting "for a full-blown investigation and for invalidation of the entire proceedings and/or remedial
action under Section 11, Rule 139-B, Revised Rules of Court," alleging that he had evidence that Commissioner
"The complainants thumb mark together with her visibly unstable shaky signature lends credence to Cunanans report was drafted by the lawyers of complainant, Attys. Antonio Cope and Rita Linda Jimeno. He
her claim that she was detained in the far flung barrio of Culianan under instructions of respondent presented two unsigned anonymous letters allegedly coming from a disgruntled employee of Attys. Cope and
while her ancestral home was demolished and the lot sold to one Tion Suy Ong. Jimeno. He claimed to have received these letters in his mailbox. 3

"It appears that respondent felt compelled to over-ensure complainants consent by getting her to Respondents motion alleging that Attys. Antonio Cope and Rita Linda Jimeno drafted Commissioner Cunanans
affix her thumb mark in addition to her signature. report was accompanied by a complaint praying for the disbarment of said lawyers including Commissioner
Cunanan. The complaint was docketed as CBD Case No. 99-658. After Attys. Cope and Jimeno and
Commissioner Cunanan filed their answers, a hearing was conducted by the Investigating Committee of the IBP
"7. Respondent likewise denies that he also acted as Corporate Secretary in addition to being the
Board of Governors.
Chairman, President and Treasurer of the corporation. Yet, respondent submitted to this commission
documents which are supported to be in the possession of the Corporate Secretary such as the stock
and transfer book and minutes of meetings. On May 26, 2001, the IBP Board of Governors issued a resolution 4 dismissing for lack of merit the complaint for
disbarment against Attys. Cope and Jimeno and Commissioner Cunanan. And in Adm. Case No. 2797, the Board
adopted and approved the report and recommendation of Commissioner Cunanan, and meted against herein
"The foregoing findings of this Commission are virtual smoking guns that prove on no uncertain terms
respondent Balicanta the penalty of suspension from the practice of law for 5 years "for commission of acts of
that respondent, who was the legal counsel of complainant in the latter part of the settlement of the
misconduct and disloyalty by taking undue and unfair advantage of his legal knowledge as a lawyer to gain
estate of her deceased husband, committed unlawful, immoral and deceitful conduct proscribed by
material benefit for himself at the expense of complainant Rosaura P. Jaldon-Cordon and caused serious
Rule 1.01 of the code of professional responsibility.
damage to the complainant."5

"Likewise, respondent clearly committed a violation of Canon 15 of the same code which provides that
To support its decision, the Board uncovered respondents fraudulent acts in the very same documents he
A lawyer should observe candor fairness and loyalty in all his dealings and transactions with his
presented to exonerate himself. It also took note of respondents contradictory and irreconcilable statements in
client.
the pleadings and position papers he submitted. However, it regarded the penalty of disbarment as too severe
for respondents misdeeds, considering that the same were his first offense. 6
"Respondents acts gravely diminish the publics respect for the integrity of the profession of law for
which this Commission recommends that he be meted the penalty of disbarment.
Pursuant to Section 12 (b), Rule 139-B of the Rules of Court, 7 the said resolution in Administrative Case No.
2797 imposing the penalty of suspension for 5 years on respondent was automatically elevated to this Court for
"The pendency of the cases at the SEC and the Regional Trial Court of Zamboanga filed by final action. On the other hand, the dismissal of the complaint for disbarment against Attys. Cope and Jimeno
complainant against respondent does not preclude a determination of respondents culpability as a and Commissioner Cunanan, docketed as CBD Case No. 99-658, became final in the absence of any petition for
lawyer. review.

"This Commission cannot further delay the resolution of this complaint filed in 1985 by complainant, This Court confirms the duly supported findings of the IBP Board that respondent committed condemnable acts
and old widow who deserves to find hope and recover her confidence in the judicial system. of deceit against his client. The fraudulent acts he carried out against his client followed a well thought of plan
to misappropriate the corporate properties and funds entrusted to him. At the very outset, he embarked on his
"The findings of this office, predominantly based on documents adduced by both parties lead to only devious scheme by making himself the President, Chairman of the Board, Director and Treasurer of the
one rather unpalatable conclusion. That respondent Atty. Jesus F. Balicanta, in his professional corporation, although he knew he was prohibited from assuming the position of President and Treasurer at the
relations with herein complainant did in fact employ unlawful, dishonest, and immoral conduct same time.8 As Treasurer, he accepted in behalf of the corporation the 19 titles that complainant and her
proscribed in no uncertain terms by Rule 1.01 of the Code of Professional Responsibility. In addition, daughter co-owned. The other treasurer appointed, Farnacio Bucoy, did not appear to be a stockholder or
respondents actions clearly violated Canon 15 to 16 of the same Code. director in the corporate records. The minutes of the meetings supposedly electing him and Bucoy as officers of
Set V Corporation Code * Doctrine of Peircing Corpo Veil Cases*Page 92 of 164

the corporation actually bore the signatures of respondent and the secretary only, contrary to his claim that respondents position paper maintained that there was no accounting because the part-time bookkeeper of the
they were signed by the directors and stockholders. corporation connived with complainant and her daughter in keeping the corporate records.

He likewise misled the IBP investigating commission in claiming that the mortgage of 9 of the properties of the Fourth, respondents claim that complainant and her daughter took control of the operations of the corporation
corporation previously belonging to complainant and her daughter was ratified by the stockholders owning two- in 1986 is belied by the fact that complainant and her daughter were not even present in the alleged meeting
thirds or 67% of the outstanding capital stock when in fact only three stockholders owning 111 out of 1,750 of the board (which took place after 1986) to discuss the foreclosure of the mortgaged properties. The truth is
outstanding shares or 6.3% assented thereto. The alleged authorization granting him the power to contract the that he never informed them of such meeting and he never gave control of the corporation to them.
LBP loan for Two Million Two Hundred Twenty Pesos (P2,220,000) was also not approved by the required
minimum of two-thirds of the outstanding capital stock despite respondents claim to the contrary. In all these Fifth, Commissioner Cunanan found that:
transactions, complainant and her daughter who both owned 1,711 out of the 1,750 outstanding shares of the
corporation or 97.7% never had any participation. Neither were they informed thereof.
"5. on the matter of the receipts issued by respondent evidencing payment to him of rentals by lessees of the
corporation, attached to the complaint as Annexes H to H-17, respondent claims that the receipts are
Clearly, there was no quorum for a valid meeting for the discussion and approval of these transactions. temporary in nature and that subsequently regular corporate receipts were issued. On their face however the
receipts clearly appear to be official receipts, printed and numbered duly signed by the respondent bearing his
Respondent cannot take refuge in the contested voting trust agreement supposedly executed by complainant printed name.
and her daughter for the reason that it authorized respondent to represent complainant for only 266 shares.
"It is difficult to believe that a lawyer of respondents stature would issue official receipts to lessees if he only
Aside from the dishonest transactions he entered into under the cloak of sham resolutions, he failed to explain meant to issue temporary ones."10
several discrepancies in his version of the facts. We hereby reiterate some of these statements noted by
Commissioner Cunanan in his findings. Sixth, respondent denies that he acted as Corporate Secretary aside from being the Chairman, President and
Treasurer of the corporation. Yet respondent submitted to the investigating commission documents which were
First, respondent blamed the directors and the stockholders who failed to convene for the required annual supposed to be in the official possession of the Corporate Secretary alone such as the stock and transfer book
meetings since 1982. However, respondent appeared able to convene the stockholders and directors when he and minutes of meetings.
contracted the LBP debt, when he sold to Jammang the corporations right of redemption over the foreclosed
properties of the corporation, when he sold one parcel of land covered by TCT No. 62807 to Jammang, when he Seventh, he alleged in his comment that he was the one who proposed the establishment of the corporation
mortgaged the 9 parcels of land to LBP which later foreclosed on said mortgage, and when he sold the that would invest the properties of the complainant but, in his position paper, he said that it was a certain Atty.
complainants ancestral home covered by TCT No. 72004. Rosauro Alvarez who made the proposal to put up the corporation.

Second, the factual findings of the investigating commission, affirmed by the IBP Board, disclosed that After a thorough review of the records, we find that respondent committed grave and serious misconduct that
complainant and her daughter own 1,711 out of 1,750 shares of the outstanding capital stock of the casts dishonor on the legal profession. His misdemeanors reveal a deceitful scheme to use the corporation as a
corporation, based on the Articles of Incorporation and deeds of transfer of the properties. But respondents means to convert for his own personal benefit properties left to him in trust by complainant and her daughter.
evidence showed that complainant had only 266 shares of stock in the corporation while her daughter had
none, notwithstanding the fact that there was nothing to indicate that complainant and her daughter ever
Not even his deviousness could cover up the wrongdoings he committed. The documents he thought could
conveyed their shares to others.
exculpate him were the very same documents that revealed his immoral and shameless ways. These
documents were extremely revealing in that they unmasked a man who knew the law and abused it for his
Respondent likewise did not explain why he did not return the certificates representing the 266 shares after the personal gain without any qualms of conscience. They painted an intricate web of lies, deceit and opportunism
lapse of 5 years from the time the voting trust certificate was executed in 1981. 9 beneath a carefully crafted smokescreen of corporate maneuvers.

The records show that up to now, the complainant and her daughter own 97% of the outstanding shares but The Code of Professional Responsibility mandates upon each lawyer, as his duty to society, the obligation to
respondent never bothered to explain why they were never asked to participate in or why they were never obey the laws of the land and promote respect for law and legal processes. Specifically, he is forbidden to
informed of important corporate decisions. engage in unlawful, dishonest, immoral or deceitful conduct. 11 If the practice of law is to remain an honorable
profession and attain its basic ideal, those enrolled in its ranks should not only master its tenets and principles
Third, respondent, in his comment, alleged that due to the objection of complainant and her daughter to his but should also, in their lives, accord continuing fidelity to them. 12 Thus, the requirement of good moral
proposal to hire an accountant, the corporation had no formal accounting of its revenues and income. However, character is of much greater import, as far as the general public is concerned, than the possession of legal
learning.13 Lawyers are expected to abide by the tenets of morality, not only upon admission to the Bar but also
Set V Corporation Code * Doctrine of Peircing Corpo Veil Cases*Page 93 of 164

throughout their legal career, in order to maintain ones good standing in that exclusive and honored
fraternity.14 Good moral character is more than just the absence of bad character. Such character expresses
itself in the will to do the unpleasant thing if it is right and the resolve not to do the pleasant thing if it is The petitioners question the decision of the Intermediate Appellate Court which sustained the private
wrong.15 This must be so because "vast interests are committed to his care; he is the recipient of unbounded respondent's contention that the deed of exchange whereby Delfin Pacheco and Pelagia Pacheco conveyed a
trust and confidence; he deals with his clients property, reputation, his life, his all." 16 parcel of land to Delpher Trades Corporation in exchange for 2,500 shares of stock was actually a deed of sale
which violated a right of first refusal under a lease contract.
Indeed, the words of former Presiding Justice of the Court of Appeals Pompeyo Diaz cannot find a more relevant
application than in this case: Briefly, the facts of the case are summarized as follows:

"There are men in any society who are so self-serving that they try to make law serve their selfish ends. In this In 1974, Delfin Pacheco and his sister, Pelagia Pacheco, were the owners of 27,169 square meters of real estate
group of men, the most dangerous is the man of the law who has no conscience. He has, in the arsenal of his Identified as Lot. No. 1095, Malinta Estate, in the Municipality of Polo (now Valenzuela), Province of Bulacan
knowledge, the very tools by which he can poison and disrupt society and bring it to an ignoble end." 17 (now Metro Manila) which is covered by Transfer Certificate of Title No. T-4240 of the Bulacan land registry.

Good moral standing is manifested in the duty of the lawyer "to hold in trust all moneys and properties of his On April 3, 1974, the said co-owners leased to Construction Components International Inc. the same property
client that may come into his possession." 18 He is bound "to account for all money or property collected or and providing that during the existence or after the term of this lease the lessor should he decide to sell the
received for or from the client."19 The relation between an attorney and his client is highly fiduciary in nature. property leased shall first offer the same to the lessee and the letter has the priority to buy under similar
Thus, lawyers are bound to promptly account for money or property received by them on behalf of their clients conditions (Exhibits A to A-5)
and failure to do so constitutes professional misconduct.20

On August 3, 1974, lessee Construction Components International, Inc. assigned its rights and obligations
This Court holds that respondent cannot invoke the separate personality of the corporation to absolve him from under the contract of lease in favor of Hydro Pipes Philippines, Inc. with the signed conformity and consent of
exercising these duties over the properties turned over to him by complainant. He blatantly used the corporate lessors Delfin Pacheco and Pelagia Pacheco (Exhs. B to B-6 inclusive)
veil to defeat his fiduciary obligation to his client, the complainant. Toleration of such fraudulent conduct was
never the reason for the creation of said corporate fiction.
The contract of lease, as well as the assignment of lease were annotated at he back of the title, as per
stipulation of the parties (Exhs. A to D-3 inclusive)
The massive fraud perpetrated by respondent on the complainant leaves us no choice but to set aside the veil
of corporate entity. For purposes of this action therefore, the properties registered in the name of the
On January 3, 1976, a deed of exchange was executed between lessors Delfin and Pelagia Pacheco and
corporation should still be considered as properties of complainant and her daughter. The respondent merely
defendant Delpher Trades Corporation whereby the former conveyed to the latter the leased property (TCT
held them in trust for complainant (now an ailing 83-year-old) and her daughter. The properties conveyed
No.T-4240) together with another parcel of land also located in Malinta Estate, Valenzuela, Metro Manila (TCT
fraudulently and/or without the requisite authority should be deemed as never to have been transferred, sold
No. 4273) for 2,500 shares of stock of defendant corporation with a total value of P1,500,000.00 (Exhs. C to C-
or mortgaged at all. Respondent shall be liable, in his personal capacity, to third parties who may have
5, inclusive) (pp. 44-45, Rollo)
contracted with him in good faith.

On the ground that it was not given the first option to buy the leased property pursuant to the proviso in the
Based on the aforementioned findings, this Court believes that the gravity of respondents offenses cannot be
lease agreement, respondent Hydro Pipes Philippines, Inc., filed an amended complaint for reconveyance of
adequately matched by mere suspension as recommended by the IBP. Instead, his wrongdoings deserve the
Lot. No. 1095 in its favor under conditions similar to those whereby Delpher Trades Corporation acquired the
severe penalty of disbarment, without prejudice to his criminal and civil liabilities for his dishonest
property from Pelagia Pacheco and Delphin Pacheco.
acts.WHEREFORE, respondent Attorney Jesus T. Balicanta is hereby DISBARRED. The Clerk of Court is directed
to strike out his name from the Roll of Attorneys.SO ORDERED.
After trial, the Court of First Instance of Bulacan ruled in favor of the plaintiff. The dispositive portion of the
decision reads:
G.R. No. L-69259 January 26, 1988

ACCORDINGLY, the judgment is hereby rendered declaring the valid existence of the plaintiffs
DELPHER TRADES CORPORATION, and DELPHIN PACHECO, petitioners, vs.INTERMEDIATE APPELLATE
preferential right to acquire the subject property (right of first refusal) and ordering the defendants
COURT and HYDRO PIPES PHILIPPINES, INC., respondents.
and all persons deriving rights therefrom to convey the said property to plaintiff who may offer to
acquire the same at the rate of P14.00 per square meter, more or less, for Lot 1095 whose area is
GUTIERREZ, JR., J.:
Set V Corporation Code * Doctrine of Peircing Corpo Veil Cases*Page 94 of 164

27,169 square meters only. Without pronouncement as to attorney's fees and costs. (Appendix I; Rec., Pipes Philippines. In the petitioners' motion for reconsideration, they refer to this scheme as "estate planning."
pp. 246- 247). (Appellant's Brief, pp. 1-2; p. 134, Rollo) (p. 252, Rollo)

The lower court's decision was affirmed on appeal by the Intermediate Appellate Court. Under this factual backdrop, the petitioners contend that there was actually no transfer of ownership of the
subject parcel of land since the Pachecos remained in control of the property. Thus, the petitioners allege:
The defendants-appellants, now the petitioners, filed a petition for certiorari to review the appellate court's "Considering that the beneficial ownership and control of petitioner corporation remained in the hands of the
decision. original co-owners, there was no transfer of actual ownership interests over the land when the same was
transferred to petitioner corporation in exchange for the latter's shares of stock. The transfer of ownership, if
anything, was merely in form but not in substance. In reality, petitioner corporation is a mere alter ego or
We initially denied the petition but upon motion for reconsideration, we set aside the resolution denying the
conduit of the Pacheco co-owners; hence the corporation and the co-owners should be deemed to be the same,
petition and gave it due course.
there being in substance and in effect an Identity of interest." (p. 254, Rollo)

The petitioners allege that:


The petitioners maintain that the Pachecos did not sell the property. They argue that there was no sale and that
they exchanged the land for shares of stocks in their own corporation. "Hence, such transfer is not within the
The denial of the petition will work great injustice to the petitioners, in that: letter, or even spirit of the contract. There is a sale when ownership is transferred for a price certain in money
or its equivalent (Art. 1468, Civil Code) while there is a barter or exchange when one thing is given in
1. Respondent Hydro Pipes Philippines, Inc, ("private respondent") will acquire from petitioners a consideration of another thing (Art. 1638, Civil Code)." (pp. 254-255, Rollo)
parcel of industrial land consisting of 27,169 square meters or 2.7 hectares (located right after the
Valenzuela, Bulacan exit of the toll expressway) for only P14/sq. meter, or a total of P380,366, On the other hand, the private respondent argues that Delpher Trades Corporation is a corporate entity
although the prevailing value thereof is approximately P300/sq. meter or P8.1 Million; separate and distinct from the Pachecos. Thus, it contends that it cannot be said that Delpher Trades
Corporation is the Pacheco's same alter ego or conduit; that petitioner Delfin Pacheco, having treated Delpher
2. Private respondent is allowed to exercise its right of first refusal even if there is no "sale" or transfer Trades Corporation as such a separate and distinct corporate entity, is not a party who may allege that this
of actual ownership interests by petitioners to third parties; and separate corporate existence should be disregarded. It maintains that there was actual transfer of ownership
interests over the leased property when the same was transferred to Delpher Trades Corporation in exchange
for the latter's shares of stock.
3. Assuming arguendo that there has been a transfer of actual ownership interests, private respondent
will acquire the land not under "similar conditions" by which it was transferred to petitioner Delpher
Trades Corporation, as provided in the same contractual provision invoked by private respondent. (pp. We rule for the petitioners.
251-252, Rollo)
After incorporation, one becomes a stockholder of a corporation by subscription or by purchasing stock directly
The resolution of the case hinges on whether or not the "Deed of Exchange" of the properties executed by the from the corporation or from individual owners thereof (Salmon, Dexter & Co. v. Unson, 47 Phil, 649, citing Bole
Pachecos on the one hand and the Delpher Trades Corporation on the other was meant to be a contract of sale v. Fulton [1912], 233 Pa., 609). In the case at bar, in exchange for their properties, the Pachecos acquired 2,500
which, in effect, prejudiced the private respondent's right of first refusal over the leased property included in original unissued no par value shares of stocks of the Delpher Trades Corporation. Consequently, the Pachecos
the "deed of exchange." became stockholders of the corporation by subscription "The essence of the stock subscription is an agreement
to take and pay for original unissued shares of a corporation, formed or to be formed." (Rohrlich 243, cited in
Agbayani, Commentaries and Jurisprudence on the Commercial Laws of the Philippines, Vol. III, 1980 Edition, p.
Eduardo Neria, a certified public accountant and son-in-law of the late Pelagia Pacheco testified that Delpher
430) It is significant that the Pachecos took no par value shares in exchange for their properties.
Trades Corporation is a family corporation; that the corporation was organized by the children of the two
spouses (spouses Pelagia Pacheco and Benjamin Hernandez and spouses Delfin Pacheco and Pilar Angeles) who
owned in common the parcel of land leased to Hydro Pipes Philippines in order to perpetuate their control over A no-par value share does not purport to represent any stated proportionate interest in the capital
the property through the corporation and to avoid taxes; that in order to accomplish this end, two pieces of real stock measured by value, but only an aliquot part of the whole number of such shares of the issuing
estate, including Lot No. 1095 which had been leased to Hydro Pipes Philippines, were transferred to the corporation. The holder of no-par shares may see from the certificate itself that he is only an aliquot
corporation; that the leased property was transferred to the corporation by virtue of a deed of exchange of sharer in the assets of the corporation. But this character of proportionate interest is not hidden
property; that in exchange for these properties, Pelagia and Delfin acquired 2,500 unissued no par value shares beneath a false appearance of a given sum in money, as in the case of par value shares. The capital
of stock which are equivalent to a 55% majority in the corporation because the other owners only owned 2,000 stock of a corporation issuing only no-par value shares is not set forth by a stated amount of money,
shares; and that at the time of incorporation, he knew all about the contract of lease of Lot. No. 1095 to Hydro but instead is expressed to be divided into a stated number of shares, such as, 1,000 shares. This
indicates that a shareholder of 100 such shares is an aliquot sharer in the assets of the corporation, no
matter what value they may have, to the extent of 100/1,000 or 1/10. Thus, by removing the par value
Set V Corporation Code * Doctrine of Peircing Corpo Veil Cases*Page 95 of 164

of shares, the attention of persons interested in the financial condition of a corporation is focused Q What are these advantages to the said spouses from the point of view of taxation in entering in the
upon the value of assets and the amount of its debts. (Agbayani, Commentaries and Jurisprudence on deed of exchange?
the Commercial Laws of the Philippines, Vol. III, 1980 Edition, p. 107).
A Having fulfilled the conditions in the income tax law, providing for tax free exchange of property,
Moreover, there was no attempt to state the true or current market value of the real estate. Land valued at they were able to execute the deed of exchange free from income tax and acquire a corporation.
P300.00 a square meter was turned over to the family's corporation for only P14.00 a square meter.
Q What provision in the income tax law are you referring to?
It is to be stressed that by their ownership of the 2,500 no par shares of stock, the Pachecos have control of the
corporation. Their equity capital is 55% as against 45% of the other stockholders, who also belong to the same A I refer to Section 35 of the National Internal Revenue Code under par. C-sub-par. (2) Exceptions
family group. regarding the provision which I quote: "No gain or loss shall also be recognized if a person exchanges
his property for stock in a corporation of which as a result of such exchange said person alone or
In effect, the Delpher Trades Corporation is a business conduit of the Pachecos. What they really did was to together with others not exceeding four persons gains control of said corporation."
invest their properties and change the nature of their ownership from unincorporated to incorporated form by
organizing Delpher Trades Corporation to take control of their properties and at the same time save on Q Did you explain to the spouses this benefit at the time you executed the deed of exchange?
inheritance taxes.

A Yes, sir
As explained by Eduardo Neria:

Q You also, testified during the last hearing that the decision to have no par value share in the
xxx xxx xxx defendant corporation was for the purpose of flexibility. Can you explain flexibility in connection with
the ownership of the property in question?
ATTY. LINSANGAN:
A There is flexibility in using no par value shares as the value is determined by the board of directors
Q Mr. Neria, from the point of view of taxation, is there any benefit to the spouses Hernandez and in increasing capitalization. The board can fix the value of the shares equivalent to the capital
Pacheco in connection with their execution of a deed of exchange on the properties for no par value requirements of the corporation.
shares of the defendant corporation?
Q Now also from the point of taxation, is there any flexibility in the holding by the corporation of the
A Yes, sir. property in question?

COURT: A Yes, since a corporation does not die it can continue to hold on to the property indefinitely for a
period of at least 50 years. On the other hand, if the property is held by the spouse the property will
Q What do you mean by "point of view"? be tied up in succession proceedings and the consequential payments of estate and inheritance taxes
when an owner dies.

A To take advantage for both spouses and corporation in entering in the deed of exchange.
Q Now what advantage is this continuity in relation to ownership by a particular person of certain
properties in respect to taxation?
ATTY. LINSANGAN:

A The property is not subjected to taxes on succession as the corporation does not die.
Q (What do you mean by "point of view"?) What are these benefits to the spouses of this deed of
exchange?
Q So the benefit you are talking about are inheritance taxes?

A Continuous control of the property, tax exemption benefits, and other inherent benefits in a
corporation. A Yes, sir. (pp. 3-5, tsn., December 15, 1981)
Set V Corporation Code * Doctrine of Peircing Corpo Veil Cases*Page 96 of 164

The records do not point to anything wrong or objectionable about this "estate planning" scheme resorted to by Petitioner appealed to the NLRC. On September 23, 1988, the NLRC dismissed the appeal. On October 21,
the Pachecos. "The legal right of a taxpayer to decrease the amount of what otherwise could be his taxes or 1988, petitioner's motion for reconsideration was denied.
altogether avoid them, by means which the law permits, cannot be doubted." (Liddell & Co., Inc. v. The
collector of Internal Revenue, 2 SCRA 632 citing Gregory v. Helvering, 293 U.S. 465, 7 L. ed. 596). Thus, this petition was filed on October 28, 1988, alleging that the NLRC gravely abused its discretion. On
November 10, 1988 the Court issued a temporary restraining order enjoining the enforcement of the NLRC's
The "Deed of Exchange" of property between the Pachecos and Delpher Trades Corporation cannot be decision dated September 23, 1988 and resolution dated October 21, 1988. The petition was given due course
considered a contract of sale. There was no transfer of actual ownership interests by the Pachecos to a third on June 14, 1989.
party. The Pacheco family merely changed their ownership from one form to another. The ownership remained
in the same hands. Hence, the private respondent has no basis for its claim of a light of first refusal under the After considering the undisputed facts and the arguments raised in the pleadings, the Court finds grave abuse
lease contract. of discretion on the part of the NLRC.

WHEREFORE, the instant petition is hereby GRANTED, The questioned decision and resolution of the then The action of the NLRC affirming the issuance of an alias writ of execution against petitioner, on the theory that
Intermediate Appellate Court are REVERSED and SET ASIDE. The amended complaint in Civil Case No. 885-V-79 the corporate personality of Philsa should be disregarded, was founded primarily on the following findings of
of the then Court of First Instance of Bulacan is DISMISSED. No costs.SO ORDERED. the POEA

xxx xxx xxx

THIRD DIVISION G.R. No. 85416 July 24, 1990


6. Per the certification issued by the Licensing Division of this Office, it appears that Philsa
Construction & Trading Co., Inc., with office address at 126 Pioneer St., Mandaluyong, Metro Manila,
FRANCISCO V. DEL ROSARIO, petitioner, vs.NATIONAL LABOR RELATIONS COMMISSION and represented by Mr. Francisco V. del Rosario, President and General Manager, was formerly a registered
LEONARDO V. ATIENZA, respondents. construction contractor whose authority was originally issued on July 21, 1978 but was already
delisted from the list of agencies/entities on August 15, 1986 for inactivity;
CORTES, J.:
7. Per another certification issued by the Licensing Division of this Office, it also appears that another
In POEA Case No. 85-06-0394, the Philippine Overseas Employment Administration (POEA) promulgated a corporation, Philsa International Placement & Services Corp., composed of practically the same set of
decision on February 4, 1986 dismissing the complaint for money claims for lack of merit. The decision was incorporators/stockholders, was registered as a licensed private employment agency whose license
appealed to the National Labor Relations Commission (NLRC), which on April 30, 1987 reversed the POEA was issued on November 5, 1981, represented by the same Mr. Francisco V. del Rosario as its
decision and ordered Philsa Construction and Trading Co., Inc. (the recruiter) and Arieb Enterprises (the foreign President/ General Manager.
employer) to jointly and severally pay private respondent the peso equivalent of $16,039.00, as salary
differentials, and $2,420.03, as vacation leave benefits. The case was elevated to the Supreme Court, but the and an application of the ruling of the Court in A.C. Ransom Labor Union-CCLU v. NLRC, G.R. No. 69494, June
petition was dismissed on August 31, 1987 and entry of judgment was made on September 24, 1987. 10, 1986, 142 SCRA 269.

A writ of execution was issued by the POEA but it was returned unsatisfied as Philsa was no longer operating However, we find that the NLRC's reliance on the findings of the POEA and the ruling in A. C. Ransom is totally
and was financially incapable of satisfying the judgment. Private respondent moved for the issuance of an alias misplaced.
writ against the officers of Philsa. This motion was opposed by the officers, led by petitioner, the president and
general manager of the corporation.
1. Under the law a corporation is bestowed juridical personality, separate and distinct from its stockholders
[Civil Code, Art. 44; Corporation Code, sec. 2]. But when the juridical personality of the corporation is used to
On February 12, 1988, the POEA issued a resolution, the dispositive portion of which read: defeat public convenience, justify wrong, protect fraud or defend crime, the corporation shall be considered as
a mere association of persons [Koppel (Phil.), Inc. v. Yatco, 77 Phil. 496 (1946), citing 1 Fletcher, Cyclopedia of
WHEREFORE, premises considered, let an alias writ of Execution be issued and the handling sheriff is Corporations, 135-136; see also Palay, Inc. v. Clave, G.R. No. 56076, September 21, 1983, 124 SCRA 638], and
ordered to execute against the properties of Mr. Francisco V. del -Rosario and if insufficient, against the its responsible officers and/or stockholders shall be held individually liable [Namarco v. Associated Finance Co.,
cash and/or surety bond of Bonding Company concerned for the full satisfaction of the judgment Inc., G.R. No. L-20886, April 27, 1967, 19 SCRA 962]. For the same reasons, a corporation shall be liable for the
awarded. obligations of a stockholder [Palacio v. Fely Transportation Company, G.R. No. L-15121, August 31, 1962, 5
SCRA 1011; Emilio Cano Enterprises, Inc. v. Court of Industrial Relations, G.R. No. L-20502, February 26, 1965,
13 SCRA 290], or a corporation and its successor-in-interest shall be considered as one and the liability of the
Set V Corporation Code * Doctrine of Peircing Corpo Veil Cases*Page 97 of 164

former shall attach to the latter [Koppel v. Yatco, supra; Liddell & Co. v. Collector of Internal Revenue, G.R. No. In Claparols v. Court of Industrial Relations, G.R. No. L-30822, July 31, 1975, 65 SCRA 613, the Claparols Steel
L-9687, June 30, 1961, 2 SCRA 632]. and Nail Plant, which was ordered to pay its workers backwages, ceased operations on June 30, 1957 and was
succeeded on the next day, July 1, 1957 by the Claparols Steel Corporation. Both corporations were
But for the separate juridical personality of a corporation to be disregarded, the wrongdoing must be clearly substantially owned and controlled by the same person and there was no break or cessation in operations.
and convincingly established. It cannot be presumed. Moreover, all the assets of the steel and nail plant were transferred to the new corporation.

In this regard we find the NLRC's decision wanting. The conclusion that Philsa allowed its license to expire so as 2. As earlier stated, we also find that, contrary to the NLRC'S holding, the ruling in A. C. Ransom is inapplicable
to evade payment of private respondent's claim is not supported by the facts. Philsa's corporate personality to this case. In A. C. Ransom, the Court said:
therefore remains inviolable.
... In the instant case, it would appear that RANSOM, in 1969, foreseeing the possibility or probability
Consider the following undisputed facts: of payment of back wages to the 22 strikers, organized ROSARIO to replace RANSOM, with the latter to
be eventually phased out if the 22 strikers win their case. RANSOM actually ceased operations on May
1, 1973, after the December 19, 1972 Decision of the Court of Industrial Relations was promulgated
(1) Private respondent filed his complaint with the POEA on June 4, 1985;
against RANSOM. [At p. 274.]

(2) The last renewal of Philsa's license expired on October 12, 1985;
The distinguishing marks of fraud were therefore clearly apparent in A. C. Ransom. A new corporation was
created, owned by the same family, engaging in the same business and operating in the same compound.
(3) The POEA dismissed private respondent's complaint on February 4, 1986;
Thus, considering that the non-payment of the workers was a continuing situation, the Court adjudged its
(4) Philsa was delisted for inactivity on August 15, 1986; * President, the "responsible officer" of the corporation, personally liable for the backwages awarded, he being
the chief operation officer or "manager" who could be held criminally liable for violations of Republic Act No.
(5) The dismissal of the complaint was appealed to the NLRC and it was only on April 30, 1987 that the 602 (the old Minimum Wage Law.)
judgment awarding differentials and benefits to private respondent was rendered.
In the case now before us, not only has there been a failure to establish fraud, but it has also not been shown
Thus, at the time Philsa allowed its license to lapse in 1985 and even at the time it was delisted in 1986, there that petitioner is the corporate officer responsible for private respondent's predicament. It must be emphasized
was yet no judgment in favor of private respondent. An intent to evade payment of his claims cannot therefore that the claim for differentials and benefits was actually directed against the foreign employer. Philsa became
be implied from the expiration of Philsa's license and its delisting. liable only because of its undertaking to be jointly and severally bound with the foreign employer, an
undertaking required by the rules of the POEA [Rule II, sec. 1(d) (3)], together with the filing of cash and surety
bonds [Rule 11, sec. 4], in order to ensure that overseas workers shall find satisfaction for awards in their favor.
Neither will the organization of Philsa International Placement and Services Corp. and its registration with the
POEA as a private employment agency imply fraud since it was organized and registered in 1981, several years
before private respondent filed his complaint with the POEA in 1985. The creation of the second corporation At this juncture, the Court finds it appropriate to point out that a judgment against a recruiter should initially be
could not therefore have been in anticipation of private respondent's money claims and the consequent enforced against the cash and surety bonds filed with the POEA. As provided in the POEA Rules and Regulations
adverse judgment against Philsa

Likewise, substantial identity of the incorporators of the two corporations does not necessarily imply fraud. ... The bonds shall answer for all valid and legal claims arising from violations of the conditions for the
grant and use of the license or authority and contracts of employment. The bonds shall likewise
guarantee compliance with the provisions of the Labor Code and its implementing rules and
The circumstances of this case distinguish it from those in earlier decisions of the Court in labor cases where
regulations relating to recruitment and placement, the rules of the Administration and relevant
the veil of corporate fiction was pierced.
issuances of the Ministry and all liabilities which the Administration may impose. ... [Rule II, see. 4.]

In La Campana Coffee Factory, Inc. v. Kaisahan ng Manggagawa sa La Campana (KKM) 93 Phil. 160 (1953), La
Quite evidently, these bonds do not answer for a single specific liability, but for all sorts of liabilities of the
Campana Coffee Factory, Inc. and La Campana Gaugau Packing were substantially owned by the same person.
recruiter to the worker and to the POEA. Moreover, the obligations guaranteed by the bonds are continuing.
They had one office, one management, and a single payroll for both businesses. The laborers of
Thus, the bonds are subject to replenishment when they are garnished, and failure to replenish shall cause the
the gaugaufactory and the coffee factory were also interchangeable, i.e., the workers in one factory worked
suspension or cancellation of the recruiter's license [Rule II, sec. 19]. Furthermore, a cash bond shall be
also in the other factory.
Set V Corporation Code * Doctrine of Peircing Corpo Veil Cases*Page 98 of 164

refunded to a recruiter who surrenders his license only upon posting of a surety bond of similar amount valid Costs against appellant bank.
for three (3) years [Rule II, sec. 20]. All these, to ensure recovery from the recruiter.
The dispositive portion of the trial court's2 decision dated July 10, 1991, on the other hand, is as follows:
It is therefore surprising why the POEA ordered execution "against the properties of Mr. Francisco V. del
Rosarioand if insufficient, against the cash and/or surety bond of Bonding Company concerned for the till WHEREFORE, premises considered, judgment is hereby rendered in favor of the plaintiffs and against
satisfaction of the judgment awarded" in complete disregard of the scheme outlined in the POEA Rules and the defendants as follows:
Regulations. On this score alone, the NLRC should not have affirmed the POEA.

1. Declaring the existence of a perfected contract to buy and sell over the six (6) parcels of land
WHEREFORE, the petition is GRANTED and the decision and resolution of the NLRC, dated September 23, 1988 situated at Don Jose, Sta. Rosa, Laguna with an area of 101 hectares, more or less, covered by and
and October 21, 1988, respectively, in POEA Case No. 85-06-0394 are SET ASIDE. The temporary restraining embraced in Transfer Certificates of Title Nos. T-106932 to T-106937, inclusive, of the Land Records of
order issued by the Court on November 10, 1988 is MADE PERMANENT.SO ORDERED. Laguna, between the plaintiffs as buyers and the defendant Producers Bank for an agreed price of Five
and One Half Million (P5,500,000.00) Pesos;
THIRD DIVISION G.R. No. 115849 January 24, 1996
2. Ordering defendant Producers Bank of the Philippines, upon finality of this decision and receipt from
FIRST PHILIPPINE INTERNATIONAL BANK (Formerly Producers Bank of the Philippines) and the plaintiffs the amount of P5.5 Million, to execute in favor of said plaintiffs a deed of absolute sale
MERCURIO RIVERA, petitioners, vs. over the aforementioned six (6) parcels of land, and to immediately deliver to the plaintiffs the
COURT OF APPEALS, CARLOS EJERCITO, in substitution of DEMETRIO DEMETRIA, and JOSE owner's copies of T.C.T. Nos. T-106932 to T- 106937, inclusive, for purposes of registration of the same
JANOLO,respondents. deed and transfer of the six (6) titles in the names of the plaintiffs;

DECISION 3. Ordering the defendants, jointly and severally, to pay plaintiffs Jose A. Janolo and Demetrio
Demetria the sums of P200,000.00 each in moral damages;
PANGANIBAN, J.:
4. Ordering the defendants, jointly and severally, to pay plaintiffs the sum of P100,000.00 as
In the absence of a formal deed of sale, may commitments given by bank officers in an exchange of letters exemplary damages ;
and/or in a meeting with the buyers constitute a perfected and enforceable contract of sale over 101 hectares
of land in Sta. Rosa, Laguna? Does the doctrine of "apparent authority" apply in this case? If so, may the 5. Ordering the defendants, jointly and severally, to pay the plaintiffs the amount of P400,000.00 for
Central Bank-appointed conservator of Producers Bank (now First Philippine International Bank) repudiate such and by way of attorney's fees;
"apparent authority" after said contract has been deemed perfected? During the pendency of a suit for specific
performance, does the filing of a "derivative suit" by the majority shareholders and directors of the distressed 6. Ordering the defendants to pay the plaintiffs, jointly and severally, actual and moderate damages in
bank to prevent the enforcement or implementation of the sale violate the ban against forum-shopping? the amount of P20,000.00;

Simply stated, these are the major questions brought before this Court in the instant Petition for review With costs against the defendants.
oncertiorari under Rule 45 of the Rules of Court, to set aside the Decision promulgated January 14, 1994 of the
respondent Court of Appeals 1 in CA-G.R CV No. 35756 and the Resolution promulgated June 14, 1994 denying
After the parties filed their comment, reply, rejoinder, sur-rejoinder and reply to sur-rejoinder, the petition was
the motion for reconsideration. The dispositive portion of the said Decision reads:
given due course in a Resolution dated January 18, 1995. Thence, the parties filed their respective memoranda
and reply memoranda. The First Division transferred this case to the Third Division per resolution dated October
WHEREFORE, the decision of the lower court is MODIFIED by the elimination of the damages awarded 23, 1995. After carefully deliberating on the aforesaid submissions, the Court assigned the case to the
under paragraphs 3, 4 and 6 of its dispositive portion and the reduction of the award in paragraph 5 undersigned ponente for the writing of this Decision.
thereof to P75,000.00, to be assessed against defendant bank. In all other aspects, said decision is
hereby AFFIRMED.
The Parties

All references to the original plaintiffs in the decision and its dispositive portion are deemed, herein
Petitioner First Philippine International Bank (formerly Producers Bank of the Philippines; petitioner Bank, for
and hereafter, to legally refer to the plaintiff-appellee Carlos C. Ejercito.
brevity) is a banking institution organized and existing under the laws of the Republic of the Philippines.
Set V Corporation Code * Doctrine of Peircing Corpo Veil Cases*Page 99 of 164

Petitioner Mercurio Rivera (petitioner Rivera, for brevity) is of legal age and was, at all times material to this
case, Head-Manager of the Property Management Department of the petitioner Bank.
TCT NO. AREA

Respondent Carlos Ejercito (respondent Ejercito, for brevity) is of legal age and is the assignee of original
plaintiffs-appellees Demetrio Demetria and Jose Janolo.
T-106932 113,580 sq. m.
Respondent Court of Appeals is the court which issued the Decision and Resolution sought to be set aside
through this petition.

T-106933 70,899 sq. m.


The Facts

The facts of this case are summarized in the respondent Court's Decision 3 as follows:
T-106934 52,246 sq. m.
(1) In the course of its banking operations, the defendant Producer Bank of the Philippines acquired six
parcels of land with a total area of 101 hectares located at Don Jose, Sta. Rose, Laguna, and covered
by Transfer Certificates of Title Nos. T-106932 to T-106937. The property used to be owned by BYME
T-106935 96,768 sq. m.
Investment and Development Corporation which had them mortgaged with the bank as collateral for a
loan. The original plaintiffs, Demetrio Demetria and Jose O. Janolo, wanted to purchase the property
and thus initiated negotiations for that purpose.
T-106936 187,114 sq. m.
(2) In the early part of August 1987 said plaintiffs, upon the suggestion of BYME investment's legal
counsel, Jose Fajardo, met with defendant Mercurio Rivera, Manager of the Property Management
Department of the defendant bank. The meeting was held pursuant to plaintiffs' plan to buy the
property (TSN of Jan. 16, 1990, pp. 7-10). After the meeting, plaintiff Janolo, following the advice of T-106937 481,481 sq. m.
defendant Rivera, made a formal purchase offer to the bank through a letter dated August 30, 1987
(Exh. "B"), as follows:

My offer is for PESOS: THREE MILLION FIVE HUNDRED THOUSAND (P3,500,000.00) PESOS, in cash.

August 30, 1987 Kindly contact me at Telephone Number 921-1344.

(3) On September 1, 1987, defendant Rivera made on behalf of the bank a formal reply by letter which
The Producers Bank of the Philippines is hereunder quoted (Exh. "C"):
Makati, Metro Manila

Attn. Mr. Mercurio Q. Rivera September 1, 1987


Manager, Property Management Dept.

Gentleman:
JP M-P GUTIERREZ ENTERPRISES
142 Charisma St., Doa Andres II
I have the honor to submit my formal offer to purchase your properties covered by titles listed
Rosario, Pasig, Metro Manila
hereunder located at Sta. Rosa, Laguna, with a total area of 101 hectares, more or less.

Attention: JOSE O. JANOLO


Set V Corporation Code * Doctrine of Peircing Corpo Veil Cases*Page 100 of 164

Dear Sir: Gentlemen:

Thank you for your letter-offer to buy our six (6) parcels of acquired lots at Sta. Rosa, Laguna (formerly Pursuant to our discussion last 28 September 1987, we are pleased to inform you that we are
owned by Byme Industrial Corp.). Please be informed however that the bank's counter-offer is at P5.5 accepting your offer for us to purchase the property at Sta. Rosa, Laguna, formerly owned by Byme
million for more than 101 hectares on lot basis. Investment, for a total price of PESOS: FIVE MILLION FIVE HUNDRED THOUSAND (P5,500,000.00).

We shall be very glad to hear your position on the on the matter. Thank you.

Best regards. (6) On October 12, 1987, the conservator of the bank (which has been placed under conservatorship
by the Central Bank since 1984) was replaced by an Acting Conservator in the person of defendant
(4) On September 17, 1987, plaintiff Janolo, responding to Rivera's aforequoted reply, wrote (Exh. "D"): Leonida T. Encarnacion. On November 4, 1987, defendant Rivera wrote plaintiff Demetria the following
letter (Exh. "F"):

September 17, 1987

Attention: Atty. Demetrio Demetria


Producers Bank
Paseo de Roxas
Makati, Metro Manila

Dear Sir:
Attention: Mr. Mercurio Rivera

Your proposal to buy the properties the bank foreclosed from Byme investment Corp. located at Sta.
Gentlemen:
Rosa, Laguna is under study yet as of this time by the newly created committee for submission to the
newly designated Acting Conservator of the bank.
In reply to your letter regarding my proposal to purchase your 101-hectare lot located at Sta. Rosa,
Laguna, I would like to amend my previous offer and I now propose to buy the said lot at P4.250
For your information.
million in CASH..

(7) What thereafter transpired was a series of demands by the plaintiffs for compliance by the bank
Hoping that this proposal meets your satisfaction.
with what plaintiff considered as a perfected contract of sale, which demands were in one form or
another refused by the bank. As detailed by the trial court in its decision, on November 17, 1987,
(5) There was no reply to Janolo's foregoing letter of September 17, 1987. What took place was a plaintiffs through a letter to defendant Rivera (Exhibit "G") tendered payment of the amount of P5.5
meeting on September 28, 1987 between the plaintiffs and Luis Co, the Senior Vice-President of million "pursuant to (our) perfected sale agreement." Defendants refused to receive both the payment
defendant bank. Rivera as well as Fajardo, the BYME lawyer, attended the meeting. Two days later, or and the letter. Instead, the parcels of land involved in the transaction were advertised by the bank for
on September 30, 1987, plaintiff Janolo sent to the bank, through Rivera, the following letter (Exh. sale to any interested buyer (Exh, "H" and "H-1"). Plaintiffs demanded the execution by the bank of
"E"): the documents on what was considered as a "perfected agreement." Thus:

The Producers Bank of the Philippines


Paseo de Roxas, Makati
Metro Manila
Mr. Mercurio Rivera
Manager, Producers Bank
Attention: Mr. Mercurio Rivera Paseo de Roxas, Makati
Metro Manila
Re: 101 Hectares of Land
in Sta. Rosa, Laguna
Set V Corporation Code * Doctrine of Peircing Corpo Veil Cases*Page 101 of 164

Dear Mr. Rivera: (9) The foregoing letter drew no response for more than four months. Then, on May 3, 1988, plaintiff,
through counsel, made a final demand for compliance by the bank with its obligations under the
This is in connection with the offer of our client, Mr. Jose O. Janolo, to purchase your 101-hectare lot considered perfected contract of sale (Exhibit "N"). As recounted by the trial court (Original Record, p.
located in Sta. Rosa, Laguna, and which are covered by TCT No. T-106932 to 106937. 656), in a reply letter dated May 12, 1988 (Annex "4" of defendant's answer to amended complaint),
the defendants through Acting Conservator Encarnacion repudiated the authority of defendant Rivera
and claimed that his dealings with the plaintiffs, particularly his counter-offer of P5.5 Million are
From the documents at hand, it appears that your counter-offer dated September 1, 1987 of this same
unauthorized or illegal. On that basis, the defendants justified the refusal of the tenders of payment
lot in the amount of P5.5 million was accepted by our client thru a letter dated September 30, 1987
and the non-compliance with the obligations under what the plaintiffs considered to be a perfected
and was received by you on October 5, 1987.
contract of sale.

In view of the above circumstances, we believe that an agreement has been perfected. We were also
(10) On May 16, 1988, plaintiffs filed a suit for specific performance with damages against the bank,
informed that despite repeated follow-up to consummate the purchase, you now refuse to honor your
its Manager Rivers and Acting Conservator Encarnacion. The basis of the suit was that the transaction
commitment. Instead, you have advertised for sale the same lot to others.
had with the bank resulted in a perfected contract of sale, The defendants took the position that there
was no such perfected sale because the defendant Rivera is not authorized to sell the property, and
In behalf of our client, therefore, we are making this formal demand upon you to consummate and that there was no meeting of the minds as to the price.
execute the necessary actions/documentation within three (3) days from your receipt hereof. We are
ready to remit the agreed amount of P5.5 million at your advice. Otherwise, we shall be constrained to
On March 14, 1991, Henry L. Co (the brother of Luis Co), through counsel Sycip Salazar Hernandez and
file the necessary court action to protect the interest of our client.
Gatmaitan, filed a motion to intervene in the trial court, alleging that as owner of 80% of the Bank's
outstanding shares of stock, he had a substantial interest in resisting the complaint. On July 8, 1991,
We trust that you will be guided accordingly. the trial court issued an order denying the motion to intervene on the ground that it was filed after
trial had already been concluded. It also denied a motion for reconsideration filed thereafter. From the
(8) Defendant bank, through defendant Rivera, acknowledged receipt of the foregoing letter and trial court's decision, the Bank, petitioner Rivera and conservator Encarnacion appealed to the Court
stated, in its communication of December 2, 1987 (Exh. "I"), that said letter has been "referred . . . to of Appeals which subsequently affirmed with modification the said judgment. Henry Co did not appeal
the office of our Conservator for proper disposition" However, no response came from the Acting the denial of his motion for intervention.
Conservator. On December 14, 1987, the plaintiffs made a second tender of payment (Exh. "L" and "L-
1"), this time through the Acting Conservator, defendant Encarnacion. Plaintiffs' letter reads: In the course of the proceedings in the respondent Court, Carlos Ejercito was substituted in place of Demetria
and Janolo, in view of the assignment of the latters' rights in the matter in litigation to said private respondent.
PRODUCERS BANK OF
THE PHILIPPINES On July 11, 1992, during the pendency of the proceedings in the Court of Appeals, Henry Co and several other
Paseo de Roxas, stockholders of the Bank, through counsel Angara Abello Concepcion Regala and Cruz, filed an action
Makati, Metro Manila (hereafter, the "Second Case") purportedly a "derivative suit" with the Regional Trial Court of Makati,
Branch 134, docketed as Civil Case No. 92-1606, against Encarnacion, Demetria and Janolo "to declare any
Attn.: Atty. NIDA ENCARNACION perfected sale of the property as unenforceable and to stop Ejercito from enforcing or implementing the
Central Bank Conservator sale"4 In his answer, Janolo argued that the Second Case was barred by litis pendentia by virtue of the case
then pending in the Court of Appeals. During the pre-trial conference in the Second Case, plaintiffs filed a
Motion for Leave of Court to Dismiss the Case Without Prejudice. "Private respondent opposed this motion on
We are sending you herewith, in - behalf of our client, Mr. JOSE O. JANOLO, MBTC Check No. 258387 in
the ground, among others, that plaintiff's act of forum shopping justifies the dismissal of both cases, with
the amount of P5.5 million as our agreed purchase price of the 101-hectare lot covered by TCT Nos.
prejudice."5 Private respondent, in his memorandum, averred that this motion is still pending in the Makati RTC.
106932, 106933, 106934, 106935, 106936 and 106937 and registered under Producers Bank.

In their Petition6 and Memorandum7, petitioners summarized their position as follows:


This is in connection with the perfected agreement consequent from your offer of P5.5 Million as the
purchase price of the said lots. Please inform us of the date of documentation of the sale immediately.
I.

Kindly acknowledge receipt of our payment.


The Court of Appeals erred in declaring that a contract of sale was perfected between Ejercito (in substitution
of Demetria and Janolo) and the bank.
Set V Corporation Code * Doctrine of Peircing Corpo Veil Cases*Page 102 of 164

II. 3) Assuming there was, was the said contract enforceable under the statute of frauds?

The Court of Appeals erred in declaring the existence of an enforceable contract of sale between the parties. 4) Did the bank conservator have the unilateral power to repudiate the authority of the bank officers
and/or to revoke the said contract?
III.
5) Did the respondent Court commit any reversible error in its findings of facts?
The Court of Appeals erred in declaring that the conservator does not have the power to overrule or revoke acts
of previous management. The First Issue: Was There Forum-Shopping?

IV. In order to prevent the vexations of multiple petitions and actions, the Supreme Court promulgated Revised
Circular No. 28-91 requiring that a party "must certify under oath . . . [that] (a) he has not (t)heretofore
The findings and conclusions of the Court of Appeals do not conform to the evidence on record. commenced any other action or proceeding involving the same issues in the Supreme Court, the Court of
Appeals, or any other tribunal or agency; (b) to the best of his knowledge, no such action or proceeding is
pending" in said courts or agencies. A violation of the said circular entails sanctions that include the summary
On the other hand, petitioners prayed for dismissal of the instant suit on the ground 8 that:
dismissal of the multiple petitions or complaints. To be sure, petitioners have included a
VERIFICATION/CERTIFICATION in their Petition stating "for the record(,) the pendency of Civil Case No. 92-1606
I. before the Regional Trial Court of Makati, Branch 134, involving a derivative suit filed by stockholders of
petitioner Bank against the conservator and other defendants but which is the subject of a pending Motion to
Petitioners have engaged in forum shopping. Dismiss Without Prejudice.9

II. Private respondent Ejercito vigorously argues that in spite of this verification, petitioners are guilty of actual
forum shopping because the instant petition pending before this Court involves "identical parties or interests
represented, rights asserted and reliefs sought (as that) currently pending before the Regional Trial Court,
The factual findings and conclusions of the Court of Appeals are supported by the evidence on record and may
Makati Branch 134 in the Second Case. In fact, the issues in the two cases are so interwined that a judgement
no longer be questioned in this case.
or resolution in either case will constitute res judicata in the other." 10

III. 11
On the other hand, petitioners explain that there is no forum-shopping because:

The Court of Appeals correctly held that there was a perfected contract between Demetria and Janolo
1) In the earlier or "First Case" from which this proceeding arose, the Bank was impleaded as
(substituted by; respondent Ejercito) and the bank.
a defendant, whereas in the "Second Case" (assuming the Bank is the real party in interest in a
derivative suit), it wasplaintiff;
IV.

2) "The derivative suit is not properly a suit for and in behalf of the corporation under the
The Court of Appeals has correctly held that the conservator, apart from being estopped from repudiating the circumstances";
agency and the contract, has no authority to revoke the contract of sale.

3) Although the CERTIFICATION/VERIFICATION (supra) signed by the Bank president and attached to
The Issues the Petition identifies the action as a "derivative suit," it "does not mean that it is one" and "(t)hat is a
legal question for the courts to decide";
From the foregoing positions of the parties, the issues in this case may be summed up as follows:
4) Petitioners did not hide the Second Case at they mentioned it in the said
1) Was there forum-shopping on the part of petitioner Bank? VERIFICATION/CERTIFICATION.

2) Was there a perfected contract of sale between the parties? We rule for private respondent.
Set V Corporation Code * Doctrine of Peircing Corpo Veil Cases*Page 103 of 164

To begin with, forum-shopping originated as a concept in private international law. 12, where non-resident condemned by Justice Andres R. Narvasa (now Chief Justice) in Minister of Natural Resources, et al., vs. Heirs of
litigants are given the option to choose the forum or place wherein to bring their suit for various reasons or Orval Hughes, et al.,"as a reprehensible manipulation of court processes and proceedings . . ." 17 when does
excuses, including to secure procedural advantages, to annoy and harass the defendant, to avoid overcrowded forum shopping take place?
dockets, or to select a more friendly venue. To combat these less than honorable excuses, the principle
of forum non conveniens was developed whereby a court, in conflicts of law cases, may refuse impositions on There is forum-shopping whenever, as a result of an adverse opinion in one forum, a party seeks a
its jurisdiction where it is not the most "convenient" or available forum and the parties are not precluded from favorable opinion (other than by appeal or certiorari) in another. The principle applies not only with
seeking remedies elsewhere. respect to suits filed in the courts but also in connection with litigations commenced in the courts
while an administrative proceeding is pending, as in this case, in order to defeat administrative
In this light, Black's Law Dictionary 13 says that forum shopping "occurs when a party attempts to have his processes and in anticipation of an unfavorable administrative ruling and a favorable court ruling. This
action tried in a particular court or jurisdiction where he feels he will receive the most favorable judgment or is specially so, as in this case, where the court in which the second suit was brought, has no
verdict." Hence, according to Words and Phrases14, "a litigant is open to the charge of "forum shopping" jurisdiction.18
whenever he chooses a forum with slight connection to factual circumstances surrounding his suit, and litigants
should be encouraged to attempt to settle their differences without imposing undue expenses and vexatious The test for determining whether a party violated the rule against forum shopping has been laid dawn in the
situations on the courts". 1986 case of Buan vs. Lopez 19, also by Chief Justice Narvasa, and that is, forum shopping exists where the
elements of litis pendentia are present or where a final judgment in one case will amount to res judicata in the
In the Philippines, forum shopping has acquired a connotation encompassing not only a choice of venues, as it other, as follows:
was originally understood in conflicts of laws, but also to a choice of remedies. As to the first (choice of
venues), the Rules of Court, for example, allow a plaintiff to commence personal actions "where the defendant There thus exists between the action before this Court and RTC Case No. 86-36563 identity of parties,
or any of the defendants resides or may be found, or where the plaintiff or any of the plaintiffs resides, at the or at least such parties as represent the same interests in both actions, as well as identity of rights
election of the plaintiff" (Rule 4, Sec, 2 [b]). As to remedies, aggrieved parties, for example, are given a choice asserted and relief prayed for, the relief being founded on the same facts, and the identity on the two
of pursuing civil liabilities independently of the criminal, arising from the same set of facts. A passenger of a preceding particulars is such that any judgment rendered in the other action, will, regardless of which
public utility vehicle involved in a vehicular accident may sue on culpa contractual, culpa aquiliana or culpa party is successful, amount to res adjudicata in the action under consideration: all the requisites, in
criminal each remedy being available independently of the others although he cannot recover more than fine, of auter action pendant.
once.

xxx xxx xxx


In either of these situations (choice of venue or choice of remedy), the litigant actually shops for a
forum of his action, This was the original concept of the term forum shopping.
As already observed, there is between the action at bar and RTC Case No. 86-36563, an identity as
regards parties, or interests represented, rights asserted and relief sought, as well as basis thereof, to
Eventually, however, instead of actually making a choice of the forum of their actions, litigants, a degree sufficient to give rise to the ground for dismissal known as auter action pendant or lis
through the encouragement of their lawyers, file their actions in all available courts, or invoke all pendens. That same identity puts into operation the sanction of twin dismissals just mentioned. The
relevant remedies simultaneously. This practice had not only resulted to (sic) conflicting adjudications application of this sanction will prevent any further delay in the settlement of the controversy which
among different courts and consequent confusion enimical (sic) to an orderly administration of justice. might ensue from attempts to seek reconsideration of or to appeal from the Order of the Regional Trial
It had created extreme inconvenience to some of the parties to the action. Court in Civil Case No. 86-36563 promulgated on July 15, 1986, which dismissed the petition upon
grounds which appear persuasive.
Thus, "forum shopping" had acquired a different concept which is unethical professional legal
practice. And this necessitated or had given rise to the formulation of rules and canons discouraging Consequently, where a litigant (or one representing the same interest or person) sues the same party against
or altogether prohibiting the practice. 15 whom another action or actions for the alleged violation of the same right and the enforcement of the same
relief is/are still pending, the defense of litis pendencia in one case is bar to the others; and, a final judgment in
What therefore originally started both in conflicts of laws and in our domestic law as a legitimate device for one would constitute res judicata and thus would cause the dismissal of the rest. In either case, forum shopping
solving problems has been abused and mis-used to assure scheming litigants of dubious reliefs. could be cited by the other party as a ground to ask for summary dismissal of the two 20 (or more) complaints
or petitions, and for imposition of the other sanctions, which are direct contempt of court, criminal prosecution,
To avoid or minimize this unethical practice of subverting justice, the Supreme Court, as already mentioned, and disciplinary action against the erring lawyer.
promulgated Circular 28-91. And even before that, the Court had prescribed it in the Interim Rules and
Guidelines issued on January 11, 1983 and had struck down in several cases 16 the inveterate use of this
insidious malpractice. Forum shopping as "the filing of repetitious suits in different courts" has been
Set V Corporation Code * Doctrine of Peircing Corpo Veil Cases*Page 104 of 164

Applying the foregoing principles in the case before us and comparing it with the Second Case, it is obvious facts, The adoption of this latter recourse renders the petitioners amenable to disciplinary action and
that there exist identity of parties or interests represented, identity of rights or causes and identity of reliefs both their actions, in this Court as well as in the Court a quo, dismissible.
sought.
In the instant case before us, there is also identity of parties, or at least, of interests represented. Although the
Very simply stated, the original complaint in the court a quo which gave rise to the instant petition was filed by plaintiffs in the Second Case (Henry L. Co. et al.) are not name parties in the First Case, they represent the
the buyer (herein private respondent and his predecessors-in-interest) against the seller (herein petitioners) to same interest and entity, namely, petitioner Bank, because:
enforce the alleged perfected sale of real estate. On the other hand, the complaint 21 in the Second Case seeks
to declare such purported sale involving the same real property "as unenforceable as against the Bank", which Firstly, they are not suing in their personal capacities, for they have no direct personal interest in the matter in
is the petitioner herein. In other words, in the Second Case, the majority stockholders, in representation of the controversy. They are not principally or even subsidiarily liable; much less are they direct parties in the assailed
Bank, are seeking to accomplish what the Bank itself failed to do in the original case in the trial court. In brief, contract of sale; and
the objective or the relief being sought, though worded differently, is the same, namely, to enable the
petitioner Bank to escape from the obligation to sell the property to respondent. In Danville Maritime, Inc. vs.
Secondly, the allegations of the complaint in the Second Case show that the stockholders are bringing a
Commission on Audit. 22, this Court ruled that the filing by a party of two apparently different actions, but with
"derivative suit". In the caption itself, petitioners claim to have brought suit "for and in behalf of the Producers
the same objective, constituted forum shopping:
Bank of the Philippines" 24. Indeed, this is the very essence of a derivative suit:

In the attempt to make the two actions appear to be different, petitioner impleaded different
An individual stockholder is permitted to institute a derivative suit on behalf of the corporation
respondents therein PNOC in the case before the lower court and the COA in the case before this
wherein he holdsstock in order to protect or vindicate corporate rights, whenever the officials of the
Court and sought what seems to be different reliefs. Petitioner asks this Court to set aside the
corporation refuse to sue, or are the ones to be sued or hold the control of the corporation. In such
questioned letter-directive of the COA dated October 10, 1988 and to direct said body to approve the
actions, the suing stockholder is regarded as a nominal party, with the corporation as the real party in
Memorandum of Agreement entered into by and between the PNOC and petitioner, while in the
interest. (Gamboa v. Victoriano, 90 SCRA 40, 47 [1979]; emphasis supplied).
complaint before the lower court petitioner seeks to enjoin the PNOC from conducting a rebidding and
from selling to other parties the vessel "T/T Andres Bonifacio", and for an extension of time for it to
comply with the paragraph 1 of the memorandum of agreement and damages. One can see that In the face of the damaging admissions taken from the complaint in the Second Case, petitioners, quite
although the relief prayed for in the two (2) actions are ostensibly different, the ultimate objective in strangely, sought to deny that the Second Case was a derivative suit, reasoning that it was brought, not by the
both actions is the same, that is, approval of the sale of vessel in favor of petitioner and to overturn minority shareholders, but by Henry Co et al., who not only own, hold or control over 80% of the outstanding
the letter-directive of the COA of October 10, 1988 disapproving the sale. (emphasis supplied). capital stock, but also constitute the majority in the Board of Directors of petitioner Bank. That being so, then
they really represent the Bank. So, whether they sued "derivatively" or directly, there is undeniably an identity
23 of interests/entity represented.
In an earlier case but with the same logic and vigor, we held:

Petitioner also tried to seek refuge in the corporate fiction that the personality Of the Bank is separate and
In other words, the filing by the petitioners of the instant special civil action for certiorari and
distinct from its shareholders. But the rulings of this Court are consistent: "When the fiction is urged as a
prohibition in this Court despite the pendency of their action in the Makati Regional Trial Court, is a
means of perpetrating a fraud or an illegal act or as a vehicle for the evasion of an existing obligation, the
species of forum-shopping. Both actions unquestionably involve the same transactions, the same
circumvention of statutes, the achievement or perfection of a monopoly or generally the perpetration of
essential facts and circumstances. The petitioners' claim of absence of identity simply because the
knavery or crime, the veil with which the law covers and isolates the corporation from the members or
PCGG had not been impleaded in the RTC suit, and the suit did not involve certain acts which
stockholders who compose it will be lifted to allow for its consideration merely as an aggregation of
transpired after its commencement, is specious. In the RTC action, as in the action before this Court,
individuals." 25
the validity of the contract to purchase and sell of September 1, 1986, i.e., whether or not it had been
efficaciously rescinded, and the propriety of implementing the same (by paying the pledgee banks the
amount of their loans, obtaining the release of the pledged shares, etc.) were the basic issues. So, too, In addition to the many cases 26 where the corporate fiction has been disregarded, we now add the instant
the relief was the same: the prevention of such implementation and/or the restoration of the status case, and declare herewith that the corporate veil cannot be used to shield an otherwise blatant violation of the
quo ante. When the acts sought to be restrained took place anyway despite the issuance by the Trial prohibition against forum-shopping. Shareholders, whether suing as the majority in direct actions or as the
Court of a temporary restraining order, the RTC suit did not become functus oficio. It remained an minority in a derivative suit, cannot be allowed to trifle with court processes, particularly where, as in this case,
effective vehicle for obtention of relief; and petitioners' remedy in the premises was plain and patent: the corporation itself has not been remiss in vigorously prosecuting or defending corporate causes and in using
the filing of an amended and supplemental pleading in the RTC suit, so as to include the PCGG as and applying remedies available to it. To rule otherwise would be to encourage corporate litigants to use their
defendant and seek nullification of the acts sought to be enjoined but nonetheless done. The remedy shareholders as fronts to circumvent the stringent rules against forum shopping.
was certainly not the institution of another action in another forum based on essentially the same
Set V Corporation Code * Doctrine of Peircing Corpo Veil Cases*Page 105 of 164

Finally, petitioner Bank argued that there cannot be any forum shopping, even assuming arguendo that there is The Second Issue: Was The Contract Perfected?
identity of parties, causes of action and reliefs sought, "because it (the Bank) was the defendant in the (first)
case while it was the plaintiff in the other (Second Case)",citing as authority Victronics Computers, Inc., The respondent Court correctly treated the question of whether or not there was, on the basis of the facts
vs. Regional Trial Court, Branch 63, Makati, etc. et al., 27 where Court held: established, a perfected contract of sale as the ultimate issue. Holding that a valid contract has been
established, respondent Court stated:
The rule has not been extended to a defendant who, for reasons known only to him, commences a
new action against the plaintiff instead of filing a responsive pleading in the other case setting There is no dispute that the object of the transaction is that property owned by the defendant bank as
forth therein, as causes of action, specific denials, special and affirmative defenses or even acquired assets consisting of six (6) parcels of land specifically identified under Transfer Certificates of
counterclaims, Thus, Velhagen's and King's motion to dismiss Civil Case No. 91-2069 by no means Title Nos. T-106932 to T-106937. It is likewise beyond cavil that the bank intended to sell the property.
negates the charge of forum-shopping as such did not exist in the first place. (emphasis supplied) As testified to by the Bank's Deputy Conservator, Jose Entereso, the bank was looking for buyers of
the property. It is definite that the plaintiffs wanted to purchase the property and it was precisely for
Petitioner pointed out that since it was merely the defendant in the original case, it could not have chosen the this purpose that they met with defendant Rivera, Manager of the Property Management Department
forum in said case. of the defendant bank, in early August 1987. The procedure in the sale of acquired assets as well as
the nature and scope of the authority of Rivera on the matter is clearly delineated in the testimony of
Respondent, on the other hand, replied that there is a difference in factual setting between Victronics and the Rivera himself, which testimony was relied upon by both the bank and by Rivera in their appeal briefs.
present suit. In the former, as underscored in the above-quoted Court ruling, the defendants did not file Thus (TSN of July 30, 1990. pp. 19-20):
anyresponsive pleading in the first case. In other words, they did not make any denial or raise any defense or
counter-claim therein In the case before us however, petitioners filed a responsive pleading to the complaint A: The procedure runs this way: Acquired assets was turned over to me and then I published
as a result of which, the issues were joined. it in the form of an inter-office memorandum distributed to all branches that these are
acquired assets for sale. I was instructed to advertise acquired assets for sale so on that
Indeed, by praying for affirmative reliefs and interposing counterclaims in their responsive pleadings, the basis, I have to entertain offer; to accept offer, formal offer and upon having been offered, I
petitioners became plaintiffs themselves in the original case, giving unto themselves the very remedies they present it to the Committee. I provide the Committee with necessary information about the
repeated in the Second Case. property such as original loan of the borrower, bid price during the foreclosure, total claim of
the bank, the appraised value at the time the property is being offered for sale and then the
information which are relative to the evaluation of the bank to buy which the Committee
Ultimately, what is truly important to consider in determining whether forum-shopping exists or not is the
considers and it is the Committee that evaluate as against the exposure of the bank and it is
vexation caused the courts and parties-litigant by a party who asks different courts and/or administrative
also the Committee that submit to the Conservator for final approval and once approved, we
agencies to rule on the same or related causes and/or to grant the same or substantially the same reliefs, in
have to execute the deed of sale and it is the Conservator that sign the deed of sale, sir.
the process creating the possibility of conflicting decisions being rendered by the different fora upon the same
issue. In this case, this is exactly the problem: a decision recognizing the perfection and directing the
enforcement of the contract of sale will directly conflict with a possible decision in the Second Case barring the The plaintiffs, therefore, at that meeting of August 1987 regarding their purpose of buying the
parties front enforcing or implementing the said sale. Indeed, a final decision in one would constitute res property, dealt with and talked to the right person. Necessarily, the agenda was the price of the
judicata in the other 28. property, and plaintiffs were dealing with the bank official authorized to entertain offers, to accept
offers and to present the offer to the Committee before which the said official is authorized to discuss
information relative to price determination. Necessarily, too, it being inherent in his authority, Rivera is
The foregoing conclusion finding the existence of forum-shopping notwithstanding, the only sanction possible
the officer from whom official information regarding the price, as determined by the Committee and
now is the dismissal of both cases with prejudice, as the other sanctions cannot be imposed because
approved by the Conservator, can be had. And Rivera confirmed his authority when he talked with the
petitioners' present counsel entered their appearance only during the proceedings in this Court, and the
plaintiff in August 1987. The testimony of plaintiff Demetria is clear on this point (TSN of May 31,1990,
Petition's VERIFICATION/CERTIFICATION contained sufficient allegations as to the pendency of the Second Case
pp. 27-28):
to show good faith in observing Circular 28-91. The Lawyers who filed the Second Case are not before us; thus
the rudiments of due process prevent us from motu propio imposing disciplinary measures against them in this
Decision. However, petitioners themselves (and particularly Henry Co, et al.) as litigants are admonished to Q: When you went to the Producers Bank and talked with Mr. Mercurio Rivera, did you ask him
strictly follow the rules against forum-shopping and not to trifle with court proceedings and processes They are point-blank his authority to sell any property?
warned that a repetition of the same will be dealt with more severely.
A: No, sir. Not point blank although it came from him, (W)hen I asked him how long it would
Having said that, let it be emphasized that this petition should be dismissed not merely because of forum- take because he was saying that the matter of pricing will be passed upon by the committee.
shopping but also because of the substantive issues raised, as will be discussed shortly. And when I asked him how long it will take for the committee to decide and he said the
committee meets every week. If I am not mistaken Wednesday and in about two week's ( sic)
Set V Corporation Code * Doctrine of Peircing Corpo Veil Cases*Page 106 of 164

time, in effect what he was saying he was not the one who was to decide. But he would refer authority (Francisco v. GSIS, 7 SCRA 577, 583-584; PNB v. Court of Appeals, 94 SCRA 357, 369-370;
it to the committee and he would relay the decision of the committee to me. Prudential Bank v. Court of Appeals, G.R. No. 103957, June 14, 1993). 29

Q Please answer the question. Article 1318 of the Civil Code enumerates the requisites of a valid and perfected contract as follows: "(1)
Consent of the contracting parties; (2) Object certain which is the subject matter of the contract; (3) Cause of
A He did not say that he had the authority (.) But he said he would refer the matter to the the obligation which is established."
committee and he would relay the decision to me and he did just like that.
There is no dispute on requisite no. 2. The object of the questioned contract consists of the six (6) parcels of
"Parenthetically, the Committee referred to was the Past Due Committee of which Luis Co was the land in Sta. Rosa, Laguna with an aggregate area of about 101 hectares, more or less, and covered by Transfer
Head, with Jose Entereso as one of the members. Certificates of Title Nos. T-106932 to T-106937. There is, however, a dispute on the first and third requisites.

What transpired after the meeting of early August 1987 are consistent with the authority and the Petitioners allege that "there is no counter-offer made by the Bank, and any supposed counter-offer which
duties of Rivera and the bank's internal procedure in the matter of the sale of bank's assets. As Rivera (or Co) may have made is unauthorized. Since there was no counter-offer by the Bank, there was
advised by Rivera, the plaintiffs made a formal offer by a letter dated August 20, 1987 stating that nothing for Ejercito (in substitution of Demetria and Janolo) to accept." 30 They disputed the factual basis of the
they would buy at the price of P3.5 Million in cash. The letter was for the attention of Mercurio Rivera respondent Court's findings that there was an offer made by Janolo for P3.5 million, to which the Bank counter-
who was tasked to convey and accept such offers. Considering an aspect of the official duty of Rivera offered P5.5 million. We have perused the evidence but cannot find fault with the said Court's findings of fact.
as some sort of intermediary between the plaintiffs-buyers with their proposed buying price on one Verily, in a petition under Rule 45 such as this, errors of fact if there be any - are, as a rule, not reviewable.
hand, and the bank Committee, the Conservator and ultimately the bank itself with the set price on The mere fact that respondent Court (and the trial court as well) chose to believe the evidence presented by
the other, and considering further the discussion of price at the meeting of August resulting in a respondent more than that presented by petitioners is not by itself a reversible error. In fact, such findings
formal offer of P3.5 Million in cash, there can be no other logical conclusion than that when, on merit serious consideration by this Court, particularly where, as in this case, said courts carefully and
September 1, 1987, Rivera informed plaintiffs by letter that "the bank's counter-offer is at P5.5 Million meticulously discussed their findings. This is basic.
for more than 101 hectares on lot basis," such counter-offer price had been determined by the Past
Due Committee and approved by the Conservator after Rivera had duly presented plaintiffs' offer for Be that as it may, and in addition to the foregoing disquisitions by the Court of Appeals, let us review the
discussion by the Committee of such matters as original loan of borrower, bid price during foreclosure, question of Rivera's authority to act and petitioner's allegations that the P5.5 million counter-offer was
total claim of the bank, and market value. Tersely put, under the established facts, the price of P5.5 extinguished by the P4.25 million revised offer of Janolo. Here, there are questions of law which could be drawn
Million was, as clearly worded in Rivera's letter (Exh. "E"), the official and definitive price at which the from the factual findings of the respondent Court. They also delve into the contractual elements of consent and
bank was selling the property. cause.

There were averments by defendants below, as well as before this Court, that the P5.5 Million price The authority of a corporate officer in dealing with third persons may be actual or apparent. The doctrine of
was not discussed by the Committee and that price. As correctly characterized by the trial court, this "apparent authority", with special reference to banks, was laid out in Prudential Bank vs. Court of Appeals31,
is not credible. The testimonies of Luis Co and Jose Entereso on this point are at best equivocal and where it was held that:
considering the gratuitous and self-serving character of these declarations, the bank's submission on
this point does not inspire belief. Both Co ad Entereso, as members of the Past Due Committee of the Conformably, we have declared in countless decisions that the principal is liable for obligations
bank, claim that the offer of the plaintiff was never discussed by the Committee. In the same vein, contracted by the agent. The agent's apparent representation yields to the principal's true
both Co and Entereso openly admit that they seldom attend the meetings of the Committee. It is representation and the contract is considered as entered into between the principal and the third
important to note that negotiations on the price had started in early August and the plaintiffs had person (citing National Food Authority vs. Intermediate Appellate Court, 184 SCRA 166).
already offered an amount as purchase price, having been made to understand by Rivera, the official
in charge of the negotiation, that the price will be submitted for approval by the bank and that the
A bank is liable for wrongful acts of its officers done in the interests of the bank or in the
bank's decision will be relayed to plaintiffs. From the facts, the official bank price. At any rate, the
course of dealings of the officers in their representative capacity but not for acts outside the
bank placed its official, Rivera, in a position of authority to accept offers to buy and negotiate the sale
scape of their authority (9 C.J.S., p. 417). A bank holding out its officers and agents as worthy
by having the offer officially acted upon by the bank. The bank cannot turn around and later say, as it
of confidence will not be permitted to profit by the frauds they may thus be enabled to
now does, that what Rivera states as the bank's action on the matter is not in fact so. It is a familiar
perpetrate in the apparent scope of their employment; nor will it be permitted to shirk its
doctrine, the doctrine of ostensible authority, that if a corporation knowingly permits one of its
responsibility for such frauds even though no benefit may accrue to the bank therefrom (10
officers, or any other agent, to do acts within the scope of an apparent authority, and thus holds him
Am Jur 2d, p. 114). Accordingly, a banking corporation is liable to innocent third persons
out to the public as possessing power to do those acts, the corporation will, as against any one who
where the representation is made in the course of its business by an agent acting within the
has in good faith dealt with the corporation through such agent, he estopped from denying his
Set V Corporation Code * Doctrine of Peircing Corpo Veil Cases*Page 107 of 164

general scope of his authority even though, in the particular case, the agent is secretly In the very recent case of Limketkai Sons Milling, Inc. vs. Court of Appeals, et. al.32, the Court, through Justice
abusing his authority and attempting to perpetrate a fraud upon his principal or some other Jose A. R. Melo, affirmed the doctrine of apparent authority as it held that the apparent authority of the officer
person, for his own ultimate benefit (McIntosh v. Dakota Trust Co., 52 ND 752, 204 NW 818, of the Bank of P.I. in charge of acquired assets is borne out by similar circumstances surrounding his dealings
40 ALR 1021). with buyers.

Application of these principles is especially necessary because banks have a fiduciary relationship with To be sure, petitioners attempted to repudiate Rivera's apparent authority through documents and testimony
the public and their stability depends on the confidence of the people in their honesty and efficiency. which seek to establish Rivera's actual authority. These pieces of evidence, however, are inherently weak as
Such faith will be eroded where banks do not exercise strict care in the selection and supervision of its they consist of Rivera's self-serving testimony and various inter-office memoranda that purport to show
employees, resulting in prejudice to their depositors. his limited actual authority, of which private respondent cannot be charged with knowledge. In any event, since
the issue is apparent authority, the existence of which is borne out by the respondent Court's findings, the
From the evidence found by respondent Court, it is obvious that petitioner Rivera has apparent or implied evidence of actual authority is immaterial insofar as the liability of a corporation is concerned 33.
authority to act for the Bank in the matter of selling its acquired assets. This evidence includes the following:
Petitioners also argued that since Demetria and Janolo were experienced lawyers and their "law firm" had once
(a) The petition itself in par. II-i (p. 3) states that Rivera was "at all times material to this case, acted for the Bank in three criminal cases, they should be charged with actual knowledge of Rivera's limited
Manager of the Property Management Department of the Bank". By his own admission, Rivera was authority. But the Court of Appeals in its Decision (p. 12) had already made a factual finding that the buyers
already the person in charge of the Bank's acquired assets (TSN, August 6, 1990, pp. 8-9); had no notice of Rivera's actual authority prior to the sale. In fact, the Bank has not shown that they acted as
its counsel in respect to any acquired assets; on the other hand, respondent has proven that Demetria and
Janolo merely associated with a loose aggrupation of lawyers (not a professional partnership), one of whose
(b) As observed by respondent Court, the land was definitely being sold by the Bank. And during the
members (Atty. Susana Parker) acted in said criminal cases.
initial meeting between the buyers and Rivera, the latter suggested that the buyers' offer should be
no less than P3.3 million (TSN, April 26, 1990, pp. 16-17);
Petitioners also alleged that Demetria's and Janolo's P4.25 million counter-offer in the letter dated September
17, 1987 extinguished the Bank's offer of P5.5 million 34 .They disputed the respondent Court's finding that
(c) Rivera received the buyers' letter dated August 30, 1987 offering P3.5 million (TSN, 30 July 1990,
"there was a meeting of minds when on 30 September 1987 Demetria and Janolo through Annex "L" (letter
p.11);
dated September 30, 1987) "accepted" Rivera's counter offer of P5.5 million under Annex "J" (letter dated
September 17, 1987)",citing the late Justice Paras 35, Art. 1319 of the Civil Code 36 and related Supreme Court
(d) Rivera signed the letter dated September 1, 1987 offering to sell the property for P5.5 million (TSN, rulings starting with Beaumont vs. Prieto 37.
July 30, p. 11);
However, the above-cited authorities and precedents cannot apply in the instant case because, as found by the
(e) Rivera received the letter dated September 17, 1987 containing the buyers' proposal to buy the respondent Court which reviewed the testimonies on this point, what was "accepted" by Janolo in his letter
property for P4.25 million (TSN, July 30, 1990, p. 12); dated September 30, 1987 was the Bank's offer of P5.5 million as confirmed and reiterated to Demetria and
Atty. Jose Fajardo by Rivera and Co during their meeting on September 28, 1987. Note that the said letter of
(f) Rivera, in a telephone conversation, confirmed that the P5.5 million was the final price of the Bank September 30, 1987 begins with"(p)ursuant to our discussion last 28 September 1987 . . .
(TSN, January 16, 1990, p. 18);
Petitioners insist that the respondent Court should have believed the testimonies of Rivera and Co that the
(g) Rivera arranged the meeting between the buyers and Luis Co on September 28, 1994, during September 28, 1987 meeting "was meant to have the offerors improve on their position of P5.5.
which the Bank's offer of P5.5 million was confirmed by Rivera (TSN, April 26, 1990, pp. 34-35). At said million."38However, both the trial court and the Court of Appeals found petitioners' testimonial evidence "not
meeting, Co, a major shareholder and officer of the Bank, confirmed Rivera's statement as to the credible", and we find no basis for changing this finding of fact.
finality of the Bank's counter-offer of P5.5 million (TSN, January 16, 1990, p. 21; TSN, April 26, 1990, p.
35); Indeed, we see no reason to disturb the lower courts' (both the RTC and the CA) common finding that private
respondents' evidence is more in keeping with truth and logic that during the meeting on September 28,
(h) In its newspaper advertisements and announcements, the Bank referred to Rivera as the officer 1987, Luis Co and Rivera "confirmed that the P5.5 million price has been passed upon by the Committee and
acting for the Bank in relation to parties interested in buying assets owned/acquired by the Bank. In could no longer be lowered (TSN of April 27, 1990, pp. 34-35)" 39. Hence, assuming arguendo that the counter-
fact, Rivera was the officer mentioned in the Bank's advertisements offering for sale the property in offer of P4.25 million extinguished the offer of P5.5 million, Luis Co's reiteration of the said P5.5 million price
question (cf. Exhs. "S" and "S-1"). during the September 28, 1987 meeting revived the said offer. And by virtue of the September 30, 1987 letter
accepting thisrevived offer, there was a meeting of the minds, as the acceptance in said letter was absolute
and unqualified.
Set V Corporation Code * Doctrine of Peircing Corpo Veil Cases*Page 108 of 164

We note that the Bank's repudiation, through Conservator Encarnacion, of Rivera's authority and action, . . . Of course, the bank's letter of September 1, 1987 on the official price and the plaintiffs'
particularly the latter's counter-offer of P5.5 million, as being "unauthorized and illegal" came only on May 12, acceptance of the price on September 30, 1987, are not, in themselves, formal contracts of sale. They
1988 or more than seven (7) months after Janolo' acceptance. Such delay, and the absence of any are however clear embodiments of the fact that a contract of sale was perfected between the parties,
circumstance which might have justifiably prevented the Bank from acting earlier, clearly characterizes the such contract being binding in whatever form it may have been entered into (case citations omitted).
repudiation as nothing more than a last-minute attempt on the Bank's part to get out of a binding contractual Stated simply, the banks' letter of September 1, 1987, taken together with plaintiffs' letter dated
obligation. September 30, 1987, constitute in law a sufficient memorandum of a perfected contract of sale.

Taken together, the factual findings of the respondent Court point to an implied admission on the part of the The respondent Court could have added that the written communications commenced not only from September
petitioners that the written offer made on September 1, 1987 was carried through during the meeting of 1, 1987 but from Janolo's August 20, 1987 letter. We agree that, taken together, these letters constitute
September 28, 1987. This is the conclusion consistent with human experience, truth and good faith. sufficient memoranda since they include the names of the parties, the terms and conditions of the contract,
the price and a description of the property as the object of the contract.
It also bears noting that this issue of extinguishment of the Bank's offer of P5.5 million was raised for the first
time on appeal and should thus be disregarded. But let it be assumed arguendo that the counter-offer during the meeting on September 28, 1987 did
constitute a "new" offer which was accepted by Janolo on September 30, 1987. Still, the statute of frauds will
This Court in several decisions has repeatedly adhered to the principle that points of law, theories, not apply by reason of the failure of petitioners to object to oral testimony proving petitioner Bank's counter-
issues of fact and arguments not adequately brought to the attention of the trial court need not be, offer of P5.5 million. Hence, petitioners by such utter failure to object are deemed to have waived any
and ordinarily will not be, considered by a reviewing court, as they cannot be raised for the first time defects of the contract under the statute of frauds, pursuant to Article 1405 of the Civil Code:
on appeal (Santos vs. IAC, No. 74243, November 14, 1986, 145 SCRA 592). 40
Art. 1405. Contracts infringing the Statute of Frauds, referred to in No. 2 of article 1403, are ratified by
. . . It is settled jurisprudence that an issue which was neither averred in the complaint nor raised the failure to object to the presentation of oral evidence to prove the same, or by the acceptance of
during the trial in the court below cannot be raised for the first time on appeal as it would be offensive benefits under them.
to the basic rules of fair play, justice and due process (Dihiansan vs. CA, 153 SCRA 713 [1987];
Anchuelo vs. IAC, 147 SCRA 434 [1987]; Dulos Realty & Development Corp. vs. CA, 157 SCRA 425 As private respondent pointed out in his Memorandum, oral testimony on the reaffirmation of the counter-offer
[1988]; Ramos vs. IAC, 175 SCRA 70 [1989]; Gevero vs. IAC, G.R. 77029, August 30, 1990). 41 of P5.5 million is a plenty and the silence of petitioners all throughout the presentation makes the evidence
binding on them thus;
Since the issue was not raised in the pleadings as an affirmative defense, private respondent was not given an
opportunity in the trial court to controvert the same through opposing evidence. Indeed, this is a matter of due A Yes, sir, I think it was September 28, 1987 and I was again present because Atty. Demetria told me
process. But we passed upon the issue anyway, if only to avoid deciding the case on purely procedural to accompany him we were able to meet Luis Co at the Bank.
grounds, and we repeat that, on the basis of the evidence already in the record and as appreciated by the
lower courts, the inevitable conclusion is simply that there was a perfected contract of sale. xxx xxx xxx

The Third Issue: Is the Contract Enforceable? Q Now, what transpired during this meeting with Luis Co of the Producers Bank?

The petition alleged42: A Atty. Demetria asked Mr. Luis Co whether the price could be reduced, sir.

Even assuming that Luis Co or Rivera did relay a verbal offer to sell at P5.5 million during the meeting Q What price?
of 28 September 1987, and it was this verbal offer that Demetria and Janolo accepted with their letter
of 30 September 1987, the contract produced thereby would be unenforceable by action there
A The 5.5 million pesos and Mr. Luis Co said that the amount cited by Mr. Mercurio Rivera is the final
being no note, memorandum or writing subscribed by the Bank to evidence such contract. (Please see
price and that is the price they intends (sic) to have, sir.
article 1403[2], Civil Code.)

Q What do you mean?.


Upon the other hand, the respondent Court in its Decision (p, 14) stated:

A That is the amount they want, sir.


Set V Corporation Code * Doctrine of Peircing Corpo Veil Cases*Page 109 of 164

Q What is the reaction of the plaintiff Demetria to Luis Co's statement ( sic) that the defendant Rivera's A It was not discussed by the Committee but it was discussed initially by Luis Co and the group of Atty.
counter-offer of 5.5 million was the defendant's bank (sic) final offer? Demetrio Demetria and Atty. Pajardo (sic) in that September 28, 1987 meeting, sir.

A He said in a day or two, he will make final acceptance, sir. [Direct testimony of Mercurio Rivera, TSN, 30 July 1990, pp. 14-15.]

Q What is the response of Mr. Luis Co?. The Fourth Issue: May the Conservator Revoke
the Perfected and Enforceable Contract.
A He said he will wait for the position of Atty. Demetria, sir.
It is not disputed that the petitioner Bank was under a conservator placed by the Central Bank of the
[Direct testimony of Atty. Jose Fajardo, TSN, January 16, 1990, at pp. 18-21.] Philippines during the time that the negotiation and perfection of the contract of sale took place. Petitioners
energetically contended that the conservator has the power to revoke or overrule actions of the management
or the board of directors of a bank, under Section 28-A of Republic Act No. 265 (otherwise known as the Central
Q What transpired during that meeting between you and Mr. Luis Co of the defendant Bank?
Bank Act) as follows:

A We went straight to the point because he being a busy person, I told him if the amount of P5.5
Whenever, on the basis of a report submitted by the appropriate supervising or examining
million could still be reduced and he said that was already passed upon by the committee. What the
department, the Monetary Board finds that a bank or a non-bank financial intermediary performing
bank expects which was contrary to what Mr. Rivera stated. And he told me that is the final offer of the
quasi-banking functions is in a state of continuing inability or unwillingness to maintain a state of
bank P5.5 million and we should indicate our position as soon as possible.
liquidity deemed adequate to protect the interest of depositors and creditors, the Monetary Board may
appoint a conservator to take charge of the assets, liabilities, and the management of that institution,
Q What was your response to the answer of Mr. Luis Co? collect all monies and debts due said institution and exercise all powers necessary to preserve the
assets of the institution, reorganize the management thereof, and restore its viability. He shall have
A I said that we are going to give him our answer in a few days and he said that was it. Atty. Fajardo the power to overrule or revoke the actions of the previous management and board of directors of the
and I and Mr. Mercurio [Rivera] was with us at the time at his office. bank or non-bank financial intermediary performing quasi-banking functions, any provision of law to
the contrary notwithstanding, and such other powers as the Monetary Board shall deem necessary.

Q For the record, your Honor please, will you tell this Court who was with Mr. Co in his Office in
Producers Bank Building during this meeting? In the first place, this issue of the Conservator's alleged authority to revoke or repudiate the perfected contract
of sale was raised for the first time in this Petition as this was not litigated in the trial court or Court of
Appeals. As already stated earlier, issues not raised and/or ventilated in the trial court, let alone in the Court of
A Mr. Co himself, Mr. Rivera, Atty. Fajardo and I.
Appeals, "cannot be raised for the first time on appeal as it would be offensive to the basic rules of fair play,
justice and due process."43
Q By Mr. Co you are referring to?

In the second place, there is absolutely no evidence that the Conservator, at the time the contract was
A Mr. Luis Co. perfected, actually repudiated or overruled said contract of sale. The Bank's acting conservator at the time,
Rodolfo Romey, never objected to the sale of the property to Demetria and Janolo. What petitioners are really
Q After this meeting with Mr. Luis Co, did you and your partner accede on (sic) the counter offer by the referring to is the letter of Conservator Encarnacion, who took over from Romey after the sale was perfected on
bank? September 30, 1987 (Annex V, petition) which unilaterally repudiated not the contract but the authority of
Rivera to make a binding offer and which unarguably came months after the perfection of the contract. Said
letter dated May 12, 1988 is reproduced hereunder:
A Yes, sir, we did.? Two days thereafter we sent our acceptance to the bank which offer we accepted,
the offer of the bank which is P5.5 million.

[Direct testimony of Atty. Demetria, TSN, 26 April 1990, at pp. 34-36.] May 12, 1988

Q According to Atty. Demetrio Demetria, the amount of P5.5 million was reached by the Committee
and it is not within his power to reduce this amount. What can you say to that statement that the
amount of P5.5 million was reached by the Committee?
Set V Corporation Code * Doctrine of Peircing Corpo Veil Cases*Page 110 of 164

Atty. Noe C. Zarate (Sgd.) Leonida T. Encarnacion


Zarate Carandang Perlas & Ass. LEONIDA T. EDCARNACION
Suite 323 Rufino Building Acting Conservator
Ayala Avenue, Makati, Metro-Manila
In the third place, while admittedly, the Central Bank law gives vast and far-reaching powers to the conservator
of a bank, it must be pointed out that such powers must be related to the "(preservation of) the assets of the
bank, (the reorganization of) the management thereof and (the restoration of) its viability." Such powers,
Dear Atty. Zarate: enormous and extensive as they are, cannot extend to the post-facto repudiation of perfected transactions,
otherwise they would infringe against the non-impairment clause of the Constitution 44. If the legislature itself
cannot revoke an existing valid contract, how can it delegate such non-existent powers to the conservator
This pertains to your letter dated May 5, 1988 on behalf of Attys. Janolo and Demetria regarding the
under Section 28-A of said law?
six (6) parcels of land located at Sta. Rosa, Laguna.

Obviously, therefore, Section 28-A merely gives the conservator power to revoke contracts that are, under
We deny that Producers Bank has ever made a legal counter-offer to any of your clients nor perfected
existing law, deemed to be defective i.e., void, voidable, unenforceable or rescissible. Hence, the
a "contract to sell and buy" with any of them for the following reasons.
conservator merely takes the place of a bank's board of directors. What the said board cannot do such as
repudiating a contract validly entered into under the doctrine of implied authority the conservator cannot do
In the "Inter-Office Memorandum" dated April 25, 1986 addressed to and approved by former Acting either. Ineluctably, his power is not unilateral and he cannot simply repudiate valid obligations of the Bank. His
Conservator Mr. Andres I. Rustia, Producers Bank Senior Manager Perfecto M. Pascua detailed the authority would be only to bring court actions to assail such contracts as he has already done so in the
functions of Property Management Department (PMD) staff and officers (Annex A.), you will instant case. A contrary understanding of the law would simply not be permitted by the Constitution. Neither by
immediately read that Manager Mr. Mercurio Rivera or any of his subordinates has no authority, power common sense. To rule otherwise would be to enable a failing bank to become solvent, at the expense of third
or right to make any alleged counter-offer. In short, your lawyer-clients did not deal with the parties, by simply getting the conservator to unilaterally revoke all previous dealings which had one way or
authorized officers of the bank. another or come to be considered unfavorable to the Bank, yielding nothing to perfected contractual rights nor
vested interests of the third parties who had dealt with the Bank.
Moreover, under Sec. 23 and 36 of the Corporation Code of the Philippines (Bates Pambansa Blg. 68.)
and Sec. 28-A of the Central Bank Act (Rep. Act No. 265, as amended), only the Board of The Fifth Issue: Were There Reversible Errors of Facts?
Directors/Conservator may authorize the sale of any property of the corportion/bank..
Basic is the doctrine that in petitions for review under Rule 45 of the Rules of Court, findings of fact by the
Our records do not show that Mr. Rivera was authorized by the old board or by any of the bank Court of Appeals are not reviewable by the Supreme Court. In Andres vs. Manufacturers Hanover & Trust
conservators (starting January, 1984) to sell the aforesaid property to any of your clients. Apparently, Corporation,45, we held:
what took place were just preliminary discussions/consultations between him and your clients, which
everyone knows cannotbind the Bank's Board or Conservator.
. . . The rule regarding questions of fact being raised with this Court in a petition for certiorari under
Rule 45 of the Revised Rules of Court has been stated in Remalante vs. Tibe, G.R. No. 59514, February
We are, therefore, constrained to refuse any tender of payment by your clients, as the same is 25, 1988, 158 SCRA 138, thus:
patently violative of corporate and banking laws. We believe that this is more than sufficient legal
justification for refusing said alleged tender.
The rule in this jurisdiction is that only questions of law may be raised in a petition for certiorari under
Rule 45 of the Revised Rules of Court. "The jurisdiction of the Supreme Court in cases brought to it
Rest assured that we have nothing personal against your clients. All our acts are official, legal and in from the Court of Appeals is limited to reviewing and revising the errors of law imputed to it, its
accordance with law. We also have no personal interest in any of the properties of the Bank. findings of the fact being conclusive " [Chan vs. Court of Appeals, G.R. No. L-27488, June 30, 1970, 33
SCRA 737, reiterating a long line of decisions]. This Court has emphatically declared that "it is not the
Please be advised accordingly. function of the Supreme Court to analyze or weigh such evidence all over again, its jurisdiction being
limited to reviewing errors of law that might have been committed by the lower court" (Tiongco v. De
la Merced, G. R. No. L-24426, July 25, 1974, 58 SCRA 89; Corona vs. Court of Appeals, G.R. No. L-
Very truly yours,
62482, April 28, 1983, 121 SCRA 865; Baniqued vs. Court of Appeals, G. R. No. L-47531, February 20,
1984, 127 SCRA 596). "Barring, therefore, a showing that the findings complained of are totally devoid
of support in the record, or that they are so glaringly erroneous as to constitute serious abuse of
discretion, such findings must stand, for this Court is not expected or required to examine or contrast
Set V Corporation Code * Doctrine of Peircing Corpo Veil Cases*Page 111 of 164

the oral and documentary evidence submitted by the parties" [Santa Ana, Jr. vs. Hernandez, G. R. No. Conservator after Rivera had duly presented plaintiffs' offer for discussion by the Committee . . .
L-16394, December 17, 1966, 18 SCRA 973] [at pp. 144-145.] Tersely put, under the established fact, the price of P5.5 Million was, as clearly worded in Rivera's
letter (Exh. "E"), the official and definitive price at which the bank was selling the property. (p. 11, CA
Likewise, in Bernardo vs. Court of Appeals 46
, we held: Decision)

The resolution of this petition invites us to closely scrutinize the facts of the case, relating to the xxx xxx xxx
sufficiency of evidence and the credibility of witnesses presented. This Court so held that it is not the
function of the Supreme Court to analyze or weigh such evidence all over again. The Supreme Court's . . . The argument deserves scant consideration. As pointed out by plaintiff, during the meeting of
jurisdiction is limited to reviewing errors of law that may have been committed by the lower court. The September 28, 1987 between the plaintiffs, Rivera and Luis Co, the senior vice-president of the bank,
Supreme Court is not a trier of facts. . . . where the topic was the possible lowering of the price, the bank official refused it and confirmed that
the P5.5 Million price had been passed upon by the Committee and could no longer be lowered (TSN of
As held in the recent case of Chua Tiong Tay vs. Court of Appeals and Goldrock Construction and Development April 27, 1990, pp. 34-35) (p. 15, CA Decision).
Corp. 47:
The respondent Court did not believe the evidence of the petitioners on this point, characterizing it as "not
The Court has consistently held that the factual findings of the trial court, as well as the Court of credible" and "at best equivocal and considering the gratuitous and self-serving character of these
Appeals, are final and conclusive and may not be reviewed on appeal. Among the exceptional declarations, the bank's submissions on this point do not inspire belief."
circumstances where a reassessment of facts found by the lower courts is allowed are when the
conclusion is a finding grounded entirely on speculation, surmises or conjectures; when the inference To become credible and unequivocal, petitioners should have presented then Conservator Rodolfo Romey to
made is manifestly absurd, mistaken or impossible; when there is grave abuse of discretion in the testify on their behalf, as he would have been in the best position to establish their thesis. Under the rules on
appreciation of facts; when the judgment is premised on a misapprehension of facts; when the evidence 51, such suppression gives rise to the presumption that his testimony would have been adverse, if
findings went beyond the issues of the case and the same are contrary to the admissions of both produced.
appellant and appellee. After a careful study of the case at bench, we find none of the above grounds
present to justify the re-evaluation of the findings of fact made by the courts below. The second point was squarely raised in the Court of Appeals, but petitioners' evidence was deemed
insufficient by both the trial court and the respondent Court, and instead, it was respondent's submissions that
In the same vein, the ruling of this Court in the recent case of South Sea Surety and Insurance Company were believed and became bases of the conclusions arrived at.
Inc. vs.Hon. Court of Appeals, et al. 48 is equally applicable to the present case:
In fine, it is quite evident that the legal conclusions arrived at from the findings of fact by the lower courts are
We see no valid reason to discard the factual conclusions of the appellate court, . . . (I)t is not the valid and correct. But the petitioners are now asking this Court to disturb these findings to fit the conclusion
function of this Court to assess and evaluate all over again the evidence, testimonial and they are espousing, This we cannot do.
documentary, adduced by the parties, particularly where, such as here, the findings of both the trial
court and the appellate court on the matter coincide. (emphasis supplied) To be sure, there are settled exceptions where the Supreme Court may disregard findings of fact by the Court of
Appeals 52. We have studied both the records and the CA Decision and we find no such exceptions in this case.
Petitioners, however, assailed the respondent Court's Decision as "fraught with findings and conclusions which On the contrary, the findings of the said Court are supported by a preponderance of competent and credible
were not only contrary to the evidence on record but have no bases at all," specifically the findings that (1) the evidence. The inferences and conclusions are seasonably based on evidence duly identified in the Decision.
"Bank's counter-offer price of P5.5 million had been determined by the past due committee and approved by Indeed, the appellate court patiently traversed and dissected the issues presented before it, lending credibility
conservator Romey, after Rivera presented the same for discussion" and (2) "the meeting with Co was not to and dependability to its findings. The best that can be said in favor of petitioners on this point is that the
scale down the price and start negotiations anew, but a meeting on the already determined price of P5.5 factual findings of respondent Court did not correspond to petitioners' claims, but were closer to the evidence
million" Hence,citing Philippine National Bank vs. Court of Appeals 49, petitioners are asking us to review and as presented in the trial court by private respondent. But this alone is no reason to reverse or ignore such
reverse such factual findings. factual findings, particularly where, as in this case, the trial court and the appellate court were in common
agreement thereon. Indeed, conclusions of fact of a trial judge as affirmed by the Court of Appeals are
The first point was clearly passed upon by the Court of Appeals 50
, thus: conclusive upon this Court, absent any serious abuse or evident lack of basis or capriciousness of any kind,
because the trial court is in a better position to observe the demeanor of the witnesses and their courtroom
manner as well as to examine the real evidence presented.
There can be no other logical conclusion than that when, on September 1, 1987, Rivera informed
plaintiffs by letter that "the bank's counter-offer is at P5.5 Million for more than 101 hectares on lot
basis, "such counter-offer price had been determined by the Past Due Committee and approved by the Epilogue.
Set V Corporation Code * Doctrine of Peircing Corpo Veil Cases*Page 112 of 164

In summary, there are two procedural issues involved forum-shopping and the raising of issues for the first time This petition for review on certiorari, under Rule 45 of the Rules of Court, seeks to annul the decision 1 of the
on appeal [viz., the extinguishment of the Bank's offer of P5.5 million and the conservator's powers to Court of Appeals in C.A. G.R. CV No. 10014 affirming the decision rendered by Branch 135, Regional Trial Court
repudiate contracts entered into by the Bank's officers] which per se could justify the dismissal of the of Makati, Metro Manila. The procedural antecedents of this petition are as follows:
present case. We did not limit ourselves thereto, but delved as well into the substantive issues the perfection
of the contract of sale and its enforceability, which required the determination of questions of fact. While the On January 23, 1985, petitioner filed a complaint 2 against private respondents to recover three thousand four
Supreme Court is not a trier of facts and as a rule we are not required to look into the factual bases of hundred twelve and six centavos (P3,412.06), representing the balance of the jeep body purchased by the
respondent Court's decisions and resolutions, we did so just the same, if only to find out whether there is Manuels from petitioner; an additional sum of twenty thousand four hundred fifty-four and eighty centavos
reason to disturb any of its factual findings, for we are only too aware of the depth, magnitude and vigor by (P20,454.80) representing the unpaid balance on the cost of repair of the vehicle; and six thousand pesos
which the parties through their respective eloquent counsel, argued their positions before this Court. (P6,000.00) for cost of suit and attorney's fees. 3 To the original balance on the price of jeep body were added
the costs of repair. 4 In their answer, private respondents interposed a counterclaim for unpaid legal services by
We are not unmindful of the tenacious plea that the petitioner Bank is operating abnormally under a Gregorio Manuel in the amount of fifty thousand pesos (P50,000) which was not paid by the incorporators,
government-appointed conservator and "there is need to rehabilitate the Bank in order to get it back on its feet directors and officers of the petitioner. The trial court decided the case on June 26, 1985, in favor of petitioner
. . . as many people depend on (it) for investments, deposits and well as employment. As of June 1987, the in regard to the petitioner's claim for money, but also allowed the counter-claim of private respondents. Both
Bank's overdraft with the Central Bank had already reached P1.023 billion . . . and there were (other) offers to parties appealed. On April 15, 1991, the Court of Appeals sustained the trial court's decision. 5 Hence, the
buy the subject properties for a substantial amount of money." 53 present petition.

While we do not deny our sympathy for this distressed bank, at the same time, the Court cannot emotionally For our review in particular is the propriety of the permissive counterclaim which private respondents filed
close its eyes to overriding considerations of substantive and procedural law, like respect for perfected together with their answer to petitioner's complaint for a sum of money. Private respondent Gregorio Manuel
contracts, non-impairment of obligations and sanctions against forum-shopping, which must be upheld under alleged as an affirmative defense that, while he was petitioner's Assistant Legal Officer, he represented
the rule of law and blind justice. members of the Francisco family in the intestate estate proceedings of the late Benita Trinidad. However, even
after the termination of the proceedings, his services were not paid. Said family members, he said, were also
This Court cannot just gloss over private respondent's submission that, while the subject properties may incorporators, directors and officers of petitioner. Hence to petitioner's collection suit, he filed a counter
currently command a much higher price, it is equally true that at the time of the transaction in 1987, the price permissive counterclaim for the unpaid attorney's fees. 6
agreed upon of P5.5 million was reasonable, considering that the Bank acquired these properties at a
foreclosure sale for no more than P3.5 million 54. That the Bank procrastinated and refused to honor its For failure of petitioner to answer the counterclaim, the trial court declared petitioner in default on this score,
commitment to sell cannot now be used by it to promote its own advantage, to enable it to escape its binding and evidence ex-parte was presented on the counterclaim. The trial court ruled in favor of private respondents
obligation and to reap the benefits of the increase in land values. To rule in favor of the Bank simply because and found that Gregorio Manuel indeed rendered legal services to the Francisco family in Special Proceedings
the property in question has algebraically accelerated in price during the long period of litigation is to reward Number 7803 "In the Matter of Intestate Estate of Benita Trinidad". Said court also found that his legal
lawlessness and delays in the fulfillment of binding contracts. Certainly, the Court cannot stamp its imprimatur services were not compensated despite repeated demands, and thus ordered petitioner to pay him the amount
on such outrageous proposition. of fifty thousand (P50,000.00) pesos. 7

WHEREFORE, finding no reversible error in the questioned Decision and Resolution, the Court hereby DENIES Dissatisfied with the trial court's order, petitioner elevated the matter to the Court of Appeals, posing the
the petition. The assailed Decision is AFFIRMED. Moreover, petitioner Bank is REPRIMANDED for engaging in following issues:
forum-shopping and WARNED that a repetition of the same or similar acts will be dealt with more severely.
Costs against petitioners.SO ORDERED. I.

SECOND DIVISION G.R. No. 100812 June 25, 1999 WHETHER OR NOT THE DECISION RENDERED BY THE LOWER COURT IS NULL AND VOID AS IT NEVER ACQUIRED
JURISDICTION OVER THE PERSON OF THE DEFENDANT.
FRANCISCO MOTORS CORPORATION, petitioner, vs.
COURT OF APPEALS and SPOUSES GREGORIO and LIBRADA MANUEL, respondents. II.

WHETHER OR NOT PLAINTIFF-APPELLANT NOT BEING A REAL PARTY IN THE ALLEGED PERMISSIVE
COUNTERCLAIM SHOULD BE HELD LIABLE TO THE CLAIM OF DEFENDANT-APPELLEES.
QUISUMBING, J.:
III.
Set V Corporation Code * Doctrine of Peircing Corpo Veil Cases*Page 113 of 164

WHETHER OR NOT THERE IS FAILURE ON THE PART OF PLAINTIFF-APPELLANT TO ANSWER THE ALLEGED Now before us, petitioner assigns the following errors:
PERMISSIVE COUNTERCLAIM. 8
I.
Petitioner contended that the trial court did not acquire jurisdiction over it because no summons was validly
served on it together with the copy of the answer containing the permissive counterclaim. Further, petitioner THE COURT OF APPEALS ERRED IN APPLYING THE DOCTRINE OF PIERCING THE VEIL OF CORPORATE ENTITY.
questions the propriety of its being made party to the case because it was not the real party in interest but the
individual members of the Francisco family concerned with the intestate case.
II.

In its assailed decision now before us for review, respondent Court of Appeals held that a counterclaim must be
THE COURT OF APPEALS ERRED IN AFFIRMING THAT THERE WAS JURISDICTION OVER PETITIONER WITH
answered in ten (10) days, pursuant to Section 4, Rule 11, of the Rules of Court; and nowhere does it state in
RESPECT TO THE COUNTERCLAIM. 13
the Rules that a party still needed to be summoned anew if a counterclaim was set up against him. Failure to
serve summons, said respondent court, did not effectively negate trial court's jurisdiction over petitioner in the
matter of the counterclaim. It likewise pointed out that there was no reason for petitioner to be excused from
answering the counterclaim. Court records showed that its former counsel, Nicanor G. Alvarez, received the
copy of the answer with counterclaim two (2) days prior to his withdrawal as counsel for petitioner. Moreover Petitioner submits that respondent court should not have resorted to piercing the veil of corporate fiction
when petitioner's new counsel, Jose N. Aquino, entered his appearance, three (3) days still remained within the because the transaction concerned only respondent Gregorio Manuel and the heirs of the late Benita Trinidad.
period to file an answer to the counterclaim. Having failed to answer, petitioner was correctly considered in According to petitioner, there was no cause of action by said respondent against petitioner; personal concerns
default by the trial of the heirs should be distinguished from those involving corporate affairs. Petitioner further contends that the
court. 9 Even assuming that the trial court acquired no jurisdiction over petitioner, respondent court also said, present case does not fall among the instances wherein the courts may look beyond the distinct personality of
but having filed a motion for reconsideration seeking relief from the said order of default, petitioner was a corporation. According to petitioner, the services for which respondent Gregorio Manuel seeks to collect fees
estopped from further questioning the trial court's jurisdiction. 10 from petitioner are personal in nature. Hence, it avers the heirs should have been sued in their personal
capacity, and not involve the corporation. 14
On the question of its liability for attorney's fees owing to private respondent Gregorio Manuel, petitioner
argued that being a corporation, it should not be held liable therefor because these fees were owed by the With regard to the permissive counterclaim, petitioner also insists that there was no proper service of the
incorporators, directors and officers of the corporation in their personal capacity as heirs of Benita Trinidad. answer containing the permissive counterclaim. It claims that the counterclaim is a separate case which can
Petitioner stressed that the personality of the corporation, vis-a-vis the individual persons who hired the only be properly served upon the opposing party through summons. Further petitioner states that by nature, a
services of private respondent, is separate and distinct, 11 hence, the liability of said individuals did not become permissive counterclaim is one which does not arise out of nor is necessarily connected with the subject of the
an obligation chargeable against petitioner. opposing party's claim. Petitioner avers that since there was no service of summons upon it with regard to the
counterclaim, then the court did not acquire jurisdiction over petitioner. Since a counterclaim is considered an
Nevertheless, on the foregoing issue, the Court of Appeals ruled as follows: action independent from the answer, according to petitioner, then in effect there should be two simultaneous
actions between the same parties: each party is at the same time both plaintiff and defendant with respect to
the other,15 requiring in each case separate summonses.
However, this distinct and separate personality is merely a fiction created by law for convenience and
to promote justice. Accordingly, this separate personality of the corporation may be disregarded, or
the veil of corporate fiction pierced, in cases where it is used as a cloak or cover for found (sic) In their Comment, private respondents focus on the two questions raised by petitioner. They defend the
illegality, or to work an injustice, or where necessary to achieve equity or when necessary for the propriety of piercing the veil of corporate fiction, but deny the necessity of serving separate summonses on
protection of creditors. (Sulo ng Bayan, Inc. vs. Araneta, Inc., 72 SCRA 347) Corporations are petitioner in regard to their permissive counterclaim contained in the answer.
composed of natural persons and the legal fiction of a separate corporate personality is not a shield
for the commission of injustice and inequity. (Chemplex Philippines, Inc. vs. Pamatian, 57 SCRA 408). Private respondents maintain both trial and appellate courts found that respondent Gregorio Manuel was
employed as assistant legal officer of petitioner corporation, and that his services were solicited by the
In the instant case, evidence shows that the plaintiff-appellant Francisco Motors Corporation is incorporators, directors and members to handle and represent them in Special Proceedings No. 7803,
composed of the heirs of the late Benita Trinidad as directors and incorporators for whom defendant concerning the Intestate Estate of the late Benita Trinidad. They assert that the members of petitioner
Gregorio Manuel rendered legal services in the intestate estate case of their deceased mother. corporation took advantage of their positions by not compensating respondent Gregorio Manuel after the
Considering the aforestated principles and circumstances established in this case, equity and justice termination of the estate proceedings despite his repeated demands for payment of his services. They cite
demands plaintiff-appellant's veil of corporate identity should be pierced and the defendant be findings of the appellate court that support piercing the veil of corporate identity in this particular case. They
compensated for legal services rendered to the heirs, who are directors of the plaintiff-appellant assert that the corporate veil may be disregarded when it is used to defeat public convenience, justify wrong,
corporation. 12 protect fraud, and defend crime. It may also be pierced, according to them, where the corporate entity is being
Set V Corporation Code * Doctrine of Peircing Corpo Veil Cases*Page 114 of 164

used as an alter ego, adjunct, or business conduit for the sole benefit of the stockholders or of another as an employee of petitioner from whom it could be deduced he was also receiving a salary. His move to
corporate entity. In these instances, they aver, the corporation should be treated merely as an association of recover unpaid legal fees through a counterclaim against Francisco Motors Corporation, to offset the unpaid
individual persons. 16 balance of the purchase and repair of a jeep body could only result from an obvious misapprehension that
petitioner's corporate assets could be used to answer for the liabilities of its individual directors, officers, and
Private respondents dispute petitioner's claim that its right to due process was violated when respondents' incorporators. Such result if permitted could easily prejudice the corporation, its own creditors, and even other
counterclaim was granted due course, although no summons was served upon it. They claim that no provision stockholders; hence, clearly inequitous to petitioner.
in the Rules of Court requires service of summons upon a defendant in a counterclaim. Private respondents
argue that when the petitioner filed its complaint before the trial court it voluntarily submitted itself to the Furthermore, considering the nature of the legal services involved, whatever obligation said incorporators,
jurisdiction of the court. As a consequence, the issuance of summons on it was no longer necessary. Private directors and officers of the corporation had incurred, it was incurred in their personal capacity. When directors
respondents say they served a copy of their answer with affirmative defenses and counterclaim on petitioner's and officers of a corporation are unable to compensate a party for a personal obligation, it is far-fetched to
former counsel, Nicanor G. Alvarez. While petitioner would have the Court believe that respondents served said allege that the corporation is perpetuating fraud or promoting injustice, and be thereby held liable therefor by
copy upon Alvarez after he had withdrawn his appearance as counsel for the petitioner, private respondents piercing its corporate veil. While there are no hard and fast rules on disregarding separate corporate identity,
assert that this contention is utterly baseless. Records disclose that the answer was received two (2) days we must always be mindful of its function and purpose. A court should be careful in assessing the milieu where
before the former counsel for petitioner withdrew his appearance, according to private respondents. They the doctrine of piercing the corporate veil may be applied. Otherwise an injustice, although unintended, may
maintain that the present petition is but a form of dilatory appeal, to set off petitioner's obligations to the result from its erroneous application.
respondents by running up more interest it could recover from them. Private respondents therefore claim
damages against petitioner. 17 The personality of the corporation and those of its incorporators, directors and officers in their personal
capacities ought to be kept separate in this case. The claim for legal fees against the concerned individual
To resolve the issues in this case, we must first determine the propriety of piercing the veil of corporate fiction. incorporators, officers and directors could not be properly directed against the corporation without violating
basic principles governing corporations. Moreover, every action including a counterclaim must be
Basic in corporation law is the principle that a corporation has a separate personality distinct from its prosecuted or defended in the name of the real party in interest. 20 It is plainly an error to lay the claim for legal
stockholders and from other corporations to which it may be connected. 18 However, under the doctrine of fees of private respondent Gregorio Manuel at the door of petitioner (FMC) rather than individual members of
piercing the veil of corporate entity, the corporation's separate juridical personality may be disregarded, for the Francisco family.
example, when the corporate identity is used to defeat public convenience, justify wrong, protect fraud, or
defend crime. Also, where the corporation is a mere alter ego or business conduit of a person, or where the However, with regard to the procedural issue raised by petitioner's allegation, that it needed to be summoned
corporation is so organized and controlled and its affairs are so conducted as to make it merely an anew in order for the court to acquire jurisdiction over it, we agree with respondent court's view to the contrary.
instrumentality, agency, conduit or adjunct of another corporation, then its distinct personality may be Section 4, Rule 11 of the Rules of Court provides that a counterclaim or cross-claim must be answered within
ignored. 19 In these circumstances, the courts will treat the corporation as a mere aggrupation of persons and ten (10) days from service. Nothing in the Rules of Court says that summons should first be served on the
the liability will directly attach to them. The legal fiction of a separate corporate personality in those cited defendant before an answer to counterclaim must be made. The purpose of a summons is to enable the court
instances, for reasons of public policy and in the interest of justice, will be justifiably set aside. to acquire jurisdiction over the person of the defendant. Although a counterclaim is treated as an entirely
distinct and independent action, the defendant in the counterclaim, being the plaintiff in the original complaint,
In our view, however, given the facts and circumstances of this case, the doctrine of piercing the corporate veil has already submitted to the jurisdiction of the court. Following Rule 9, Section 3 of the 1997 Rules of Civil
has no relevant application here. Respondent court erred in permitting the trial court's resort to this doctrine. Procedure, 21 if a defendant (herein petitioner) fails to answer the counterclaim, then upon motion of plaintiff,
The rationale behind piercing a corporation's identity in a given case is to remove the barrier between the the defendant may be declared in default. This is what happened to petitioner in this case, and this Court finds
corporation from the persons comprising it to thwart the fraudulent and illegal schemes of those who use the no procedural error in the disposition of the appellate court on this particular issue. Moreover, as noted by the
corporate personality as a shield for undertaking certain proscribed activities. However, in the case at bar, respondent court, when petitioner filed its motion seeking to set aside the order of default, in effect it
instead of holding certain individuals or persons responsible for an alleged corporate act, the situation has submitted itself to the jurisdiction of the court. As well said by respondent court:
been reversed. It is the petitioner as a corporation which is being ordered to answer for the personal liability of
certain individual directors, officers and incorporators concerned. Hence, it appears to us that the doctrine has Further on the lack of jurisdiction as raised by plaintiff-appellant[,] [t]he records show that upon its
been turned upside down because of its erroneous invocation. Note that according to private respondent request, plaintiff-appellant was granted time to file a motion for reconsideration of the disputed
Gregorio Manuel his services were solicited as counsel for members of the Francisco family to represent them decision. Plaintiff-appellant did file its motion for reconsideration to set aside the order of default and
in the intestate proceedings over Benita Trinidad's estate. These estate proceedings did not involve any the judgment rendered on the counterclaim.
business of petitioner.
Thus, even if the court acquired no jurisdiction over plaintiff-appellant on the counterclaim, as it
Note also that he sought to collect legal fees not just from certain Francisco family members but also from vigorously insists, plaintiff-appellant is considered to have submitted to the court's jurisdiction when it
petitioner corporation on the claims that its management had requested his services and he acceded thereto filed the motion for reconsideration seeking relief from the court. (Soriano vs. Palacio, 12 SCRA 447). A
Set V Corporation Code * Doctrine of Peircing Corpo Veil Cases*Page 115 of 164

party is estopped from assailing the jurisdiction of a court after voluntarily submitting himself to its petitioners amended their complaint to implead private respondent Well World Toys, Inc. (Well World for
jurisdiction. (Tejones vs. Gironella, 159 SCRA 100). Estoppel is a bar against any claims of lack of brevity), a corporation also engaged in the manufacture of stuffed toys for export with principal office located
jurisdiction. (Balais vs. Balais, 159 SCRA 37). 22 at Las Pias, Manila.

WHEREFORE, the petition is hereby GRANTED and the assailed decision is hereby REVERSED insofar only as it In their complaint, petitioners basically alleged that they were original probationary employees 10 of Well World
held Francisco Motors Corporation liable for the legal obligation owing to private respondent Gregorio Manuel; but were later laid off in 1989 "for starting to organize themselves into a union". 11 They applied with and were
but this decision is without prejudice to his filing the proper suit against the concerned members of the thereafter hired by April. On February 2, 1990, and while under the employ of April, petitioners conducted a
Francisco family in their personal capacity. No pronouncement as to costs.1wphi certification election where their union, Alyansang Likha ng mga Anak ng Bayan (ALAB), won as the exclusive
bargaining agent for the workers. Petitioners thereafter submitted a Collective Bargaining Agreement proposal
1. THIRD DIVISION which April rejected in view of its cessation of operation. The closure, petitioners declared, is April's clever ploy
to "defeat their right to self organization". 12 Petitioners further alleged that the original incorporators and
principal officers of April were likewise the original incorporators of Well World, thus both corporations should
G.R. No. 108936 October 4, 1996
be treated as one corporation liable for their claims. In his decision dated December 20, 1991, the Labor Arbiter
found as valid the closure of April, and treated April and Well World as two distinct corporations. While the
SOL LAGUIO, RENE LAOLAO, ANNALIZA ENSANDO, EDELIZA ASAS, LILIA MARAY, EVELYN UNTALAN,* seventy-seven complainants were ruled to be the employees of April, the Labor Arbiter, nevertheless, ordered
ROSARIO CHICO, REYNALDO GARCIA, MERLITA DE LOS SANTOS,* JOSEPHINE DERONG,* GEMMA Well World to give financial assistance to its former forty-nine probationary employees who were found to have
TIBALAO BANTOLO, LUCY ALMONTE,* CRISPINA VANQUARDIA, NARCISA VENZON, NORMA been laid off in 1989 due to business losses. April was likewise ordered to pay its separated employees their
ELEGANTE,* AMELITA MORENO,* ABNER PETILOS, NARCISO HILAPO, DOLORES OLAES, MELINDA separation pay and, together with Well World, assessed for attorney's fees. Petitioners appealed before the
LLADOC, ERNA AZARCON, and APRIL TOY, INC. WORKERS UNION ALAB, petitioners, National Labor Relations Commission (NLRC), but to no avail. Hence, this petition, supported by the Office of
vs. the Solicitor General, anchored solely on the NLRC's purported grave abuse of discretion in not finding April and
NATIONAL LABOR RELATIONS COMMISSION, WELL WORLD TOYS, INC., APRIL TOYS, INC., YU SHENG Well World as one corporation liable for their grievances.
LING, JENN L. WANG, EUCLIFF CHENG, CHI SHENG LIN, NENITA C. AGUIRRE, MA. THERESA R.
CADIENTE and GLICERIA R. AGUIRRE, respondents.
To bolster their claim that April and Well World are one and the same corporation, petitioners argue
that both corporations have the same set of incorporators. Thus
RESOLUTION
Incorporators of Well World Incorporators of April

Name Citizenship No. of Name Citizenship No. of


FRANCISCO, J.:p Shares Shares

ntSO ORDERED. Eucliff Cheng Filipino 148 Nenita C. Aguirre Filipino 2,797
Jenn Li Wang Chinese 25 Ma. Theresa Cadiente Filipino 800
Private respondent April Toy, Inc. (April for brevity) is a domestic corporation incorporated on January 6, 1989, Yu-Sheng Ling Chinese 25 Gliceria R. Aguirre Filipino 400
for the purpose of "manufacturing, importing, exporting, buying , selling, sub-contracting or otherwise dealing Chia-Sheng Lin Chinese 25 Pacifico R. Cadiente Filipino 1
in, at wholesale and retail," 1 stuffed toys, with principal place of business at Paraaque, Manila. On December Chia-Yu-Yen Lin Chinese 25 Emalyn A. Fernandez Filipino 1
20, 1989, or after almost a year of operation, April posted a memorandum 2 within its premises and circulated a Ma. Theresa Cadiente Filipino 1 Erlinda M. Hizon Filipino 1
copy of the same among its employees informing them of its dire financial condition. To avert further business Gliceria Aguirre Filipino 1
reverses, April decided to shorten its corporate term "up to February 28, 1990," 3 submitted a notice of 4,000
dissolution to the Securities and Exchange Commission and published the same in a newspaper of general 250
circulation 4. April also notified its employees, the Department of Labor and Employment, 5 the Social Security
System, 6 the Board of Investments, 7 the Bureau of Internal Revenue, 8 and the Municipality of Paraaque of its (Petition, pp. 4-5; Rollo, pp. 5-6; Memorandum pp. 7-8, Rollo, 242-243.)
dissolution.
Petitioners also insist that the two corporations "are being managed by Mr. Jean Li Wang" 13 and that
In view of April's cessation of operations, petitioners who initially composed of seventy-seven employees below their articles of incorporation, general information sheets and certificates of increase of capital stock
filed a complaint for "illegal shutdown/retrenchment/dismissal and unfair labor practice." 9 On June 21, 1990, were notarized by the same Notary Public. Additionally, petitioners aver that when some of them
Set V Corporation Code * Doctrine of Peircing Corpo Veil Cases*Page 116 of 164

transferred from Well World to April they were not given their separation pay, a factor which April Toy, Inc. were incorporators and minority stockholders of Well-World Toy, Inc. Hence it does not
presumably proves that April is a mere conduit of Well World. Petitioners likewise assert that their mean that the two (2) corporations are adjunct and conduit. There is not express provision under the
transfer from one corporation to another was made at the time that they were on the process of Corporation Law prohibiting stockholders or incorporators of a corporation to be a stockholder or
organizing a union. Finally, petitioners allege that April and Well World were engaged in the same line incorporator of another corporation.
of business, with the latter also supplying the former raw materials and machineries. These
circumstances, petitioners claim, make their case akin to the case of La Campana Coffee Factory The fiction that a corporation was a distinct and separate personality shall not be used as a subterfuge
Inc. v. Kaisahan ng mga Manggagawa sa La Campana (KKM), 93 Phil. 160, where the Court considered to commit injustice and circumvent the law does not apply in the present case. There is no conclusive
two corporations, i.e., La Campana Coffee Factory, Inc. and La Campana Gaugau Packing, as one and evidence to convince us that respondent April Toy, Inc. was established and later on closed to defeat
the same. We are not persuaded. the rights of the workers of Well-World Toy, Inc. which would otherwise support the charge of unfair
labor practice. Hence, we find that the two (2) corporations are separate and distinct entities. 17
A cursory examination of the composition of April and Well World's incorporators and the number of
shares they own hardly supports petitioners' asseveration. In fact, petitioners' allegation that both and, on appeal, by public respondent NLRC, thus:
corporations were managed by a single individual, Mr. Jen Li Weng, contradicts paragraphs 7 and 8 of
their petition which state:
[R]elative to the closure of April Toy, it is clear from the records that as early as December 1989 or
long before a certification election was conducted among its rank-and-file employees on February 2,
7. Respondents Yu-Sheng Ling, Jen Li Weng (Alias James Wang), Eucliff Cheng and Chia Sheng 1990, the employees were already aware that April Toy was suffering from financial crisis. It further
Lin are the President, Managing Director, Treasurer and Secretary respectively of respondent appearing that April Toy continued to suffer losses as evidenced by its financial statements ending
Well World Toy, Inc., all of whom are holding office at 399-B Real St., Talon, Las Pias, Metro December 31, 1989 and its balance sheet ending March 31, 1990, the Labor Arbiter a quo correctly
Manila. . . ruled that the eventual closure of its business on February 27, 1990, is valid.

8. Respondents Nenita C. Aguirre, Ma. Theresa R. Cadiente and Gliceria R. Aguirre are the Anent the question of whether or not April Toy and Well-World Toy are one and the same, with the facts
President, Treasurer and Secretary, respectively of respondent April Toy, Inc. all of whom are and circumstances showing that the owners of April Toy are different from those of Well-World, the
holding office at No. 6-C Ascie Avenue, Severina Industrial Estate, Km. 16 South management of one being different from the other, and the office of April Toy is situated more than
Superhighway Paraaque. . . 14 ten kilometers away from Well-World, plus the fact that the closure of April Toy was for valid reasons,
the Labor Arbiter likewise correctly opined that the two corporations are separate and distinct from
What clearly appears therefrom is that the two corporations have two different set of officers managing their each other, and that there is no basis for piercing the veil of corporate fiction. 18
respective affairs in two separate offices.
Furthermore, the petition hinges on the factual findings of both the Labor Arbiter and the NLRC. It
It is basic that a corporation is invested by law with a personality separate and distinct from those of the should be stressed that the factual findings of quasi-judicial agencies like the NLRC are generally
persons composing it as well as from that of any other legal entity to which it may be related. Mere substantial accorded not only respect but, at times, finality if such are supported by substantial evidence 19.
identity of the incorporators of the two corporations does not necessarily imply fraud, 15 nor warrant the Judicial review by this Court in labor cases does not go so far as to require this Court to evaluate the
piercing of the veil of corporate fiction. In the absence of clear and convincing evidence that April and Well sufficiency of the evidence upon which the Labor Arbiter and respondent NLRC based their
World's corporate personalities were used to perpetuate fraud, or circumvent the law said corporations were determination as our review is limited to issues of jurisdiction or grave abuse of discretion. In the
rightly treated as distinct and separate from each other. Further, petitioners' emphatic reliance with the case of instant suit, the findings of the Labor Arbiter was duly affirmed by respondent NLRC, findings amply
La Campana is misplaced. In La Campana, unlike in this case, the two corporations, i.e., La Campana Coffee supported by substantial evidence on record. We find no cogent reason, as none was presented, to
Factory, Inc. and La Campana Gaugau Packing, were not only owned by the same person, but moreover have a deviate from the same.
single management, business office and a single payroll for both businesses. Indeed, the workers of La
Campana Gaugau Packing "were interchangeable, that is, the laborers from gaugau factory were sometimes ACCORDINGLY, finding no grave abuse of discretion on the part of respondent NLRC in rendering the
transferred to the coffee factory and vice-versa." 16 assailed resolution, the instant petition is hereby DISMISSED for lack of merit.SO ORDERED.

We thus quote with approval the observations made by the Labor Arbiter as follows: SECOND DIVISION G.R. No. 124715 January 24, 2000

We can not fully subscribe to the above contention of the complainants. We do not believe that the RUFINA LUY LIM, petitioner, vs.
circumstances related by the complainants are sufficient indicia that the two corporations are one and COURT OF APPEALS, AUTO TRUCK TBA CORPORATION, SPEED DISTRIBUTING, INC., ACTIVE
the same corporation although it appears that two of the original incorporators and stockholders of DISTRIBUTORS, ALLIANCE MARKETING CORPORATION, ACTION COMPANY, INC. respondents.
Set V Corporation Code * Doctrine of Peircing Corpo Veil Cases*Page 117 of 164

BUENA, J.: xxx xxx xxx

Alliance Block 3, Lot 6, Dacca BF Homes,


May a corporation, in its universality, be the proper subject of and be included in the inventory of the estate of
Marketing, Inc. Paraaque, Metro Manila.
a deceased person?
xxx xxx xxx
Petitioner disputes before us through the instant petition for review on certiorari, the decision1 of the Court of Speed
Appeals promulgated on 18 April 1996, in CA-GR SP No. 38617, which nullified and set aside the orders dated 910 Barrio Niog, Aguinaldo
Distributing
Highway, Bacoor, Cavite.
04 July 19952, 12 September 19953 and 15 September 19954 of the Regional Trial Court of Quezon City, Branch Inc.
93, sitting as a probate court.
xxx xxx xxx

Petitioner Rufina Luy Lim is the surviving spouse of late Pastor Y. Lim whose estate is the subject of probate Auto Truck TBA 2251 Roosevelt Avenue, Quezon
proceedings in Special Proceedings Q-95-23334, entitled, "In Re: Intestate Estate of Pastor Y. Lim Rufina Luy Corp. City.
Lim, represented by George Luy, Petitioner".1wphi1.nt xxx xxx xxx

Active
Private respondents Auto Truck Corporation, Alliance Marketing Corporation, Speed Distributing, Inc., Active Block 3, Lot 6, Dacca BF Homes,
Distributors,
Distributing, Inc. and Action Company are corporations formed, organized and existing under Philippine laws Paraaque, Metro Manila.
Inc.
and which owned real properties covered under the Torrens system.
xxx xxx xxx
On 11 June 1994, Pastor Y. Lim died intestate. Herein petitioner, as surviving spouse and duly represented by Action 100 20th Avenue Murphy, Quezon
her nephew George Luy, fried on 17 March 1995, a joint petition 5 for the administration of the estate of Pastor Y. Company City or 92-D Mc-Arthur Highway
Lim before the Regional Trial Court of Quezon City. Valenzuela Bulacan.

Private respondent corporations, whose properties were included in the inventory of the estate of Pastor Y. Lim, 3.1 Although the above business entities dealt and engaged in business with the public as
then filed a motion6 for the lifting of lis pendens and motion7 for exclusion of certain properties from the estate corporations, all their capital, assets and equity were however, personally owned by the late
of the decedent. Pastor Y Lim. Hence the alleged stockholders and officers appearing in the respective articles
of incorporation of the above business entities were mere dummies of Pastor Y. Lim, and they
In an order8 dated 08 June 1995, the Regional Trial Court of Quezon City, Branch 93, sitting as a probate court, were listed therein only for purposes of registration with the Securities and Exchange
granted the private respondents' twin motions, in this wise: Commission.

4. Pastor Lim, likewise, had Time, Savings and Current Deposits with the following banks: (a)
Metrobank, Grace Park, Caloocan City and Quezon Avenue, Quezon City Branches and (b) First
Intestate Bank (formerly Producers Bank), Rizal Commercial Banking Corporation and in other banks
Wherefore, the Register of Deeds of Quezon City is hereby ordered to lift, expunge or delete the
whose identities are yet to be determined.
annotation of lis pendens on Transfer Certificates of Title Nos. 116716, 116717, 116718, 116719 and
5182 and it is hereby further ordered that the properties covered by the same titles as well as those
properties by (sic) Transfer Certificate of Title Nos. 613494, 363123, 236236 and 263236 are excluded 5. That the following real properties, although registered in the name of the above entities, were
from these proceedings.SO ORDERED. actually acquired by Pastor Y. Lim during his marriage with petitioner, to wit:

Subsequently, Rufina Luy Lim filed a verified amended petition 9 which contained the following averments: Corporation Title Location

xxx xxx xxx


3. The late Pastor Y. Lim personally owned during his lifetime the following business entities, to wit:
k. Auto Truck TCT No. 617726 Sto. Domingo TBA
Corporation Cainta, Rizal
Business
Address:
Entity
Set V Corporation Code * Doctrine of Peircing Corpo Veil Cases*Page 118 of 164

q. Alliance Marketing TCT No. 27896 Prance, Metro Manila On 15 September 1995, the probate court acting on an ex parte motion filed by petitioner, issued an
order13 the dispositive portion of which reads:

Wherefore, the parties and the following banks concerned herein under enumerated are hereby
Copies of the above-mentioned Transfer Certificate of Title and/or Tax Declarations are hereto attached
ordered to comply strictly with this order and to produce and submit to the special administrators,
as Annexes "C" to "W".
through this Honorable Court within (5) five days from receipt of this order their respective records of
the savings/current accounts/time deposits and other deposits in the names of Pastor Lim and/or
xxx xxx xxx corporations above-mentioned, showing all the transactions made or done concerning savings/current
accounts from January 1994 up to their receipt of this court order.
7. The aforementioned properties and/or real interests left by the late Pastor Y. Lim, are all conjugal in
nature, having been acquired by him during the existence of his marriage with petitioner. xxx xxx xxx

8. There are other real and personal properties owned by Pastor Y. Lim which petitioner could not as SO ORDERED.
yet identify. Petitioner, however will submit to this Honorable Court the identities thereof and the
necessary documents covering the same as soon as possible.
Private respondent filed a special civil action for certiorari14, with an urgent prayer for a restraining order or writ
of preliminary injunction, before the Court of Appeals questioning the orders of the Regional Trial Court, sitting
On 04 July 1995, the Regional Trial Court acting on petitioner's motion issued an order 10, thus: as a probate court.

Wherefore, the order dated 08 June 1995 is hereby set aside and the Registry of Deeds of Quezon City On 18 April 1996, the Court of Appeals, finding in favor of herein private respondents, rendered the assailed
is hereby directed to reinstate the annotation of lis pendens in case said annotation had already been decision15, the decretal portion of which declares:
deleted and/or cancelled said TCT Nos. 116716, 116717, 116718, 116719 and 51282.

Wherefore, premises considered, the instant special civil action for certiorari is hereby granted, The
Further more (sic), said properties covered by TCT Nos. 613494, 365123, 236256 and 236237 by impugned orders issued by respondent court on July 4, 1995 and September 12, 1995 are hereby
virtue of the petitioner are included in the instant petition.SO ORDERED. nullified and set aside. The impugned order issued by respondent on September 15, 1995 is nullified
insofar as petitioner corporations" bank accounts and records are concerned.SO ORDERED.
On 04 September 1995, the probate court appointed Rufina Lim as special administrator 11 and Miguel Lim and
Lawyer Donald Lee, as co-special administrators of the estate of Pastor Y. Lim, after which letters of Through the expediency of Rule 45 of the Rules of Court, herein petitioner Rufina Luy Lim now comes before us
administration were accordingly issued. with a lone assignment of error16:

In an order12 dated 12 September 1995, the probate court denied anew private respondents' motion for The respondent Court of Appeals erred in reversing the orders of the lower court which merely allowed
exclusion, in this wise: the preliminary or provisional inclusion of the private respondents as part of the estate of the late
deceased (sic) Pastor Y. Lim with the respondent Court of Appeals arrogating unto itself the power to
The issue precisely raised by the petitioner in her petition is whether the corporations are the mere repeal, to disobey or to ignore the clear and explicit provisions of Rules 81,83,84 and 87 of the Rules
alter egos or instrumentalities of Pastor Lim, Otherwise (sic) stated, the issue involves the piercing of of Court and thereby preventing the petitioner, from performing her duty as special administrator of
the corporate veil, a matter that is clearly within the jurisdiction of this Honorable Court and not the the estate as expressly provided in the said Rules.
Securities and Exchange Commission. Thus, in the case of Cease vs. Court of Appeals, 93 SCRA 483,
the crucial issue decided by the regular court was whether the corporation involved therein was the Petitioner's contentions tread on perilous grounds.
mere extension of the decedent. After finding in the affirmative, the Court ruled that the assets of the
corporation are also assets of the estate.
In the instant petition for review, petitioner prays that we affirm the orders issued by the probate court which
were subsequently set aside by the Court of Appeals.
A reading of P.D. 902, the law relied upon by oppositors, shows that the SEC's exclusive (sic) applies
only to intra-corporate controversy. It is simply a suit to settle the intestate estate of a deceased
Yet, before we delve into the merits of the case, a review of the rules on jurisdiction over probate proceedings
person who, during his lifetime, acquired several properties and put up corporations as his
is indeed in order.
instrumentalities.SO ORDERED.
Set V Corporation Code * Doctrine of Peircing Corpo Veil Cases*Page 119 of 164

The provisions of Republic Act 769117, which introduced amendments to Batas Pambansa Blg. 129, are This Court, in PASTOR, JR. vs. COURT OF APPEALS,18 held:
pertinent:
. . . As a rule, the question of ownership is an extraneous matter which the probate court cannot
Sec. 1. Section 19 of Batas Pambansa Blg. 129, otherwise known as the "Judiciary Reorganization Act of 1980", resolve with finality. Thus, for the purpose of determining whether a certain property should or should
is hereby amended to read as follows: not be included in the inventory of estate properties, the Probate Court may pass upon the title
thereto, but such determination is provisional, not conclusive, and is subject to the final decision in a
Sec. 19. Jurisdiction in civil cases. Regional Trial Courts shall exercise exclusive jurisdiction: separate action to resolve title.

xxx xxx xxx

(4) In all matters of probate, both testate and intestate, where the gross value of the estate exceeds We reiterated the rule in PEREIRA vs. COURT OF APPEALS19:
One Hundred Thousand Pesos (P100,000) or, in probate matters in Metro Manila, where such gross
value exceeds Two Hundred Thousand Pesos (P200,000); . . . The function of resolving whether or not a certain property should be included in the inventory or
list of properties to be administered by the administrator is one clearly within the competence of the
xxx xxx xxx probate court. However, the court's determination is only provisional in character, not conclusive, and
is subject to the final decision in a separate action which may be instituted by the parties.

Sec. 3. Section 33 of the same law is hereby amended to read as follows:


Further, in MORALES vs. CFI OF CAVITE20 citing CUIZON vs. RAMOLETE21, We made an exposition on the probate
court's limited jurisdiction:
Sec. 33. Jurisdiction of Metropolitan Trial Courts, Municipal Trial Courts and Municipal Circuit Trial Courts in Civil
Cases. Metropolitan Trial Courts, Municipal Trial Courts and Municipal Circuit Trial Courts shall exercise:
It is a well-settled rule that a probate court or one in charge of proceedings whether testate or
intestate cannot adjudicate or determine title to properties claimed to be a part of the estate and
1. Exclusive original jurisdiction over civil actions and probate proceedings, testate and intestate, including the
which are equally claimed to belong to outside parties. All that the said court could do as regards said
grant of provisional remedies in proper cases, where the value of the personal property, estate or amount of
properties is to determine whether they should or should not be included in the inventory or list of
the demand does not exceed One Hundred Thousand Pesos (P100,000) or, in Metro Manila where such personal
properties to be administered by the administrator. If there is no dispute, well and good; but if there is,
property, estate or amount of the demand does not exceed Two Hundred Thousand Pesos (P200,000), exclusive
then the parties, the administrator and the opposing parties have to resort to an ordinary action for a
of interest, damages of whatever kind, attorney's fees, litigation expenses and costs, the amount of which must
final determination of the conflicting claims of title because the probate court cannot do so.
be specifically alleged, Provided, that interest, damages of whatever kind, attorney's, litigation expenses and
costs shall be included in the determination of the filing fees, Provided further, that where there are several
claims or causes of actions between the same or different parties, embodied in the same complaint, the Again, in VALERA vs. INSERTO22, We had occasion to elucidate, through Mr. Justice Andres Narvasa 23:
amount of the demand shall be the totality of the claims in all the causes of action, irrespective of whether the
causes of action arose out of the same or different transactions; Settled is the rule that a Court of First Instance (now Regional Trial Court), acting as a probate court,
exercises but limited jurisdiction, and thus has no power to take cognizance of and determine the
xxx xxx xxx issue of title to property claimed by a third person adversely to the decedent, unless the claimant and
all other parties having legal interest in the property consent, expressly or impliedly, to the submission
of the question to the probate court for adjudgment, or the interests of third persons are not thereby
Simply put, the determination of which court exercises jurisdiction over matters of probate depends upon the
prejudiced, the reason for the exception being that the question of whether or not a particular matter
gross value of the estate of the decedent.
should be resolved by the court in the exercise of its general jurisdiction or of its limited jurisdiction as
a special court (e.g. probate, land registration, etc.), is in reality not a jurisdictional but in essence of
As to the power and authority of the probate court, petitioner relies heavily on the principle that a probate procedural one, involving a mode of practice which may be waived. . . .
court may pass upon title to certain properties, albeit provisionally, for the purpose of determining whether a
certain property should or should not be included in the inventory.
. . . . These considerations assume greater cogency where, as here, the Torrens title is not in the
decedent's name but in others, a situation on which this Court has already had occasion to rule . . . .
In a litany of cases, We defined the parameters by which the court may extend its probing arms in the (emphasis Ours)
determination of the question of title in probate proceedings.
Set V Corporation Code * Doctrine of Peircing Corpo Veil Cases*Page 120 of 164

Petitioner, in the present case, argues that the parcels of land covered under the Torrens system and registered of corporate fiction, the presumption of conclusiveness of said titles in favor of private respondents should
in the name of private respondent corporations should be included in the inventory of the estate of the stand undisturbed.
decedent Pastor Y. Lim, alleging that after all the determination by the probate court of whether these
properties should be included or not is merely provisional in nature, thus, not conclusive and subject to a final Accordingly, the probate court was remiss in denying private respondents' motion for exclusion. While it may
determination in a separate action brought for the purpose of adjudging once and for all the issue of title. be true that the Regional Trial Court, acting in a restricted capacity and exercising limited jurisdiction as a
probate court, is competent to issue orders involving inclusion or exclusion of certain properties in the
Yet, under the peculiar circumstances, where the parcels of land are registered in the name of private inventory of the estate of the decedent, and to adjudge, albeit, provisionally the question of title over
respondent corporations, the jurisprudence pronounced in BOLISAY vs., ALCID 24 is of great essence and finds properties, it is no less true that such authority conferred upon by law and reinforced by jurisprudence, should
applicability, thus: be exercised judiciously, with due regard and caution to the peculiar circumstances of each individual case.

It does not matter that respondent-administratrix has evidence purporting to support her claim of Notwithstanding that the real properties were duly registered under the Torrens system in the name of private
ownership, for, on the other hand, petitioners have a Torrens title in their favor, which under the law is respondents, and as such were to be afforded the presumptive conclusiveness of title, the probate court
endowed with incontestability until after it has been set aside in the manner indicated in the law itself, obviously opted to shut its eyes to this gleamy fact and still proceeded to issue the impugned orders.
which of course, does not include, bringing up the matter as a mere incident in special proceedings for
the settlement of the estate of deceased persons. . . . By its denial of the motion for exclusion, the probate court in effect acted in utter disregard of the presumption
of conclusiveness of title in favor of private respondents. Certainly, the probate court through such brazen act
. . . . In regard to such incident of inclusion or exclusion, We hold that if a property covered by Torrens transgressed the clear provisions of law and infringed settled jurisprudence on this matter.
title is involved, the presumptive conclusiveness of such title should be given due weight, and in the
absence of strong compelling evidence to the contrary, the holder thereof should be considered as the Moreover, petitioner urges that not only the properties of private respondent corporations are properly part of
owner of the property in controversy until his title is nullified or modified in an appropriate ordinary the decedent's estate but also the private respondent corporations themselves. To rivet such flimsy contention,
action, particularly, when as in the case at bar, possession of the property itself is in the persons petitioner cited that the late Pastor Y. Lim during his lifetime, organized and wholly-owned the five corporations,
named in the title. . . . which are the private respondents in the instant case. 25 Petitioner thus attached as Annexes "F" 26 and "G"27 of
the petition for review affidavits executed by Teresa Lim and Lani Wenceslao which among others, contained
A perusal of the records would reveal that no strong compelling evidence was ever presented by petitioner to averments that the incorporators of Uniwide Distributing, Inc. included on the list had no actual and
bolster her bare assertions as to the title of the deceased Pastor Y. Lim over the properties. Even so, P.D. 1529, participation in the organization and incorporation of the said corporation. The affiants added that the persons
otherwise known as, "The Property Registration Decree", proscribes collateral attack on Torrens Title, hence: whose names appeared on the articles of incorporation of Uniwide Distributing, Inc., as incorporators thereof,
are mere dummies since they have not actually contributed any amount to the capital stock of the corporation
xxx xxx xxx and have been merely asked by the late Pastor Y. Lim to affix their respective signatures thereon.

Sec. 48. Certificate not subject to collateral attack. A certificate of title shall not be subject to It is settled that a corporation is clothed with personality separate and distinct from that of the persons
collateral attack. It cannot be altered, modified or cancelled except in a direct proceeding in composing it. It may not generally be held liable for that of the persons composing it. It may not be held liable
accordance with law. for the personal indebtedness of its stockholders or those of the entities connected with it. 28

In CUIZON vs. RAMOLETE, where similarly as in the case at bar, the property subject of the controversy was Rudimentary is the rule that a corporation is invested by law with a personality distinct and separate from its
duly registered under the Torrens system, We categorically stated: stockholders or members. In the same vein, a corporation by legal fiction and convenience is an entity shielded
by a protective mantle and imbued by law with a character alien to the persons comprising it.

. . . Having been apprised of the fact that the property in question was in the possession of third
parties and more important, covered by a transfer certificate of title issued in the name of such third Nonetheless, the shield is not at all times invincible. Thus, in FIRST PHILIPPINE INTERNATIONAL BANK vs.COURT
parties, the respondent court should have denied the motion of the respondent administrator and OF APPEALS29, We enunciated:
excluded the property in question from the inventory of the property of the estate. It had no authority
to deprive such third persons of their possession and ownership of the property. . . . . . . When the fiction is urged as a means of perpetrating a fraud or an illegal act or as a vehicle for the
evasion of an existing obligation, the circumvention of statutes, the achievement or perfection of a
Inasmuch as the real properties included in the inventory of the estate of the Late Pastor Y. Lim are in the monopoly or generally the perpetration of knavery or crime, the veil with which the law covers and
possession of and are registered in the name of private respondent corporations, which under the law possess isolates the corporation from the members or stockholders who compose it will be lifted to allow for its
a personality separate and distinct from their stockholders, and in the absence of any cogency to shred the veil consideration merely as an aggregation of individuals. . . .
Set V Corporation Code * Doctrine of Peircing Corpo Veil Cases*Page 121 of 164

Piercing the veil of corporate entity requires the court to see through the protective shroud which exempts its corporations.WHEREFORE, in view of the foregoing disquisitions, the instant petition is hereby DISMISSED for
stockholders from liabilities that ordinarily, they could be subject to, or distinguishes one corporation from a lack of merit and the decision of the Court of Appeals which nullified and set aside the orders issued by the
seemingly separate one, were it not for the existing corporate fiction. 30 Regional Trial Court, Branch 93, acting as a probate court, dated 04 July 1995 and 12 September 1995 is
AFFIRMED.1wphi1.ntSO ORDERED.
The corporate mask may be lifted and the corporate veil may be pierced when a corporation is just but the
alter ego of a person or of another corporation. Where badges of fraud exist, where public convenience is SECOND DIVISION G.R. No. 98310 October 24, 1996
defeated; where a wrong is sought to be justified thereby, the corporate fiction or the notion of legal entity
should come to naught.31 MATUGUINA INTEGRATED WOOD PRODUCTS, INC., petitioner, vs.
The HON. COURT OF APPEALS, DAVAO ENTERPRISES CORPORATION, The HON. MINISTER, (NOW
Further, the test in determining the applicability of the doctrine of piercing the veil of corporate fiction is as SECRETARY) of NATURAL RESOURCES AND PHILLIP CO, respondents.
follows: 1) Control, not mere majority or complete stock control, but complete domination, not only of finances
but of policy and business practice in respect to the transaction attacked so that the corporate entity as to this TORRES, JR., J.:p
transaction had at the time no separate mind, will or existence of its own; (2) Such control must have been
used by the defendant to commit fraud or wrong, to perpetuate the violation of a statutory or other positive
legal duty, or dishonest and unjust act in contravention of plaintiffs legal right; and (3) The aforesaid control
and breach of duty must proximately cause the injury or unjust loss complained of. The absence of any of these
elements prevent "piercing the corporate veil".32 Matuguina Integrated Wood Products Inc. (MIWPI, for brevity) filed this action for Prohibition, Damages and
Injunction, in order to prevent the respondent Minister (now Secretary) of Natural Resources from enforcing its
Order of Execution against it, for liability arising from an alleged encroachment of the petitioner over the timber
Mere ownership by a single stockholder or by another corporation of all or nearly all of the capital stock of a
concession of respondent DAVENCOR located in Mati, Davao Oriental.
corporation is not of itself a sufficient reason for disregarding the fiction of separate corporate personalities. 33

The Regional Trial Court, Branch 17, Davao City, ruled in favor of the petitioner, but on appeal, was reversed by
Moreover, to disregard the separate juridical personality of a corporation, the wrong-doing must be clearly and
the respondent Court of Appeals in its decision dated February 25, 1991, which found MIWPI, as an alter ego of
convincingly established. It cannot be presumed.34
Milagros Matuguina and/or Matuguina Logging Enterprises (MLE), to be liable to DAVENCOR for the illegal
encroachment.
Granting arguendo that the Regional Trial Court in this case was not merely acting in a limited capacity as a
probate court, petitioner nonetheless failed to adduce competent evidence that would have justified the court
The following are the antecedent facts:
to impale the veil of corporate fiction. Truly, the reliance reposed by petitioner on the affidavits executed by
Teresa Lim and Lani Wenceslao is unavailing considering that the aforementioned documents possess no
weighty probative value pursuant to the hearsay rule. Besides it is imperative for us to stress that such On June 28, 1973, the Acting Director of the Bureau of Forest Development issued Provisional Timber License
affidavits are inadmissible in evidence inasmuch as the affiants were not at all presented during the course of (PTL) No. 30, covering an area of 5,400 hectares to Ms. Milagros Matuguina who was then doing business under
the proceedings in the lower court. To put it differently, for this Court to uphold the admissibility of said the name of MLE, a sole proprietorship venture. A portion, covering 1,900 hectares, of the said area was
documents would be to relegate from Our duty to apply such basic rule of evidence in a manner consistent with located within the territorial boundary of Gov. Generoso in Mati, Davao Oriental, and adjoined the timber
the law and jurisprudence. concession of Davao Enterprises Corporation (DAVENCOR), the private respondent in this case.

Our pronouncement in PEOPLE BANK AND TRUST COMPANY vs. LEONIDAS35 finds pertinence: On July 10, 1974, petitioner Matuguina Integrated Wood Products, Inc. (MIWPI), was incorporated, having an
authorized capital stock of Ten Million Pesos (P10,000,000.00). 1 The incorporators/stockholders of MIWPI, and
their stock subscriptions were as follows:
Affidavits are classified as hearsay evidence since they are not generally prepared by the affiant but
by another who uses his own language in writing the affiant's statements, which may thus be either
omitted or misunderstood by the one writing them. Moreover, the adverse party is deprived of the Name No. Of Shares Amount of Capital
opportunity to cross-examine the affiants. For this reason, affidavits are generally rejected for being Subscribed Stock Subscribed
hearsay, unless the affiant themselves are placed on the witness stand to testify thereon.
1. Henry Wee 1,160,000 1,160,000.00
As to the order36 of the lower court, dated 15 September 1995, the Court of Appeals correctly observed that the 2. Ma. Milagros Matuguina 400,000 400,000.00
Regional Trial Court, Branch 93 acted without jurisdiction in issuing said order; The probate court had no 3. Alejandro Chua Chun 200,000 200,000.00
authority to demand the production of bank accounts in the name of the private respondent 4. Bernadita Chua 120,000 120,000.00
Set V Corporation Code * Doctrine of Peircing Corpo Veil Cases*Page 122 of 164

5. Domingo Herrera 40,000 40,000.00 For our Resolution is the appeal by MATUGUINA LOGGING ENTERPRISES (MLR, for short) of the Order
6. Manuel Hernaez 40,000 40,000.00 dated 15 July 1991 of the Director of Forest Development finding and declaring MLE to have
7. Luis Valderama 40,000 40,000.00 encroached upon, and conducted illegal logging operations within the license or concession area of
DAVAO ENTERPRISES CORPORATION. The aforesaid Order dispositively states:
2,000,000 2,000,000.00
======== ========= Wherefore, there being a clear and convincing proof that Matuguina Conducted illegal
operation within the license area of DAVENCOR, above named respondent is hereby ordered
Milagros Matuguina became the majority stockholder of MIWPI on September 24, 1974, when the latter's Board to pay to the complainant the equivalent value in pesos of 2,352.04 cubic meters of timber
of Directors approved by Resolution the transfer of 1,000,000 shares from Henry Wee to Milagros Matuguina, based on the market price obtaining, at the logpond of the respondent at the time of cutting,
thus giving her seventy percent (70%) stock ownership of MIWPI. minus the cost of production, or to restitute to the complainant equal volume of 2,352.04
cubic meters of logs owned by respondent to be taken at respondent's logpond. The
In an undated letter 2 to the Director of Forest Development (BFD) on November 26, 1974, Milagros Matuguina respondent is hereby directed to comply with this Order within a period of ninety (90) days
requested the Director for a change of name and transfer of management of PTL No. 30 from a single from receipt of this Order and after the lapse of the said period, no compliance has been
proprietorship under her name, to that of MIWPI. made by the respondent, its logging operations shallipso facto become automatically
suspended until respondent shall have complied as directed.

This request was favorably endorsed on December 2, 1974 3 by the BFD's Acting Director, Jose Viado to
respondent Secretary of Natural Resources, who approved the same on September 5, 1975. 4 The Regional Director of Region II, Davao City is hereby instructed to implement this Order
and to submit his compliance report within ten (10) days after the lapse of the ninety (90)
days period within which the respondent is directed to comply with this Order.
On July 17, 1975, Milagros Matuguina and petitioner MIWPI executed a Deed of Transfer 5 transferring all of the
former's rights, interests, ownership and participation in Provincial Timber License No. 30 to the latter for and in
consideration of 148,000 shares of stocks in MIWPI. And that the dispositive portion of the said decision states:

A copy of said deed was submitted to the Director of Forest Development and petitioner MIWPI had since been WHEREFORE, the Order dated 15 July 1981 of the Director of Forest Development is hereby
acting as holder and licensee of PTL No. 30 AFFIRMED.

On July 28, 1975, pending approval of the request to transfer the PTL to MIWPI, DAVENCOR, through its When the Decision of the Minister of Natural Resources became final and executory, Philip Co and DAVENCOR
Assistant General Manager, complained to the District Forester at Mati, Davao Oriental that Milagros requested the respondent Minister on October 30, 1986 to issue immediately a writ of execution against MLE
Matuguina/MLE had encroached into and was conducting logging operations in DAVENCOR's timber concession. and/or MIWPI. 9 The Order of Execution 10 was issued on January 6, 1987 by the Minister through the latter's
Assistant on Legal Affairs. The said Order directed the issuance of a writ of execution, not only against MLE, but
likewise against MIWPI. The dispositive portion of the order provides:
After investigation of DAVENCOR's complaint, the Investigating Committee which looked into DAVENCOR's
complaint submitted its report to the Director, finding that MLE had encroached on the concession area of
DAVENCOR. In line with this, the Director of Forest Development issued an Order 6 on July 15, 1981, finding and WHEREFORE, let a Writ of Execution be issued against Matuguina Logging Enterprises and/or
declaring MLE to have encroached upon, and conducted illegal logging operations within the licensed or Matuguina Integrated Wood Products, Inc. For the satisfaction of the Decision of the Bureau of Forest
concession area of DAVENCOR. Development dated 15 July 1981, and the Order of this office dated 1 October 1986.SO ORDERED.

11
MLE appealed the Order to the Ministry of Natural Resources, which appeal was docketed as MNR CASE No. Subsequently, a writ of execution dated January 8, 1987 was issued in favor of the respondent DAVENCOR,
6540. During the pendency of the appealed case with the Minister of Natural Resources, Ma. Milagros which states:
Matuguina disposed of her shares in petitioner MIWPI, thereby ceasing to be a stockholder of the petitioner as
of March 16, 1986. 7 The City/Provincial Sheriff
Davao City
On October 1, 1986, The Minister of Natural Resources, Hon. Ernesto M. Maceda rendered his
Decision, 8affirming the aforesaid order of the Director of Foreign Development, stating thus: GREETINGS:

DECISION
Set V Corporation Code * Doctrine of Peircing Corpo Veil Cases*Page 123 of 164

You are hereby directed to enforce, implement and execute the Order of Execution dated 06 June 1987 12. That the plaintiff is willing and able to file the necessary bond executed to the defendants, in an
of this Office in the above-entitled case against Matuguina Logging Enterprises and/or Matuguina amount to be fixed by the court, to the effect that the plaintiff will pay to the defendants all damages
Integrated Wood Products, Inc. Its officers or any person or corporation in its behalf and conformably which they may sustain by reason of the injunction if the court should finally decide that the plaintiff
with the Order dated 15 July 1981 of the Director of Forest Development, stating dispositively. was not entitled thereto.

xxx xxx xxx MIWPI, likewise, alleges that in wantonly and imprudently procuring the Writ of Execution against it, which
DAVENCOR and Philip Co seek to enforce a 2.5 Million Peso liability of plaintiff, the latter has been constrained
You are hereby requested to submit your return to this Office within the period of sixty (60) days from to bring the present action, thereby incurring damages in the sum of P500,000.00 in concept of actual and
your receipt hereof as to action taken hereon.SO ORDERED. compensatory damages, and P250,000.00 in attorney's fees, which amount petitioner now seeks to recover.

On February 11, 1987, MIWPI filed the instant complaint 12 for prohibition, damages and injunction, with prayer The trial court issued a temporary restraining order the next day, February 12, 1987, restraining and/or
for restraining order, which case was docketed as Civil Case No. 18,457-87 in the Regional Trial Court Davao enjoining the private respondents and the Hon. Secretary of Natural Resources from enforcing, implementing
City, Branch 17. MIWPI stated its primary cause of action, the relevant portion of which reads, viz.: and/or carrying into effect, the decision of the respondent Secretary dated October 1, 1986, as well as the
order of execution dated January 6, 1987.

5. That plaintiff which has a distinct and separate personality of its own under the law, and was never
a party to the case between DAVENCOR and MLE, suddenly became a party to the case after the On February 17, 1987, private respondents filed a Motion to Dismiss 13 alleging that the trial court had no
decision became final and executory with the issuance of Annex "B" hereof for reasons known to the jurisdiction over the case under Presidential Decree No. 705, to which Motion to Dismiss, petitioner filed an
defendants alone: Opposition14 dated February 1987. On March 9, 1987, the trial court issued an order 15 denying private
respondent's Motion to Dismiss. Hence, private respondents filed their Answer 16 dated March 13, 1987 and an
Amended Answer 17 dated July 16, 1987.
6. That the issuance of Annex "B" hereof (the order of execution) by the defendant Minister has been
made not only without or in excess of his authority but that the same was issued patently without any
factual or legal basis, hence, a gross violation of plaintiff's constitutional rights under the due process In the latter pleading, private respondents raised the following special and affirmative defenses:
clause;
7. That neither Milagros Matuguina nor Matuguina Integrated Wood Products, Inc. advised defendant
7. That plaintiff, in the face of the order (Annex "B") complained of, there being no appeal or any plain, Davencor of the change of name, and transfer of management of PTL No. 30 from Milagros Matuguina
speedy, and adequate remedy in the ordinary course of law, does not have any alternative but to to Matuguina Integrated Wood Products, Inc., during the pendency of MNR Case No. 6540 before the
ventilate the present recourse; Bureau of Forest Development and the Ministry of Natural Resources, notwithstanding that the lawyer
of Matuguina Integrated Wood Products, Inc., who was also a stockholder thereof, had appeared for
Milagros Matuguina in said administrative case.
8. That defendant Minister is doing, threatens or is about to do, or is procuring or suffering to be done,
some act which definitely is in violation of the plaintiff's rights respecting the subject matter of the
action, and unless said act or acts are restrained or prohibited at least during the pendency of this 8. That plaintiff has acted in bad faith and is now in estoppel from questioning the Writ of Execution
case, said act or acts would probably work not only injustice to plaintiff but would tend to render the issued against Milagros Matuguina (now Matuguina Integrated Wood Products, Inc.) to satisfy the
judgment of this Honorable Court ineffectual; judgment in MNR Case No. 6540.

9. That the commission or continuance of the acts complained of during the present litigation would 9. This Honorable Court has no jurisdiction over the nature and subject matter of this action, especially
not only cause great and irreparable injury, but will also work injustice to the plaintiff, and would because:
complicate, aggravate and multiply the issues in this case;
(a) The plaintiff has not exhausted administrative remedies available to it before initiating
10. That the plaintiff is entitled to the relief demanded, and the whole or part of such relief consists in this action;
restraining the commission or continuance of the acts complained of, or in the performance of acts,
either for a limited period or perpetually; (b) In the guise of entertaining an action for damages, this Court is being misled by the
plaintiff into deciding questions properly for the Department of Natural Resources to decide
11. That great and irreparable injury would inevitably result to the plaintiff before the matter can be exclusively in the lawful exercise of its regulatory jurisdiction;
heard on notice, hence, immediate issuance of a restraining order is necessary and proper;
Set V Corporation Code * Doctrine of Peircing Corpo Veil Cases*Page 124 of 164

(c) The plaintiff is now precluded and estopped from filing this action. WHEREFORE, premises considered, the decision appealed from is reversed and set aside and the
Order of Execution issued by the Minister of Natural Resources dated January 6, 1987 is affirmed.
10. The plaintiff has no cause of action against the defendants and has not stated any in its complaint, Without pronouncement as to costs.SO ORDERED.
especially because:
In due time, petitioner filed a motion for reconsideration. 21 Private respondents filed their opposition 22 to the
(a) Having failed to exhaust administrative remedies, plaintiff is without a ripe cause of action same on April 2, 1991. In a Resolution 23 dated April 12, 1991, the motion was denied by the respondent Court.
that can be pleaded before this Honorable Court;
Not content with the court's pronouncement, petitioner is now before us on a Petition for Review
(b) In substance, there is no justiciable question raised under the facts and circumstances of on Certiorari, 24alleging that the respondent court acted with grave abuse of discretion in rendering the
this case. questioned decision and its companion resolution, denying the motion for reconsideration.

Meanwhile, on June 2, 1987, the trial court issued on order 18 granting the petitioner's prayer for the issuance of The reasons relied upon by the Petitioner in filing its petition are hereby restated:
a writ of preliminary injunction against the private respondents and the Secretary of Natural Resources,
ordering them to desist, refrain and prevent from enforcing respondent Secretary's Decision dated October 1, I
1986 as well as the writ of execution dated January 8, 1987.
PETITIONER WAS DENIED DUE PROCESS OF LAW WHEN IT WAS MADE LIABLE BY RESPONDENT SECRETARY OF
19
On May 10, 1989, the trial court rendered its Decision in favor of the petitioner, disposing of the action as NATURAL RESOURCES IN HIS ORDER OF EXECUTION DATED 06 JANUARY 1987 (EXHIBIT "B" OF ATTACHMENT
follows: "O") ISSUED IN MNR CASE NO. 6540 DESPITE THE FACT THAT PETITIONER WAS NEVER A PARTY NOR A
PARTICIPANT IN THE SAID CASE: IN FACT, PETITIONER NEVER HAD NOTICE OF THE PROCEEDINGS IN MNR CASE
WHEREFORE, in view of the foregoing, finding the evidence of plaintiff, Matuguina Integrated Wood NO. 6540.
Products, Inc. sufficient to sustain a preponderance of evidence, showing that the order of execution
dated January 6, 1987, issued by the Minister of Natural Resources, through Alexander C. Castro, II
Assistant Minister for Legal Affairs, included therein, plaintiff Matuguina Integrated Wood Products,
Inc., despite non-inclusion of plaintiff in the decision of the then Minister of Natural Resources, dated THE FAILURE TO AFFORD PETITIONER THE OPPORTUNITY TO BE HEARD IN THE ADMINISTRATIVE LEVEL (MNR
October 1, 1986, already final and executory before the issuance of the order and execution, said CASE NO. 6540) COULD NOT HAVE BEEN CURED BY THE INSTITUTION OF THE ACTION FOR PROHIBITION IN THE
order or execution is hereby declared null and void and without any legal effect. TRIAL COURT BECAUSE SAID COURT HAD NO JURISDICTION TO DETERMINE WHETHER PETITIONER WAS GUILTY
OF ENCROACHMENT ON PRIVATE RESPONDENT DAVENCOR'S TIMBER CONCESSION; FURTHERMORE, THE
As a consequence thereof, the writ of preliminary injunction issued by this court, dated June 2, 1987 is QUESTION ON WHETHER PETITIONER WAS GUILTY OF ENCROACHMENT WAS NEVER PUT IN ISSUE IN THE CASE
hereby made permanent. BEFORE THE TRIAL COURT.

Moreover, as a result of the filing of this case, defendant Philip Co and Davencor Corporation, are III
ordered to jointly and severally pay the amount of P100,000.00 as actual and compensatory damages,
along with another amount of P20,000.00 as attorney's fees and costs of this action, in favor of THE LIABILITY OF MILAGROS/MLE AS FOUND BY RESPONDENT SECRETARY IN ITS DECISION DATED 01 OCTOBER
plaintiff Matuguina Integrated Wood Products, Inc.SO ORDERED. 1986 (EXHIBIT "A" OF THE ATTACHMENT "0") CANNOT BE IMPUTED AGAINST PETITIONER SINCE THE LATTER IS
A CORPORATION HAVING A PERSONALITY SEPARATE AND DISTINCT FROM MILAGROS/MLE.
Private respondents appealed the trial court's decision on May 19, 1989. Their notice of appeal was approved
by the trial court. The appealed case was docketed with respondent Honorable Court of Appeals as CA-G.R. SP IV
No. 19887.

PETITIONER CANNOT BE MADE LIABLE TO PRIVATE RESPONDENTS UNDER THE DEED OF TRANSFER DATED 18
20
On February 25, 1991, the respondent Court rendered its Decision, reversing the lower court's JULY 1975 (EXHIBIT "3" OF ATTACHMENT "P") AND SECTION 61 OF THE REVISED FORESTRY CODE OF THE
pronouncement. The dispositive portion of the Decision reads: PHILIPPINES (P.D. 705, AS AMENDED):

A. THE ALLEGED TRANSFER OF PTL NO. 30 FROM MILAGROS/MLE TO PETITIONER NEVER BECAME BINDING AND
EFFECTIVE SINCE PTL NO. 30 REMAINED IN THE NAME OF MILAGROS/MLE UNTIL ITS EXPIRATION ON 30 JUNE
Set V Corporation Code * Doctrine of Peircing Corpo Veil Cases*Page 125 of 164

1977: THIS IS DUE TO THE FACT THAT SAID TRANSFER WAS NEVER APPROVED BY THE SECRETARY OF NATURAL The writ of execution must conform to the judgment which is to be executed, as it may not vary the terms of
RESOURCES. the judgment it seeks to enforce. 30 Nor may it go beyond the terms of the judgment sought to be executed.
Where the execution is not in harmony with the judgment which gives it life and exceeds it, it has pro tanto no
B. GRANTING ARGUENDO THAT THERE WAS AN EFFECTIVE TRANSFER OF PTL NO. 30 FROM validity. To maintain otherwise would be to ignore the constitutional provision against depriving a person of his
MILAGROS/MLE TO PETITIONER, THE TRANSFER COULD NOT MAKE PETITIONER LIABLE FOR THE property without due process of law. 31
ALLEGED ENCROACHMENT OF PRIVATE RESPONDENT DAVENCOR'S TIMBER CONCESSION, SINCE:
The writ of execution issued by the Secretary of Natural Resources on January 8, 1987 clearly varies the term of
1. SAID TRANSFER WAS EXECUTED PRIOR TO THE COMMISSION OF THE ALLEGED his Decision of October 1, 1986, inasmuch as the Writ includes the MIWPI as party liable whereas the Decision
ENCROACHMENT AND THE FILING THE ADMINISTRATIVE COMPLAINT FOR ENCROACHMENT only mentions Milagros Matuguina/MLE.
DATED 28 JULY 1975; THUS, PETITIONER CANNOT BE MADE LIABLE FOR OBLIGATIONS OF
MILAGROS/MLE WHICH WERE INCURRED AFTER THE DATE OF THE SAID TRANSFER. There is no basis for the issuance of the Order of Execution against the petitioner. The same was issued without
giving the petitioner an opportunity to defend itself and oppose the request of DAVENCOR for the issuance of a
2. SAID TRANSFER COVERED ONLY FORESTRY CHARGES AND OTHER GOVERNMENT FEES, writ of execution against it. In fact, it does not appear that petitioner was at all furnished with a copy of
AND DID NOT INCLUDE THE PERSONAL LIABILITY OF MILAGROS/MLE THAT AROSE FROM THE DAVENCOR's letter requesting for the Execution of the Honorable Secretary's decision against it. Petitioner was
ENCROACHMENT OF THE TIMBER CONCESSION OF RESPONDENT DAVENCOR. 25 suddenly made liable upon the order of execution by the respondent Secretary's expedient conclusions that
MLE and MIWPI are one and the same, apparently on the basis merely of DAVENCOR's letter requesting for the
Order, and without hearing or impleading MIWPI. Until the issuance of the Order of execution, petitioner was
Private Respondents DAVENCOR and the public respondent Hon. Minister (now Secretary) of Natural Resources
not included or mentioned in the proceedings as having any participation in the encroachment in DAVENCOR's
filed separate Comments 26 on September 5, 1991 and June 8, 1992 respectively.
timber concession. This action of the respondent Secretary disregards the most basis tenets of due process and
elementary fairness.
The essential issues of the present controversy boil down to the following:
The liberal atmosphere which pervades the procedure in administrative proceedings does not empower the
Was the Petitioner denied due process when it was adjudged liable with MLE for encroaching upon the timber presiding officer to make conclusions of fact before hearing all the parties concerned. 32 In Police Commission
concession of DAVENCOR in the respondent Minister's Order of Execution? vs.Hon. Judge Lood, 33 we held that the formalities usually attendant in court hearings need not be present in
an administrative investigation, provided that the parties are heard given the opportunity to adduce their
Is the petitioner a transferee of MLE's interest, as to make it liable for the latter's illegal logging operations in evidence. The right to notice and hearing is essential to due process and its non-observance will, as a rule,
DAVENCOR's timber concession, or more specially, is it possible to pierce the veil of MIWPI's corporate invalidate the administrative proceedings.
existence, making it a mere conduit or successor of MLE?
As observed by the appellate court, to writ:
Generally accepted is the principle that no man shall be affected by any proceeding to which he is a stranger,
and strangers to a case not bound by judgment rendered by the court. In the same manner an execution can the appellant should have filed a Motion with the Minister with Notice to the appellee to include the
be issued only against a party and not against one who did not have his day in court. In Lorenzo vs. Cayetano, latter as party liable for the judgment in order to afford the appellee an opportunity to be heard on its
78 SCRA 485 [1987], this Court held that only real parties in interest in an action are bound by judgment liability for the judgment rendered against Ma. Milagros Matuguina doing business under the name
therein and by writs of execution and demolition issued pursuant thereto. 27 Matuguina Logging Enterprises. 34

Indeed a judgment cannot bind persons who are not parties to the Continuing, the said court stated further that:
action. 28 It is elementary that strangers to a case are not bound by the judgment rendered by the court and
such judgment is not available as an adjudication either against or in favor of such other person. A decision of a
Nevertheless, the failure to comply with the procedure in order to satisfy the requirements of due
court will not operate to divest the rights of a person who has not and has never been a party to a litigation,
process was cured by the present action for prohibition where the liability of appellee has been
either as plaintiff or as defendant. Execution of a judgment can only be issued against one who is a party to the
ventilated.
action, and not against one who, not being a party in the action has not yet had his day in court. 29

We do not agree. Essential, Prohibition is a remedy to prevent inferior courts, corporations, boards or persons
from usurping or exercising a jurisdiction or power with which they have not been vested by law 35 As we have
held inMafinco Trading Corporation vs. Ople, et al, 36 in a certiorari or prohibition case, only issues affecting the
jurisdiction of the tribunal, board and offices involved may be resolved on the basis of undisputed facts.
Set V Corporation Code * Doctrine of Peircing Corpo Veil Cases*Page 126 of 164

The issue of whether or not petitioner is an alter ego of Milagros Matuguina/MLE, is one of fact, and which Defendant's arguments on this peripheral aspect of corporate existence, do not at all indicate that
should have been threshed out in the administrative proceedings, and not in the prohibition proceedings in the such a legal fiction, was granted.
trial court, where it is precisely the failure of the respondent Minister of Natural Resources to proceed as
mandated by law in the execution of its order which is under scrutiny. In the first place, the alleged control of plaintiff corporation was not evident in any particular corporate
acts of plaintiff corporation, wherein Maria Milagros Matuguina Logging Enterprises using plaintiff
Assuming, arguendo, that prohibition is the proper remedy for determining the propriety of piercing the corporation, executed acts or powers directly involving plaintiff corporation.
separate personality of petitioner with its stockholders, the evidence presented at said trial does not warrant
such action. Neither was there any evidence of defendants, that Maria Milagros Matuguina Logging Enterprises,
using the facilities and resources of plaintiff corporation, involved itself in transaction using both single
It is settled that a corporation is clothed with personality separate and distinct from that of the persons proprietorship and plaintiff corporation in such particular line of business undertakings.
composing it. It may not generally be held liable for that of the persons composing it. It may not be held liable
for the personal indebtedness of its stockholders or those of the entities connected with it. Conversely, a As stated by this court in resolving plaintiff's prayer for issuance of a writ or preliminary injunction,
stockholder cannot be made to answer for any of its financial obligations even if he should be its said:
president. 37 But when the juridical personality of the corporation is used to defeat public convenience, justify
wrong, protect fraud or defend crime, the corporation shall be considered as a mere association of persons
There is actually, no evidence presented by defendant, showing that sometime on March 15,
(Koppel, Inc. vs. Yatco, 77 Phil 496, Palay, Inc. vs. Clave, G.R. No. 56076, September 21, 1983, 124 SCRA 638),
1986, to January 1987, during which period, the subject decision of Hon. Secretary of Natural
and its responsible officers and/or stockholders shall be individually liable (Namarco vs. Associated Finance Co.,
Resources and corresponding writ of execution, Maria Milagros Matuguina was a stockholder
Inc., G.R. No. L-20886, April 27, 1967, 19 SCRA 962). For the same reasons, a corporation shall be liable for the
of plaintiff corporation in such amount or was she an officer of plaintiff corporation in
obligations of a stockholder (Palacio vs. Fely Transportation Co., G.R No. L-15121, August 31, 1963, 5 SCRA
whatever capacity.
1011), or a corporation and its successor-in-interest shall be considered as one and the liability of the former
shall attach to the latter. 38
The above circumstances is relevant and significant to assume any such justification of including
plaintiff corporation in the subject writ of execution, otherwise, as maintained by defendants, what
But for the separate juridical personality of a corporation to be disregarded, the wrongdoing must be clearly
matters most was the control of Milagros Matuguina Logging Enterprises of plaintiff corporation in
and convincingly established. It cannot be presumed. 39
1974 and 1975, when the administrative case was pending, this circumstance alone without formally
including plaintiff corporation in said case, will not create any valid and sufficient justification for
In the case at bar, there is, insufficient basis for the appellate court's ruling that MIWPI is the same as plaintiff corporation, to have been supposedly included in the suit against defendants and Maria
Matuguina. The trial court's observation is enlightening. Milagros Matuguina Logging Enterprises, in the administrative case.

Despite apparently opposing evidence of both parties, the Court gathered and finds, that defendant's Yet, granting as claimed by defendants, that in 1974 or in 1975, Maria Milagros Matuguina became the
attempt to pierce the veil of corporate personality of plaintiff corporation, as to consider plaintiff controlling stockholder of plaintiff corporation, on account of the change of name and transfer of
corporations merely an adjunct or alter ego of Maria Milagros Matuguina Logging Enterprises, to justify management of PTL No. 30, this circumstance, we repeat, does not of itself prove that plaintiff
defendant's claim against plaintiff corporation, suffers heavily from insufficiency of evidence. corporation was the alter ego of Maria Milagros Matuguina Logging Enterprises, as enunciated in
various decisions of this Court, to writ:
It is the vehement contention of defendants, to bolster its claim, that plaintiff corporation is the alter
ego of Maria Milagros Matuguina Logging Enterprises, because when Milagros Matuguina became the It is important to bear in mind that mere ownership by a single stockholder or by
Chairman of the Board of Directors of plaintiff corporation, she requested for the change of name and another corporation of all or nearly all of the capital stocks of the corporation, is not
transfer of management of PTL No. 30, from her single proprietorship, to plaintiff corporation. itself a sufficient warrant for disregarding the fiction of separate personality (Liddel
and Co. vs. Collector of Internal Revenue, G.R. No. 9687, June 30, 1961).
Secondly, when Milagros Matuguina executed the deed of transfer, transferring her forest concession
under PTL No. 30, together with all the structures and improvements therein, to plaintiff corporation, It is recognized as lawful to obtain a corporation charter, even with a single substantial
for a consideration of P14,800.00 representing 148,000 shares of stocks of plaintiff corporation stockholder, to engage in specific activity and such activity may co-exist with other private
actually all existing shares of stocks of Milagros Matuguina, in plaintiff corporation represents 77.4% activities of the stockholder.
therein; suffice to say that plaintiff corporation practically became an alter ego of Milagros Matuguina.

If the corporation is substantial one, conducted lawfully; without fraud on another, its
separate identity is to be respected. 40
Set V Corporation Code * Doctrine of Peircing Corpo Veil Cases*Page 127 of 164

In this jurisdiction, it is a settled rule that conclusions and findings of fact by the trial court are entitled to great Even if it is mandated in the abovestated provision that "the transferee shall assume all the obligations of the
weight on appeal and should not be disturbed unless for strong and cogent reasons because the trial court is in transferor" this does not mean that all obligations are assumed, indiscriminately.
a better position to examine real evidence, as well as to observe the demeanor of the witnesses while testifying
in the case. 41 Invariably, it is not the letter, but the spirit of the law and intent of the legislature that is important. When the
interpretation of a statute according to the exact and literal import of its words would lead to absurdity, it
It is likewise improper to state that the MIWPI is the privy or the successor-in-interest of MLE, as the liability for should be construed according to the spirit and reason, disregarding if necessary the letter of the law. 45
the encroachment over DAVENCOR's timber concession is concerned, by reason of the transfer of interest in
PTL No. 30 from MLE to MIWPI. In construing statutes, the terms used therein are generally to be given their ordinary meaning, that is, such
meaning which is ascribed to them when they are commonly used, to the end that absurdity in the law must be
First of all, it does not appear indubitable that the said transfer ever became effective, since PTL No. 30 avoided. 46 The term "obligations" as used in the final clause of the second paragraph of Section 61 of P.D. 705
remained in the name of Milagros Matuguina/MLE until it expired on June 30, 1977. 42 is construed to mean those obligations incurred by the transferor in the ordinary course of business. It cannot
be construed to mean those obligations or liabilities incurred by the transferor as a result of transgressions of
More importantly, even if it is deemed that there was a valid change of name and transfer of interest in the PTL the law, as these are personal obligations of the transferor, and could not have been included in the term
No. 30, this only signifies a transfer of authority, from MLE to MIWPI, to conduct logging operations in the area "obligations" absent any modifying provision to that effect.
covered by PTL No. 30. It does not show indubitable proof that MIWPI was a mere conduit or successor of
Milagros Matuguina/MLE, as far the latter's liability for the encroachment upon DAVENCOR's concession is In the September 16, 1975 letters of Acting Director of the Bureau of Forest Development of Milagros
concerned. This is the only conclusion which we can discern from the language of Section 61 of P.D. 750, 43 and Matuguina and MIWPI informing them of the approval of Matuguina's request for the change of name and
the letters of the Acting Minister of Natural Resources to Milagros Matuguina/MLE and to MIWPI, on September transfer of management of PTL No. 30, the following statements were made by the Acting Director:
16, 1975. 44 In Soriano vs. Court of Appeals, this Court stated in clear language, that
In view hereof, (Matuguina Integrated Wood Products, Inc.) shall assume the responsibility of paying
It is the general rule that the protective mantle of a corporation's separate and distinct personality whatever pending liabilities and/or accounts remaining unsettled, if any, by the former licensee,
could only be pierced and liability attached directly to its officers and/or members stockholders, Milagros Matuguina, with the government. (Emphasis ours) 47
when the same is used for fraudulent, unfair, or illegal purpose. In the case at bar, there is no showing
that the Association entered into the transaction with the private respondent for the purpose of Accordingly, the letter's language implies that the obligations which MIWPI are to assume as transferee of
defrauding the latter of his goods or the payment thereof. . . . Therefore, the general rule on corporate Milagros Matuguina/MLE are those obligations in favor of the government only, and not to any other entity.
liability, not the exception, should be applied in resolving this case. (G.R. No. 49834, June 22, 1989) Thus this would include Forestry Charges, Taxes, Fees, and similar accountabilities.

The respondents cite Section 61 of P.D. 705 to establish MIWPI's succession to the liability of Milagros In sum, the Court makes the following pronouncements:
Matuguina/MLE:

(a) The respondent Honorable Minister of Natural Resources gravely abused its discretion when it issued its
Sec. 61. Transfers. Unless authorized by the Department Head, no licensee, lessee, or permittee Order of Execution on January 6, 1987, including therein as one of the parties liable the petitioner Matuguina
may transfer, exchange, sell, or convey his license agreement, license, lease or permit, or any of his Integrated Wood Products, Inc., which was never a party to the assailed proceeding resulting in the issuance of
rights or interests therein, or any of his assets used in connection therewith. such Order and, without affording the same an opportunity to be heard before it was adjudged liable.

The licensee, lessee, or permittee shall be allowed to transfer or convey his license agreement, (b) The petitioner is a corporate entity separate and distinct from Milagros Matuguina/Matuguina Logging
license, lease, or permit only if he has not violated any forestry law, rule or regulation; has been Enterprises, there being no clear basis for considering it as a mere conduit or alter ego of Matuguina/MLE, and
faithfully complying with the terms and conditions of the license agreement, license, lease or permit; therefore, cannot be made liable for the obligations of the same for encroachment over the timber concession
the transferee has all the qualifications and none of the disqualifications to hold a license agreement, of private respondent DAVENCOR.
license, lease or permit; there is no evidence that such transfer or conveyance is being made for
purposes of speculation; and the transferee shall assume all the obligations of the transferor.
IN VIEW OF THE FOREGOING, the Petition is hereby GRANTED, and the Decision dated February 25, 1991, is SET
ASIDE. The decision of the Regional Trial Court is hereby REINSTATED, and correspondingly, Order of Execution
The transferor shall forever be barred from acquiring another license agreement, license, lease or of the respondent Secretary of Natural Resources is declared NULL and VOID and without effect.No
permit. pronouncement as to costs.SO ORDERED.
Set V Corporation Code * Doctrine of Peircing Corpo Veil Cases*Page 128 of 164

FIRST DIVISION G.R. No. 120077 October 13, 2000 During his employment with the Mazoon Printing Press in the Sultanate of Oman, respondent Santos received a
letter dated May 2, 1988 from Mr. Gerhard R. Shmidt, General Manager, Palace Hotel, Beijing, China. Mr.
THE MANILA HOTEL CORP. AND MANILA HOTEL INTL. LTD., petitioners, vs. Schmidt informed respondent Santos that he was recommended by one Nestor Buenio, a friend of his.
NATIONAL LABOR RELATIONS COMMISSION, ARBITER CEFERINA J. DIOSANA AND MARCELO G.
SANTOS, respondents. Mr. Shmidt offered respondent Santos the same position as printer, but with a higher monthly salary and
increased benefits. The position was slated to open on October 1, 1988. 11
PARDO, J.:
On May 8, 1988, respondent Santos wrote to Mr. Shmidt and signified his acceptance of the offer.
The case before the Court is a petition for certiorari1 to annul the following orders of the National Labor
Relations Commission (hereinafter referred to as "NLRC") for having been issued without or with excess On May 19, 1988, the Palace Hotel Manager, Mr. Hans J. Henk mailed a ready to sign employment contract to
jurisdiction and with grave abuse of discretion:2 respondent Santos. Mr. Henk advised respondent Santos that if the contract was acceptable, to return the same
to Mr. Henk in Manila, together with his passport and two additional pictures for his visa to China.
(1) Order of May 31, 1993.3 Reversing and setting aside its earlier resolution of August 28, 1992. 4 The
questioned order declared that the NLRC, not the Philippine Overseas Employment Administration On May 30, 1988, respondent Santos resigned from the Mazoon Printing Press, effective June 30, 1988, under
(hereinafter referred to as "POEA"), had jurisdiction over private respondent's complaint; the pretext that he was needed at home to help with the family's piggery and poultry business.

(2) Decision of December 15, 1994.5 Directing petitioners to jointly and severally pay private On June 4, 1988, respondent Santos wrote the Palace Hotel and acknowledged Mr. Henk's letter. Respondent
respondent twelve thousand and six hundred dollars (US$ 12,600.00) representing salaries for the Santos enclosed four (4) signed copies of the employment contract (dated June 4, 1988) and notified them that
unexpired portion of his contract; three thousand six hundred dollars (US$3,600.00) as extra four he was going to arrive in Manila during the first week of July 1988.
months salary for the two (2) year period of his contract, three thousand six hundred dollars
(US$3,600.00) as "14th month pay" or a total of nineteen thousand and eight hundred dollars The employment contract of June 4, 1988 stated that his employment would commence September 1, 1988 for
(US$19,800.00) or its peso equivalent and attorney's fees amounting to ten percent (10%) of the total a period of two years. 12 It provided for a monthly salary of nine hundred dollars (US$900.00) net of taxes,
award; and payable fourteen (14) times a year.13

(3) Order of March 30, 1995.6 Denying the motion for reconsideration of the petitioners. On June 30, 1988, respondent Santos was deemed resigned from the Mazoon Printing Press.

In May, 1988, private respondent Marcelo Santos (hereinafter referred to as "Santos") was an overseas worker On July 1, 1988, respondent Santos arrived in Manila.
employed as a printer at the Mazoon Printing Press, Sultanate of Oman. Subsequently, in June 1988, he was
directly hired by the Palace Hotel, Beijing, People's Republic of China and later terminated due to retrenchment.
On November 5, 1988, respondent Santos left for Beijing, China. He started to work at the Palace Hotel. 14

Petitioners are the Manila Hotel Corporation (hereinafter referred to as "MHC") and the Manila Hotel
Subsequently, respondent Santos signed an amended "employment agreement" with the Palace Hotel,
International Company, Limited (hereinafter referred to as "MHICL").
effective November 5, 1988. In the contract, Mr. Shmidt represented the Palace Hotel. The Vice President
(Operations and Development) of petitioner MHICL Miguel D. Cergueda signed the employment agreement
When the case was filed in 1990, MHC was still a government-owned and controlled corporation duly organized under the word "noted".
and existing under the laws of the Philippines.

From June 8 to 29, 1989, respondent Santos was in the Philippines on vacation leave. He returned to China and
MHICL is a corporation duly organized and existing under the laws of Hong Kong. 7 MHC is an "incorporator" of reassumed his post on July 17, 1989.
MHICL, owning 50% of its capital stock.8

On July 22, 1989, Mr. Shmidt's Executive Secretary, a certain Joanna suggested in a handwritten note that
By virtue of a "management agreement" 9 with the Palace Hotel (Wang Fu Company Limited), MHICL 10 trained respondent Santos be given one (1) month notice of his release from employment.
the personnel and staff of the Palace Hotel at Beijing, China.

Now the facts.


Set V Corporation Code * Doctrine of Peircing Corpo Veil Cases*Page 129 of 164

On August 10, 1989, the Palace Hotel informed respondent Santos by letter signed by Mr. Shmidt that his The Palace Hotel and Mr. Shmidt were not served with summons and neither participated in the proceedings
employment at the Palace Hotel print shop would be terminated due to business reverses brought about by the before the Labor Arbiter.18
political upheaval in China.15 We quote the letter:16
On June 27, 1991, Labor Arbiter Ceferina J. Diosana, decided the case against petitioners, thus: 19
"After the unfortunate happenings in China and especially Beijing (referring to Tiannamen Square
incidents), our business has been severely affected. To reduce expenses, we will not open/operate "WHEREFORE, judgment is hereby rendered:
printshop for the time being.

"1. directing all the respondents to pay complainant jointly and severally;
"We sincerely regret that a decision like this has to be made, but rest assured this does in no way
reflect your past performance which we found up to our expectations."
"a) $20,820 US dollars or its equivalent in Philippine currency as unearned salaries;

"Should a turnaround in the business happen, we will contact you directly and give you priority on
"b) P50,000.00 as moral damages;
future assignment."

"c) P40,000.00 as exemplary damages; and


On September 5, 1989, the Palace Hotel terminated the employment of respondent Santos and paid all benefits
due him, including his plane fare back to the Philippines.
"d) Ten (10) percent of the total award as attorney's fees."SO ORDERED."
On October 3, 1989, respondent Santos was repatriated to the Philippines.
On July 23, 1991, petitioners appealed to the NLRC, arguing that the POEA, not the NLRC had jurisdiction over
the case.
On October 24, 1989, respondent Santos, through his lawyer, Atty. Ednave wrote Mr. Shmidt, demanding full
compensation pursuant to the employment agreement.
On August 28, 1992, the NLRC promulgated a resolution, stating: 20
On November 11, 1989, Mr. Shmidt replied, to wit: 17
"WHEREFORE, let the appealed Decision be, as it is hereby, declared null and void for want of
jurisdiction. Complainant is hereby enjoined to file his complaint with the POEA."SO ORDERED."
His service with the Palace Hotel, Beijing was not abruptly terminated but we followed the one-month
notice clause and Mr. Santos received all benefits due him.
On September 18, 1992, respondent Santos moved for reconsideration of the afore-quoted resolution. He
argued that the case was not cognizable by the POEA as he was not an "overseas contract worker." 21
"For your information the Print Shop at the Palace Hotel is still not operational and with a low business
outlook, retrenchment in various departments of the hotel is going on which is a normal management
practice to control costs. On May 31, 1993, the NLRC granted the motion and reversed itself. The NLRC directed Labor Arbiter Emerson
Tumanon to hear the case on the question of whether private respondent was retrenched or dismissed. 22
"When going through the latest performance ratings, please also be advised that his performance was
below average and a Chinese National who is doing his job now shows a better approach. On January 13, 1994, Labor Arbiter Tumanon completed the proceedings based on the testimonial and
documentary evidence presented to and heard by him. 23
"In closing, when Mr. Santos received the letter of notice, he hardly showed up for work but still
enjoyed free accommodation/laundry/meals up to the day of his departure." Subsequently, Labor Arbiter Tumanon was re-assigned as trial Arbiter of the National Capital Region, Arbitration
Branch, and the case was transferred to Labor Arbiter Jose G. de Vera. 24
On February 20, 1990, respondent Santos filed a complaint for illegal dismissal with the Arbitration Branch,
National Capital Region, National Labor Relations Commission (NLRC). He prayed for an award of nineteen On November 25, 1994, Labor Arbiter de Vera submitted his report. 25 He found that respondent Santos was
thousand nine hundred and twenty three dollars (US$19,923.00) as actual damages, forty thousand pesos illegally dismissed from employment and recommended that he be paid actual damages equivalent to his
(P40,000.00) as exemplary damages and attorney's fees equivalent to 20% of the damages prayed for. The salaries for the unexpired portion of his contract.26
complaint named MHC, MHICL, the Palace Hotel and Mr. Shmidt as respondents.
On December 15, 1994, the NLRC ruled in favor of private respondent, to wit: 27
Set V Corporation Code * Doctrine of Peircing Corpo Veil Cases*Page 130 of 164

"WHEREFORE, finding that the report and recommendations of Arbiter de Vera are supported by The employment contract. Respondent Santos was hired directly by the Palace Hotel, a foreign employer,
substantial evidence, judgment is hereby rendered, directing the respondents to jointly and severally through correspondence sent to the Sultanate of Oman, where respondent Santos was then employed. He was
pay complainant the following computed contractual benefits: (1) US$12,600.00 as salaries for the hired without the intervention of the POEA or any authorized recruitment agency of the government. 36
unexpired portion of the parties' contract; (2) US$3,600.00 as extra four (4) months salary for the two
(2) years period (sic) of the parties' contract; (3) US$3,600.00 as "14th month pay" for the aforesaid Under the rule of forum non conveniens, a Philippine court or agency may assume jurisdiction over the case if it
two (2) years contract stipulated by the parties or a total of US$19,800.00 or its peso equivalent, plus chooses to do so provided: (1) that the Philippine court is one to which the parties may conveniently resort to;
(4) attorney's fees of 10% of complainant's total award."SO ORDERED." (2) that the Philippine court is in a position to make an intelligent decision as to the law and the facts; and (3)
that the Philippine court has or is likely to have power to enforce its decision. 37 The conditions are unavailing in
On February 2, 1995, petitioners filed a motion for reconsideration arguing that Labor Arbiter de Vera's the case at bar.
recommendation had no basis in law and in fact. 28
Not Convenient. We fail to see how the NLRC is a convenient forum given that all the incidents of the case
On March 30, 1995, the NLRC denied the motion for reconsideration. 29 from the time of recruitment, to employment to dismissal occurred outside the Philippines. The inconvenience
is compounded by the fact that the proper defendants, the Palace Hotel and MHICL are not nationals of the
Hence, this petition.30 Philippines. Neither .are they "doing business in the Philippines." Likewise, the main witnesses, Mr. Shmidt and
Mr. Henk are non-residents of the Philippines.

On October 9, 1995, petitioners filed with this Court an urgent motion for the issuance of a temporary
restraining order and/or writ of preliminary injunction and a motion for the annulment of the entry of judgment No power to determine applicable law. Neither can an intelligent decision be made as to the law governing
of the NLRC dated July 31, 1995.31 the employment contract as such was perfected in foreign soil. This calls to fore the application of the principle
of lex loci contractus (the law of the place where the contract was made). 38

On November 20, 1995, the Court denied petitioner's urgent motion. The Court required respondents to file
their respective comments, without giving due course to the petition. 32 The employment contract was not perfected in the Philippines. Respondent Santos signified his acceptance by
writing a letter while he was in the Republic of Oman. This letter was sent to the Palace Hotel in the People's
Republic of China.
On March 8, 1996, the Solicitor General filed a manifestation stating that after going over the petition and its
annexes, they can not defend and sustain the position taken by the NLRC in its assailed decision and orders.
The Solicitor General prayed that he be excused from filing a comment on behalf of the NLRC 33 No power to determine the facts. Neither can the NLRC determine the facts surrounding the alleged illegal
dismissal as all acts complained of took place in Beijing, People's Republic of China. The NLRC was not in a
position to determine whether the Tiannamen Square incident truly adversely affected operations of the Palace
On April 30,1996, private respondent Santos filed his comment. 34
Hotel as to justify respondent Santos' retrenchment.

On June 26, 1996, the Court granted the manifestation of the Solicitor General and required the NLRC to file its
Principle of effectiveness, no power to execute decision. Even assuming that a proper decision could be
own comment to the petition.35
reached by the NLRC, such would not have any binding effect against the employer, the Palace Hotel. The
Palace Hotel is a corporation incorporated under the laws of China and was not even served with summons.
On January 7, 1997, the NLRC filed its comment. Jurisdiction over its person was not acquired.

The petition is meritorious. This is not to say that Philippine courts and agencies have no power to solve controversies involving foreign
employers. Neither are we saying that we do not have power over an employment contract executed in a
I. Forum Non-Conveniens foreign country. If Santos were an "overseas contract worker", a Philippine forum, specifically the POEA, not the
NLRC, would protect him.39 He is not an "overseas contract worker" a fact which he admits with conviction. 40

The NLRC was a seriously inconvenient forum.


Even assuming that the NLRC was the proper forum, even on the merits, the NLRC's decision cannot be
sustained.
We note that the main aspects of the case transpired in two foreign jurisdictions and the case involves purely
foreign elements. The only link that the Philippines has with the case is that respondent Santos is a Filipino
citizen. The Palace Hotel and MHICL are foreign corporations. Not all cases involving our citizens can be tried II. MHC Not Liable
here.
Set V Corporation Code * Doctrine of Peircing Corpo Veil Cases*Page 131 of 164

Even if we assume two things: (1) that the NLRC had jurisdiction over the case, and (2) that MHICL was liable Second, and more importantly, there was no existing employer-employee relationship between Santos and
for Santos' retrenchment, still MHC, as a separate and distinct juridical entity cannot be held liable. MHICL. In determining the existence of an employer-employee relationship, the following elements are
considered:51
True, MHC is an incorporator of MHICL and owns fifty percent (50%) of its capital stock. However, this is not
enough to pierce the veil of corporate fiction between MHICL and MHC. "(1) the selection and engagement of the employee;

Piercing the veil of corporate entity is an equitable remedy. It is resorted to when the corporate fiction is used "(2) the payment of wages;
to defeat public convenience, justify wrong, protect fraud or defend a crime. 41 It is done only when a
corporation is a mere alter ego or business conduit of a person or another corporation. "(3) the power to dismiss; and

In Traders Royal Bank v. Court of Appeals,42 we held that "the mere ownership by a single stockholder or by "(4) the power to control employee's conduct."
another corporation of all or nearly all of the capital stock of a corporation is not of itself a sufficient reason for
disregarding the fiction of separate corporate personalities."
MHICL did not have and did not exercise any of the aforementioned powers. It did not select respondent Santos
as an employee for the Palace Hotel. He was referred to the Palace Hotel by his friend, Nestor Buenio. MHICL
The tests in determining whether the corporate veil may be pierced are: First, the defendant must have control did not engage respondent Santos to work. The terms of employment were negotiated and finalized through
or complete domination of the other corporation's finances, policy and business practices with regard to the correspondence between respondent Santos, Mr. Schmidt and Mr. Henk, who were officers and representatives
transaction attacked. There must be proof that the other corporation had no separate mind, will or existence of the Palace Hotel and not MHICL. Neither did respondent Santos adduce any proof that MHICL had the power
with respect the act complained of. Second, control must be used by the defendant to commit fraud or to control his conduct. Finally, it was the Palace Hotel, through Mr. Schmidt and not MHICL that terminated
wrong. Third, the aforesaid control or breach of duty must be the proximate cause of the injury or loss respondent Santos' services.
complained of. The absence of any of the elements prevents the piercing of the corporate veil. 43

Neither is there evidence to suggest that MHICL was a "labor-only contractor." 52 There is no proof that MHICL
It is basic that a corporation has a personality separate and distinct from those composing it as well as from "supplied" respondent Santos or even referred him for employment to the Palace Hotel.
that of any other legal entity to which it may be related. 44 Clear and convincing evidence is needed to pierce
the veil of corporate fiction.45 In this case, we find no evidence to show that MHICL and MHC are one and the
Likewise, there is no evidence to show that the Palace Hotel and MHICL are one and the same entity. The fact
same entity.
that the Palace Hotel is a member of the "Manila Hotel Group" is not enough to pierce the corporate veil
between MHICL and the Palace Hotel.
III. MHICL not Liable

IV. Grave Abuse of Discretion


Respondent Santos predicates MHICL's liability on the fact that MHICL "signed" his employment contract with
the Palace Hotel. This fact fails to persuade us.
Considering that the NLRC was forum non-conveniens and considering further that no employer-employee
relationship existed between MHICL, MHC and respondent Santos, Labor Arbiter Ceferina J. Diosana clearly had
First, we note that the Vice President (Operations and Development) of MHICL, Miguel D. Cergueda signed the no jurisdiction over respondent's claim in NLRC NCR Case No. 00-02-01058-90.
employment contract as a mere witness. He merely signed under the word "noted".

Labor Arbiters have exclusive and original jurisdiction only over the following: 53
When one "notes" a contract, one is not expressing his agreement or approval, as a party would. 46 In Sichangco
v. Board of Commissioners of Immigration,47 the Court recognized that the term "noted" means that the person
"1. Unfair labor practice cases;
so noting has merely taken cognizance of the existence of an act or declaration, without exercising a judicious
deliberation or rendering a decision on the matter.
"2. Termination disputes;
Mr. Cergueda merely signed the "witnessing part" of the document. The "witnessing part" of the document is
that which, "in a deed or other formal instrument is that part which comes after the recitals, or where there are "3. If accompanied with a claim for reinstatement, those cases that workers may file involving wages,
no recitals, after the parties (emphasis ours)."48 As opposed to a party to a contract, a witness is simply one rates of pay, hours of work and other terms and conditions of employment;
who, "being present, personally sees or perceives a thing; a beholder, a spectator, or eyewitness." 49 One who
"notes" something just makes a "brief written statement" 50 a memorandum or observation.
Set V Corporation Code * Doctrine of Peircing Corpo Veil Cases*Page 132 of 164

"4. Claims for actual, moral, exemplary and other forms of damages arising from employer-employee IN VIEW OF THE FOREGOING, we find no legal basis to support the assessment in question against
relations; petitioner. If at all, the assessment should have been directed against JACKBILT, the manufacturer.
Accordingly, the decision appealed from is reversed, and the surety bond filed to guarantee payment
"5. Cases arising from any violation of Article 264 of this Code, including questions involving legality of of said assessment is ordered cancelled. No pronouncement as to costs.
strikes and lockouts; and
Norton and Harrison is a corporation organized in 1911, (1) to buy and sell at wholesale and retail, all kinds of
"6. Except claims for Employees Compensation, Social Security, Medicare and maternity benefits, all goods, wares, and merchandise; (2) to act as agents of manufacturers in the United States and foreign
other claims, arising from employer-employee relations, including those of persons in domestic or countries; and (3) to carry on and conduct a general wholesale and retail mercantile establishment in the
household service, involving an amount exceeding five thousand pesos (P5,000.00) regardless of Philippines. Jackbilt is, likewise, a corporation organized on February 16, 1948 primarily for the purpose of
whether accompanied with a claim for reinstatement." making, producing and manufacturing concrete blocks. Under date of July 27, 1948. Norton and Jackbilt entered
into an agreement whereby Norton was made the sole and exclusive distributor of concrete blocks
manufactured by Jackbilt. Pursuant to this agreement, whenever an order for concrete blocks was received by
In all these cases, an employer-employee relationship is an indispensable jurisdictional requirement.
the Norton & Harrison Co. from a customer, the order was transmitted to Jackbilt which delivered the
merchandise direct to the customer. Payment for the goods is, however, made to Norton, which in turn pays
The jurisdiction of labor arbiters and the NLRC under Article 217 of the Labor Code is limited to disputes arising Jackbilt the amount charged the customer less a certain amount, as its compensation or profit. To exemplify the
from an employer-employee relationship which can be resolved by reference to the Labor Code, or other labor sales procedures adopted by the Norton and Jackbilt, the following may be cited. In the case of the sale of 420
statutes, or their collective bargaining agreements.54 pieces of concrete blocks to the American Builders on April 1, 1952, the purchaser paid to Norton the sum of
P189.00 the purchase price. Out of this amount Norton paid Jackbilt P168.00, the difference obviously being its
"To determine which body has jurisdiction over the present controversy, we rely on the sound judicial principle compensation. As per records of Jackbilt, the transaction was considered a sale to Norton. It was under this
that jurisdiction over the subject matter is conferred by law and is determined by the allegations of the procedure that the sale of concrete blocks manufactured by Jackbilt was conducted until May 1, 1953, when the
complaint irrespective of whether the plaintiff is entitled to all or some of the claims asserted therein." 55 agency agreement was terminated and a management agreement between the parties was entered into. The
management agreement provided that Norton would sell concrete blocks for Jackbilt, for a fixed monthly fee of
P2,000.00, which was later increased to P5,000.00.
The lack of jurisdiction of the Labor Arbiter was obvious from the allegations of the complaint. His failure to
dismiss the case amounts to grave abuse of discretion. 56
During the existence of the distribution or agency agreement, or on June 10, 1949, Norton & Harrison acquired
by purchase all the outstanding shares of stock of Jackbilt. Apparently, due to this transaction, the
V. The Fallo
Commissioner of Internal Revenue, after conducting an investigation, assessed the respondent Norton &
Harrison for deficiency sales tax and surcharges in the amount of P32,662.90, making as basis thereof the sales
WHEREFORE, the Court hereby GRANTS the petition for certiorari and ANNULS the orders and resolutions of the of Norton to the Public. In other words, the Commissioner considered the sale of Norton to the public as
National Labor Relations Commission dated May 31, 1993, December 15, 1994 and March 30, 1995 in NLRC the original sale and not the transaction from Jackbilt. The period covered by the assessment was from July 1,
NCR CA No. 002101-91 (NLRC NCR Case No. 00-02-01058-90).No costs.SO ORDERED. 1949 to May 31, 1953. As Norton and Harrison did not conform with the assessment, the matter was brought to
the Court of Tax Appeals.
EN BANC
The Commissioner of Internal Revenue contends that since Jackbilt was owned and controlled by Norton &
G.R. No. L-17618 August 31, 1964 Harrison, the corporate personality of the former (Jackbilt) should be disregarded for sales tax purposes, and
the sale of Jackbilt blocks by petitioner to the public must be considered as the original sales from which the
sales tax should be computed. The Norton & Harrison Company contended otherwise that is, the transaction
COMMISSIONER OF INTERNAL REVENUE, petitioner,
subject to tax is the sale from Jackbilt to Norton.
vs.
NORTON and HARRISON COMPANY, respondent.
Wherefore, the parties respectfully pray that the foregoing stipulation of facts be admitted and approved by
this Honorable Court, without prejudice to the parties adducing other evidence to prove their case not covered
PAREDES, J.:
by this stipulation of facts. 1wph1.t

This is an appeal interposed by the Commissioner of Internal Revenue against the following judgment of the
The majority of the Tax Court, in relieving Norton & Harrison of liability under the assessment, made the
Court of Tax Appeals:
following observations:
Set V Corporation Code * Doctrine of Peircing Corpo Veil Cases*Page 133 of 164

The law applicable to the case is Section 186 of the National Internal Revenue Code which imposes a identities of the two companies should be disregarded. Among these circumstances, which we find not
percentage tax of 7% on every original sale of goods, wares or merchandise, such tax to be based on successfully refuted by appellee Norton are: (a) Norton and Harrison owned all the outstanding stocks of
the gross selling price of such goods, wares or merchandise. The term "original sale" has been defined Jackbilt; of the 15,000 authorized shares of Jackbilt on March 31, 1958, 14,993 shares belonged to Norton and
as the first sale by every manufacturer, producer or importer. (Sec. 5, Com. Act No. 503.) Subsequent Harrison and one each to seven others; (b) Norton constituted Jackbilt's board of directors in such a way as to
sales by persons other than the manufacturer, producer or importer are not subject to the sales tax. enable it to actually direct and manage the other's affairs by making the same officers of the board for both
companies. For instance, James E. Norton is the President, Treasurer, Director and Stockholder of Norton. He
If JACKBILT actually sold concrete blocks manufactured by it to petitioner under the distributorship or also occupies the same positions in Jackbilt corporation, the only change being, in the Jackbilt, he is merely a
agency agreement of July 27, 1948, such sales constituted the original sales which are taxable under nominal stockholder. The same is true with Mr. Jordan, F. M. Domingo, Mr. Mantaring, Gilbert Golden and
Section 186 of the Revenue Code, while the sales made to the public by petitioner are subsequent Gerardo Garcia, while they are merely employees of the North they are Directors and nominal stockholders of
sales which are not taxable. But it appears to us that there was no such sale by JACKBILT to petitioner. the Jackbilt (c) Norton financed the operations of the Jackbilt, and this is shown by the fact that the loans
Petitioner merely acted as agent for JACKBILT in the marketing of its products. This is shown by the obtained from the RFC and Bank of America were used in the expansion program of Jackbilt, to pay advances
fact that petitioner merely accepted orders from the public for the purchase of JACKBILT blocks. The for the purchase of equipment, materials rations and salaries of employees of Jackbilt and other sundry
purchase orders were transmitted to JACKBILT which delivered the blocks to the purchaser directly. expenses. There was no limit to the advances given to Jackbilt so much so that as of May 31, 1956, the unpaid
There was no instance in which the blocks ordered by the purchasers were delivered to the petitioner. advances amounted to P757,652.45, which were not paid in cash by Jackbilt, but was offset by shares of stock
Petitioner never purchased concrete blocks from JACKBILT so that it never acquired ownership of such issued to Norton, the absolute and sole owner of Jackbilt; (d) Norton treats Jackbilt employees as its own.
concrete blocks. This being so, petitioner could not have sold JACKBILT blocks for its own account. It Evidence shows that Norton paid the salaries of Jackbilt employees and gave the same privileges as Norton
did so merely as agent of JACKBILT. The distributorship agreement of July 27, 1948, is denominated by employees, an indication that Jackbilt employees were also Norton's employees. Furthermore service rendered
the parties themselves as an "agency for marketing" JACKBILT products. ... . in any one of the two companies were taken into account for purposes of promotion; (e) Compensation given to
board members of Jackbilt, indicate that Jackbilt is merely a department of Norton. The income tax return of
Norton for 1954 shows that as President and Treasurer of Norton and Jackbilt, he received from Norton
xxx xxx xxx
P56,929.95, but received from Jackbilt the measly amount of P150.00, a circumstance which points out that
remuneration of purported officials of Jackbilt are deemed included in the salaries they received from Norton.
Therefore, the taxable selling price of JACKBILT blocks under the aforesaid agreement is the price The same is true in the case of Eduardo Garcia, an employee of Norton but a member of the Board of Jackbilt.
charged to the public and not the amount billed by JACKBILT to petitioner. The deficiency sales tax His Income tax return for 1956 reveals that he received from Norton in salaries and bonuses P4,220.00, but
should have been assessed against JACKBILT and not against petitioner which merely acted as the received from Jackbilt, by way of entertainment, representation, travelling and transportation allowances
former's agent. P3,000.00. However, in the withholding statement (Exh. 28-A), it was shown that the total of P4,200.00 and
P3,000.00 (P7,220.00) was received by Garcia from Norton, thus portraying the oneness of the two companies.
xxx xxx xxx The Income Tax Returns of Albert Golden and Dioscoro Ramos both employees of Norton but board members of
Jackbilt, also disclose the game method of payment of compensation and allowances. The offices of Norton and
Jackbilt are located in the same compound. Payments were effected by Norton of accounts for Jackbilt and vice
Presiding Judge Nable of the same Court expressed a partial dissent, stating:
versa. Payments were also made to Norton of accounts due or payable to Jackbilt and vice versa.

Upon the aforestated circumstances, which disclose Norton's control over and direction of Jackbilt's
Norton and Harrison, while not denying the presence of the set up stated above, tried to explain that the
affairs, the corporate personality of Jackbilt should be disregarded, and the transactions between
control over the affairs of Jackbilt was not made in order to evade payment of taxes; that the loans obtained by
these two corporations relative to the concrete blocks should be ignored in determining the
it which were given to Jackbilt, were necessary for the expansion of its business in the manufacture of concrete
percentage tax for which Norton is liable. Consequently, the percentage tax should be computed on
blocks, which would ultimately benefit both corporations; that the transactions and practices just mentioned,
the basis of the sales of Jackbilt blocks to the public.
are not unusual and extraordinary, but pursued in the regular course of business and trade; that there could be
no confusion in the present set up of the two corporations, because they have separate Boards, their cash
The majority opinion is now before Us on appeal by the Commissioner of Internal Revenue, on four (4) assigned assets are entirely and strictly separate; cashiers and official receipts and bank accounts are distinct and
errors, all of which pose the following propositions: (1) whether the acquisition of all the stocks of the Jackbilt different; they have separate income tax returns, separate balance sheets and profit and loss statements.
by the Norton & Harrison Co., merged the two corporations into a single corporation; (2) whether the basis of These explanations notwithstanding an over-all appraisal of the circumstances presented by the facts of the
the computation of the deficiency sales tax should be the sale of the blocks to the public and not to Norton. case, yields to the conclusion that the Jackbilt is merely an adjunct, business conduit or alter ego, of Norton and
Harrison and that the fiction of corporate entities, separate and distinct from each, should be disregarded. This
It has been settled that the ownership of all the stocks of a corporation by another corporation does not is a case where the doctrine of piercing the veil of corporate fiction, should be made to apply. In the case
necessarily breed an identity of corporate interest between the two companies and be considered as a of Liddell & Co. Inc. v. Coll. of Int. Rev., supra, it was held:
sufficient ground for disregarding the distinct personalities (Liddell & Co., Inc. v. Coll. of Int. Rev. L-9687, June
30, 1961). However, in the case at bar, we find sufficient grounds to support the theory that the separate
Set V Corporation Code * Doctrine of Peircing Corpo Veil Cases*Page 134 of 164

There are quite a series of conspicuous circumstances that militates against the separate and distinct basis and their returns are accurate, we would have the following result: Jackbilt declared a taxable net income
personality of Liddell Motors Inc., from Liddell & Co. We notice that the bulk of the business of Liddell of P161,202.31 in which the income tax due was computed at P37,137.00 (Exh. 8); whereas Norton declared as
& Co. was channel Red through Liddell Motors, Inc. On the other hand, Liddell Motors Inc. pursued no taxable, a net income of P120,101.59, on which the income tax due was computed at P25,628.00. The total of
activities except to secure cars, trucks, and spare parts from Liddell & Co., Inc. and then sell them to these liabilities is P50,764.84. On the other hand, if the net taxable earnings of both corporations are
the general public. These sales of vehicles by Liddell & Co, to Liddell Motors. Inc. for the most part combined, during the same taxable year, the tax due on their total which is P281,303.90 would be P70,764.00.
were shown to have taken place on the same day that Liddell Motors, Inc. sold such vehicles to the So that, even on the question of income tax alone, it would be to the advantages of Norton that the
public. We may even say that the cars and trucks merely touched the hands of Liddell Motors, Inc. as a corporations should be regarded as separate entities.
matter of formality.
WHEREFORE, the decision appealed from should be as it is hereby reversed and another entered making the
xxx xxx xxx appellee Norton & Harrison liable for the deficiency sales taxes assessed against it by the appellant
Commissioner of Internal Revenue, plus 25% surcharge thereon. Costs against appellee Norton & Harrison.
Accordingly, the mere fact that Liddell & Co. and Liddell Motors, Inc. are corporations owned and
controlled by Frank Liddell directly or indirectly is not by itself sufficient to justify the disregard of the FIRST DIVISION G.R. No. 129459 September 29, 1998
separate corporate identity of one from the other. There is however, in this instant case, a peculiar
sequence of the organization and activities of Liddell Motors, Inc. SAN JUAN STRUCTURAL AND STEEL FABRICATORS, INC., petitioner, vs.
COURT OF APPEALS, MOTORICH SALES CORPORATION, NENITA LEE GRUENBERG, ACL
As opined in the case of Gregory v. Helvering "the legal right of a tax payer to decrease the amount of DEVELOPMENT CORP. and JNM REALTY AND DEVELOPMENT CORP., respondents.
what otherwise would be his taxes, or altogether avoid them, by means which the law permits, cannot
be doubted". But as held in another case, "where a corporation is a dummy, is unreal or a sham and PANGANIBAN, J.:
serves no business purpose and is intended only as a blind, the corporate form may be ignored for the
law cannot countenance a form that is bald and a mischievous fictions".
May corporate treasurer, by herself and without any authorization from he board of directors, validly sell a
parcel of land owned by the corporation?. May the veil of corporate fiction be pierced on the mere ground that
... a taxpayer may gain advantage of doing business thru a corporation if he pleases, but the revenue almost all of the shares of stock of the corporation are owned by said treasurer and her husband?
officers in proper cases, may disregard the separate corporate entity where it serves but as a shield
for tax evasion and treat the person who actually may take benefits of the transactions as the person
The Case
accordingly taxable.

These questions are answered in the negative by this Court in resolving the Petition for Review
... to allow a taxpayer to deny tax liability on the ground that the sales were made through another
on Certioraribefore us, assailing the March 18, 1997 Decision 1 of the Court of Appeals 2 in CA GR CV No. 46801
and distinct corporation when it is proved that the latter is virtually owned by the former or that they
which, in turn, modified the July 18, 1994 Decision of the Regional Trial Court of Makati, Metro Manila, Branch
are practically one and the same is to sanction a circumvention of our tax laws. (and cases cited
63 3 in Civil Case No. 89-3511. The RTC dismissed both the Complaint and the Counterclaim filed by the parties.
therein.)
On the other hand, the Court of Appeals ruled:

In the case of Yutivo Sons Hardware Co. v. Court of Tax Appeals, L-13203, Jan. 28, 1961, this Court made a
WHEREFORE, premises considered, the appealed decision is AFFIRMED WITH MODIFICATION ordering
similar ruling where the circumstances of unity of corporate identities have been shown and which are identical
defendant-appellee Nenita Lee Gruenberg to REFUND or return to plaintiff-appellant the downpayment
to those obtaining in the case under consideration. Therein, this Court said:
of P100,000.00 which she received from plaintiff-appellant. There is no pronouncement as to costs. 4

We are, however, inclined to agree with the court below that SM was actually owned and controlled by 5
The petition also challenges the June 10, 1997 CA Resolution denying reconsideration.
petitioner as to make it a mere subsidiary or branch of the latter created for the purpose of selling the
vehicles at retail (here concrete blocks) ... .
The Facts
It may not be amiss to state in this connection, the advantages to Norton in maintaining a semblance of
separate entities. If the income of Norton should be considered separate from the income of Jackbilt, then each The facts as found by the Court of Appeals are as follows:
would declare such earning separately for income tax purposes and thus pay lesser income tax. The combined
taxable Norton-Jackbilt income would subject Norton to a higher tax. Based upon the 1954-1955 income tax
return of Norton and Jackbilt (Exhs. 7 & 8), and assuming that both of them are operating on the same fiscal
Set V Corporation Code * Doctrine of Peircing Corpo Veil Cases*Page 135 of 164

Plaintiff-appellant San Juan Structural and Steel Fabricators, Inc.'s amended complaint alleged that on 14 understanding between Mrs. Gruenberg and plaintiff-appellant that the Transfer of Rights/Deed of
February 1989, plaintiff-appellant entered into an agreement with defendant-appellee Motorich Sales Assignment will be signed only upon receipt of cash payment; thus they agreed that if the payment be
Corporation for the transfer to it of a parcel of land identified as Lot 30, Block 1 of the Acropolis Greens in check, they will meet at a bank designated by plaintiff-appellant where they will encash the check
Subdivision located in the District of Murphy, Quezon City. Metro Manila, containing an area of Four Hundred and sign the Transfer of Rights/Deed. However, plaintiff-appellant informed Mrs. Gruenberg of the
Fourteen (414) square meters, covered by TCT No. (362909) 2876: that as stipulated in the Agreement of 14 alleged availability of the check, by phone, only after banking hours.
February 1989, plaintiff-appellant paid the downpayment in the sum of One Hundred Thousand (P100,000.00)
Pesos, the balance to be paid on or before March 2, 1989; that on March 1, 1989. Mr. Andres T. Co, president of On the basis of the evidence, the court a quo rendered the judgment appealed from[,] dismissing
plaintiff-appellant corporation, wrote a letter to defendant-appellee Motorich Sales Corporation requesting for a plaintiff-appellant's complaint, ruling that:
computation of the balance to be paid: that said letter was coursed through defendant-appellee's broker. Linda
Aduca, who wrote the computation of the balance: that on March 2, 1989, plaintiff-appellant was ready with the
The issue to be resolved is: whether plaintiff had the right to compel defendants to execute a
amount corresponding to the balance, covered by Metrobank Cashier's Check No. 004223, payable to
deed of absolute sale in accordance with the agreement of February 14, 1989: and if so,
defendant-appellee Motorich Sales Corporation; that plaintiff-appellant and defendant-appellee Motorich Sales
whether plaintiff is entitled to damage.
Corporation were supposed to meet in the office of plaintiff-appellant but defendant-appellee's treasurer,
Nenita Lee Gruenberg, did not appear; that defendant-appellee Motorich Sales Corporation despite repeated
demands and in utter disregard of its commitments had refused to execute the Transfer of Rights/Deed of As to the first question, there is no evidence to show that defendant Nenita Lee Gruenberg
Assignment which is necessary to transfer the certificate of title; that defendant ACL Development Corp. is was indeed authorized by defendant corporation. Motorich Sales, to dispose of that property
impleaded as a necessary party since Transfer Certificate of Title No. (362909) 2876 is still in the name of said covered by T.C.T. No. (362909) 2876. Since the property is clearly owned by the corporation.
defendant; while defendant JNM Realty & Development Corp. is likewise impleaded as a necessary party in view Motorich Sales, then its disposition should be governed by the requirement laid down in Sec.
of the fact that it is the transferor of right in favor of defendant-appellee Motorich Sales Corporation: that on 40. of the Corporation Code of the Philippines, to wit:
April 6, 1989, defendant ACL Development Corporation and Motorich Sales Corporation entered into a Deed of
Absolute Sale whereby the former transferred to the latter the subject property; that by reason of said transfer, Sec. 40, Sale or other disposition of assets. Subject to the provisions of
the Registry of Deeds of Quezon City issued a new title in the name of Motorich Sales Corporation, represented existing laws on illegal combination and monopolies, a corporation may by
by defendant-appellee Nenita Lee Gruenberg and Reynaldo L. Gruenberg, under Transfer Certificate of Title No. a majority vote of its board of directors . . . sell, lease, exchange, mortgage,
3571; that as a result of defendants-appellees Nenita Lee Gruenberg and Motorich Sales Corporation's bad faith pledge or otherwise dispose of all or substantially all of its property and
in refusing to execute a formal Transfer of Rights/Deed of Assignment, plaintiff-appellant suffered moral and assets including its goodwill . . . when authorized by the vote of the
nominal damages which may be assessed against defendants-appellees in the sum of Five Hundred Thousand stockholders representing at least two third (2/3) of the outstanding capital
(500,000.00) Pesos; that as a result of defendants-appellees Nenita Lee Gruenberg and Motorich Sales stock . . .
Corporation's unjustified and unwarranted failure to execute the required Transfer of Rights/Deed of
Assignment or formal deed of sale in favor of plaintiff-appellant, defendants-appellees should be assessed
No such vote was obtained by defendant Nenita Lee Gruenberg for that proposed sale[;]
exemplary damages in the sum of One Hundred Thousand (P100,000.00) Pesos; that by reason of defendants-
neither was there evidence to show that the supposed transaction was ratified by the
appellees' bad faith in refusing to execute a Transfer of Rights/Deed of Assignment in favor of plaintiff-
corporation. Plaintiff should have been on the look out under these circumstances. More so,
appellant, the latter lost the opportunity to construct a residential building in the sum of One Hundred
plaintiff himself [owns] several corporations (tsn dated August 16, 1993, p. 3) which makes
Thousand (P100,000.00) Pesos; and that as a consequence of defendants-appellees Nenita Lee Gruenberg and
him knowledgeable on corporation matters.
Motorich Sales Corporation's bad faith in refusing to execute a deed of sale in favor of plaintiff-appellant, it has
been constrained to obtain the services of counsel at an agreed fee of One Hundred Thousand (P100,000.00)
Pesos plus appearance fee for every appearance in court hearings. Regarding the question of damages, the Court likewise, does not find substantial evidence to
hold defendant Nenita Lee Gruenberg liable considering that she did not in anyway
misrepresent herself to be authorized by the corporation to sell the property to plaintiff (tsn
In its answer, defendants-appellees Motorich Sales Corporation and Nenita Lee Gruenberg interposed
dated September 27, 1991, p. 8).
as affirmative defense that the President and Chairman of Motorich did not sign the agreement
adverted to in par. 3 of the amended complaint; that Mrs. Gruenberg's signature on the agreement
(ref: par. 3 of Amended Complaint) is inadequate to bind Motorich. The other signature, that of Mr. In the light of the foregoing, the Court hereby renders judgment DISMISSING the complaint at
Reynaldo Gruenberg, President and Chairman of Motorich, is required: that plaintiff knew this from the instance for lack of merit.
very beginning as it was presented a copy of the Transfer of Rights (Annex B of amended complaint) at
the time the Agreement (Annex B of amended complaint) was signed; that plaintiff-appellant itself "Defendants" counterclaim is also DISMISSED for lack of basis. (Decision, pp. 7-8; Rollo, pp.
drafted the Agreement and insisted that Mrs. Gruenberg accept the P100,000.00 as earnest money; 34-35)
that granting, without admitting, the enforceability of the agreement, plaintiff-appellant nonetheless
failed to pay in legal tender within the stipulated period (up to March 2, 1989); that it was the
For clarity, the Agreement dated February 14, 1989 is reproduced hereunder:
Set V Corporation Code * Doctrine of Peircing Corpo Veil Cases*Page 136 of 164

AGREEMENT The transferor warrants that he [sic] is the lawful owner of the above-described property and
that there [are] no existing liens and/or encumbrances of whatsoever nature;
KNOW ALL MEN BY THESE PRESENTS:
In case of failure by the Transferee to pay the balance on the date specified on 1, (b), the
This Agreement, made and entered into by and between: earnest money shall be forfeited in favor of the Transferor.

That upon full payment of the balance, the TRANSFEROR agrees to execute a TRANSFER OF
RIGHTS/DEED OF ASSIGNMENT in favor of the TRANSFEREE.

MOTORICH SALES CORPORATION, a corporation duly organized and existing under and by virtue of
Philippine Laws, with principal office address at 5510 South Super Hi-way cor. Balderama St., Pio del IN WITNESS WHEREOF, the parties have hereunto set their hands this 14th day of February,
Pilar. Makati, Metro Manila, represented herein by its Treasurer, NENITA LEE GRUENBERG, hereinafter 1989 at Greenhills, San Juan, Metro Manila, Philippines.
referred to as the TRANSFEROR;
MOTORICH SALES CORPORATION SAN JUAN STRUCTURAL & STEEL FABRICATORS
and
TRANSFEROR TRANSFEREE
SAN JUAN STRUCTURAL & STEEL FABRICATORS, a corporation duly organized and existing under and
by virtue of the laws of the Philippines, with principal office address at Sumulong Highway, Barrio [SGD.] [SGD.]
Mambungan, Antipolo, Rizal, represented herein by its President, ANDRES T. CO, hereinafter referred
to as the TRANSFEREE. By. NENITA LEE GRUENBERG By: ANDRES T. CO

WITNESSETH, That: Treasurer President

WHEREAS, the TRANSFEROR is the owner of a parcel of land identified as Lot 30 Block 1 of the Signed In the presence of:
ACROPOLIS GREENS SUBDIVISION located at the District of Murphy, Quezon City, Metro Manila,
containing an area of FOUR HUNDRED FOURTEEN (414) SQUARE METERS, covered by a TRANSFER OF
[SGD.] [SGD.]
RIGHTS between JNM Realty & Dev. Corp. as the Transferor and Motorich Sales Corp. as the Transferee;

6
NOW, THEREFORE, for and in consideration of the foregoing premises, the parties have
agreed as follows:
In its recourse before the Court of Appeals, petitioner insisted:
1. That the purchase price shall be at FIVE THOUSAND TWO HUNDRED
PESOS (P5,200.00) per square meter; subject to the following terms: 1. Appellant is entitled to compel the appellees to execute a Deed of Absolute Sale in accordance with
the Agreement of February 14, 1989,
a. Earnest money amounting to ONE HUNDRED THOUSAND PESOS
7
(P100,000.00), will be paid upon the execution of this agreement and shall 2. Plaintiff is entitled to damages.
form part of the total purchase price;
As stated earlier, the Court of Appeals debunked petitioner's arguments and affirmed the Decision of the RTC
b. Balance shall be payable on or before March 2, 1989; with the modification that Respondent Nenita Lee Gruenberg was ordered to refund P100,000 to petitioner, the
amount remitted as "downpayment" or "earnest money." Hence, this petition before us. 8
2. That the monthly amortization for the month of February 1989 shall be for the
account of the Transferor; and that the monthly amortization starting March 21, The Issues
1989 shall be for the account of the Transferee;
Before this Court, petitioner raises the following issues:
Set V Corporation Code * Doctrine of Peircing Corpo Veil Cases*Page 137 of 164

I. Whether or not the doctrine of piercing the veil of corporate fiction is applicable in the instant case A corporation is a juridical person separate and distinct from its stockholders or members. Accordingly, the
property of the corporation is not the property of its stockholders or members and may not be sold by the
II. Whether or not the appellate court may consider matters which the parties failed to raise in the stockholders or members without express authorization from the corporation's board of directors. 10 Section 23
lower court of BP 68, otherwise known as the Corporation Code of the Philippines, provides;

III. Whether or not there is a valid and enforceable contract between the petitioner and the respondent Sec. 23. The Board of Directors or Trustees. Unless otherwise provided in this Code, the corporate
corporation powers of all corporations formed under this Code shall be exercised, all business conducted and all
property of such corporations controlled and held by the board of directors or trustees to be elected
from among the holders of stocks, or where there is no stock, from among the members of the
IV. Whether or not the Court of Appeals erred in holding that there is a valid correction/substitution of
corporation, who shall hold office for one (1) year and until their successors are elected and qualified.
answer in the transcript of stenographic note[s].

9
Indubitably, a corporation may act only through its board of directors or, when authorized either by its bylaws
V. Whether or not respondents are liable for damages and attorney's fees
or by its board resolution, through its officers or agents in the normal course of business. The general principles
of agency govern the relation between the corporation and its officers or agents, subject to the articles of
The Court synthesized the foregoing and will thus discuss them seriatim as follows: incorporation, bylaws, or relevant provisions of law. 11 Thus, this Court has held that "a corporate officer or
agent may represent and bind the corporation in transactions with third persons to the extent that the
1. Was there a valid contract of sale between petitioner and Motorich? authority to do so has been conferred upon him, and this includes powers which have been intentionally
conferred, and also such powers as, in the usual course of the particular business, are incidental to, or may be
implied from, the powers intentionally conferred, powers added by custom and usage, as usually pertaining to
2. May the doctrine of piercing the veil of corporate fiction be applied to Motorich?
the particular officer or agent, and such apparent powers as the corporation has caused persons dealing with
the officer or agent to believe that it has conferred." 12
3. Is the alleged alteration of Gruenberg's testimony as recorded in the transcript of stenographic
notes material to the disposition of this case?
Furthermore, the Court has also recognized the rule that "persons dealing with an assumed agent, whether the
assumed agency be a general or special one bound at their peril, if they would hold the principal liable, to
4. Are respondents liable for damages and attorney's fees? ascertain not only the fact of agency but also the nature and extent of authority, and in case either is
controverted, the burden of proof is upon them to establish it (Harry Keeler v. Rodriguez, 4 Phil. 19)." 13 Unless
The Court's Ruling duly authorized, a treasurer, whose powers are limited, cannot bind the corporation in a sale of its assets. 14

The petition is devoid of merit. In the case at bar, Respondent Motorich categorically denies that it ever authorized Nenita Gruenberg, its
treasurer, to sell the subject parcel of land. 15 Consequently, petitioner had the burden of proving that Nenita
Gruenberg was in fact authorized to represent and bind Motorich in the transaction. Petitioner failed to
First Issue: Validity of Agreement
discharge this burden. Its offer of evidence before the trial court contained no proof of such authority. 16 It has
not shown any provision of said respondent's articles of incorporation, bylaws or board resolution to prove that
Petitioner San Juan Structural and Steel Fabricators, Inc. alleges that on February 14, 1989, it entered through Nenita Gruenberg possessed such power.
its president, Andres Co, into the disputed Agreement with Respondent Motorich Sales Corporation, which was
in turn allegedly represented by its treasurer, Nenita Lee Gruenberg. Petitioner insists that "[w]hen Gruenberg
That Nenita Gruenberg is the treasurer of Motorich does not free petitioner from the responsibility of
and Co affixed their signatures on the contract they both consented to be bound by the terms thereof." Ergo,
ascertaining the extent of her authority to represent the corporation. Petitioner cannot assume that she, by
petitioner contends that the contract is binding on the two corporations. We do not agree.
virtue of her position, was authorized to sell the property of the corporation. Selling is obviously foreign to a
corporate treasurer's function, which generally has been described as "to receive and keep the funds of the
True, Gruenberg and Co signed on February 14, 1989, the Agreement, according to which a lot owned by corporation, and to disburse them in accordance with the authority given him by the board or the properly
Motorich Sales Corporation was purportedly sold. Such contract, however, cannot bind Motorich, because it authorized officers." 17
never authorized or ratified such sale.

Neither was such real estate sale shown to be a normal business activity of Motorich. The primary purpose of
Motorich is marketing, distribution, export and import in relation to a general merchandising
business. 18 Unmistakably, its treasurer is not cloaked with actual or apparent authority to buy or sell real
property, an activity which falls way beyond the scope of her general authority.
Set V Corporation Code * Doctrine of Peircing Corpo Veil Cases*Page 138 of 164

Art. 1874 and 1878 of the Civil Code of the Philippines provides: Petitioner also argues that the veil of corporate fiction of Motorich should be pierced, because the latter is a
close corporation. Since "Spouses Reynaldo L. Gruenberg and Nenita R. Gruenberg owned all or almost all or
Art. 1874. When a sale of a piece of land or any interest therein is through an agent, the authority of 99.866% to be accurate, of the subscribed capital stock" 25 of Motorich, petitioner argues that Gruenberg
the latter shall be in writing: otherwise, the sale shall be void. needed no authorization from the board to enter into the subject contract. 26 It adds that, being solely owned
by the Spouses Gruenberg, the company can treated as a close corporation which can be bound by the acts of
its principal stockholder who needs no specific authority. The Court is not persuaded.
Art. 1878. Special powers of attorney are necessary in the following case:

First, petitioner itself concedes having raised the issue belatedly, 27 not having done so during the trial, but
xxx xxx xxx
only when it filed its sur-rejoinder before the Court of Appeals. 28 Thus, this Court cannot entertain said issue at
this late stage of the proceedings. It is well-settled the points of law, theories and arguments not brought to the
(5) To enter any contract by which the ownership of an immovable is transmitted or acquired either attention of the trial court need not be, and ordinarily will not be, considered by a reviewing court, as they
gratuitously or for a valuable consideration; cannot be raised for the first time on appeal. 29 Allowing petitioner to change horses in midstream, as it were, is
to run roughshod over the basic principles of fair play, justice and due process.
xxx xxx xxx.
Second, even if the above mentioned argument were to be addressed at this time, the Court still finds no
Petitioner further contends that Respondent Motorich has ratified said contract of sale because of its reason to uphold it. True, one of the advantages of a corporate form of business organization is the limitation of
"acceptance of benefits," as evidenced by the receipt issued by Respondent Gruenberg. 19 Petitioner is an investor's liability to the amount of the investment. 30 This feature flows from the legal theory that a
clutching at straws. corporate entity is separate and distinct from its stockholders. However, the statutorily granted privilege of a
corporate veil may be used only for legitimate purposes. 31 On equitable considerations, the veil can be
disregarded when it is utilized as a shield to commit fraud, illegality or inequity; defeat public convenience;
As a general rule, the acts of corporate officers within the scope of their authority are binding on the
confuse legitimate issues; or serve as a mere alter ego or business conduit of a person or an instrumentality,
corporation. But when these officers exceed their authority, their actions "cannot bind the corporation, unless it
agency or adjunct of another corporation. 32
has ratified such acts or is estopped from disclaiming them." 20

Thus, the Court has consistently ruled that "[w]hen the fiction is used as a means of perpetrating a fraud or an
In this case, there is a clear absence of proof that Motorich ever authorized Nenita Gruenberg, or made it
illegal act or as vehicle for the evasion of an existing obligation, the circumvention of statutes, the achievement
appear to any third person that she had the authority, to sell its land or to receive the earnest money. Neither
or perfection of a monopoly or generally the perpetration of knavery or crime, the veil with which the law
was there any proof that Motorich ratified, expressly or impliedly, the contract. Petitioner rests its argument on
covers and isolates the corporation from the members or stockholders who compose it will be lifted to allow for
the receipt which, however, does not prove the fact of ratification. The document is a hand-written one, not a
its consideration merely as an aggregation of individuals." 33
corporate receipt, and it bears only Nenita Gruenberg's signature. Certainly, this document alone does not
prove that her acts were authorized or ratified by Motorich.
We stress that the corporate fiction should be set aside when it becomes a shield against liability for fraud,
illegality or inequity committed on third persons. The question of piercing the veil of corporate fiction is
Art. 1318 of the Civil Code lists the requisites of a valid and perfected contract: "(1) consent of the contracting
essentially, then, a matter of proof. In the present case, however, the Court finds no reason to pierce the
parties; (2) object certain which is the subject matter of the contract; (3) cause of the obligation which is
corporate veil of Respondent Motorich. Petitioner utterly failed to establish that said corporation was formed, or
established." As found by the trial court 21 and affirmed by the Court of Appeals, 22 there is no evidence that
that it is operated, for the purpose of shielding any alleged fraudulent or illegal activities of its officers or
Gruenberg was authorized to enter into the contract of sale, or that the said contract was ratified by Motorich.
stockholders; or that the said veil was used to conceal fraud, illegality or inequity at the expense of third
This factual finding of the two courts is binding on this Court. 23 As the consent of the seller was not obtained,
persons like petitioner.
no contract to bind the obligor was perfected. Therefore, there can be no valid contract of sale between
petitioner and Motorich.
Petitioner claims that Motorich is a close corporation. We rule that it is not. Section 96 of the Corporation Code
defines a close corporation as follows:
Because Motorich had never given a written authorization to Respondent Gruenberg to sell its parcel of land,
we hold that the February 14, 1989 Agreement entered into by the latter with petitioner is void under Article
1874 of the Civil Code. Being inexistent and void from the beginning, said contract cannot be ratified. 24 Sec. 96. Definition and Applicability of Title. A close corporation, within the meaning of this Code, is
one whose articles of incorporation provide that: (1) All of the corporation's issued stock of all classes,
exclusive of treasury shares, shall be held of record by not more than a specified number of persons,
Second Issue:
not exceeding twenty (20); (2) All of the issued stock of all classes shall be subject to one or more
Piercing the Corporate Veil Not Justified
specified restrictions on transfer permitted by this Title; and (3) The corporation shall not list in any
stock exchange or make any public offering of any of its stock of any class. Notwithstanding the
Set V Corporation Code * Doctrine of Peircing Corpo Veil Cases*Page 139 of 164

foregoing, a corporation shall be deemed not a close corporation when at least two-thirds (2/3) of its Petitioner calls our attention to the following excerpt of the transcript of stenographic notes (TSN):
voting stock or voting rights is owned or controlled by another corporation which is not a close
corporation within the meaning of this Code. . . . . Q Did you ever represent to Mr. Co that you were authorized by the corporation to sell the property?

The articles of incorporation 34 of Motorich Sales Corporation does not contain any provision stating that (1) the A Yes, sir. 45

number of stockholders shall not exceed 20, or (2) a preemption of shares is restricted in favor of any
stockholder or of the corporation, or (3) listing its stocks in any stock exchange or making a public offering of
Petitioner claims that the answer "Yes" was crossed out, and, in its place was written a "No" with an initial
such stocks is prohibited. From its articles, it is clear that Respondent Motorich is not a close
scribbled above it. 46 This, however, is insufficient to prove that Nenita Gruenberg was authorized to represent
corporation. 35 Motorich does not become one either, just because Spouses Reynaldo and Nenita Gruenberg
Respondent Motorich in the sale of its immovable property. Said excerpt be understood in the context of her
owned 99.866% of its subscribed capital stock. The "[m]ere ownership by a single stockholder or by another
whole testimony. During her cross-examination. Respondent Gruenberg testified:
corporation of all or capital stock of a corporation is not of itself sufficient ground for disregarding the separate
corporate personalities." 36 So, too, a narrow distribution of ownership does not, by itself, make a close
corporation. Q So, you signed in your capacity as the treasurer?

Petitioner cites Manuel R. Dulay Enterprises, Inc. v. Court of Appeals 37 wherein the Court ruled that ". . . [A] Yes, sir.
petitioner corporation is classified as a close corporation and, consequently, a board resolution authorizing the
sale or mortgage of the subject property is not necessary to bind the corporation for the action of its Q Even then you kn[e]w all along that you [were] not authorized?
president." 38 But the factual milieu inDulay is not on all fours with the present case. In Dulay, the sale of real
property was contracted by the president of a close corporation with the knowledge and acquiescence of its
A Yes, sir.
board of directors. 39 In the present case, Motorich is not a close corporation, as previously discussed, and the
agreement was entered into by the corporate treasurer without the knowledge of the board of directors.
Q You stated on direct examination that you did not represent that you were authorized to sell the
property?
The Court is not unaware that there are exceptional cases where "an action by a director, who singly is the
controlling stockholder, may be considered as a binding corporate act and a board action as nothing more than
a mere formality." 40 The present case, however, is not one of them. A Yes, sir.

As stated by petitioner, Spouses Reynaldo and Nenita Gruenberg own "almost 99.866%" of Respondent Q But you also did not say that you were not authorized to sell the property, you did not tell that to Mr.
Motorich.41 Since Nenita is not the sole controlling stockholder of Motorich, the aforementioned exception does Co, is that correct?
not apply. Granting arguendo that the corporate veil of Motorich is to be disregarded, the subject parcel of land
would then be treated as conjugal property of Spouses Gruenberg, because the same was acquired during their A That was not asked of me.
marriage. There being no indication that said spouses, who appear to have been married before the effectivity
of the Family Code, have agreed to a different property regime, their property relations would be governed by
Q Yes, just answer it.
conjugal partnership of gains. 42 As a consequence, Nenita Gruenberg could not have effected a sale of the
subject lot because "[t]here is no co-ownership between the spouses in the properties of the conjugal
partnership of gains. Hence, neither spouse can alienate in favor of another his or interest in the partnership or A I just told them that I was the treasurer of the corporation and it [was] also the president who [was]
in any property belonging to it; neither spouse can ask for a partition of the properties before the partnership also authorized to sign on behalf of the corporation.
has been legally dissolved." 43
Q You did not say that you were not authorized nor did you say that you were authorized?
Assuming further, for the sake of argument, that the spouses' property regime is the absolute community of
property, the sale would still be invalid. Under this regime, "alienation of community property must have the A Mr. Co was very interested to purchase the property and he offered to put up a P100,000.00 earnest
written consent of the other spouse or he authority of the court without which the disposition or encumbrance money at that time. That was our first meeting. 47
is void." 44 Both requirements are manifestly absent in the instant case.
Clearly then, Nenita Gruenberg did not testify that Motorich had authorized her to sell its property. On the other
Third Issue: Challenged Portion of TSN Immaterial hand, her testimony demonstrates that the president of Petitioner Corporation, in his great desire to buy the
property, threw caution to the wind by offering and paying the earnest money without first verifying
Gruenberg's authority to sell the lot.
Set V Corporation Code * Doctrine of Peircing Corpo Veil Cases*Page 140 of 164

Fourth Issue: principle embodied in Article 2154 of Civil Code. 55 Although there was no binding relation between them,
Damages and Attorney's Fees petitioner paid Gruenberg on the mistaken belief that she had the authority to sell the property of
Motorich. 56 Article 2155 of Civil Code provides that "[p]ayment by reason of a mistake in the contruction or
Finally, petitioner prays for damages and attorney's fees, alleging that "[i]n an utter display of malice and bad application of a difficult question of law may come within the scope of the preceding article."WHEREFORE, the
faith, respondents attempted and succeeded in impressing on the trial court and [the] Court of Appeals that petition is hereby DENIED and the assailed Decision is AFFIRMED.SO ORDERED.
Gruenberg did not represent herself as authorized by Respondent Motorich despite the receipt issued by the
former specifically indicating that she was signing on behalf of Motorich Sales Corporation. Respondent SECOND DIVISIONG.R. No. L-41337 June 30, 1988
Motorich likewise acted in bad faith when it claimed it did not authorize Respondent Gruenberg and that the
contract [was] not binding, [insofar] as it [was] concerned, despite receipt and enjoyment of the proceeds of TAN BOON BEE & CO., INC., petitioner, vs.
Gruenberg's act." 48Assuming that Respondent Motorich was not a party to the alleged fraud, petitioner THE HONORABLE HILARION U. JARENCIO, PRESIDING JUDGE OF BRANCH XVIII of the Court of First
maintains that Respondent Gruenberg should be held liable because she "acted fraudulently and in bad faith Instance of Manila, GRAPHIC PUBLISHING, INC., and PHILIPPINE AMERICAN CAN DRUG
[in] representing herself as duly authorized by [R]espondent [C]orporation." 49 COMPANY,respondents.

As already stated, we sustain the findings of both the trial and the appellate courts that the foregoing PARAS, J.:
allegations lack factual bases. Hence, an award of damages or attorney's fees cannot be justified. The amount
paid as "earnest money" was not proven to have redounded to the benefit of Respondent Motorich. Petitioner
This is a petition for certiorari, with prayer for preliminary injunction, to annul and set aside the March 26, 1975
claims that said amount was deposited to the account of Respondent Motorich, because "it was deposited with
Order of the then Court of First Instance of Manila, Branch XXIII, setting aside the sale of "Heidelberg" cylinder
the account of Aren Commercial c/o Motorich Sales Corporation." 50 Respondent Gruenberg, however, disputes
press executed by the sheriff in favor of the herein petitioner, as well as the levy on the said property, and
the allegations of petitioner. She testified as follows:
ordering the sheriff to return the said machinery to its owner, herein private respondent Philippine American
Drug Company.
Q You voluntarily accepted the P100,000.00, as a matter of fact, that was encashed, the
check was encashed.
Petitioner herein, doing business under the name and style of Anchor Supply Co., sold on credit to herein
private respondent Graphic Publishing, Inc. (GRAPHIC for short) paper products amounting to P55,214.73. On
A Yes. sir, the check was paid in my name and I deposit[ed] it. December 20, 1972, GRAPHIC made partial payment by check to petitioner in the total amount of P24,848.74;
and on December 21, 1972, a promissory note was executed to cover the balance of P30,365.99. In the said
Q In your account? promissory note, it was stipulated that the amount will be paid on monthly installments and that failure to pay
any installment would make the amount immediately demandable with an interest of 12% per annum. On
A Yes, sir. 51 September 6, 1973, for failure of GRAPHIC to pay any installment, petitioner filed with the then Court of First
Instance of Manila, Branch XXIII, presided over by herein respondent judge, Civil Case No. 91857 for a Sum of
Money (Rollo, pp. 36-38). Respondent judge declared GRAPHIC in default for failure to file its answer within the
In any event, Gruenberg offered to return the amount to petitioner ". . . since the sale did not push
reglementary period and plaintiff (petitioner herein) was allowed to present its evidence ex parte. In a Decision
through." 52
dated January 18, 1974 (Ibid., pp. 39-40), the trial court ordered GRAPHIC to pay the petitioner the sum of
P30,365.99 with 12% interest from March 30, 1973 until fully paid, plus the costs of suit. On motion of
Moreover, we note that Andres Co is not a neophyte in the world of corporate business. He has been the petitioner, a writ of execution was issued by respondent judge; but the aforestated writ having expired without
president of Petitioner Corporation for more than ten years and has also served as chief executive of two other the sheriff finding any property of GRAPHIC, an alias writ of execution was issued on July 2, 1974.
corporate entities. 53Co cannot feign ignorance of the scope of the authority of a corporate treasurer such as
Gruenberg. Neither can he be oblivious to his duty to ascertain the scope of Gruenberg's authorization to enter
Pursuant to the said issued alias writ of execution, the executing sheriff levied upon one (1) unit printing
into a contract to sell a parcel of land belonging to Motorich.
machine Identified as "Original Heidelberg Cylinder Press" Type H 222, NR 78048, found in the premises of
GRAPHIC. In a Notice of Sale of Execution of Personal Property dated July 29, 1974, said printing machine was
Indeed, petitioner's claim of fraud and bad faith is unsubstantiated and fails to persuade the Court. Indubitably, scheduled for auction sale on July 26, 1974 at 10:00 o'clock at 14th St., Cor. Atlanta St., Port Area, Manila
petitioner appears to be the victim of its own officer's negligence in entering into a contract with and paying an (lbid., p. 45); but in a letter dated July 19, 1974, herein private respondent, Philippine American Drug Company
unauthorized officer of another corporation. (PADCO for short) had informed the sheriff that the printing machine is its property and not that of GRAPHIC,
and accordingly, advised the sheriff to cease and desist from carrying out the scheduled auction sale on July
As correctly ruled by the Court of Appeals, however, Nenita Gruenberg should be ordered to return to petitioner 26, 1974. Notwithstanding the said letter, the sheriff proceeded with the scheduled auction sale, sold the
the amount she received as earnest money, as "no one shall enrich himself at the expense of another." 54 a property to the petitioner, it being the highest bidder, and issued a Certificate of Sale in favor of petitioner
(Rollo, p. 48). More than five (5) hours after the auction sale and the issuance of the certificate of sale, PADCO
Set V Corporation Code * Doctrine of Peircing Corpo Veil Cases*Page 141 of 164

filed an "Affidavit of Third Party Claim" with the Office of the City Sheriff ( Ibid., p. 47). Thereafter, on July In other words, constitution, Section 17 of Rule 39 of the Revised Rules of Court, the rights of third-
30,1974, PADCO filed with the Court of First Instance of Manila, Branch XXIII, a Motion to Nullify Sale on party claimants over certain properties levied upon by the sheriff to satisfy the judgment should not
Execution (With Injunction) (Ibid., pp, 49-55), which was opposed by the petitioner (Ibid., pp. 5668). Respondent be decided inthe action where the third-party claims have been presented, but in the separate action
judge, in an Order dated March 26, 1975 (Ibid., pp. 64-69), ruled in favor of PADCO. The decretal portion of the instituted by the claimants.
said order, reads:
... Otherwise stated, the court issuing a writ of execution is supposed to enforce the authority only
WHEREFORE, the sale of the 'Heidelberg cylinder press executed by the Sheriff in favor of the plaintiff over properties of the judgment debtor, and should a third party appeal- to claim the property levied
as well as the levy on the said property is hereby set aside and declared to be without any force and upon by the sheriff, the procedure laid down by the Rules is that such claim should be the subject of a
effect. The Sheriff is ordered to return the said machinery to its owner, the Philippine American Drug separate and independent action.
Co.
xxx xxx xxx
Petitioner filed a Motion For Reconsideration (Ibid., pp. 7093) and an Addendum to Motion for Reconsideration
(Ibid., pp. 94-08), but in an Order dated August 13, 1975, the same was denied for lack of merit ( Ibid., p. 109). ... This rule is dictated by reasons of convenience, as "intervention is more likely to inject confusion
Hence, the instant petition. into the issues between the parties in the case . . . with which the third-party claimant has nothing to
do and thereby retard instead of facilitate the prompt dispatch of the controversy which is the
In a Resolution dated September 12, 1975, the Second Division of this Court resolved to require the underlying objective of the rules of pleading and practice." Besides, intervention may not be permitted
respondents to comment, and to issue a temporary restraining order (Rollo, p. 111 ). After submission of the after trial has been concluded and a final judgment rendered in the case.
parties' Memoranda, the case was submitted for decision in the Resolution of November 28, 1975 ( Ibid., p.
275). However, the fact that petitioner questioned the jurisdiction of the court during the initial hearing of the case
but nevertheless actively participated in the trial, bars it from questioning now the court's jurisdiction. A party
Petitioner, to support its stand, raised two (2) issues, to wit: who voluntarily participated in the trial, like the herein petitioner, cannot later on raise the issue of the court's
lack of jurisdiction (Philippine National Bank vs. Intermediate Appellate Court, 143 SCRA [1986]).
I
As to the second issue (the non-piercing of PADCO's corporate Identity) the decision of respondent judge is as
THE RESPONDENT JUDGE GRAVELY EXCEEDED, IF NOT ACTED WITHOUT JURISDICTION WHEN HE ACTED UPON follows:
THE MOTION OF PADCO, NOT ONLY BECAUSE SECTION 17, RULE 39 OF THE RULES OF COURT WAS NOT
COMPLIED WITH, BUT ALSO BECAUSE THE CLAIMS OF PADCO WHICH WAS NOT A PARTY TO THE CASE COULD The plaintiff, however, contends that the controlling stockholders of the Philippine American Drug Co.
NOT BE VENTILATED IN THE CASE BEFORE HIM BUT IN INDEPENDENT PROCEEDING. are also the same controlling stockholders of the Graphic Publishing, Inc. and, therefore, the levy upon
the said machinery which was found in the premises occupied by the Graphic Publishing, Inc. should
II be upheld. This contention cannot be sustained because the two corporations were duly incorporated
under the Corporation Law and each of them has a juridical personality distinct and separate from the
other and the properties of one cannot be levied upon to satisfy the obligation of the other. This legal
THE RESPONDENT JUDGE GRAVELY ABUSED HIS DISCRETION WHEN HE REFUSED TO PIERCE THE PADCO'S
preposition is elementary and fundamental.
(IDENTITY) AND DESPITE THE ABUNDANCE OF EVIDENCE CLEARLY SHOWING THAT PADCO WAS CONVENIENTLY
SHIELDING UNDER THE THEORY OF CORPORATE PETITION.
It is true that a corporation, upon coming into being, is invested by law with a personality separate and distinct
from that of the persons composing it as well as from any other legal entity to which it may be related (Yutivo &
Petitioner contends that respondent judge gravely exceeded, if not, acted without jurisdiction, in nullifying the
Sons Hardware Company vs. Court of Tax Appeals, 1 SCRA 160 [1961]; and Emilio Cano Enterprises, Inc. vs.
sheriffs sale not only because Section 17, Rule 39 of the Rules of Court was not complied with, but more
CIR, 13 SCRA 290 [1965]). As a matter of fact, the doctrine that a corporation is a legal entity distinct and
importantly because PADCO could not have litigated its claim in the same case, but in an independent civil
separate from the members and stockholders who compose it is recognized and respected in all cases which
proceeding.
are within reason and the law (Villa Rey Transit, Inc. vs. Ferrer, 25 SCRA 845 [1968]). However, this separate
and distinct personality is merely a fiction created by law for convenience and to promote justice (Laguna
This contention is well-taken. Transportation Company vs. SSS, 107 Phil. 833 [1960]). Accordingly, this separate personality of the
corporation may be disregarded, or the veil of corporate fiction pierced, in cases where it is used as a cloak or
In the case of Bayer Philippines, Inc. vs. Agana (63 SCRA 355, 366-367 [1975]), this Court categorically ruled as cover for fraud or illegality, or to work an injustice, or where necessary to achieve equity or when necessary for
follows: the protection of creditors (Sulo ng Bayan, Inc. vs. Araneta, Inc., 72 SCRA 347 [1976]). Corporations are
Set V Corporation Code * Doctrine of Peircing Corpo Veil Cases*Page 142 of 164

composed of natural persons and the legal fiction of a separate corporate personality is not a shield for the MELENCIO-HERRERA, J.:1wph1.t
commission of injustice and inequity (Chenplex Philippines, Inc., et al. vs. Hon. Pamatian et al., 57 SCRA 408
(19741). Likewise, this is true when the corporation is merely an adjunct, business conduit or alter ego of These certiorari proceedings stem from the award rendered against petitioner Telephone Engineering and
another corporation. In such case, the fiction of separate and distinct corporation entities should be Services, Co., Inc. (TESCO) on October 6, 1967 by the Acting Referee of Regional Office No. 4, Quezon City Sub-
disregarded (Commissioner of Internal Revenue vs. Norton & Harrison, 11 SCRA 714 [1964]). Regional Office, Workmen's Compensation Section, in favor of respondent Leonila S. Gatus and her children,
dependents of the deceased employee Pacifico L. Gatus. The principal contention is that the award was
In the instant case, petitioner's evidence established that PADCO was never engaged in the printing business; rendered without jurisdiction as there was no employer-employee relationship between petitioner and the
that the board of directors and the officers of GRAPHIC and PADCO were the same; and that PADCO holds 50% deceased.
share of stock of GRAPHIC. Petitioner likewise stressed that PADCO's own evidence shows that the printing
machine in question had been in the premises of GRAPHIC since May, 1965, long before PADCO even acquired Petitioner is a domestic corporation engaged in the business of manufacturing telephone equipment with
its alleged title on July 11, 1966 from Capitol Publishing. That the said machine was allegedly leased by PADCO offices at Sheridan Street, Mandaluyong, Rizal. Its Executive Vice-President and General Manager is Jose Luis
to GRAPHIC on January 24, 1966, even before PADCO purchased it from Capital Publishing on July 11, 1966, Santiago. It has a sister company, the Utilities Management Corporation (UMACOR), with offices in the same
only serves to show that PADCO's claim of ownership over the printing machine is not only farce and sham but location. UMACOR is also under the management of Jose Luis Santiago.
also unbelievable.

On September 8, 1964, UMACOR employed the late Pacifica L. Gatus as Purchasing Agent. On May 16, 1965,
Considering the aforestated principles and the circumstances established in this case, respondent judge should Pacifico L. Gatus was detailed with petitioner company. He reported back to UMACOR on August 1, 1965. On
have pierced PADCO's veil of corporate Identity. January 13, 1967, he contracted illness and although he retained to work on May 10, 1967, he died
nevertheless on July 14, 1967 of "liver cirrhosis with malignant degeneration."
Respondent PADCO argues that if respondent judge erred in not piercing the veil of its corporate fiction, the
error is merely an error of judgment and not an error of jurisdiction correctable by appeal and not by certiorari. On August 7, 1967, his widow, respondent Leonila S. Gatus, filed a "Notice and Claim for Compensation" with
Regional Office No. 4, Quezon City Sub-Regional Office, Workmen's Compensation Section, alleging therein that
To this argument of respondent, suffice it to say that the same is a mere technicality. In the case of Rubio vs. her deceased husband was an employee of TESCO, and that he died of liver cirrhosis. 1 On August 9, 1967, and
Mariano (52 SCRA 338, 343 [1973]), this Court ruled: Office wrote petitioner transmitting the Notice and for Compensation, and requiring it to submit an Employer's
Report of Accident or Sickness pursuant to Section 37 of the Workmen's Compensation Act (Act No. 3428). 2 An
While We recognize the fact that these movants the MBTC, the Phillips spouses, the Phillips "Employer's Report of Accident or Sickness" was thus submitted with UMACOR indicated as the employer of the
corporation and the Hacienda Benito, Inc. did raise in their respective answers the issue as to the deceased. The Report was signed by Jose Luis Santiago. In answer to questions Nos. 8 and 17, the employer
propriety of the instant petition for certiorari on the ground that the remedy should have been appeal stated that it would not controvert the claim for compensation, and admitted that the deceased employee
within the reglementary period, We considered such issue as a mere technicality which would have contracted illness "in regular occupation." 3 On the basis of this Report, the Acting Referee awarded death
accomplished nothing substantial except to deny to the petitioner the right to litigate the matters he benefits in the amount of P5,759.52 plus burial expenses of P200.00 in favor of the heirs of Gatus in a letter-
raised ... award dated October 6, 1967 4 against TESCO.

Litigations should, as much as possible, be decided on their merits and not on technicality (De las Alas vs. Replying on October 27, 1967, TESCO, through Jose Luis Santiago, informed the Acting Referee that it would
Court of Appeals, 83 SCRA 200, 216 [1978]). Every party-litigant must be afforded the amplest opportunity for avail of the 15-days-notice given to it to state its non-conformity to the award and contended that the cause of
the proper and just determination of his cause, free from the unacceptable plea of technicalities (Heirs of the illness contracted by Gatus was in no way aggravated by the nature of his work. 5
Ceferino Morales vs. Court of Appeals, 67 SCRA 304, 310 [1975]).PREMISES CONSIDERED, the March 26,1975
Order of the then Court of First Instance of Manila, is ANNULLED and SET ASIDE, and the Temporary Restraining On November 6, 1967, TESCO requested for an extension of ten days within which to file a Motion for
Order issued is hereby made permanent.SO ORDERED. Reconsideration, 6 and on November 15, 1967, asked for an additional extension of five days. 7 TESCO filed its
"Motion for Reconsideration and/or Petition to Set Aside Award" on November 18, 1967, alleging as grounds
FIRST DIVISION G.R. No. L-28694 May 13, 1981 therefor, that the admission made in the "Employer's Report of Accident or Sickness" was due to honest
mistake and/or excusable negligence on its part, and that the illness for which compensation is sought is not an
occupational disease, hence, not compensable under the law. 8 The extension requested was denied. The
TELEPHONE ENGINEERING & SERVICE COMPANY, INC., petitioner, vs.
Motion for Reconsideration was likewise denied in an Order issued by the Chief of Section of the Regional Office
WORKMEN'S COMPENSATION COMMISSION, PROVINCIAL SHERIFF OF RIZAL and LEONILA SANTOS
dated December 28, 1967 9 predicated on two grounds: that the alleged mistake or negligence was not
GATUS, for herself and in behalf of her minor children, Teresita, Antonina and Reynaldo, all
excusable, and that the basis of the award was not the theory of direct causation alone but also on that of
surnamed GATUS, respondents.
aggravation. On January 28, 1968, an Order of execution was issued by the same Office.
Set V Corporation Code * Doctrine of Peircing Corpo Veil Cases*Page 143 of 164

On February 3, 1968, petitioner filed an "Urgent Motion to Compel Referee to Elevate the Records to the and/or Petition to Set Aside Award," and in its "Urgent Motion to Compel the Referee to Elevate Records to the
Workmen's Compensation Commission for Review." 10 Meanwhile, the Provincial Sheriff of Rizal levied on and Commission for Review," petitioner represented and defended itself as the employer of the deceased. Nowhere
attached the properties of TESCO on February 17, 1968, and scheduled the sale of the same at public auction in said documents did it allege that it was not the employer. Petitioner even admitted that TESCO and UMACOR
on February 26, 1968. On February 28, 1968, the Commission issued an Order requiring petitioner to submit are sister companies operating under one single management and housed in the same building. Although
verified or true copies of the Motion for Reconsideration and/or Petition to Set Aside Award and Order of respect for the corporate personality as such, is the general rule, there are exceptions. In appropriate cases,
December 28, 1967, and to show proof that said Motion for Reconsideration was filed within the reglementary the veil of corporate fiction may be pierced as when the same is made as a shield to confuse the legitimate
period, with the warning that failure to comply would result in the dismissal of the Motion. However, before this issues. 16
Order could be released, TESCO filed with this Court, on February 22, 1968, The present petition for "Certiorari
with Preliminary Injunction" seeking to annul the award and to enjoin the Sheriff from levying and selling its While, indeed, jurisdiction cannot be conferred by acts or omission of the parties, TESCO'S denial at this stage
properties at public auction. that it is the employer of the deceased is obviously an afterthought, a devise to defeat the law and evade its
obligations. 17 This denial also constitutes a change of theory on appeal which is not allowed in this
On February 29, 1968, this Court required respondents to answer the Petition but denied Injunction. 11 TESCO'S jurisdiction. 18Moreover, issues not raised before the Workmen's Compensation Commission cannot be raised
Urgent Motion dated April 2, 1968, for the issuance of a temporary restraining order to enjoin the Sheriff from for the first time on appeal. 19 For that matter, a factual question may not be raised for the first time on appeal
proceeding with the auction sale of its properties was denied in our Resolution dated May 8, 1968. to the Supreme Court. 20

TESCO asserts: 1wph1.t This certiorari proceeding must also be held to have been prematurely brought. Before a petition for certiorari
can be instituted, all remedies available in the trial Court must be exhausted first. 21 certiorari cannot be
I. That the respondent Workmen's Compensation Commission has no jurisdiction nor authority to resorted to when the remedy of appeal is present. 22 What is sought to be annulled is the award made by the
render the award (Annex 'D', Petition) against your petitioner there being no employer-employee Referee. However, TESCO did not pursue the remedies available to it under Rules 23, 24 and 25 of the Rules of
relationship between it and the deceased Gatus; the Workmen's Compensation Commission, namely, an appeal from the award of the Referee, within fifteen
days from notice, to the Commission; a petition for reconsideration of the latter's resolution, if adverse, to the
Commission en banc; and within ten days from receipt of an unfavorable decision by the latter, an appeal to
II. That petitioner can never be estopped from questioning the jurisdiction of respondent commission
this Court. As petitioner had not utilized these remedies available to it, certiorari win not he, it being
especially considering that jurisdiction is never conferred by the acts or omission of the parties;
prematurely filed. As this Court ruled in the case of Manila Jockey Club, Inc. vs. Del Rosario, 2 SCRA 462
(1961). 1wph1.t
III. That this Honorable Court has jurisdiction to nullify the award of respondent commission.
An aggrieved party by the decision of a Commissioner should seek a reconsideration of the decision
TESCO takes the position that the Commission has no jurisdiction to render a valid award in this suit as there by the Commission en banc. If the decision is adverse to him, he may appeal to the Supreme Court.
was no employer-employee relationship between them, the deceased having been an employee of UMACOR An appeal brought to the Supreme Court without first resorting to the remedy referred to is premature
and not of TESCO. In support of this contention, petitioner submitted photostat copies of the payroll of UMACOR and may be dismissed.
for the periods May 16-31, 1967 and June 1-15, 1967 12 showing the name of the deceased as one of the three
employees listed under the Purchasing Department of UMACOR. It also presented a photostat copy of a check
Although this rule admits of exceptions, as where public welfare and the advancement of public policy so
of UMACOR payable to the deceased representing his salary for the period June 14 to July 13, 1967. 13
dictate, the broader interests of justice so require, or where the Orders complained of were found to be
completely null and void or that the appeal was not considered the appropriate remedy, 23 the case at bar does
Both public and private respondents contend, on the other hand, that TESCO is estopped from claiming lack of not fan within any of these exceptions. WHEREFORE, this Petition is hereby dismissed.SO ORDERED.
employer employee relationship.
SECOND DIVISION G.R. No. 89561 September 13, 1990
To start with, a few basic principles should be re-stated the existence of employer-employee relationship is the
jurisdictional foundation for recovery of compensation under the Workmen's Compensation Law. 14 The lack of
BUENAFLOR C. UMALI, MAURICIA M. VDA. DE CASTILLO, VICTORIA M. CASTILLO, BERTILLA C. RADA,
employer-employee relationship, however, is a matter of defense that the employer should properly raise in the
MARIETTA C. ABAEZ, LEOVINA C. JALBUENA and SANTIAGO M. RIVERA, petitioners, vs.
proceedings below. The determination of this relationship involves a finding of fact, which is conclusive and
COURT OF APPEALS, BORMAHECO, INC. and PHILIPPINE MACHINERY PARTS MANUFACTURING CO.,
binding and not subject to review by this Court. 15
INC., respondents.

Viewed in the light of these criteria, we note that it is only in this Petition before us that petitioner denied, for
REGALADO, J.:
the first time, the employer-employee relationship. In fact, in its letter dated October 27, 1967 to the Acting
Referee, in its request for extension of time to file Motion for Reconsideration, in its "Motion for Reconsideration
Set V Corporation Code * Doctrine of Peircing Corpo Veil Cases*Page 144 of 164

This is a petition to review the decision of respondent Court of Appeals, dated August 3, 1989, in CA-GR CV No. and of her children, namely: Buenaflor, Bertilla, Victoria, Marietta and Leovina, all surnamed Castillo
15412, entitled "Buenaflor M. Castillo Umali, et al. vs. Philippine Machinery Parts Manufacturing Co., Inc., et has (sic) been issued, namely: TCT No. T-12113 (Exhibit E ); TCT No. T-13113 (Exhibit F); TCT No. T-
al.,"1 the dispositive portion whereof provides: 13116 (Exhibit G ) and TCT No. T13117 (Exhibit H )

WHEREFORE, viewed in the light of the entire record, the judgment appealed from must be, as it is d) That mentioned parcels of land were submitted as guaranty in the Agreement of Counter-Guaranty
hereby REVERSED. In lieu thereof, a judgment is hereby rendered- with Chattel-Real Estate Mortgage executed on 24 October 1970 between Insurance Corporation of the
Philippines and Slobec Realty Corporation represented by Santiago Rivera (Exhibit 1);
1) Dismissing the complaint, with cost against plaintiffs;
e) That based on the Certificate of Sale issued by the Sheriff of the Province of Quezon in favor of
2) Ordering plaintiffs-appellees to vacate the subject properties; and Insurance Corporation of the Philippines it was able to transfer to itself the titles over the lots in
question, namely: TCT No. T-23705 (Exhibit M), TCT No. T 23706 (Exhibit N ), TCT No. T-23707 (Exhibit
0) and TCT No. T 23708 (Exhibit P);
3) Ordering plaintiffs-appellees to pay upon defendants' counterclaims:

f) That on 10 April 1975, the Insurance Corporation of the Philippines sold to PM Parts the immovables
a) To defendant-appellant PM Parts: (i) damages consisting of the value of the fruits in the
in question (per Exhibit 6 for PM Parts) and by reason thereof, succeeded in transferring unto itself the
subject parcels of land of which they were deprived in the sum of P26,000.00 and (ii)
titles over the lots in dispute, namely: per TCT No. T-24846 (Exhibit Q ), per TCT No. T-24847 (Exhibit
attorney's fees of P15,000.00
R ), TCT No. T-24848 (Exhibit), TCT No. T-24849 (Exhibit T );

b) To defendant-appellant Bormaheco: (i) expenses of litigation in the amount of P5,000.00


g) On 26 August l976, Mauricia Meer Vda. de Castillo' genther letter to Modesto N. Cervantes stating
and (ii) attorney's fees of P15,000.00.
that she and her children refused to comply with his demands (Exhibit V-2);

SO ORDERED.
h) That from at least the months of October, November and December 1970 and January 1971,
Modesto N. Cervantes was the Vice-President of Bormaheco, Inc. later President thereof, and also he is
The original complaint for annulment of title filed in the court a quo by herein petitioners included as party one of the Board of Directors of PM Parts; on the other hand, Atty. Martin M. De Guzman was the legal
defendants the Philippine Machinery Parts Manufacturing Co., Inc. (PM Parts), Insurance Corporation of the counsel of Bormaheco, Inc., later Executive Vice-President thereof, and who also is the legal counsel of
Philippines (ICP), Bormaheco, Inc., (Bormaheco) and Santiago M. Rivera (Rivera). A Second Amended Complaint Insurance Corporation of the Philippines and PM Parts; that Modesto N. Cervantes served later on as
was filed, this time impleading Santiago M. Rivera as party plaintiff. President of PM Parts, and that Atty. de Guzman was retained by Insurance Corporation of the
Philippines specifically for foreclosure purposes only;
During the pre-trial conference, the parties entered into the following stipulation of facts:
i) Defendant Bormaheco, Inc. on November 25, 1970 sold to Slobec Realty and Development, Inc.,
As between all parties: Plaintiff Buenaflor M. Castillo is the judicial administratrix of the estate of Felipe represented by Santiago Rivera, President, one (1) unit Caterpillar Tractor D-7 with Serial No. 281114
Castillo in Special Proceeding No. 4053, pending before Branch IX, CFI of Quezon (per Exhibit A) which evidenced by a contract marked Exhibit J and Exhibit I for Bormaheco, Inc.;
intestate proceedings was instituted by Mauricia Meer Vda. de Castillo, the previous administratrix of
the said proceedings prior to 1970 (per exhibits A-1 and A-2) which case was filed in Court way back in j) That the Surety Bond No. 14010 issued by co-defendant ICP was likewise secured by an Agreement
1964; with Counter-Guaranty with Real Estate Mortgage executed by Slobec Realty & Development, Inc.,
Mauricia Castillo Meer, Buenaflor Castillo, Bertilla Castillo, Victoria Castillo, Marietta Castillo and
b) The four (4) parcels of land described in paragraph 3 of the Complaint were originally covered by Leovina Castillo, as mortgagors in favor of ICP which document was executed and ratified before
TCT No. T-42104 and Tax Dec. No. 14134 with assessed value of P3,100.00; TCT No. T 32227 and Tax notary public Alberto R. Navoa of the City of Manila on October 24,1970;
Dec. No. 14132, with assessed value of P5,130,00; TCT No. T-31762 and Tax Dec. No. 14135, with
assessed value of P6,150.00; and TCT No. T-42103 with Tax Dec. No. 14133, with assessed value of k) That the property mortgaged consisted of four (4) parcels of land situated in Lucena City and
P3,580.00 (per Exhibits A-2 and B, B-1 to B-3 C, C-1 -to C3 covered by TCT Nos. T-13114, T13115,
T-13116 and T-13117 of the Register of Deeds of Lucena City;
c) That the above-enumerated four (4) parcels of land were the subject of the Deed of Extra-Judicial
Partition executed by the heirs of Felipe Castillo (per Exhibit D) and by virtue thereof the titles thereto
has (sic) been cancelled and in lieu thereof, new titles in the name of Mauricia Meer Vda. de Castillo
Set V Corporation Code * Doctrine of Peircing Corpo Veil Cases*Page 145 of 164

l) That the tractor sold by defendant Bormaheco, Inc. to Slobec Realty & Development, Inc. was now President of Bormaheco, Inc., sent his letter dated 9 August 1976 to Mauricia Meer Vda. de
delivered to Bormaheco, Inc. on or about October 2,1973, by Mr. Menandro Umali for purposes of Castillo (Exhibit V), demanding that she and her children should vacate the premises;
repair;
u) That the Caterpillar Crawler Tractor Model CAT D-7 which was received by Slobec Realty
m) That in August 1976, PM Parts notified Mrs. Mauricia Meer about its ownership and the assignment Development Corporation was actually reconditioned and repainted. " 2
of Mr. Petronilo Roque as caretaker of the subject property;
We cull the following antecedents from the decision of respondent Court of Appeals:
n) That plaintiff and other heirs are harvest fruits of the property (daranghita) which is worth no less
than Pl,000.00 per harvest. Plaintiff Santiago Rivera is the nephew of plaintiff Mauricia Meer Vda. de Castillo. The Castillo family
are the owners of a parcel of land located in Lucena City which was given as security for a loan from
As between plaintiffs and the Development Bank of the Philippines. For their failure to pay the amortization, foreclosure of the
defendant Bormaheco, Inc said property was about to be initiated. This problem was made known to Santiago Rivera, who
proposed to them the conversion into subdivision of the four (4) parcels of land adjacent to the
o) That on 25 November 1970, at Makati, Rizal, Same Rivera, in representation of the Slobec Realty & mortgaged property to raise the necessary fund. The Idea was accepted by the Castillo family and to
Development Corporation executed in favor of Bormaheco, Inc., represented by its Vice-President carry out the project, a Memorandum of Agreement (Exh. U p. 127, Record) was executed by and
Modesto N. Cervantes a Chattel Mortgage concerning one unit model CAT D7 Caterpillar Crawler between Slobec Realty and Development, Inc., represented by its President Santiago Rivera and the
Tractor as described therein as security for the payment in favor of the mortgagee of the amount of Castillo family. In this agreement, Santiago Rivera obliged himself to pay the Castillo family the sum of
P180,000.00 (per Exhibit K) that Id document was superseded by another chattel mortgage dated P70,000.00 immediately after the execution of the agreement and to pay the additional amount of
January 23, 1971 (Exhibit 15); P400,000.00 after the property has been converted into a subdivision. Rivera, armed with the
agreement, Exhibit U , approached Mr. Modesto Cervantes, President of defendant Bormaheco, and
proposed to purchase from Bormaheco two (2) tractors Model D-7 and D-8 Subsequently, a Sales
p) On 18 December 1970, at Makati, Rizal, the Bormaheco, Inc., represented by its Vice-President
Agreement was executed on December 28,1970 (Exh. J, p. 22, Record).
Modesto Cervantes and Slobec Realty Corporation represented by Santiago Rivera executed the sales
agreement concerning the sale of one (1) unit Model CAT D7 Caterpillar Crawler Tractor as described
therein for the amount of P230,000.00 (per Exhibit J) which document was superseded by the Sales On January 23, 1971, Bormaheco, Inc. and Slobec Realty and Development, Inc., represented by its
Agreement dated January 23,1971 (Exhibit 16); President, Santiago Rivera, executed a Sales Agreement over one unit of Caterpillar Tractor D-7 with
Serial No. 281114, as evidenced by the contract marked Exhibit '16'. As shown by the contract, the
price was P230,000.00 of which P50,000.00 was to constitute a down payment, and the balance of
q) Although it appears on the document entitled Chattel Mortgage (per Exhibit K) that it was executed
P180,000.00 payable in eighteen monthly installments. On the same date, Slobec, through Rivera,
on 25 November 1970, and in the document entitled Sales Agreement (per Exhibit J) that it was
executed in favor of Bormaheco a Chattel Mortgage (Exh. K, p. 29, Record) over the said equipment as
executed on 18 December 1970, it appears in the notarial register of the notary public who notarized
security for the payment of the aforesaid balance of P180,000.00. As further security of the
them that those two documents were executed on 11 December 1970. The certified xerox copy of the
aforementioned unpaid balance, Slobec obtained from Insurance Corporation of the Phil. a Surety
notarial register of Notary Public Guillermo Aragones issued by the Bureau of Records Management is
Bond, with ICP (Insurance Corporation of the Phil.) as surety and Slobec as principal, in favor of
hereto submitted as Exhibit BB That said chattel mortgage was superseded by another document
Bormaheco, as borne out by Exhibit '8' (p. 111, Record). The aforesaid surety bond was in turn secured
dated January 23, 1971;
by an Agreement of Counter-Guaranty with Real Estate Mortgage (Exhibit I, p. 24, Record) executed by
Rivera as president of Slobec and Mauricia Meer Vda. de Castillo, Buenaflor Castillo Umali, Bertilla
r) That on 23 January 1971, Slobec Realty Development Corporation, represented by Santiago Rivera, Castillo-Rada, Victoria Castillo, Marietta Castillo and Leovina Castillo Jalbuena, as mortgagors and
received from Bormaheco, Inc. one (1) tractor Caterpillar Model D-7 pursuant to Invoice No. 33234 Insurance Corporation of the Philippines (ICP) as mortgagee. In this agreement, ICP guaranteed the
(Exhibits 9 and 9-A, Bormaheco, Inc.) and delivery receipt No. 10368 (per Exhibits 10 and 10-A for obligation of Slobec with Bormaheco in the amount of P180,000.00. In giving the bond, ICP required
Bormaheco, Inc that the Castillos mortgage to them the properties in question, namely, four parcels of land covered by
TCTs in the name of the aforementioned mortgagors, namely TCT Nos. 13114, 13115, 13116 and
s) That on 28 September 1973, Atty. Martin M. de Guzman, as counsel of Insurance Corporation of the 13117 all of the Register of Deeds for Lucena City.
Philippines purchased at public auction for said corporation the four (4) parcels of land subject of tills
case (per Exhibit L), and which document was presented to the Register of Deeds on 1 October 1973; On the occasion of the execution on January 23, 1971, of the Sales Agreement Exhibit '16', Slobec,
represented by Rivera received from Bormaheco the subject matter of the said Sales Agreement,
t) Although it appears that the realties in issue has (sic) been sold by Insurance Corporation of the namely, the aforementioned tractor Caterpillar Model D-7 as evidenced by Invoice No. 33234 (Exhs. 9
Philippines in favor of PM Parts on 1 0 April 1975, Modesto N. Cervantes, formerly Vice- President and and 9-A, p. 112, Record) and Delivery Receipt No. 10368 (Exhs. 10 and 10-A, p. 113). This tractor was
Set V Corporation Code * Doctrine of Peircing Corpo Veil Cases*Page 146 of 164

known by Rivera to be a reconditioned and repainted one [Stipulation of Facts, Pre-trial Order, par. After trial, the court a quo rendered judgment, with the following decretal portion:
(u)].
WHEREFORE, judgment is hereby rendered in favor of the plaintiffs and against the defendants,
Meanwhile, for violation of the terms and conditions of the Counter-Guaranty Agreement (Exh. 1), the declaring the following documents:
properties of the Castillos were foreclosed by ICP As the highest bidder with a bid of P285,212.00, a
Certificate of Sale was issued by the Provincial Sheriff of Lucena City and Transfer Certificates of Title Agreement of Counter-Guaranty with Chattel-Real Estate Mortgage dated October
over the subject parcels of land were issued by the Register of Deeds of Lucena City in favor of ICP 24,1970 (Exhibit 1);
namely, TCT Nos. T-23705, T 23706, T-23707 and T-23708 (Exhs. M to P, pp. 38-45). The mortgagors
had one (1) year from the date of the registration of the certificate of sale, that is, until October 1,
Sales Agreement dated December 28, 1970 (Exhibit J)
1974, to redeem the property, but they failed to do so. Consequently, ICP consolidated its ownership
over the subject parcels of land through the requisite affidavit of consolidation of ownership dated
October 29, 1974, as shown in Exh. '22'(p. 138, Rec.). Pursuant thereto, a Deed of Sale of Real Estate Chattel Mortgage dated November 25, 1970 (Exhibit K)
covering the subject properties was issued in favor of ICP (Exh. 23, p. 139, Rec.).
Sales Agreement dated January 23, 1971 (Exhibit 16);
On April 10, 1975, Insurance Corporation of the Phil. ICP sold to Phil. Machinery Parts Manufacturing
Co. (PM Parts) the four (4) parcels of land and by virtue of said conveyance, PM Parts transferred unto Chattel Mortgage dated January 23, 1971 (Exhibit 17);
itself the titles over the lots in dispute so that said parcels of land are now covered by TCT Nos. T-
24846, T-24847, T-24848 and T-24849 (Exhs. Q-T, pp. 46-49, Rec.).
Certificate of Sale dated September 28, 1973 executed by the Provincial Sheriff of
Quezon in favor of Insurance Corporation of the Philippines (Exhibit L);
Thereafter, PM Parts, through its President, Mr. Modesto Cervantes, sent a letter dated August 9,1976
addressed to plaintiff Mrs. Mauricia Meer Castillo requesting her and her children to vacate the subject
null and void for being fictitious, spurious and without consideration. Consequently, Transfer
property, who (Mrs. Castillo) in turn sent her reply expressing her refusal to comply with his demands.
Certificates of Title Nos. T 23705, T-23706, T23707 and T-23708 (Exhibits M, N, O and P) issued in the
name of Insurance Corporation of the Philippines, are likewise null and void.
On September 29, 1976, the heirs of the late Felipe Castillo, particularly plaintiff Buenaflor M. Castillo
Umali as the appointed administratrix of the properties in question filed an action for annulment of
The sale by Insurance Corporation of the- Philippines in favor of defendant Philippine Machinery Parts
title before the then Court of First Instance of Quezon and docketed thereat as Civil Case No. 8085.
Manufacturing Co., Inc., over Id four (4) parcels of land and Transfer Certificates of Title Nos. T 24846,
Thereafter, they filed an Amended Complaint on January 10, 1980 (p. 444, Record). On July 20, 1983,
T-24847, T-24848 and T-24849 subsequently issued by virtue of said sale in the name of Philippine
plaintiffs filed their Second Amended Complaint, impleading Santiago M. Rivera as a party plaintiff (p.
Machinery Parts Manufacturing Co., Inc., are similarly declared null and void, and the Register of
706, Record). They contended that all the aforementioned transactions starting with the Agreement of
Deeds of Lucena City is hereby directed to issue, in lieu thereof, transfer certificates of title in the
Counter-Guaranty with Real Estate Mortgage (Exh. I), Certificate of Sale (Exh. L) and the Deeds of
names of the plaintiffs, except Santiago Rivera.
Authority to Sell, Sale and the Affidavit of Consolidation of Ownership (Annexes F, G, H, I) as well as
the Deed of Sale (Annexes J, K, L and M) are void for being entered into in fraud and without the
consent and approval of the Court of First Instance of Quezon, (Branch IX) before whom the Orders the defendants jointly and severally to pay the plaintiffs moral damages in the sum of
administration proceedings has been pending. Plaintiffs pray that the four (4) parcels of land subject P10,000.00, exemplary damages in the amount of P5,000.00, and actual litigation expenses in the
hereof be declared as owned by the estate of the late Felipe Castillo and that all Transfer Certificates sum of P6,500.00.
of Title Nos. 13114,13115,13116,13117, 23705, 23706, 23707, 23708, 24846, 24847, 24848 and
24849 as well as those appearing as encumbrances at the back of the certificates of title mentioned Defendants are likewise ordered to pay the plaintiffs, jointly and severally, the sum of P10,000.00 for
be declared as a nullity and defendants to pay damages and attorney's fees (pp. 71071 1, Record). and as attomey's fees. With costs against the defendants.SO ORDERED. 4

In their amended answer, the defendants controverted the complaint and alleged, by way of As earlier stated, respondent court reversed the aforequoted decision of the trial court and rendered the
affirmative and special defenses that the complaint did not state facts sufficient to state a cause of judgment subject of this petition-
action against defendants; that plaintiffs are not entitled to the reliefs demanded; that plaintiffs are
estopped or precluded from asserting the matters set forth in the Complaint; that plaintiffs are guilty
Petitioners contend that respondent Court of Appeals erred:
of laches in not asserting their alleged right in due time; that defendant PM Parts is an innocent
purchaser for value and relied on the face of the title before it bought the subject property (p. 744,
Record). 3
Set V Corporation Code * Doctrine of Peircing Corpo Veil Cases*Page 147 of 164

1. In holding and finding that the actions entered into between petitioner Rivera with Cervantes are all There is absolute simulation, which renders the contract null and void, when the parties do not intend to be
fair and regular and therefore binding between the parties thereto; bound at all by the same. 9 The basic characteristic of this type of simulation of contract is the fact that the
apparent contract is not really desired or intended to either produce legal effects or in any way alter the
2. In reversing the decision of the lower court, not only based on erroneous conclusions of facts, juridical situation of the parties. The subsequent act of Rivera in receiving and making use of the tractor
erroneous presumptions not supported by the evidence on record but also, holding valid and binding subject matter of the Sales Agreement and Chattel Mortgage, and the simultaneous issuance of a surety bond
the supposed payment by ICP of its obligation to Bormaheco, despite the fact that the surety bond in favor of Bormaheco, concomitant with the execution of the Agreement of Counter-Guaranty with Chattel/Real
issued it had already expired when it opted to foreclose extrajudically the mortgage executed by the Estate Mortgage, conduce to the conclusion that petitioners had every intention to be bound by these
petitioners; contracts. The occurrence of these series of transactions between petitioners and private respondents is a
strong indication that the parties actually intended, or at least expected, to exact fulfillment of their respective
obligations from one another.
3. In aside the finding of the lower court that there was necessity to pierce the veil of corporate
existence; and
Neither will an allegation of fraud prosper in this case where petitioners failed to show that they were induced
5
to enter into a contract through the insidious words and machinations of private respondents without which the
4. In reversing the decision of the lower court of affirming the same
former would not have executed such contract. To set aside a document solemnly executed and voluntarily
delivered, the proof of fraud must be clear and convincing. 10 We are not persuaded that such quantum of proof
I. Petitioners aver that the transactions entered into between Santiago M. Rivera, as President of Slobec Realty exists in the case at bar.
and Development Company (Slobec) and Mode Cervantes, as Vice-President of Bormaheco, such as the Sales
Agreement, 6 Chattel Mortgage 7 and the Agreement of Counter-Guaranty with Chattel/Real Estate
The fact that it was Bormaheco which paid the premium for the surety bond issued by ICP does not per
Mortgage, 8 are all fraudulent and simulated and should, therefore, be declared nun and void. Such allegation is
se affect the validity of the bond. Petitioners themselves admit in their present petition that Rivera executed a
premised primarily on the fact that contrary to the stipulations agreed upon in the Sales Agreement (Exhibit J),
Deed of Sale with Right of Repurchase of his car in favor of Bormaheco and agreed that a part of the proceeds
Rivera never made any advance payment, in the alleged amount of P50,000.00, to Bormaheco; that the tractor
thereof shall be used to pay the premium for the bond. 11 In effect, Bormaheco accepted the payment of the
was received by Rivera only on January 23, 1971 and not in 1970 as stated in the Chattel Mortgage (Exhibit K);
premium as an agent of ICP The execution of the deed of sale with a right of repurchase in favor of Bormaheco
and that when the Agreement of Counter-Guaranty with Chattel/Real Estate Mortgage was executed on October
under such circumstances sufficiently establishes the fact that Rivera recognized Bormaheco as an agent of ICP
24, 1970, to secure the obligation of ICP under its surety bond, the Sales Agreement and Chattel Mortgage had
Such payment to the agent of ICP is, therefore, binding on Rivera. He is now estopped from questioning the
not as yet been executed, aside from the fact that it was Bormaheco, and not Rivera, which paid the premium
validity of the suretyship contract.
for the surety bond issued by ICP

II. Under the doctrine of piercing the veil of corporate entity, when valid grounds therefore exist, the legal
At the outset, it will be noted that petitioners submission under the first assigned error hinges purely on
fiction that a corporation is an entity with a juridical personality separate and distinct from its members or
questions of fact. Respondent Court of Appeals made several findings to the effect that the questioned
stockholders may be disregarded. In such cases, the corporation will be considered as a mere association of
documents are valid and binding upon the parties, that there was no fraud employed by private respondents in
persons. The members or stockholders of the corporation will be considered as the corporation, that is, liability
the execution thereof, and that, contrary to petitioners' allegation, the evidence on record reveals that
will attach directly to the officers and stockholders. 12 The doctrine applies when the corporate fiction is used to
petitioners had every intention to be bound by their undertakings in the various transactions had with private
defeat public convenience, justify wrong, protect fraud, or defend crime, 13 or when it is made as a shield to
respondents. It is a general rule in this jurisdiction that findings of fact of said appellate court are final and
confuse the legitimate issues 14 or where a corporation is the mere alter ego or business conduit of a person, or
conclusive and, thus, binding on this Court in the absence of sufficient and convincing proof, inter alia, that the
where the corporation is so organized and controlled and its affairs are so conducted as to make it merely an
former acted with grave abuse of discretion. Under the circumstances, we find no compelling reason to deviate
instrumentality, agency, conduit or adjunct of another corporation. 15
from this long-standing jurisprudential pronouncement.

In the case at bar, petitioners seek to pierce the V621 Of corporate entity of Bormaheco, ICP and PM Parts,
In addition, the alleged failure of Rivera to pay the consideration agreed upon in the Sales Agreement, which
alleging that these corporations employed fraud in causing the foreclosure and subsequent sale of the real
clearly constitutes a breach of the contract, cannot be availed of by the guilty party to justify and support an
properties belonging to petitioners While we do not discount the possibility of the existence of fraud in the
action for the declaration of nullity of the contract. Equity and fair play dictates that one who commits a breach
foreclosure proceeding, neither are we inclined to apply the doctrine invoked by petitioners in granting the
of his contract may not seek refuge under the protective mantle of the law.
relief sought. It is our considered opinion that piercing the veil of corporate entity is not the proper remedy in
order that the foreclosure proceeding may be declared a nullity under the circumstances obtaining in the legal
The evidence of record, on an overall calibration, does not convince us of the validity of petitioners' contention case at bar.
that the contracts entered into by the parties are either absolutely simulated or downright fraudulent.
In the first place, the legal corporate entity is disregarded only if it is sought to hold the officers and
stockholders directly liable for a corporate debt or obligation. In the instant case, petitioners do not seek to
Set V Corporation Code * Doctrine of Peircing Corpo Veil Cases*Page 148 of 164

impose a claim against the individual members of the three corporations involved; on the contrary, it is these Surety Bond No. B-1401 0 which was issued by ICP in favor of Bormaheco, wherein ICP and Slobec undertook to
corporations which desire to enforce an alleged right against petitioners. Assuming that petitioners were indeed guarantee the payment of the balance of P180,000.00 payable in eighteen (18) monthly installments on one
defrauded by private respondents in the foreclosure of the mortgaged properties, this fact alone is not, under unit of Model CAT D-7 Caterpillar Crawler Tractor, pertinently provides in part as follows:
the circumstances, sufficient to justify the piercing of the corporate fiction, since petitioners do not intend to
hold the officers and/or members of respondent corporations personally liable therefor. Petitioners are merely 1. The liability of INSURANCE CORPORATION OF THE PHILIPPINES, under this BOND will expire Twelve (I
seeking the declaration of the nullity of the foreclosure sale, which relief may be obtained without having to 2) months from date hereof. Furthermore, it is hereby agreed and understood that the INSURANCE
disregard the aforesaid corporate fiction attaching to respondent corporations. Secondly, petitioners failed to CORPORATION OF THE PHILIPPINES will not be liable for any claim not presented in writing to the
establish by clear and convincing evidence that private respondents were purposely formed and operated, and Corporation within THIRTY (30) DAYS from the expiration of this BOND, and that the obligee hereby
thereafter transacted with petitioners, with the sole intention of defrauding the latter. waives his right to bring claim or file any action against Surety and after the termination of one (1)
year from the time his cause of action accrues. 18
The mere fact, therefore, that the businesses of two or more corporations are interrelated is not a justification
for disregarding their separate personalities, 16 absent sufficient showing that the corporate entity was The surety bond was dated October 24, 1970. However, an annotation on the upper part thereof states: "NOTE:
purposely used as a shield to defraud creditors and third persons of their rights. EFFECTIVITY DATE OF THIS BOND SHALL BE ON JANUARY 22, 1971." 19

III. The main issue for resolution is whether there was a valid foreclosure of the mortgaged properties by ICP On the other hand, the Sales Agreement dated January 23, 1971 provides that the balance of P180,000.00 shall
Petitioners argue that the foreclosure proceedings should be declared null and void for two reasons, viz.: (1) no be payable in eighteen (18) monthly installments. 20 The Promissory Note executed by Slobec on even date in
written notice was furnished by Bormaheco to ICP anent the failure of Slobec in paying its obligation with the favor of Bormaheco further provides that the obligation shall be payable on or before February 23, 1971 up to
former, plus the fact that no receipt was presented to show the amount allegedly paid by ICP to Bormaheco; July 23, 1972, and that non-payment of any of the installments when due shall make the entire obligation
and (b) at the time of the foreclosure of the mortgage, the liability of ICP under the surety bond had already immediately due and demandable. 21
expired.

It is basic that liability on a bond is contractual in nature and is ordinarily restricted to the obligation expressly
Respondent court, in finding for the validity of the foreclosure sale, declared: assumed therein. We have repeatedly held that the extent of a surety's liability is determined only by the
clause of the contract of suretyship as well as the conditions stated in the bond. It cannot be extended by
Now to the question of whether or not the foreclosure by the ICP of the real estate mortgage was in implication beyond the terms the contract. 22
the exercise of a legal right, We agree with the appellants that the foreclosure proceedings instituted
by the ICP was in the exercise of a legal right. First, ICP has in its favor the legal presumption that it Fundamental likewise is the rule that, except where required by the provisions of the contract, a demand or
had indemnified Bormaheco by reason of Slobec's default in the payment of its obligation under the notice of default is not required to fix the surety's liability. 23 Hence, where the contract of suretyship stipulates
Sales Agreement, especially because Bormaheco consented to ICPs foreclosure of the mortgage. This that notice of the principal's default be given to the surety, generally the failure to comply with the condition
presumption is in consonance with pars. R and Q Section 5, Rule 5, * New Rules of Court which will prevent recovery from the surety. There are certain instances, however, when failure to comply with the
provides that it is disputably presumed that private transactions have been fair and regular. likewise, it condition will not extinguish the surety's liability, such as a failure to give notice of slight defaults, which are
is disputably presumed that the ordinary course of business has been followed: Second, ICP had the waived by the obligee; or on mere suspicion of possible default; or where, if a default exists, there is excuse or
right to proceed at once to the foreclosure of the mortgage as mandated by the provisions of Art. 2071 provision in the suretyship contract exempting the surety for liability therefor, or where the surety already has
Civil Code for these further reasons: Slobec, the principal debtor, was admittedly insolvent; Slobec's knowledge or is chargeable with knowledge of the default. 24
obligation becomes demandable by reason of the expiration of the period of payment; and its
authorization to foreclose the mortgage upon Slobec's default, which resulted in the accrual of ICPS
In the case at bar, the suretyship contract expressly provides that ICP shag not be liable for any claim not filed
liability to Bormaheco. Third, the Agreement of Counter-Guaranty with Real Estate Mortgage (Exh. 1)
in writing within thirty (30) days from the expiration of the bond. In its decision dated May 25 1987, the court a
expressly grants to ICP the right to foreclose the real estate mortgage in the event of 'non-payment or
quocategorically stated that '(n)o evidence was presented to show that Bormaheco demanded payment from
non-liquidation of the entire indebtedness or fraction thereof upon maturity as stipulated in the
ICP nor was there any action taken by Bormaheco on the bond posted by ICP to guarantee the payment of
contract'. This is a valid and binding stipulation in the absence of showing that it is contrary to law,
plaintiffs obligation. There is nothing in the records of the proceedings to show that ICP indemnified Bormaheco
morals, good customs, public order or public policy. (Art. 1306, New Civil Code). 17
for the failure of the plaintiffs to pay their obligation. " 25 The failure, therefore, of Bormaheco to notify ICP in
writing about Slobec's supposed default released ICP from liability under its surety bond. Consequently, ICP
1. Petitioners asseverate that there was no notice of default issued by Bormaheco to ICP which would have could not validly foreclose that real estate mortgage executed by petitioners in its favor since it never incurred
entitled Bormaheco to demand payment from ICP under the suretyship contract. any liability under the surety bond. It cannot claim exemption from the required written notice since its case
does not fall under any of the exceptions hereinbefore enumerated.
Set V Corporation Code * Doctrine of Peircing Corpo Veil Cases*Page 149 of 164

Furthermore, the allegation of ICP that it has paid Bormaheco is not supported by any documentary evidence. qualification in the stipulation of facts submitted by the parties before the trial court. Hence, the defense of
Section 1, Rule 131 of the Rules of Court provides that the burden of evidence lies with the party who asserts good faith may not be resorted to by private respondent PM Parts which is charged with knowledge of the true
an affirmative allegation. Since ICP failed to duly prove the fact of payment, the disputable presumption that relations existing between Bormaheco, ICP and herein petitioners. Accordingly, the transfer certificates of title
private transactions have been fair and regular, as erroneously relied upon by respondent Court of Appeals, issued in its name, as well as the certificate of sale, must be declared null and void since they cannot be
finds no application to the case at bar. considered altogether free of the taint of bad faith.

2. The liability of a surety is measured by the terms of his contract, and, while he is liable to the full extent WHEREFORE, the decision of respondent Court of Appeals is hereby REVERSED and SET ASIDE, and judgment is
thereof, such liability is strictly limited to that assumed by its terms. 26 While ordinarily the termination of a hereby rendered declaring the following as null and void: (1) Certificate of Sale, dated September 28,1973,
surety's liability is governed by the provisions of the contract of suretyship, where the obligation of a surety is, executed by the Provincial Sheriff of Quezon in favor of the Insurance Corporation of the Philippines; (2)
under the terms of the bond, to terminate at a specified time, his obligation cannot be enlarged by an Transfer Certificates of Title Nos. T-23705, T-23706, T-23707 and T-23708 issued in the name of the Insurance
unauthorized extension thereof. 27This is an exception to the general rule that the obligation of the surety Corporation of the Philippines; (3) the sale by Insurance Corporation of the Philippines in favor of Philippine
continues for the same period as that of the principal debtor. 28 Machinery Parts Manufacturing Co., Inc. of the four (4) parcels of land covered by the aforesaid certificates of
title; and (4) Transfer Certificates of Title Nos. T-24846, T-24847, T-24848 and T24849 subsequently issued by
It is possible that the period of suretyship may be shorter than that of the principal obligation, as where the virtue of said sale in the name of the latter corporation.
principal debtor is required to make payment by installments. 29 In the case at bar, the surety bond issued by
ICP was to expire on January 22, 1972, twelve (1 2) months from its effectivity date, whereas Slobec's The Register of Deeds of Lucena City is hereby directed to cancel Transfer Certificates of Title Nos. T-24846, T-
installment payment was to end on July 23, 1972. Therefore, while ICP guaranteed the payment by Slobec of 24847, T24848 and T-24849 in the name of Philippine Machinery Parts Manufacturing Co., Inc. and to issue in
the balance of P180,000.00, such guaranty was valid only for and within twelve (1 2) months from the date of lieu thereof the corresponding transfer certificates of title in the name of herein petitioners, except Santiago
effectivity of the surety bond, or until January 22, 1972. Thereafter, from January 23, 1972 up to July 23, 1972, Rivera.The foregoing dispositions are without prejudice to such other and proper legal remedies as may be
the liability of Slobec became an unsecured obligation. The default of Slobec during this period cannot be a available to respondent Bormaheco, Inc. against herein petitioners.SO ORDERED.
valid basis for the exercise of the right to foreclose by ICP since its surety contract had already been
terminated. Besides, the liability of ICP was extinguished when Bormaheco failed to file a written claim against THIRD DIVISION G.R. Nos. 121662-64 July 6, 1999
it within thirty (30) days from the expiration of the surety bond. Consequently, the foreclosure of the mortgage,
after the expiration of the surety bond under which ICP as surety has not incurred any liability, should be
VLASON ENTERPRISES CORPORATION, petitioner, vs.
declared null and void.
COURT OF APPEALS and DURAPROOF SERVICES, represented by its General Manager, Cesar Urbino
Sr.,respondents.
3. Lastly, it has been held that where The guarantor holds property of the principal as collateral surety for his
personal indemnity, to which he may resort only after payment by himself, until he has paid something as such
PANGANIBAN, J.:
guarantor neither he nor the creditor can resort to such collaterals. 30

The Agreement of Counter-Guaranty with Chattel/Real Estate Mortgage states that it is being issued for and in
consideration of the obligations assumed by the Mortgagee-Surety Company under the terms and conditions of
ICP Bond No. 14010 in behalf of Slobec Realty Development Corporation and in favor of Bormaheco, Summons to a domestic or resident corporation should be served on officers, agents or employees, who are
Inc. 31 There is no doubt that said Agreement of Counter-Guaranty is issued for the personal indemnity of ICP responsible enough to warrant the presumption that they will transmit to the corporation notice of the filing of
Considering that the fact of payment by ICP has never been established, it follows, pursuant to the doctrine the action against it. Rules on the service of motions should be liberally construed in order to promote the ends
above adverted to, that ICP cannot foreclose on the subject properties, of substantial justice. A rigid application that will result in the manifest injustice should be avoided. A default
judgment against several defendants cannot affect the rights of one who was never declared in default. In any
event, such judgment cannot include award not prayed for in the complaint, even if proven ex parte.
IV. Private respondent PM Parts posits that it is a buyer in good faith and, therefore, it acquired a valid title over
the subject properties. The submission is without merit and the conclusion is specious
The Case
We have stated earlier that the doctrine of piercing the veil of corporate fiction is not applicable in this case.
However, its inapplicability has no bearing on the good faith or bad faith of private respondent PM Parts. It must These principles were used by this Court in resolving this Petition for Review on Certiorari before us, assailing
be noted that Modesto N. Cervantes served as Vice-President of Bormaheco and, later, as President of PM Parts. the July 19, 1993 Decision 1 and the August 15 Resolution 2 promulgated by the Court of Appeals. The assailed
On this fact alone, it cannot be said that PM Parts had no knowledge of the aforesaid several transactions Decision disposed as follows: 3
executed between Bormaheco and petitioners. In addition, Atty. Martin de Guzman, who is the Executive Vice-
President of Bormaheco, was also the legal counsel of ICP and PM Parts. These facts were admitted without
Set V Corporation Code * Doctrine of Peircing Corpo Veil Cases*Page 150 of 164

ACCORDINGLY, in view of the foregoing disquisitions, all the three (3) consolidated petitions While seizure proceedings were ongoing, La Union was hit by three typhoons, and the vessel ran aground and
forcertiorari are hereby GRANTED. was abandoned. On June 8, 1989, its authorized representative, Frank Cadacio, entered into a salvage
agreement with private respondent to secure and repair the vessel at the agreed consideration of $1 million
THE assailed Order of respondent Judge Arsenio Gonong of the Regional Trial Court of Manila, Branch and "fifty percent (50%) [of] the cargo after all expenses, cost and taxes." 6
8, dated April 5, 1991, in the first petition for certiorari (CA-G.R. SP No. 24669); the assailed Order of
Judge Bernardo Pardo, Executive Judge of the Regional Trial Court of Manila, Branch 8, dated July 6, Finding that no fraud was committed, the District Collector of Customs, Aurelio M. Quiray, lifted the warrant of
1992, in the second petition for certiorari (CA-G.R. SP No. 28387); and finally, the assailed order or seizure on July 16, 1989. 7 However, in a Second Indorsement dated November 11, 1989, then Customs
Resolution en banc of the respondent Court of Tax Appeals Judges Ernesto Acosta, Ramon de Veyra Commissioner Salvador M. Mison declined to issue a clearance for Quiray's Decision; instead, he forfeited the
and Manuel Gruba, under date of October 5, 1992, in the third petition for certiorari(CA-G.R. SP No. vessel and its cargo in accordance with Section 2530 of the Tariff and Customs Code. 8 Accordingly, acting
29317) are all hereby NULLIFIED and SET ASIDE thereby giving way to the entire decision dated District Collector of Customs John S. Sy issued a Decision decreeing the forfeiture and the sale of the cargo in
February 18, 1991 of the respondent Regional Trial Court of Manila, Branch 8, in Civil Case No. 89- favor of the government. 9
51451 which remains valid, final and executory, if not yet wholly executed.
To enforce its preferred salvor's lien, herein Private Respondent Duraproof Services filed with the Regional Trial
THE writ of preliminary injunction heretofore issued by this Court on March 6, 1992 and reiterated on Court of Manila a Petition for Certiorari, Prohibition and Mandamus 10 assailing the actions of Commissioner
July 22, 1992 and this date against the named respondents specified in the dispositive portion of the Mison and District Collector Sy. Also impleaded as respondents were PPA Representative Silverio Mangaoang
judgment of the respondent Regional Trial Court of Manila, Branch 8 in the first petition for certiorari, and Med Line Philippines, Inc.
which remains valid, existing and enforceable, is hereby MADE PERMANENT without prejudice (1) to
the [private respondent's] remaining unpaid obligations to the herein party-intervenor in accordance On January 10, 1989, private respondent amended its Petition 11 to include former District Collector Quiray; PPA
with the Compromise Agreement or in connection with the decision of the respondent lower court in Port Manager Adolfo Ll. Amor Jr; Petitioner Vlason Enterprises as represented by its president, Vicente
CA-G.R. SP No. 24669 and (2) to the government, in relation to the forthcoming decision of the Angliongto; Singkong Trading Company as represented by Atty. Eddie Tamondong; Banco Du Brasil; Dusit
respondent Court of Tax Appeals on the amount of taxes, charges, assessments or obligations that are International Co., Inc.; Thai-Nan Enterprises Ltd. and Thai-United Trading Co., Ltd. 12 In both Petitions, private
due, as totally secured and fully guaranteed payment by the [private respondent's] bond, subject to respondent plainly failed to include any allegation pertaining to petitioner, or any prayer for relief against
the relevant rulings of the Department of Finance and other prevailing laws and jurisprudence. it.1wphi1.nt

The assailed Resolution ruled: Summonses for the amended Petition were served on Atty. Joseph Capuyan for Med Line Philippines: Angliongto
(through his secretary, Betty Bebero), Atty. Tamondong and Commissioner Mison. 13 Upon motion of the private
ACCORDINGLY, in the light of the foregoing disquisitions, as well as considering these clarifications, the respondent, the trial court allowed summons by publication to be served upon the alien defendants who were
three (3) motions aforementioned are hereby DENIED. not residents and had no direct representatives in the country. 14

The Facts On January 29, 1990, private respondent moved to declare respondents in default, but the trial court denied
the motion in its February 23, 1990 Order, 15 because Mangaoang and Amor had jointly filed a Motion to
Poro Point Shipping Services, then acting as the local agent of Omega Sea Transport Company of Honduras & Dismiss, while Mison and Med Line had moved separately for an extension to file a similar motion. 16 Later it
Panama, a Panamanian company, (hereafter referred to as Omega), requested permission for its vessel M/V rendered an Order dated July 2, 1990, giving due course to the motions to dismiss filed by Mangaoang and
Star Ace, which had engine trouble, to unload its cargo and to store it at the Philippine Ports Authority (PPA) Amor on the ground of litis pendentia, and by the commissioner and district collector of customs on the ground
compound in San Fernando, La Union while awaiting transshipment to Hongkong. The request was approved by of lack of jurisdiction. 17 In another Order, the trial court dismissed the action against Med Line Philippines on
the Bureau of Customs. 4 Despite the approval, the customs personnel boarded the vessel when it docked on the ground of litis pendentia. 18
January 7, 1989, on suspicion that it was the hijacked M/V Silver Med owned by Med Line Philippines Co., and
that its cargo would be smuggled into the country. 5 The district customs collector seized said vessel and its On two other occasions, private respondent again moved to declare the following in default: petitioner, Quiray,
cargo pursuant to Section 2301, Tariff and Customs Code. A notice of hearing of SFLU Seizure Identification No. Sy and Mison on March 26, 1990; 19 and Banco Du Brazil, Dusit International Co., Inc., Thai-Nan Enterprises Ltd.
3-89 was served on its consignee, Singkong Trading Co. of Hongkong, and its shipper, Dusit International Co., and Thai-United Trading Co., Ltd. on August 24, 1990. 20 There is no record, however, that the trial court acted
Ltd. of Thailand. upon the motions. On September 18, 1990, petitioner filed another Motion for leave to amend the
petition, 21 alleging that its counsel failed to include the following "necessary and/or indispensable parties":
Omega represented by Cadacio; and M/V Star Ace represented by Capt. Nahon Rada, relief captain. Aside from
impleading these additional respondents, private respondent also alleged in the Second (actually, third)
Amended
Set V Corporation Code * Doctrine of Peircing Corpo Veil Cases*Page 151 of 164

Petition 22 that the owners of the vessel intended to transfer and alienate their rights and interests over the c. Preservation, securing and guarding fees on the vessel in the amount of $225,000.00;
vessel and its cargo, to the detriment of the private respondent.
d. Maintenance fees in the amount P2,685,000.00;
The trial court granted leave to private respondent to amend its Petition, but only to exclude the customs
commissioner and the district collector. 23 Instead, private respondent filed the "Second Amended Petition with e. Salaries of the crew from August 16, 1989 to December 1989 in the amount of $43,000.00
Supplemental Petition" against Singkong Trading Company; and Omega and M/V Star Ace, 24 to which Cadacio and unpaid salaries from January 1990 up to the present.
and Rada filed a Joint Answer. 25

f. Attorney's fees in the amount of P656,000.00;


Declared in default in an Order issued by the trial court on January 23, 1991, were the following: Singkong
Trading Co., Commissioner Mison, M/V Star Ace and Omega. 26 Private respondent filed, and the trial court
3. [Vlason] Enterprises to pay [private respondent] in the amount of P3,000,000.00 for damages;
granted, an ex parte Motion to present evidence against the defaulting respondents. 27 Only private
respondent, Atty. Tamondong, Commissioner Mison, Omega and M/V Star Ace appeared in the next pretrial
hearing; thus, the trial court declared the other respondents in default and allowed private respondent to 4. Banco [Du] Brazil to pay [private respondent] in the amount of $300,000.00 in damages; and
present evidence against them. 28 Cesar Urbino, general manager of private respondent, testified and adduced finally,
evidence against the other respondents, including herein petitioner. As regards petitioner, he declared: "Vlason
Enterprises represented by Atty. Sy and Vicente Angliongto thru constant intimidation and harassment of 5. Costs of [s]uit.
utilizing the PPA Management of San Fernando, La Union . . . further delayed, and [private respondent] incurred
heavy overhead expenses due to direct and incidental expenses . . . causing irreparable damages of about
Subsequently, upon the motion of Omega, Singkong Trading Co. and private respondent, the trial court
P3,000,000 worth of ship tackles, rigs, and appurtenances including radar antennas and apparatuses, which
approved a Compromise Agreement 31 among the movants, reducing by 20 percent the amounts adjudged. For
were taken surreptitiously by persons working for Vlason Enterprises or its agents[.] 29
their part, respondents-movants agreed not to appeal the Decision. 32 On March 8, 1991, private respondent
moved for the execution of judgment, claiming that the trial court Decision had already become final and
On December 29, 1990, private respondent and Rada, representing Omega, entered into a Memorandum of executory. 33 The Motion was granted 34 and a Writ of Execution was issued. 35 To satisfy the Decision, Sheriffs
Agreement stipulating that Rada would write and notify Omega regarding the demand for salvage fees of Jorge Victorino, Amado Sevilla and Dionisio Camagon were deputized on March 13, 1991 to levy and to sell on
private respondent; and that if Rada did not receive any instruction from his principal, he would assign the execution the defendant's vessel and personal property.
vessel in favor of the salvor. 30

On March 14, 1991, petitioner filed, by special appearance, a Motion for Reconsideration on the grounds that it
On February 18, 1991, the trial court disposed as follows: was allegedly not impleaded as a defendant, served summons or declared in default; that private respondent
was not authorized to present evidence against it in default; that the judgment in default was fatally defective,
WHEREFORE, IN VIEW OF THE FOREGOING, based on the allegations, prayer and evidence adduced, because private respondent had not paid filing fees for the award; and that private respondent had not prayed
both testimonial and documentary, the Court is convinced, that, indeed, defendants/respondents are for such award. 36 Private respondent opposed the Motion, arguing that it was a mere scrap of paper due to its
liable to [private respondent] in the amount as prayed for in the petition for which it renders judgment defective notice of hearing.
as follows:
On March 18, 1991, the Bureau of Customs also filed an ex parte Motion to recall the execution, and to quash
1. Respondent M/V Star Ace, represented by Capt. Nahum Rada, [r]elief [c]aptain of the vessel and the notice of levy and the sale on execution. 37 Despite this Motion, the auction sale was conducted on March
Omega Sea Transport Company, Inc., represented by Frank Cadacio[,] is ordered to refrain from 21, 1991 by Sheriff Camagon, with private respondent submitting the winning
alienating or transferring the vessel M/V Star Ace to any third parties; bid. 38 The trial court ordered the deputy sheriffs to cease and desist from implementing the Writ of Execution
and from levying on the personal property of the defendants. 39 Nevertheless, Sheriff Camagon issued the
corresponding Certificate of Sale on March 27, 1991. 40
2. Singkong Trading Company to pay the following:

On April 12, 1991, 41 private respondent filed with the Court of Appeals (CA) a Petition for Certiorari and
a. Taxes due the government;
Prohibition to nullify the cease and desist orders of the trial court. 42 Respondent Court issued on April 26, 1991
a Resolution which reads: 43
b. Salvage fees on the vessel in the amount of $1,000,000.00 based on . . . Form of Salvage
Agreement;
MEANWHILE, in order to preserve the status quo and so as not to render the present petition moot and
academic, a TEMPORARY RESTRAINING ORDER is hereby ISSUED enjoining the respondent Judge, the
Set V Corporation Code * Doctrine of Peircing Corpo Veil Cases*Page 152 of 164

Honorable Arsenio M. Gonong, from enforcing and/or implementing the Orders dated 22 March 1991 Length: 66.92 ms. Breadth: 11.28 ms.
and 5 April 1991 which ordered respondent Sheriff to cease and desist from implementing the writ of
execution and the return thereof, the quashing of the levy . . . on [the] execution [and sale] of the Depth: 4.52 m.s. Gross Tons: 1,029.56
properties levied upon and sold at public auction by the Sheriff, for reason of grave abuse of discretion
and in excess of jurisdiction, until further orders from this Court.
Net Tons: 1,027/43 Official Number: 708069

WITHIN ten (10) days from notice hereof, respondents [petitioner included] are also required to SHOW
Material: Steel Class License: Coastwise
CAUSE why the prayer for a writ of preliminary injunction should not be granted.

License No. 81-0059


On May 8, 1991, petitioner received from Camagon a notice to pay private respondent P3 million to satisfy the
trial court Decision. Not having any knowledge of the CA case to which it was not impleaded, petitioner filed
with the trial court a Motion to Dismiss ex abutandi ad cautelam on the grounds that (1) the Petition of private Petitioner also filed a special appearance before the CA. It prayed for the lifting of the levy on its properties or,
respondent stated no cause of action against it, (2) the trial court had no jurisdiction over the case, and (3) litis alternatively, for a temporary restraining order against their auction until its Motion for Reconsideration was
pendentia barred the suit. 44 resolved by the trial court. 46

On May 10, 1991, Camagon levied on petitioner's properties, which were scheduled for auction later on May Acting on petitioner's Motion for Reconsideration, the trial court reversed its Decision of February 18, 1991,
16, 1991. Specific descriptions of the properties are as follows: 45 holding in its May 22, 1991 Resolution as follows: 47

a) Motor Tugboat "DEN DEN" ex Emerson-l. . . . [T]hat . . . Motion for Reconsideration [of petitioner] was filed on March 14, 1991 (see: page 584,
records, Vol. 2) indubitably showing that it was seasonably filed within the 15-day time-frame.
Therefore, . . . said default-judgment ha[d] not yet become final and executory when the Writ of
Length: 35.67 ms. Breadth: 7.33 ms.
Execution was issued on March 13, 1991 . . . The rules [provide] that [the e]xecution shall issue as a
matter of right upon the expiration of the period of appeal from a judgment if no appeal has been duly
Depth: 3.15 ms Gross Tons: 205.71 perfected (Sec. 1, R-39, RRC). That being the case, VEC has all the right to file as it
did . . . the aforementioned reconsideration motion calling [the] attention of the Court and pointing
Net tons: 67.48 ms Official Number: 213551 therein its supposed error and its correction if, indeed, any [error was] committed. It is in this light that
this Court made an in-depth reflection and assessment of the premises or reasons raised by
Material: Steel Class license: CWL [petitioner], and after a re-examination of the facts and evidence spread on the records, it has come
to the considered conclusion that the questioned default-judgment has been improvidently issued. By
the records, the claim of [private respondent] that his January 29, 1990 Ex-Parte Motion To Declare
License No. 4424
Defendants In Default (pp. 174-177, records, Vol. 1) including VEC had been granted is belied by the
February 23, 1990 Order (pp. 214-215, records, ibid) par. 2, thereof, reading to wit:
b) Barge "FC99" ex YD-153
By the foregoing, for reasons stated thereunder respectively, this Court, in the
Length: 34.15 ms. Breadth: 15.85 m.s. exercise of its judicious discretion, in the sense that the rules should be liberally
construed in order to promote their object and to assist the parties, resolves to DENY
Depth: 2.77 m.s. Gross Tons: 491.70 petitioner's Motion to have the Commissioner of Customs AND OTHER ENUMERATED
RESPONDENTS DECLARED IN DEFAULT. [Emphasis ours].

Net Tons: 491.70 Official Number: 227236


Not even [private respondent's] November 23, 1990 "Ex-Parte Motion To Present [Evidence]
Against Defaulting Defendants" (page 489, records, Vol. 2) [can] be deemed as a remedy of
Material: Steel Class License: CWL
the fact that there never was issued an order of default against respondents including
[petitioner] VEC. Having thus established that there [had] been no order of default against
License No. 83-0012 VEC as contemplated by Sec. 1, Rule 18, in relation to Sec. 9, Rule 13, Revised Rules of Court,
there could not have been any valid default-judgment rendered against it. The issuance of an
c) Barge "LAWIN" ex "Sea Lion 2". order of default is a condition sine qua nonin order [that] a judgment by default be clothed
Set V Corporation Code * Doctrine of Peircing Corpo Veil Cases*Page 153 of 164

with validity. Further, records show that this Court never had authorized [private respondent] To enjoin the CTA from enforcing said Order, private respondent filed before the Court of Appeals another
to adduce evidence ex-parte against [petitioner] VEC. In sum, the February 18, 1991 decision Petition for Certiorari, 56 which was later also consolidated with CA-GR SP No. 24669.
by default is null and void as against [petitioner] VEC. With this considered conclusion of
nullity of said default judgment in question, this Court feels there is no more need for it to On July 19, 1993, the CA rendered the assailed Decision. Petitioner filed (1) a Motion for Clarification, praying
resolve Arguments I-A & I-B, as well as III-A & III-B, of the March 14, 1991 Motion for for a declaration that the trial court Decision against it was not valid; and (2) a partial Motion for
Reconsideration. The Court agrees, however, with said discussions on the non-compliance Reconsideration, seeking to set aside the assailed Decision insofar as the latter affected it.
[with] Sec. 2, Rule 7 (Title of Complaint) and Sec. I, Rule 8 on the requirement of indicating in
the complaint the ultimate facts on which the party pleading relies for his claim of defense 57
On July 5, 1995, the Court of Appeals issued the following Resolution:
[--] which is absent in the January 9, Amended Petition (pp. 122-141, records, Vol. I) [--] for it
merely mentioned [petitioner] VEC in par. 5 thereof and no more. It abides, likewise, with
[Argument] III-B that the Decision in suit award[ed] amounts never asked for in instant Pending resolution of the motions for reconsideration, filed by Vlason Enterprises Corporation and
petition as regards VEC (Sec. 5, Rule 18, RRC). . . . . Banco [Du] Brazil, and considering [private respondent's] Motion for Entry of Judgment with respect to
respondent PPA having already been granted by this Court as far back as June 17, 1994, pursuant to
the resolution of the Supreme Court dated December 8, 1993 in G.R. No. 111270-72 (Philippine Ports
WHEREFORE, in view of the foregoing consideration, and as prayed for, the February 18, 1991
Authority vs. Court of Appeals, et al.) informing the parties in the said case that the judgment sought
Judgment by Default is hereby reconsidered and SET ASIDE.
to be reviewed has now become final and executory, the lower court may now take appropriate action
on the urgent ex-parte motion for issuance a writ of execution, filed by [private respondent] on July 15,
On June 26, 1992, then Executive Judge Bernardo P. Pardo 48 of the Regional Trial Court of Manila issued an 1994.
Order49 annulling the Sheriff's Report/Return dated April 1, 1991, and all proceedings taken by Camagon.

On August 28, 1995, the Regional Trial Court of Manila, Branch 26, issued a Writ of Possession which resulted in
The CA granted private respondent's Motion to file a Supplemental Petition impleading petitioner in CA-GR private respondent taking possession of petitioner's barge Lawin (formerly Sea Lion 2) on September 1,
24669.50 In view of the rampant pilferage of the cargo deposited at the PPA compound, private respondent 1995. 58
obtained from the appellate court a Writ of Preliminary Injunction dated March 6, 1992. The Writ: reads: 51
59
Hence, this Petition.
ACCORDINGLY, in view of the foregoing disquisitions, the urgent verified motion for preliminary
injunction dated February 11, 1992 is hereby GRANTED. Therefore, let a writ of preliminary injunction
Ruling of the Respondent Court
forthwith issue against the respondents and all persons or agents acting in their behalf, enjoining
them not to interfere in the transferring of the aforementioned vessel and its cargoes, or in removing
said cargoes . . . from [the] PPA compound. As already adverted to, Respondent Court granted the Petition for Certiorari of the private respondent, which
was consolidated with the latter's two other Petitions. The court a quo issued the following rulings:
On September 15, 1992, Sheriff Amado Sevilla seized petitioner's motor tugboat Den Den by virtue of the
Order 52dated April 3, 1992, issued by the RTC of Manila, Branch 26. 53 1. The trial court had jurisdiction over the salvor's claim or admiralty case pursuant to Batas
Pambansa Bilang 129.
On August 6, 1992, the CA consolidated CA-GR SP No. 28387 54 with CA-GR SP No. 24669. 55 The Court of Tax
Appeals issued on October 5, 1992, a Resolution in CTA Case Nos. 4492, 4494 and 4500, which disposed as 2. Since the Decision of the trial court became final and executory, never having been disputed or
follows: appealed to a higher court, the trial judge committed grave abuse of discretion in recalling the Writ of
Execution and in quashing the levy and the execution of the sale of M/V Star Ace and its cargo.
Confirming the order in open court on October 5, 1992, the Court hereby RESOLVES to:
2. Such acts constituted an alteration or a modification of a final and executory judgment and could
never be justified under law and jurisprudence.
1. Order Respondent Commissioner of Customs to assign or detail [a] sufficient number of customs
police and guards aboard, and around the vicinity of, the vessel "M/V Star Ace" now in anchor at
Mariveles, Bataan or elsewhere, in order to ensure its safety during the pendency of these cases; 3. Civil Case 59-51451 dealt only with the salvor's claim without passing upon the legality or the
validity of the undared Decision of the Commissioner of Customs in the seizure proceeding.
2. Direct him to assign personnel and/or representatives to conduct an inventory of part of the vessel's
cargo now in the possession of Mr. Cesar S. Urbino, Sr. at 197 Heroes del "96 Street, Caloocan City, 4. Petitioner and his co-respondents could not invoke the jurisdiction of a court to secure affirmative
which inventory may be participated in by all the parties interested in said cargo." relief against their opponent and, after failing to obtain such relief, question the court's jurisdiction.
Set V Corporation Code * Doctrine of Peircing Corpo Veil Cases*Page 154 of 164

5. Petitioner had no recourse through any of the following judicially accepted means to question the At any rate, the decision dated July 19, 1993 of this Court on the main petition for certiorari is not yet final
final judgment: (except with respect to respondent PPA), the Bureau of Customs having filed a petition forcertiorari and
prohibition, under Rule 65 of the Rules of Court, with the Supreme Court, necessitating prudence on Our part to
a. a petition for relief from judgment under Rule 38, await its final verdict. 60

b. a direct action to annul and enjoin the enforcement of the questioned judgment, Assignment of Errors
and
61
Before us, petitioner submits the following assignment of errors on the part of Respondent Court:
c. a collateral attack against the questioned judgment which appears void on its
face. I

6. A court which has already acquired jurisdiction over a case cannot be ousted by a coequal court; The Court of Appeals committed serious error in ruling that the entire decision of the trial court in Civil
the res in this case the vessel and its cargo were placed under the control of the trial court ahead Case No. 89-51451 dated 18 February 1991 became final and executory because it "was never
of the CTA. disputed or appealed".

7. The admiralty Decision had attained finality while the issue of the validity of the seizure
proceedings was still under determination.
A VEC filed a motion for reconsideration of the said decision two days before
In the assailed Resolution, Respondent Court clarified that there was no need to serve summons anew on deadline, which motion was granted by the trial court.
petitioner, since it had been served summons when the Second Amended Petition (the third) was filed; and that
petitioner's Motion for Reconsideration was defective and void, because it contained no notice of hearing
addressed to the counsel of private respondent in violation of Rule 16, Section 4 of the Rules of Court.

B The trial court correctly granted VEC's motion for reconsideration and set aside the
To this second motion, [private respondent] contends that there was no need to serve summons anew 18 February 1991 decision . . . against VEC, for:
to VEC when the second amended petition was filed impleading VEC, pursuant to the ruling of the
Supreme Court in Asiatic Travel Corp. vs. CA (164 SCRA 623); and that finally, the decision of the
1. The trial court never acquired jurisdiction over the person of VEC as to
court a quo o[n] February 18, 1991 became final and executory, notwithstanding the timely filing of
enable it to render any judgment against it:
the motion for reconsideration of VEC for the reason that the said motion for reconsideration was
defective or void, there being no notice of hearing addressed to the counsel of petitioner. In fact, no
motion such as this instant one can be acted upon by the Court without proof of service of the notice (i) VEC was not impleaded as a respondent in Civil Case No. 89-
thereof, pursuant to Rule 16, Section 4 of the Rules of Court. 51451;

xxx xxx xxx (ii) Summons was not served on VEC;

Finally, we should never lose sight of the fact that the instant petition for certiorari is proper only to correct 2. The trial court improperly rendered judgment by default against VEC;
errors of jurisdiction committed by the lower court, or grave abuse of discretion which is tantamount to lack of
jurisdiction Where the error is not one of jurisdiction but an error of law or of fact which is a mistake of (i) The trial court never issued an order of default against VEC;
judgment, appeal is the remedy (Salas vs. Castro. 216 SCRA 198). Here, respondents failed to appeal. Hence,
the decision dated February 18, 1991 of the lower court has long become final, executory and unappealable.
(ii) The trial court never authorized ex-parte presentation of
We do not and cannot therefore review the instant case as if it were on appeal and direct actions on these
evidence against VEC.
motions. While the proper remedy is appeal, the action for certiorari will not be entertained. Indeed, certiorari
is not a substitute for lapsed appeal.
3. The Judgment by default was fatally defective because:
Set V Corporation Code * Doctrine of Peircing Corpo Veil Cases*Page 155 of 164

(i) No filing fee was paid by [private respondent) for the staggering the trial court, thirteen days after it received the Decision or two days before the lapse of the reglementary
amount of damages awarded by the trial court. period to appeal. 66 Thus, as to petitioner, the trial court Decision had not attained finality.

(ii) The 18 February 1991 decision violates the Revised Rules of Exception to the Rule
Court, which prescribe that a judgment by default cannot decree a
relief not prayed for. on Notice of Hearing

II

Since the 18 February 1991 Decision in Civil Case No. 89-51451 is void as against VEC, the Respondent Court and private respondent argue that, although timely filed, petitioner's Motion for
recall of the writ of execution was valid, as far as VEC is concerned. Reconsideration was a mere scrap of paper, because (1) it did not contain a notice of hearing addressed to the
current counsel of private respondent, and (2) the notice of hearing addressed to and served on private
The Court believes that the issues can be simplified and restated as follows: respondent's deceased counsel was not sufficient. Admittedly, this Motion contained a notice of hearing sent to
Atty. Jesus C. Concepcion who, according to private respondent, had already died and had since been
1. Has the February 18, 1991 RTC Decision become final and executory in regard to petitioner? substituted by its new counsel, Atty. Domingo Desierto. Therefore, the appellate court ruled that the said
Motion did not toll the reglementary period to appeal and that the trial court Decision became final.

2. Did the trial court acquire jurisdiction over the petitioner?


This Court disagrees. Rule 15 of the Rules of Court states:

3. Was the RTC default judgment binding on petitioner?


Sec. 4. Notice. Notice of a motion shall be served by the applicant to all parties concerned, at least
three (3) days before the hearing thereof, together with a copy of the motion, and of any affidavits
4. Was the grant of damages against petitioner procedurally proper?
and other papers accompanying it. The court, however, for good cause may hear a motion on shorter
notice, specially on matters which the court may dispose of on its own motion.
5. Was private respondent entitled to a writ of execution?
Sec. 5. Contents of notice. The notice shall be directed to the parties concerned, and shall state the
This Court's Ruling time and place for the hearing of the motion. 67

The petition is meritorious. Ideally, the foregoing Rule requires the petitioner to address and to serve on the counsel of private respondent
the notice of hearing of the Motion for Reconsideration. The case at bar, however, is far from ideal. First,
First Issue: Finality of the RTC Decision petitioner was not validly summoned and it did not participate in the trial of the case in the lower court; thus, it
was understandable that petitioner would not be familiar with the parties and their counsels. Second, Atty.
Desierto entered his appearance only as collaborating counsel, 68 who is normally not entitled to notices even
A judgment becomes "final and executory" by operation of law. Its finality becomes a fact when the
from this Court. Third, private respondent made no manifestation on record that Atty. Concepcion was already
reglementary period to appeal lapses, and no appeal is perfected within such period. 62 The admiralty case filed
dead. Besides, it was Atty. Concepcion who signed the Amended Petition, wherein petitioner was first
by private respondent with the trial court involved multiple defendants. This being the case, it necessarily
impleaded as respondent and served a copy thereof. Naturally, petitioner's attention was focused on this
follows that the period of appeal of the February 18, 1991 RTC Decision depended on the date a copy of the
pleading, and it was within its rights to assume that the signatory to such pleading was the counsel for private
judgment was received by each of the defendants. Elsewise stated, each defendant had a different period
respondent.
within which to appeal, depending on the date of receipt of the Decision. 63

The Court has consistently held that a motion which does not meet the requirements of Sections 4 and 5 of
Omega, Singkong Trading Co. and M/V Star Ace chose to enter into a compromise agreement with private
Rule 15 of the Rules of Court is considered a worthless piece of paper, which the clerk of court has no right to
respondent. As to these defendants, the trial court Decision had become final, and a writ of execution could be
receive and the trial court has no authority to act upon. Service of a copy of a motion containing a notice of the
issued against them. 64 Doctrinally, a compromise agreement is immediately final and executory. 65
time and the place of hearing of that motion is a mandatory requirement, and the failure of movants to comply
with these requirements renders their motions fatally defective. 69 However, there are exceptions to the strict
Petitioner, however, is not in the same situation. Said Decision cannot be said to have attained finality as to the application of this rule. These exceptions are as
petitioner, which a party to the compromise. Moreover, petitioner filed a timely Motion for Reconsideration with follows: 70
Set V Corporation Code * Doctrine of Peircing Corpo Veil Cases*Page 156 of 164

. . . Liberal construction of this rule has been allowed by this Court in cases (1) where a rigid The sheriff's return shows that Angliongto who was president of petitioner corporation, through his secretary
application will result in a manifest failure or miscarriage of justice; 71 especially if a party successfully Betty Bebero, was served summons on January 18, 1990. 78 Petitioner claims that this service was defective for
shows that the alleged defect in the questioned final and executory judgment is not apparent on its two reasons: (1) Bebero was an employee of Vlasons Shipping, Inc., which was an entity separate and distinct
face or from the recitals contained therein; (2) where the interest of substantial justice will be from Petitioner Vlason Enterprises Corporation (VEC); and (2) the return pertained to the service of summons
served; 72 (3) where the resolution of the motion is addressed solely to the sound and judicious for the amended Petition, not for the "Second Amended Petition with Supplemental Petition," the latter pleading
discretion of the court; 73 and (4) where the injustice to the adverse party is not commensurate [to] having superseded the former.
the degree of his thoughtlessness in not complying with the procedure prescribed. 74
A corporation may be served summons through its agents or officers who under the Rules are designated to
The present case falls under the first exception. Petitioner was not informed of any cause of action or claim accept service of process. A summons addressed to a corporation and served on the secretary of its president
against it. All of a sudden, the vessels which petitioner used in its salvaging business were levied upon and sold binds that corporation. 79 This is based on the rationale that service must be made on a representative so
in execution to satisfy a supposed judgment against it. To allow this to happen simply because of a lapse in integrated with the corporation sued, that it is safe to assume that said representative had sufficient
fulfilling the notice requirement which, as already said, was satisfactorily explained would be a manifest responsibility and discretion to realize the importance of the legal papers served and to relay the same to the
failure or miscarriage of justice. president or other responsible officer of the corporation being sued. 80 The secretary of the president satisfies
this criterion. This rule requires, however, that the secretary should be an employee of the corporation sought
A notice of hearing is conceptualized as an integral component of procedural due process intended to afford to be summoned. Only in this manner can there be an assurance that the secretary will "bring home to the
the adverse parties a chance to be heard before a motion is resolved by the court. Through such notice, the corporation [the] notice of the filing of the action" against it.
adverse party is permitted time to study and answer the arguments in the motion.
In the present case, Bebero was the secretary of Angliongto, who was president of both VSI and petitioner, but
Circumstances in the case at bar show that private respondent was not denied procedural due process, and she was an employee of VSI, not of petitioner. The piercing of the corporate veil cannot be resorted to when
that the very purpose of a notice of hearing had been served. On the day of the hearing, Atty. Desierto did not serving summons. 81 Doctrinally, a corporation is a legal entity distinct and separate from the members and
object to the said Motion for lack of notice to him; in fact, he was furnished in open court with a copy of the stockholders who compose it. However, when the corporate fiction is used as a means of perpetrating a fraud,
motion and was granted by the trial court thirty days to file his opposition to it. These circumstances clearly evading an existing obligation, circumventing a statute, achieving or perfecting a monopoly or, in generally
justify a departure from the literal application of the notice of hearing rule. 75 In other cases, after the trial court perpetrating a crime, the veil will be lifted to expose the individuals composing it. None of the foregoing
learns that a motion lacks such notice, the prompt resetting of the hearing with due notice to all the parties is exceptions has been shown to exist in the present case. Quite the contrary, the piercing of the corporate veil in
held to have cured the defect. 76 this case will result in manifest injustice. This we cannot allow. Hence, the corporate fiction remains.

Verily, the notice requirement is not a ritual to be followed blindly. Procedural due process is not based solely Effect of Amendment of
on a mechanistic and literal application that renders any deviation inexorably fatal. Instead, procedural rules
are liberally construed to promote their objective and to assist in obtaining a just, speedy and inexpensive Pleading on Jurisdiction
determination of any action and proceeding. 77 For the foregoing reasons, we believe that Respondent Court
committed reversible error in holding that the Motion for Reconsideration was a mere scrap of paper. Petitioner claims that the trial court did not acquire jurisdiction over it, because the former had not been served
summons anew for the Second Amended Petition or for the Second Amended Petition with Supplemental
Second Issue: Jurisdiction Over Petitioner Petition. In the records, it appears that only Atty. Tamondong, counsel for Singkong Trading, was furnished a
copy of the Second Amended Petition. 82 The corresponding sheriff's return indicates that only Omega, M/V Star
Service of Summons Ace and Capt. Rada were served summons and copies of said Petition. 83

on a Corporation We disagree. Although it is well-settled that an amended pleading supersedes the original one, which is thus
deemed withdrawn and no longer considered part of the record, it does not follow ipso facto that the service of
a new summons for amended petitions or complaints is required. Where the defendants have already appeared
before the trial court by virtue of a summons on the original complaint, the amended complaint may be served
upon them without need of another summons, even if new causes of action are alleged. 84 After it is acquired, a
court's jurisdiction continues until the case is finally terminated. Conversely, when defendants have not yet
appeared in court and no summons has been validly served, new summons for the amended complaint must
be served on them. 85 It is not the change of cause of action that gives rise to the need to serve another
summons for the amended complaint, but rather the acquisition of jurisdiction over the persons of the
Set V Corporation Code * Doctrine of Peircing Corpo Veil Cases*Page 157 of 164

defendants. If the trial court has not yet acquired jurisdiction over them, a new service of summons for the The general rule is allegata et probata a judgment must conform to the pleadings and the theory of the
amended complaint is required.1wphi1.nt action under which the case was tried. 87 But a court may also rule and render judgment on the basis of the
evidence before it, even though the relevant pleading has not been previously amended, so long as no surprise
In this case, the trial court obviously labored under the erroneous impression that petitioner had already been or prejudice to the adverse party is thereby caused. 88
placed under its jurisdiction since it had been served summons through the secretary of its president. Thus, it
dispensed with the service on petitioner of new summons for the subsequent amendments of the Petition. We In the case at bar, the liability of petitioner was based not on any allegation in the four Petitions filed with the
have already ruled, however, that the first service of summons on petitioner was invalid. Therefore, the trial trial court, but on the evidence presented ex parte by the private respondent. Since the trial court had not
court never acquired jurisdiction, and the said court should have required a new service of summons for the validly acquired jurisdiction over the person of petitioner, there way for the latter to have validly and knowingly
amended Petitions. waived its objection to the private respondent's presentation of evidence against it.

Impleading a Party in the Third Issue: Judgment by Default

Title of the Complaint The trial court Decision holding petitioner liable for damages is basically a default judgment. In Section 18,
judgment by default is allowed under the following condition: 89
Petitioner further claims that the trial court failed to acquire jurisdiction to render judgment against it because
(1) the title of the three Petitions filed by private respondent never included petitioner as a party-defendant, in
violation of Rule 7; and (2) the Petitions failed to state any allegation of ultimate facts constituting a cause of
action against petitioner. Sec. 1. Judgment by default. If the defendant fails to answer within the time specified in these rules,
the court shall, upon motion of the plaintiff and proof of such failure, declare the defendant in default.
We disagree with petitioner on the first ground. The judicial attitude has always been favorable and liberal in Thereupon the court shall proceed to receive the plaintiff's evidence and render judgment granting
allowing amendments to pleadings. Pleadings shall be construed liberally so as to render substantial justice to him such relief as the complaint and the facts proven may warrant. . . . .
the parties and to determine speedily and inexpensively the actual merits of the controversy with the least
regard to technicalities. 86 Thus, it becomes crucial to determine whether petitioner was declared in default, and whether the reception of
evidence ex parte against it was procedurally valid.
The inclusion of the names of all the parties in the title of a complaint is a formal requirement under Section 3,
Rule 7. However, the rules of pleadings require courts to pierce the form and go into the substance and not to Petitioner Was Never
be misled by a false or wrong name given to a pleading. The averments in the complaint, not the title,
controlling. Although the general rule requires the inclusion of the names of all the parties in the title of a
Declared In Default
complaint, the non-inclusion of one or some of them is not fatal to the cause of action of a plaintiff, provided
there is a statement in the body of the petition indicating that a defendant was made a party to such action.
Petitioner insists that the trial court never declared it in default.
Private respondent claims that petitioner has always been included in the caption of all the Petitions it filed,
which included Antonio Sy, field manager of petitioner. We checked and noted that in the caption and the body We agree. The trial court denied the January 29, 1990 Motion of private respondent to declare all the
of the Amended Petition and Second Amended Petition with Supplemental Petition, Antonio Sy alleged to be defendants in default, but it never acted on the latter's subsequent Motion to declare petitioner likewise.
representing Med Line Philippines, not petitioner. Because it was private respondent who was responsible for During the pretrial on January 23, 1993, the RTC declared in default only "Atty. Eddie Tamondong, as well as the
the errors, the Court cannot excuse it from compliance, for such action will prejudice petitioner, who had no other defendants Hon. Salvador Mison, M/V Star Ace, Omega Sea Transport Co., Inc. of Panama and Sinkong
hand in the preparation of these pleadings. In any event, we reiterate that, as a general rule, mere failure to Trading Co., [but] despite . . . due notice to them, [they] failed to appear. 90 Even private respondent cannot
include the name of a party in the title of a complaint is not fatal by itself. pinpoint which trial court order held petitioner in default.

Stating a Cause of Action More important, the trial court, in its Resolution dated May 22, 1991, admitted that it never declared petitioner
in default, viz.:
in the Complaint
. . . It is in this light that this [c]ourt made an in-depth reflection and assessment of the premises or
reasons raised by [petitioner] VEC[;] and after a re-examination of the facts and evidence spread on
the records, it has come to the considered conclusion that the questioned default-judgment has been
Set V Corporation Code * Doctrine of Peircing Corpo Veil Cases*Page 158 of 164

improvidently issued. [Based on] the records, the claim of [private respondent] that [its] January 29, Additional Filing Fees as
1990 Ex-Parte Motion to Declare Defendants In Default (pp. 174-177, records, Vol. 1) including VEC
had been granted is belied by the February 23, 1990 Order (pp. 214-215, records, ibid.) par. 2, thereof, Lien on the Judgment
...

Had the trial court validly acquired jurisdiction over petitioner, nonpayment of docket fees would not have
xxx xxx xxx prevented it from holding petitioner liable for damages. The Court, in Manchester Development Corporation v.
Court of Appeals, 92 ruled that a court acquires jurisdiction over any case only upon the payment of the
Not even petitioner's November 23, 1990 "Ex-Parte Motion To Present Evidence Against prescribed docket fee, not upon the amendment of the complaint or the payment of the docket fees based on
Defaulting Defendants" (page 489, records, Vol. 2) [can] be deemed as a remedy [for] the the amount sought in the amended pleading. This ruling, however, was modified in Sun Insurance Office, Ltd. v.
fact that there never was issued an order of default against respondents including [petitioner] Asuncion, 93 which added:
VEC. Having thus established that there ha[d] been no order of default against VEC as
contemplated by Sec. 1, Rule 18, in relation to Sec. 9, Rule 13, Revised Rules of Court, there 3. Where the trial court acquires jurisdiction over a claim [through] the filing of the appropriate
could not have been any valid default-judgment rendered against it. The issuance of an order pleading and payment of the prescribed filing fee but, subsequently, the judgment awards a claim not
[o]f default is a condition sine qua nonin order [that] a judgment by default be clothed with specified in the pleading, or if specified the same has been left for determination by the court, the
validity. Further, records show that this [c]ourt never had authorized [private respondent] to additional filing fee therefor shall constitute a lien on the judgment. It shall be the responsibility of the
adduce evidence ex-parte against [Petitioner] VEC. In sum, the February 18, 1991 decision by Clerk of Court or his duly authorized deputy to enforce said lien and assess and collect the additional
default is null and void as against [Petitioner] VEC. . . . fee.

The aforementioned default judgment refers to the February 18, 1989 Decision, not to the Order finding Filing fees for damages and awards that cannot be estimated constitute liens on the awards finally granted by
petitioner in default as contended by private respondent. Furthermore, it is a legal impossibility to declare a the trial court. Their nonpayment alone is not a ground for the invalidation of the award.
party-defendant to be in default before it was validly served summons.

Judgment by Default Cannot


Trial Court Did Not Allow

Grant Relief Prayed For


Presentation of Evidence

A declaration or order of default is issued as a punishment for unnecessary delay in joining issues. In such
Ex Parte Against Petitioner event, defendants lose their standing in court, they cannot expect the trial court to act upon their pleadings,
and they are not entitled to notice of the proceeding until the final termination of the
case. 94 Thus, the trial court proceeds with the reception of the plaintiff's evidence upon which a default
judgment is rendered.
The Order of December 10, 1990, which allowed the presentation of evidence ex parte against the defaulting
defendants, could not have included petitioner, because the trial court granted private respondent's motion Section 1 of Rule 18 provides that after the defendant has been declared in default, "the court shall proceed to
praying for the declaration of only the foreign defendants in default. So too, private respondent's ex receive the plaintiff's evidence and render judgment granting him such relief as the complaint and the facts
parte Motion to present evidence referred to the foreign defendants only. 91 proven may warrant." The reliefs that may be granted, however, are restricted by Section 5, which provides
that a judgment entered against a party in default shall not exceed the amount or be different in kind from that
Furthermore, the reception of evidence ex parte against a non-defaulting party is procedurally indefensible. prayed for.
Without a declaration that petitioner is in default as required in Section 1, Rule 18, the trial court had no
authority to order the presentation of evidence ex parte against petitioner to render judgment against it by In other words, under Section 1, a declaration of default is not an admission of the truth or the validity of the
default. The trial judge must have thought that since it failed to summons and was in default, it effectively plaintiff's claims. 95 The claimant must still prove his claim and present evidence. In this sense the law gives
waived any objection to the presentation of evidence against it. This rule, however, would have applied only if defaulting parties some measure of protection because plaintiffs, despite the default of defendants, are still
petitioner had submitted itself to the jurisdiction of the trial court. The latter correctly declared, in the required to substantiate their allegations in the complaint. The judgment of default against defendants who
Resolution just cited, that the default judgment against the former had been improvidently rendered. have not appeared or filed their answers does not imply a waiver of all their rights, except their right to be
heard and to present evidence in their favor. Their failure to answer does not imply their admission of the facts
Fourth Issue: Award Not Paid and Prayed For and the causes of action of the plaintiffs, because the latter are required to adduce evidence to support their
allegations.
Set V Corporation Code * Doctrine of Peircing Corpo Veil Cases*Page 159 of 164

Moreover, the trial court is not allowed by the Rules to receive evidence that tends to show a relief not sought declaring the plaintiff Villa Rey Transit, Inc., to be the lawful owner of the said certificates of public
or specified in the pleadings. 96 The plaintiff cannot be granted an award greater than or different in kind from convenience; and ordering the private defendants, jointly and severally, to pay to the plaintiff, the sum of
that specified in the complaint. 97 P5,000.00 as and for attorney's fees. The case against the PSC was dismissed.

This case should be distinguished, however, from that of defendants, who filed an answer but were absent The rather ramified circumstances of the instant case can best be understood by a chronological narration of
during trial. In that case, they can be held liable for an amount greater than or different from that originally the essential facts, to wit:
prayed for, provided that the award is warranted by the proven facts. This rule is premised on the theory that
the adverse party failed to object to evidence relating to an issue not raised in the pleadings. Prior to 1959, Jose M. Villarama was an operator of a bus transportation, under the business name of Villa Rey
Transit, pursuant to certificates of public convenience granted him by the Public Service Commission (PSC, for
The latter rule, however, is not applicable to the instant case. Admittedly, private respondent presented short) in Cases Nos. 44213 and 104651, which authorized him to operate a total of thirty-two (32) units on
evidence that would have been sufficient to hold petitioner liable for damages. However, it did not include in its various routes or lines from Pangasinan to Manila, and vice-versa. On January 8, 1959, he sold the
amended Petitions any prayer for damages against petitioner. Therefore, the trial court could not have validly aforementioned two certificates of public convenience to the Pangasinan Transportation Company, Inc.
held the latter liable for damages even if it were in default. (otherwise known as Pantranco), for P350,000.00 with the condition, among others, that the seller (Villarama)
"shall not for a period of 10 years from the date of this sale, apply for any TPU service identical or competing
Fifth Issue: Execution of Final Judgment with the buyer."

Section 1 of Rule 39 provides that execution shall issue only upon a judgment that finally disposes of the action Barely three months thereafter, or on March 6, 1959: a corporation called Villa Rey Transit, Inc. (which shall be
or proceeding. Such execution shall issue as a matter of right upon the expiration of the period to appeal it, if referred to hereafter as the Corporation) was organized with a capital stock of P500,000.00 divided into 5,000
no appeal has been duly perfected. 98 shares of the par value of P100.00 each; P200,000.00 was the subscribed stock; Natividad R. Villarama (wife of
Jose M. Villarama) was one of the incorporators, and she subscribed for P1,000.00; the balance of P199,000.00
was subscribed by the brother and sister-in-law of Jose M. Villarama; of the subscribed capital stock,
In the present case, however, we have already shown that the trial court's Decision has not become final and
P105,000.00 was paid to the treasurer of the corporation, who was Natividad R. Villarama.
executory against petitioner. In fact, the judgment does not even bind it. Obviously, Respondent Court
committed serious reversible errors when it allowed the execution of the said judgment against
petitioner.WHEREFORE, the appeal is hereby GRANTED, and the assailed Decision and Resolution of the Court In less than a month after its registration with the Securities and Exchange Commission (March 10, 1959), the
of Appeals are REVERSED and SET ASIDE insofar as they affect petitioner. The levy and the sale on execution of Corporation, on April 7, 1959, bought five certificates of public convenience, forty-nine buses, tools and
petitioner's properties are declared NULL and VOID. Said properties are ordered RESTORED to petitioner. No equipment from one Valentin Fernando, for the sum of P249,000.00, of which P100,000.00 was paid upon the
pronouncement as to costs. signing of the contract; P50,000.00 was payable upon the final approval of the sale by the PSC; P49,500.00 one
year after the final approval of the sale; and the balance of P50,000.00 "shall be paid by the BUYER to the
different suppliers of the SELLER."
EN BANC G.R. No. L-23893 October 29, 1968

The very same day that the aforementioned contract of sale was executed, the parties thereto immediately
VILLA REY TRANSIT, INC., plaintiff-appellant, vs.
applied with the PSC for its approval, with a prayer for the issuance of a provisional authority in favor of the
EUSEBIO E. FERRER, PANGASINAN TRANSPORTATION CO., INC. and PUBLIC SERVICE
vendee Corporation to operate the service therein involved. 1 On May 19, 1959, the PSC granted the provisional
COMMISSION,defendants.
permit prayed for, upon the condition that "it may be modified or revoked by the Commission at any time, shall
EUSEBIO E. FERRER and PANGASINAN TRANSPORTATION CO., INC., defendants-appellants.
be subject to whatever action that may be taken on the basic application and shall be valid only during the
pendency of said application." Before the PSC could take final action on said application for approval of sale,
PANGASINAN TRANSPORTATION CO., INC., third-party plaintiff-appellant, however, the Sheriff of Manila, on July 7, 1959, levied on two of the five certificates of public
vs. convenience involved therein, namely, those issued under PSC cases Nos. 59494 and 63780, pursuant to a writ
JOSE M. VILLARAMA, third-party defendant-appellee. of execution issued by the Court of First Instance of Pangasinan in Civil Case No. 13798, in favor of Eusebio
Ferrer, plaintiff, judgment creditor, against Valentin Fernando, defendant, judgment debtor. The Sheriff made
ANGELES, J.: and entered the levy in the records of the PSC. On July 16, 1959, a public sale was conducted by the Sheriff of
the said two certificates of public convenience. Ferrer was the highest bidder, and a certificate of sale was
issued in his name.
This is a tri-party appeal from the decision of the Court of First Instance of Manila, Civil Case No. 41845,
declaring null and void the sheriff's sale of two certificates of public convenience in favor of defendant Eusebio
E. Ferrer and the subsequent sale thereof by the latter to defendant Pangasinan Transportation Co., Inc.;
Set V Corporation Code * Doctrine of Peircing Corpo Veil Cases*Page 160 of 164

Thereafter, Ferrer sold the two certificates of public convenience to Pantranco, and jointly submitted for Ferrer, for his part, challenges the decision insofar as it holds that the sheriff's sale is null and void; and the
approval their corresponding contract of sale to the PSC. 2 Pantranco therein prayed that it be authorized sale of the two certificates in question by Valentin Fernando to the Corporation, is valid. He also assails the
provisionally to operate the service involved in the said two certificates. award of P5,000.00 as attorney's fees in favor of the Corporation, and the failure to award moral damages to
him as prayed for in his counterclaim.
The applications for approval of sale, filed before the PSC, by Fernando and the Corporation, Case No. 124057,
and that of Ferrer and Pantranco, Case No. 126278, were scheduled for a joint hearing. In the meantime, to wit, The Corporation, on the other hand, prays for a review of that portion of the decision awarding only P5,000.00
on July 22, 1959, the PSC issued an order disposing that during the pendency of the cases and before a final as attorney's fees, and insisting that it is entitled to an award of P100,000.00 by way of exemplary damages.
resolution on the aforesaid applications, the Pantranco shall be the one to operate provisionally the service
under the two certificates embraced in the contract between Ferrer and Pantranco. The Corporation took issue After a careful study of the facts obtaining in the case, the vital issues to be resolved are: (1) Does the
with this particular ruling of the PSC and elevated the matter to the Supreme Court, 3 which decreed, after stipulation between Villarama and Pantranco, as contained in the deed of sale, that the former "SHALL NOT FOR
deliberation, that until the issue on the ownership of the disputed certificates shall have been finally settled by A PERIOD OF 10 YEARS FROM THE DATE OF THIS SALE, APPLY FOR ANY TPU SERVICE IDENTICAL OR COMPETING
the proper court, the Corporation should be the one to operate the lines provisionally. WITH THE BUYER," apply to new lines only or does it include existing lines?; (2) Assuming that said stipulation
covers all kinds of lines, is such stipulation valid and enforceable?; (3) In the affirmative, that said stipulation is
On November 4, 1959, the Corporation filed in the Court of First Instance of Manila, a complaint for the valid, did it bind the Corporation?
annulment of the sheriff's sale of the aforesaid two certificates of public convenience (PSC Cases Nos. 59494
and 63780) in favor of the defendant Ferrer, and the subsequent sale thereof by the latter to Pantranco, against For convenience, We propose to discuss the foregoing issues by starting with the last proposition.
Ferrer, Pantranco and the PSC. The plaintiff Corporation prayed therein that all the orders of the PSC relative to
the parties' dispute over the said certificates be annulled.
The evidence has disclosed that Villarama, albeit was not an incorporator or stockholder of the Corporation,
alleging that he did not become such, because he did not have sufficient funds to invest, his wife, however,
In separate answers, the defendants Ferrer and Pantranco averred that the plaintiff Corporation had no valid was an incorporator with the least subscribed number of shares, and was elected treasurer of the Corporation.
title to the certificates in question because the contract pursuant to which it acquired them from Fernando was The finances of the Corporation which, under all concepts in the law, are supposed to be under the control and
subject to a suspensive condition the approval of the PSC which has not yet been fulfilled, and, therefore, administration of the treasurer keeping them as trust fund for the Corporation, were, nonetheless, manipulated
the Sheriff's levy and the consequent sale at public auction of the certificates referred to, as well as the sale of and disbursed as if they were the private funds of Villarama, in such a way and extent that Villarama appeared
the same by Ferrer to Pantranco, were valid and regular, and vested unto Pantranco, a superior right thereto. to be the actual owner-treasurer of the business without regard to the rights of the stockholders. The following
testimony of Villarama,4 together with the other evidence on record, attests to that effect:
Pantranco, on its part, filed a third-party complaint against Jose M. Villarama, alleging that Villarama and the
Corporation, are one and the same; that Villarama and/or the Corporation was disqualified from operating the Q. Doctor, I want to go back again to the incorporation of the Villa Rey Transit, Inc. You heard the
two certificates in question by virtue of the aforementioned agreement between said Villarama and Pantranco, testimony presented here by the bank regarding the initial opening deposit of ONE HUNDRED FIVE
which stipulated that Villarama "shall not for a period of 10 years from the date of this sale, apply for any TPU THOUSAND PESOS, of which amount Eighty-Five Thousand Pesos was a check drawn by yourself
service identical or competing with the buyer." personally. In the direct examination you told the Court that the reason you drew a check for Eighty-
Five Thousand Pesos was because you and your wife, or your wife, had spent the money of the
Upon the joinder of the issues in both the complaint and third-party complaint, the case was tried, and stockholders given to her for incorporation. Will you please tell the Honorable Court if you knew at the
thereafter decision was rendered in the terms, as above stated. time your wife was spending the money to pay debts, you personally knew she was spending the
money of the incorporators?
As stated at the beginning, all the parties involved have appealed from the decision. They submitted a joint
record on appeal. A. You know my money and my wife's money are one. We never talk about those things.

Pantranco disputes the correctness of the decision insofar as it holds that Villa Rey Transit, Inc. (Corporation) is Q. Doctor, your answer then is that since your money and your wife's money are one money and
a distinct and separate entity from Jose M. Villarama; that the restriction clause in the contract of January 8, you did not know when your wife was paying debts with the incorporator's money?
1959 between Pantranco and Villarama is null and void; that the Sheriff's sale of July 16, 1959, is likewise null
and void; and the failure to award damages in its favor and against Villarama. A. Because sometimes she uses my money, and sometimes the money given to her she gives to
me and I deposit the money.

Q. Actually, aside from your wife, you were also the custodian of some of the incorporators here, in
the beginning?
Set V Corporation Code * Doctrine of Peircing Corpo Veil Cases*Page 161 of 164

A. Not necessarily, they give to my wife and when my wife hands to me I did not know it belonged Another witness, Celso Rivera, accountant of the Corporation, testified that while in the books of the
to the incorporators. corporation there appears an entry that the treasurer received P95,000.00 as second installment of the paid-in
subscriptions, and, subsequently, also P100,000.00 as the first installment of the offer for second subscriptions
Q. It supposes then your wife gives you some of the money received by her in her capacity as worth P200,000.00 from the original subscribers, yet Villarama directed him (Rivera) to make vouchers
treasurer of the corporation? liquidating the sums.7 Thus, it was made to appear that the P95,000.00 was delivered to Villarama in payment
for equipment purchased from him, and the P100,000.00 was loaned as advances to the stockholders. The said
accountant, however, testified that he was not aware of any amount of money that had actually passed hands
A. Maybe.
among the parties involved,8 and actually the only money of the corporation was the P105,000.00 covered by
the deposit slip Exh. 23, of which as mentioned above, P85,000.00 was paid by Villarama's personal check.
Q. What did you do with the money, deposit in a regular account?
Further, the evidence shows that when the Corporation was in its initial months of operation, Villarama
A. Deposit in my account. purchased and paid with his personal checks Ford trucks for the Corporation. Exhibits 20 and 21 disclose that
the said purchases were paid by Philippine Bank of Commerce Checks Nos. 992618-B and 993621-B,
Q. Of all the money given to your wife, she did not receive any check? respectively. These checks have been sufficiently established by Fausto Abad, Assistant Accountant of Manila
Trading & Supply Co., from which the trucks were purchased 9 and Aristedes Solano, an employee of the
Philippine Bank of Commerce,10as having been drawn by Villarama.
A. I do not remember.

Exhibits 6 to 19 and Exh. 22, which are photostatic copies of ledger entries and vouchers showing that
Q. Is it usual for you, Doctor, to be given Fifty Thousand Pesos without even asking what is this?
Villarama had co-mingled his personal funds and transactions with those made in the name of the Corporation,
are very illuminating evidence. Villarama has assailed the admissibility of these exhibits, contending that no
xxx xxx xxx evidentiary value whatsoever should be given to them since "they were merely photostatic copies of the
originals, the best evidence being the originals themselves." According to him, at the time Pantranco offered
JUDGE: Reform the question. the said exhibits, it was the most likely possessor of the originals thereof because they were stolen from the
files of the Corporation and only Pantranco was able to produce the alleged photostat copies thereof.
Q. The subscription of your brother-in-law, Mr. Reyes, is Fifty-Two Thousand Pesos, did your wife
give you Fifty-two Thousand Pesos? Section 5 of Rule 130 of the Rules of Court provides for the requisites for the admissibility of secondary
evidence when the original is in the custody of the adverse party, thus: (1) opponent's possession of the
original; (2) reasonable notice to opponent to produce the original; (3) satisfactory proof of its existence; and
A. I have testified before that sometimes my wife gives me money and I do not know exactly for
(4) failure or refusal of opponent to produce the original in court. 11 Villarama has practically admitted the
what.
second and fourth requisites. 12 As to the third, he admitted their previous existence in the files of the
Corporation and also that he had seen some of them. 13 Regarding the first element, Villarama's theory is that
The evidence further shows that the initial cash capitalization of the corporation of P105,000.00 was mostly since even at the time of the issuance of the subpoena duces tecum, the originals were already missing,
financed by Villarama. Of the P105,000.00 deposited in the First National City Bank of New York, representing therefore, the Corporation was no longer in possession of the same. However, it is not necessary for a party
the initial paid-up capital of the Corporation, P85,000.00 was covered by Villarama's personal check. The seeking to introduce secondary evidence to show that the original is in the actual possession of his adversary. It
deposit slip for the said amount of P105,000.00 was admitted in evidence as Exh. 23, which shows on its face is enough that the circumstances are such as to indicate that the writing is in his possession or under his
that P20,000.00 was paid in cash and P85,000.00 thereof was covered by Check No. F-50271 of the First control. Neither is it required that the party entitled to the custody of the instrument should, on being notified
National City Bank of New York. The testimonies of Alfonso Sancho 5 and Joaquin Amansec,6 both employees of to produce it, admit having it in his possession. 14Hence, secondary evidence is admissible where he denies
said bank, have proved that the drawer of the check was Jose Villarama himself. having it in his possession. The party calling for such evidence may introduce a copy thereof as in the case of
loss. For, among the exceptions to the best evidence rule is "when the original has been lost, destroyed, or
cannot be produced in court." 15 The originals of the vouchers in question must be deemed to have been lost, as
even the Corporation admits such loss. Viewed upon this light, there can be no doubt as to the admissibility in
evidence of Exhibits 6 to 19 and 22.

Taking account of the foregoing evidence, together with Celso Rivera's testimony, 16 it would appear that:
Villarama supplied the organization expenses and the assets of the Corporation, such as trucks and
equipment;17there was no actual payment by the original subscribers of the amounts of P95,000.00 and
Set V Corporation Code * Doctrine of Peircing Corpo Veil Cases*Page 162 of 164

P100,000.00 as appearing in the books; 18 Villarama made use of the money of the Corporation and deposited incorporation that the acts and conduct of the corporation be carried out in its own corporate name because it
them to his private accounts;19 and the Corporation paid his personal accounts. 20 has its own personality.

Villarama himself admitted that he mingled the corporate funds with his own money. 21 He also admitted that The doctrine that a corporation is a legal entity distinct and separate from the members and stockholders who
gasoline purchases of the Corporation were made in his name 22 because "he had existing account with Stanvac compose it is recognized and respected in all cases which are within reason and the law. 29 When the fiction is
which was properly secured and he wanted the Corporation to benefit from the rebates that he received." 23 urged as a means of perpetrating a fraud or an illegal act or as a vehicle for the evasion of an existing
obligation, the circumvention of statutes, the achievement or perfection of a monopoly or generally the
The foregoing circumstances are strong persuasive evidence showing that Villarama has been too much perpetration of knavery or crime, 30 the veil with which the law covers and isolates the corporation from the
involved in the affairs of the Corporation to altogether negative the claim that he was only a part-time general members or stockholders who compose it will be lifted to allow for its consideration merely as an aggregation
manager. They show beyond doubt that the Corporation is his alter ego. of individuals.

It is significant that not a single one of the acts enumerated above as proof of Villarama's oneness with the Upon the foregoing considerations, We are of the opinion, and so hold, that the preponderance of evidence
Corporation has been denied by him. On the contrary, he has admitted them with offered excuses. have shown that the Villa Rey Transit, Inc. is an alter ego of Jose M. Villarama, and that the restrictive clause in
the contract entered into by the latter and Pantranco is also enforceable and binding against the said
Corporation. For the rule is that a seller or promisor may not make use of a corporate entity as a means of
Villarama has admitted, for instance, having paid P85,000.00 of the initial capital of the Corporation with the
evading the obligation of his covenant.31 Where the Corporation is substantially the alter ego of the covenantor
lame excuse that "his wife had requested him to reimburse the amount entrusted to her by the incorporators
to the restrictive agreement, it can be enjoined from competing with the covenantee. 32
and which she had used to pay the obligations of Dr. Villarama (her husband) incurred while he was still the
owner of Villa Rey Transit, a single proprietorship." But with his admission that he had received P350,000.00
from Pantranco for the sale of the two certificates and one unit,24 it becomes difficult to accept Villarama's The Corporation contends that even on the supposition that Villa Rey Transit, Inc. and Villarama are one and the
explanation that he and his wife, after consultation, 25 spent the money of their relatives (the stockholders) same, the restrictive clause in the contract between Villarama and Pantranco does not include the purchase of
when they were supposed to have their own money. Even if Pantranco paid the P350,000.00 in check to him, as existing lines but it only applies to application for the new lines. The clause in dispute reads thus:
claimed, it could have been easy for Villarama to have deposited said check in his account and issued his own
check to pay his obligations. And there is no evidence adduced that the said amount of P350,000.00 was all (4) The SELLER shall not, for a period of ten (10) years from the date of this sale apply for any TPU
spent or was insufficient to settle his prior obligations in his business, and in the light of the stipulation in the service identical or competing with the BUYER. (Emphasis supplied)
deed of sale between Villarama and Pantranco that P50,000.00 of the selling price was earmarked for the
payments of accounts due to his creditors, the excuse appears unbelievable. As We read the disputed clause, it is evident from the context thereof that the intention of the parties was to
eliminate the seller as a competitor of the buyer for ten years along the lines of operation covered by the
On his having paid for purchases by the Corporation of trucks from the Manila Trading & Supply Co. with his certificates of public convenience subject of their transaction. The word "apply" as broadly used has for frame
personal checks, his reason was that he was only sharing with the Corporation his credit with some companies. of reference, a service by the seller on lines or routes that would compete with the buyer along the routes
And his main reason for mingling his funds with that of the Corporation and for the latter's paying his private acquired by the latter. In this jurisdiction, prior authorization is needed before anyone can operate a TPU
bills is that it would be more convenient that he kept the money to be used in paying the registration fees on service,33whether the service consists in a new line or an old one acquired from a previous operator. The clear
time, and since he had loaned money to the Corporation, this would be set off by the latter's paying his bills. intention of the parties was to prevent the seller from conducting any competitive line for 10 years since,
Villarama admitted, however, that the corporate funds in his possession were not only for registration fees but anyway, he has bound himself not to apply for authorization to operate along such lines for the duration of
for other important obligations which were not specified. 26 such period.34

Indeed, while Villarama was not the Treasurer of the Corporation but was, allegedly, only a part-time If the prohibition is to be applied only to the acquisition of new certificates of public convenience thru an
manager,27he admitted not only having held the corporate money but that he advanced and lent funds for the application with the Public Service Commission, this would, in effect, allow the seller just the same to compete
Corporation, and yet there was no Board Resolution allowing it. 28 with the buyer as long as his authority to operate is only acquired thru transfer or sale from a previous
operator, thus defeating the intention of the parties. For what would prevent the seller, under the
Villarama's explanation on the matter of his involvement with the corporate affairs of the Corporation only circumstances, from having a representative or dummy apply in the latter's name and then later on
renders more credible Pantranco's claim that his control over the corporation, especially in the management transferring the same by sale to the seller? Since stipulations in a contract is the law between the contracting
and disposition of its funds, was so extensive and intimate that it is impossible to segregate and identify which parties,
money belonged to whom. The interference of Villarama in the complex affairs of the corporation, and
particularly its finances, are much too inconsistent with the ends and purposes of the Corporation law, which, Every person must, in the exercise of his rights and in the performance of his duties, act with justice,
precisely, seeks to separate personal responsibilities from corporate undertakings. It is the very essence of give everyone his due, and observe honesty and good faith. (Art. 19, New Civil Code.)
Set V Corporation Code * Doctrine of Peircing Corpo Veil Cases*Page 163 of 164

We are not impressed of Villarama's contention that the re-wording of the two previous drafts of the contract of years; and third, with respect to situs or territory, the restraint is only along the lines covered by the
sale between Villarama and Pantranco is significant in that as it now appears, the parties intended to effect the certificates sold. In view of these limitations, coupled with the consideration of P350,000.00 for
least restriction. We are persuaded, after an examination of the supposed drafts, that the scope of the final just two certificates of public convenience, and considering, furthermore, that the disputed stipulation is only
stipulation, while not as long and prolix as those in the drafts, is just as broad and comprehensive. At most, it incidental to a main agreement, the same is reasonable and it is not harmful nor obnoxious to public
can be said that the re-wording was done merely for brevity and simplicity. service.38 It does not appear that the ultimate result of the clause or stipulation would be to leave solely to
Pantranco the right to operate along the lines in question, thereby establishing monopoly or predominance
The evident intention behind the restriction was to eliminate the sellers as a competitor, and this must be, approximating thereto. We believe the main purpose of the restraint was to protect for a limited time the
considering such factors as the good will 35 that the seller had already gained from the riding public and his business of the buyer.
adeptness and proficiency in the trade. On this matter, Corbin, an authority on Contracts has this to say. 36
Indeed, the evils of monopoly are farfetched here. There can be no danger of price controls or deterioration of
When one buys the business of another as a going concern, he usually wishes to keep it going; he the service because of the close supervision of the Public Service Commission. 39 This Court had stated long
wishes to get the location, the building, the stock in trade, and the customers. He wishes to step into ago,40that "when one devotes his property to a use in which the public has an interest, he virtually grants to the
the seller's shoes and to enjoy the same business relations with other men. He is willing to pay much public an interest in that use and submits it to such public use under reasonable rules and regulations to be
more if he can get the "good will" of the business, meaning by this the good will of the customers, that fixed by the Public Utility Commission."
they may continue to tread the old footpath to his door and maintain with him the business relations
enjoyed by the seller. Regarding that aspect of the clause that it is merely ancillary or incidental to a lawful agreement, the
underlying reason sustaining its validity is well explained in 36 Am. Jur. 537-539, to wit:
... In order to be well assured of this, he obtains and pays for the seller's promise not to reopen
business in competition with the business sold. ... Numerous authorities hold that a covenant which is incidental to the sale and transfer of a trade or
business, and which purports to bind the seller not to engage in the same business in competition with
As to whether or not such a stipulation in restraint of trade is valid, our jurisprudence on the matter 37says: the purchaser, is lawful and enforceable. While such covenants are designed to prevent competition
on the part of the seller, it is ordinarily neither their purpose nor effect to stifle competition generally
in the locality, nor to prevent it at all in a way or to an extent injurious to the public. The business in
The law concerning contracts which tend to restrain business or trade has gone through a long series
the hands of the purchaser is carried on just as it was in the hands of the seller; the former merely
of changes from time to time with the changing condition of trade and commerce. With trifling
takes the place of the latter; the commodities of the trade are as open to the public as they were
exceptions, said changes have been a continuous development of a general rule. The early cases
before; the same competition exists as existed before; there is the same employment furnished to
show plainly a disposition to avoid and annul all contract which prohibited or restrained any one from
others after as before; the profits of the business go as they did before to swell the sum of public
using a lawful trade "at any time or at any place," as being against the benefit of the state. Later,
wealth; the public has the same opportunities of purchasing, if it is a mercantile business; and
however, the rule became well established that if the restraint was limited to "a certain time" and
production is not lessened if it is a manufacturing plant.
within "a certain place," such contracts were valid and not "against the benefit of the state." Later
cases, and we think the rule is now well established, have held that a contract in restraint of trade is
valid providing there is a limitation upon either time or place. A contract, however, which restrains a The reliance by the lower court on tile case of Red Line Transportation Co. v. Bachrach41 and finding that the
man from entering into business or trade without either a limitation as to time or place, will be held stipulation is illegal and void seems misplaced. In the said Red Line case, the agreement therein sought to be
invalid. enforced was virtually a division of territory between two operators, each company imposing upon itself an
obligation not to operate in any territory covered by the routes of the other. Restraints of this type, among
common carriers have always been covered by the general rule invalidating agreements in restraint of trade. 42
The public welfare of course must always be considered and if it be not involved and the restraint
upon one party is not greater than protection to the other requires, contracts like the one we are
discussing will be sustained. The general tendency, we believe, of modern authority, is to make the Neither are the other cases relied upon by the plaintiff-appellee applicable to the instant case. In Pampanga
test whether the restraint is reasonably necessary for the protection of the contracting parties. If the Bus Co., Inc. v. Enriquez,43the undertaking of the applicant therein not to apply for the lifting of restrictions
contract is reasonably necessary to protect the interest of the parties, it will be upheld. (Emphasis imposed on his certificates of public convenience was not an ancillary or incidental agreement. The restraint
supplied.) was the principal objective. On the other hand, in Red Line Transportation Co., Inc. v. Gonzaga,44 the restraint
there in question not to ask for extension of the line, or trips, or increase of equipment was not an
agreement between the parties but a condition imposed in the certificate of public convenience itself.
Analyzing the characteristics of the questioned stipulation, We find that although it is in the nature of an
agreement suppressing competition, it is, however, merely ancillary or incidental to the main agreement which
is that of sale. The suppression or restraint is only partial or limited: first, in scope, it refers only to application Upon the foregoing considerations, Our conclusion is that the stipulation prohibiting Villarama for a period of 10
for TPU by the seller in competition with the lines sold to the buyer; second, in duration, it is only for ten (10) years to "apply" for TPU service along the lines covered by the certificates of public convenience sold by him to
Pantranco is valid and reasonable. Having arrived at this conclusion, and considering that the preponderance of
Set V Corporation Code * Doctrine of Peircing Corpo Veil Cases*Page 164 of 164

the evidence have shown that Villa Rey Transit, Inc. is itself the alter ego of Villarama, We hold, as prayed for in part of a prior sale executed by Fernando in favor of the said corporation, which necessitated the latter to file
Pantranco's third party complaint, that the said Corporation should, until the expiration of the 1-year period the action to annul the sheriff's sale to Ferrer and the subsequent transfer to Pantranco, it is entitled to collect
abovementioned, be enjoined from operating the line subject of the prohibition. actual and compensatory damages, and attorney's fees in the amount of P25,000.00. The evidence on record,
however, does not clearly show that said defendants acted in bad faith in their acquisition of the certificates in
To avoid any misunderstanding, it is here to be emphasized that the 10-year prohibition upon Villarama is not question. They believed that because the bill of sale has yet to be approved by the Public Service Commission,
against his application for, or purchase of, certificates of public convenience, but merely the operation of TPU the transaction was not a consummated sale, and, therefore, the title to or ownership of the certificates was
along the lines covered by the certificates sold by him to Pantranco. Consequently, the sale between Fernando still with the seller. The award by the lower court of attorney's fees of P5,000.00 in favor of Villa Rey Transit, Inc.
and the Corporation is valid, such that the rightful ownership of the disputed certificates still belongs to the is, therefore, without basis and should be set aside.
plaintiff being the prior purchaser in good faith and for value thereof. In view of the ancient rule of caveat
emptorprevailing in this jurisdiction, what was acquired by Ferrer in the sheriff's sale was only the right which Eusebio Ferrer's charge that by reason of the filing of the action to annul the sheriff's sale, he had suffered and
Fernando, judgment debtor, had in the certificates of public convenience on the day of the sale. 45 should be awarded moral, exemplary damages and attorney's fees, cannot be entertained, in view of the
conclusion herein reached that the sale by Fernando to the Corporation was valid.
Accordingly, by the "Notice of Levy Upon Personalty" the Commissioner of Public Service was notified that "by
virtue of an Order of Execution issued by the Court of First Instance of Pangasinan, the rights, interests, or Pantranco, on the other hand, justifies its claim for damages with the allegation that when it purchased
participation which the defendant, VALENTIN A. FERNANDO in the above entitled case may have in the ViIlarama's business for P350,000.00, it intended to build up the traffic along the lines covered by the
following realty/personalty is attached or levied upon, to wit: The rights, interests and participation on the certificates but it was rot afforded an opportunity to do so since barely three months had elapsed when the
Certificates of Public Convenience issued to Valentin A. Fernando, in Cases Nos. 59494, etc. ... Lines Manila contract was violated by Villarama operating along the same lines in the name of Villa Rey Transit, Inc. It is
to Lingayen, Dagupan, etc. vice versa." Such notice of levy only shows that Ferrer, the vendee at auction of further claimed by Pantranco that the underhanded manner in which Villarama violated the contract is
said certificates, merely stepped into the shoes of the judgment debtor. Of the same principle is the provision pertinent in establishing punitive or moral damages. Its contention as to the proper measure of damages is that
of Article 1544 of the Civil Code, that "If the same thing should have been sold to different vendees, the it should be the purchase price of P350,000.00 that it paid to Villarama. While We are fully in accord with
ownership shall be transferred to the person who may have first taken possession thereof in good faith, if it Pantranco's claim of entitlement to damages it suffered as a result of Villarama's breach of his contract with it,
should be movable property." the record does not sufficiently supply the necessary evidentiary materials upon which to base the award and
there is need for further proceedings in the lower court to ascertain the proper amount.
There is no merit in Pantranco and Ferrer's theory that the sale of the certificates of public convenience in
question, between the Corporation and Fernando, was not consummated, it being only a conditional sale PREMISES CONSIDERED, the judgment appealed from is hereby modified as follows:
subject to the suspensive condition of its approval by the Public Service Commission. While section 20(g) of the
Public Service Act provides that "subject to established limitation and exceptions and saving provisions to the 1. The sale of the two certificates of public convenience in question by Valentin Fernando to Villa Rey Transit,
contrary, it shall be unlawful for any public service or for the owner, lessee or operator thereof, without the Inc. is declared preferred over that made by the Sheriff at public auction of the aforesaid certificate of public
approval and authorization of the Commission previously had ... to sell, alienate, mortgage, encumber or lease convenience in favor of Eusebio Ferrer;
its property, franchise, certificates, privileges, or rights or any part thereof, ...," the same section also provides:

2. Reversed, insofar as it dismisses the third-party complaint filed by Pangasinan Transportation Co. against
... Provided, however, That nothing herein contained shall be construed to prevent the transaction Jose M. Villarama, holding that Villa Rey Transit, Inc. is an entity distinct and separate from the personality of
from being negotiated or completed before its approval or to prevent the sale, alienation, or lease by Jose M. Villarama, and insofar as it awards the sum of P5,000.00 as attorney's fees in favor of Villa Rey Transit,
any public service of any of its property in the ordinary course of its business. Inc.;

It is clear, therefore, that the requisite approval of the PSC is not a condition precedent for the validity and 3. The case is remanded to the trial court for the reception of evidence in consonance with the above findings
consummation of the sale. as regards the amount of damages suffered by Pantranco; and

Anent the question of damages allegedly suffered by the parties, each of the appellants has its or his own 4. On equitable considerations, without costs. So ordered.
version to allege.

Villa Rey Transit, Inc. claims that by virtue of the "tortious acts" of defendants (Pantranco and Ferrer) in
acquiring the certificates of public convenience in question, despite constructive and actual knowledge on their

S-ar putea să vă placă și